AANP FNP Certification Exam with complete solution/ REAL EXAM QUESTIONS AND CORRECT ANSWERS(VERIFIED ANSWERS)|AGRADEAANP FNP Certification Exam with complete solution/

All diastolic murmurs are pathological. Murmurs Grades I-barely II-audible III- clearly audible. IV- first time thrill V-Steth edge VI-entire steth. EXAM

III first time audible, IV first time thrill

Fundal height 12 weeks

Fundal Height 12 weeks above symphysis pubis. EXAM

Fundus 16 weeks between symphysis pubis and umbilicus.
Fundus at 20 weeks is at umbilicus.
2 cm more of less from # of wk gestation is normal if more or less order US

3 month old infant with down syndrome, due to milk intolerance, mom started on goats milk; now has pale conjunctiva but otherwise healthy. Low HCT. What additional test would you order?

Iron, TIBC

3 months of synthroid, TSH increased, T4 normal, what do you do?

Increase Medication

3 ways to assess cognitive function in patient with signs/symptoms of memory loss

Mini mental exam

4 month old with strabismus, mom is worried……

tell her it is normal.

4 month old wont keep anything down, what is the main thing you look at?

Growth chart

6 month old closed anterior fontanel.

XRAY

Abnormal cells on PAP, what do you do next?

Refer for Colposcopy

CAGE ACRONYM

Cut down
Annoyed by criticism
Guilty about drinking
Eye opener drink

Causes of tachycardia

Fever
Anemia
Hypotension

Cranial nerves responsible for extraocular eye movements

CN 3,4,6

Definition of metabolic syndrome

cluster of conditions that increase risk of heart disease, stroke, diabetes.

diagnose trichomoniasis

wet prep

Elderly presents with atrophic vaginitis, small uterus, palpable 4×5 ovary, what do you do next?

Pelvic US

Epistaxis is most common in the area of the nose known as kiesselbachs triangle, where is this located?

Anterior septum

Definitive diagnosis of acute bacterial prostatitis

urinalysis and culture

GERD treatment

H2 is first line, give hs

Grade 3 cells on Pap, treatment?

LEEP
excision

Fingernail hematoma treatment?

drill hole and drain blood?

Increased risk of ectopic pregnancy

Salpingitis, or history of abortion, PID,

Koplick spots

Measles (rubeola). Grains of salt lesions inside mouth in Measles

Koplick spots

Measles (rubeola). Grains of salt lesions inside mouth in Measles

Legg-Calve-Perthes Disease

Avascular necrosis of the proximal femoral head

Lipid level of 1500, increased risk for?

Pancreatitis

Low HGB, Low HCT, High MCV indicates what?

Macrocytic anemia, B12 Def

Man with BPH, prostate feels on digital exam?

Enlarged, symmetrical, smooth

Man with HTN, CAD, present femoral pulses but absent pedal

Arterial Insufficiency

McMurray’s Sign
(+) palpable or audible click while extending with varus stress

Meniscus tears

Lachman’s Test

pivot shift test (ACL tear)

Newborn with foot turned in, what do you do?

refer to orthopedist

Osgood-Schlatter disease

Knee pain.
inflammation or irritation of the tibia at its point of attachment with the patellar tendon

Patient forgot to start Thanksgiving dinner and husband states she has trouble remembering tasks and trouble with organization. What is this indicative of?

Alzheimer’s

Pt has Barretts Esophagus, insurance no longer covers GI who was treating condition. Pt at FNP office wanting refill prescriptions. What do you do?

Refer to oncologist

Pt presents with rash on shoulder, erythematous maculopapular rash with center clearing and scaling?

Tinea Corporis

Pt presents with “bag of worms:, indicates?

Varicocele

Pt with atopic dermatitis, look for what other diseases?

Asthma

Pt with bleeding after menopause

endometrial biopsy, need to screen for cancer

Pt with hx of PID, increased rick for?

Infertility

Pt with HIV took high potency anti viral treatments and CD4 is >400, what does this indicate?

This is good. Want higher than 350

Pt with hx of htn and stroke, now having memory loss. What does this indicate?

Vascular dementia

Pregnant teacher with exposure to 5ths disease (SLAP CHEEK), what risk is there to the fetus?

slap cheek, PVB19, rash hands / feet Fetal death and birth defects

Quick assessment of patients fall risk? Timed Get up and Go

Timed Get up and Go

Red beefy tongue?

pernicious anemia

Rotator cuff injury presentation

disturbs sleep, arm weakness, dull ache

Shingles near eye

immediate referral to ophthalmology

Signs and symptoms of Roseola (6ths disease) ?

Viral infection
Can result in a maculopapular rash, but up to 70% of cases proceed without the rash stage
-Usually accompanied by a high fever (41°C or 105°F) that comes on quickly and lasts up to 3 days followed by rash
-Seizures may occur during this period.
-On the fourth day, the fever disappears and the rash appears, first on chest and trunk, then less prominently on the face and limbs.
-High fever, pink flat or raised rash

Treatment for chronic alcoholism:

12 step program

Treatment for Gonorrhea?

Rocephin 250mg IMx1 plus
Azithromycin 1 gm orally x1 to cover chlamydia. , or doxy 100 mg BID x7d. Green colored vaginal discharge, friable cervix. EXAM
Report to health department

Young female want birth control, forgets to take pills, does not want to get pregnant for at least 5 years:

IUD

Basal cell cancer

Waxy, pearly, telangiectasia, ulcer center lesion
-most common type of skin cancer caused by UV exposure. Metastatic is rare

Actinic Keratosis

Scaly red to yellow located in sun exposed area
-a precancerous skin growth that occurs on sun-damaged skin

Actinic Keratosis

Numerous round dry pink to red areas. Scaly red to yellow located in sun exposed area
-a pre squamous cell carcinoma occurs on sun-damaged skin
biopsy
cryo or 5fu cream

subungual hematoma tx

Make a hole and drain the blood

Moderate acne treatment

Without inflammation: Topical retinoid

Moderate inflammation: Topical Retinoid or benzoyl peroxide or Azaleic acic ( very expensive and hard to get covered)

PLUS ADD

Oral antibiotics: doxycycline or tetracycline or minocycline- (tetracyclines has been proven most affective for inflammatory acne)

Minocycline- long term use has been linked with pseudotumor cerebri

Oral contraceptives for hormone related

Spironolactone

heart murmur with holosystolic or pan systolic

Heart mumur with mid systolic

MR – radiate axilla, 5th ICS MCL, apex,

AS – radiate neck, 2ICS right sternal border
MR ASSH

Coarctation of Aorta

COA: bounding radial and weak femoral pulse
increase blood pressure in arms, and lower pressure in lower legs.

-congenital cardiac condition characterized by a narrowing of the aorta

Murmur Grade III – VI

Loud murmur easily heard

JVD caused by

-tension pneumothorax,
-Rt. sided heart failure,
-cardiac tamponade,
-traumatic axphysia from
Increase in portal pressure(LIVER) in venous side or cor pulmonale

pt. with gradual onset of fever, hemorrhages on nail beds, painful raised red nodules, rash on palms

endocarditis , painful red spots on fingers olser’s nodes, janeway legions rash on palms and soles.

Know the difference between Peripheral Arterial Disease and Chronic Venous Insufficiency or PVD. There was question about PAD.

PAD: Absence of pulse, decrease blow flowing down, PAIN, Dx doppler or ABI<0.9, TX exercise by walking or antiplatelet,


PVD: Volume, edema, discoloration, decrease blood going up,

Chronic Bronchitis Treatment

Smoking cessation
Pulmonary rehabilitation
Pharmacologic therapy
Supplemental oxygen

TB… PPD is positive if area of induration is:

>5 mm in an immunocompromised patient or close contact
>10 mm in immigrant, health care workers, drug user
>15 mm in a patient who lives in an area where TB is very rare.

what condition would make you order Lateral X-ray of the neck. Options include: Drooling, Unable to do ROM of the neck / stiff neck.

Croup/Epiglottitis

AV nicking (Arterioles pressing on vein of the eye)

HTN retinopathy

intraocular pressure (IOP)

Fluid pressure inside the eye; measured with tonometry

Rovsing sign

pain in the RLQ when the LLQ is palpated (indicative of appendicitis)

Pencil-like stools occur in an obstruction of what

Sigmoid descending colon
It’s a thin narrow stool and possible causes include colon cancer, diarrhea, IBS. Refer for GI colonoscopy

headache after trauma

SDH

migraine headache

-paroxysmal (sudden, periodic) attacks of mostly unilateral headache, often accompanied by disordered vision, nausea, or vomiting, lasting hours or days and caused by dilation of arteries.

4-12 hours,

abortive triptans

prophylaxis propranolol, TCA amitriptyline, anticonvulsants topiramate

Vitamin B12

Folate shares a close relationship with this other B Vitamin.

Vitamin B12

Cognitive deficits, glossitis, pernicious anemia,
Folate shares a close relationship with this other B Vitamin.

88/yr. old patient in for follow up secondary. She’s been treated with Tylenol for Joint arthritis. Her SED rate was checked after 6 weeks of treatment and it was 28. Normal range is from something to 25. How would you treat the pt.

be changed to NSAID, SED rate is a sign of inflammation

Which medication causes low sperm count for a patient

SSRI

grandiosity

Exaggerated belief in or claims about one’s importance or identity.
Bipolor

ADHD

A behavioral problem characterized by short attention span, restless movement, and impaired learning capacity.

pt. pap’s smear noted with Low Grade Squamous Intraepithelial Lesions and High Grade Squamous Epithelia Lesion noted on the report, what should NP do?

HPV test if not done. Refer for colposcopy

Chlamydia trachomatis

Doxycycline (+ ceftriaxone for gonorrhea coinfection)

Genital warts treatment

Cryotherapy
OR
Podophyllotoxin cream –
OR
Imiquimod (Aldara cream)

HIV pt. with antiviral and CD4 count still less than 200. What should NP tell the pt.

tell the pt. that he is qualified to be diagnosed with AIDS according to CDC

Most common cause of death in children

motor vehicle

Tanner 2

Tanner 2- female breast bud areola develops Male testes scrotum start to enlarge, scrotum gets darker

Tanner 4

Tanner 4- female nipples and areola become elevated from breast, secondary mound Male penis grows wider grows in length, darker scrotum

pt. expericieng memory loss, and increase in confusion and she has a history of stroke, HTN, What type of Alzheimer.

vascular dementia

presbycusis

a gradual loss of sensorineural hearing that occurs as the body ages

a pregnant female at slightly above symphysis pubic and Fundal height is 32cm (above the umbilical). What should be done

Ultrasound

Romberg test

cerebellar
-ask client to stand with feet at comfortable distance apart, arms at sides, and eyes closed
-expected finding: client should be able to stand with minimal swaying for at least 5 seconds

Direct Coombs test

r/o bilirubin

preeclampsia treatment

bed rest, laying on her side

Molluscum contagiosum treatment

Currettage, liquid nitrogen

pyloric stenosis

non bilious vomiting, olive like firm mass palpated on right upper quadrant

horizontal nystagmus that stops when eye is close to midline in a college student

Ménière’s disease ?

Nystagmus test

the involuntary jerking of the eyes as a person gazes to the side

eating, painful lump noted on the jaw that comes and go.

sialolithiasis).

Etopic Pregnancy: Risk Factors

Previous ectopic pregnancy
Prior fallopian tube surgery
Previous pelvic or abdominal surgery
Certain sexually transmitted infections (STIs)
Pelvic inflammatory disease
Endometriosis

cigarette smoking
age older than 35 years
history of infertility
use of assisted reproductive technology, such as in vitro fertilization (IVF)

Which among the list can cause increase in respiration

Options include (low oxygen, high oxygen, hypercapnia, hypocapnia)?

Which among the list can cause increase in respiration

hypercapnia

Osteoporosis Risk Factors (ACCESS)

A-lcohol Use
C-orticosteroid Use
C-alcium low
E-strogen low
S-moking
S-edentary lifestyle/s

ACCESS leads to OSTEOPOROSIS

to prevent fracture in a pt. with low vitamin d hydroxyl, high TSH and low Hct

VITAMIN D 600-800, CALCIUM 1000-1200.

OSTEOPOROSIS BONE ABSORPTION EXCEED BONE FORMATION.
LOW TSH= Hyperthyroidism- BONE DEMINERALIZATION.
NEED CALCIUM for BONE GROWTH

Carotid bruit

abnormal flow of blood through the carotid artery

Carotid bruit

abnormal flow of blood through the carotid artery due to atherosclerotic disease

Common causes of GERD

risk factors- alcohol, anticholinergic, CCB, chocolate peppermint, fatty, spicy, citrus foods, hormones, obesity, pregnancy, smoking, theophylline, exacerbated by CCB verapamil

Common causes of GERD

risk factors-
alcohol,
anticholinergic,
CCB, BB
chocolate peppermint, fatty, spicy, citrus foods,
hormones,
obesity,
pregnancy,
smoking,
theophylline

Zeprexa. What lab and intervention to put in place

CAUSES ELEVATED LIPDS, GLUCOSE, WEIGHT
monitor CBC for low WBC
weight- BMI q 3m
b/p, mental status, lips, prolactin, glucose

Weber test

Sensorineural loss Weber test(top of head) no laterization, normal finding, does not lateralize to either ear, bilateral hearing loss, if hear better in left ear, right sensorineural loss

Weber test

Sensorineural loss Weber test (top of head) no laterization- normal, does not lateralize to either ear- bilateral hearing loss, if hear better in left ear, right sensorineural loss.
SUN- sensorial lateralize unaffected ear
CAFFE- Conductive lateralize to affected ear

Assessment on patient with ascites

Dullness

Assessment on patient with ascites

Dullness to percussion

Varus Stress Test

application of a lateral force to the medial aspect of a joint in an attempt to create a gap in the lateral joint line, thereby testing the stability of the lateral aspect of the joint
LCL

Varus Stress Test

TEST LCL (lateral-vaRus)
McMurrays- Meniscus CLICK
application of a lateral force to the medial aspect of a joint in an attempt to create a gap in the lateral joint line, thereby testing the stability of the lateral aspect of the joint
LCL

German Measles (Rubella)

Pink, papular rash (similar to measles but paler) first appears on face, then spreads. Distinguished from measles by presence of neck lymphadenopathy and absence of Koplik spots.

patient with IOP of 32mmHg, what do you expect during fundoscopic exam

patient with IOP of 32mmHg, what do you expect during fundoscopic exam

increase cup-to-disc ratio
retinal hemorrhage
optic nerve asymmetry and pallor
measured w tonometry

BPH and urge incontinence

anticholinergics/oxybutynin, impamine/tricyclic/antidepressant

Anticholinergic- can’t think or blink, can’t see (Increase eye pressure) or pee, can’t spit or shit, SADCCUB sedation, anorexia, dry mouth confusion, constipation, urinary retention, BPH

BPH and urge incontinence

TX anticholinergics/oxybutynin,
impamine/tricyclic antidepressant

seasonal affective disorder (SAD)

a mood disorder caused by the body’s reaction to low levels of sunlight in the winter months

intussusception

telescoping obstruction of the intestines, cuts off blood supply, fatal, sudden loud crying, comes/goes, vomiting, blood/mucus mixed with stool, SAUSAGE LIKE
MASS

intussusception

telescoping obstruction of the intestines, cuts off blood supply, fatal, sudden loud crying, comes/goes, vomiting, blood/mucus mixed with stool, SAUSAGE LIKE
MASS CURRENT JELLY STOOL

IBS (irritable bowel syndrome)

An intestinal disorder causing pain in the belly, gas, diarrhea, and constipation.
due to Small intestinal bacterial overgrowth, or SIBO

IBS (irritable bowel syndrome)

SS pain in the belly, gas, diarrhea, and constipation. Pencil like stool.
Caused by: Small intestinal bacterial overgrowth, or SIBO
TX fiber, avoid gas foods, antispasmodics, decrease life stress

Osteoporosis treatment

TX first line is bisphosphonates alendronate, Fosamax, calcium500 mg, vitamin d thru food and supplementation, testosterone, wt bearing exercise

Osteopenia- increase calcium dark green vegetables, salmon, sardines, soy and OJ

Hormone (estrogen) replacement therapy (HRT) slows bone loss
Natural progesterone cream prompts new bone growth
Statins increase bone mineral density

Osteoporosis treatment

TX BIOPHOSPHATES alendronate, Fosamax, CALCIUM 500 mg, vitamin D food and supplementation, testosterone, WT BEARING EXERCISE

CAUCASIAN and ASIAN affected most
RISK PPI, STATIN, STEROIDS, THYROID,

Osteopenia- increase calcium dark green vegetables, salmon, sardines, soy and OJ

Hormone (estrogen) replacement therapy (HRT) slows bone loss
Natural progesterone cream prompts new bone growth
BONE DENSITY >2.5

Retinoblastoma

white reflection in child’s pupil

for staph aureus infection (skin) with pus

MRSA- TX Bactrim or tetracyclines?

hyperparathyroidism

high calcium

Hyperthyroidism treatment

methimazole, PTU-propylthiouracil (preferred in pregnancy)
Radioactive iodine, Beta blockers

Mammography Screening

-Age 45 – 54 yearly mammogram
-55 and older every 2 years

Fifth’s Disease (Erythema Infectiosum)

B19: lytic infection, respiratory transmission
Sx: flushed rash/fever in kids
Px: fever, get better in a week

pt has AOM but has hives on Amoxicillin and N/V with erythromycin, what meds to give

TREATMENT: Amoxicillin (first line), then Augmentin, Omnicef, Ceftin, Levaquin. If your patient is only PCN allergic do azithromycin or clarithromycin.

chlamydia in pregnancy

Azithromycin 1 gm PO x1 or Amoxicillin 500 mg PO TID x7d. Test of cure 3 weeks after completion of treatment (PREGO). EXAM

papilledema

optic disc swollen w/ blurred edges due to increased ICP EXAM

actinic keratoses

Precursor to squamous cell carcinoma. “numerous dry round and pink to red lesions” with a rough and scaly texture. Does not heal. Slow growing in sun exposed areas. Diagnosis: BIOPSY Golden Standard. Treatment: Sm. (cryotherapy), Lrg. (5-FU cream)- which causes ur skin to ooze, crust, scab, redness EXAM

Allergic Conjunctivitis

“stringy; increased tearing” PO antihistamines. Type I sensitivity. Typically bilateral. Rhinitis and allergic shiner.

COPD

COPD- Gold 1-2- SABA or SAMA ON EXAM.BASCIALLY ANTICHOLINERGIC FIRST LINE FOR COPD ON EXAM

Gold 1-2 that are poor controlled- LAMA or LABA. May use SABA for rescue.
Gold 3-4 LAMA first line. If poor use LAMA plus LABA. Alternative is LABA + ICS.
Gold 3-4- refer
SABA- Albuterol, levoalbuterol (terol)
LABA- Formeterol, salmeterol (Terol)
SAMA- Atrovent Ipatropium (tropium)
LAMA- Spiriva Tiotroium (tropium)
COPD long term is OXYGEN

CN IX Glossopharyngeal

– Shoulder shrug/ ROMBERG test EXAM

CN V Trigeminal

Herpes. CORNEAL ABRASION. EXAM

CN VIII Vestibulocochlear

ears 8 EXAM

CN VII Facial

BELLS EXAM

ACEI contraindicated

pregnancy

Safe to give varicella/MMR

Do not give <12 mo. EXAM QUESTION

Acne Vulgaris

common acne. Retin-A, acne worsens 4-6 weeks if no improvement in 8-12 weeks increase dose or add erythromycin, benzoyl peroxide.

Acne Rosacea

– chronic small acne like papules/pustules around nose mouth chin. TREATMENT- Metrogel, Azelex. Low dose tetracycline. Clindamycin. EXAM

Cataracts

is on EXAM in elderly night vision issues. Opaque

Kawasaki disease

– acute high fever, enlarged lymph. BRIGHT RED RASH, conjunctivitis, dry cracked lips, strawberry tongue, Swollen hands, feet, AFTER the fever resides the rash PEELS on hands/feet. Treated with high dose aspirin and gamma globulin. This is TOXIC and VASCULAR, think blood clots, heart problems etc. Treat: high dose aspirin. EXAM

Erythema migrans

Erythema Migraines- (stage 1 Lyme) Target bulls-eye, usually appears in 7-14 days POST bitten tick. Rash is hot to touch with rough texture, flu like symptoms. DX: B. Burgdorferi via ELISA, confirm with western blot. Increased ESR. TREATMENT: Less than 7 Amoxicillin or cefuroxime axetil. Older than 7 Doxycycline. EXAM

Rocky Mountain Spotted Fever

Inc. fever, chills, N/v, photophobia, myalgia, arthralgias THEN 2-5 days later you develop a petechial rash on forearms, ankles, wrists, that spreads towards trunk and becomes generalized. Think rocky NC/OK/AK/TN/MO. DX: PCR essay with Rickessetti Antigen TREATMENT- doxycycline. EXAM

Addison’s

Addison’s- deficient in cortisol (think low sodium, blood sugar, but Increase K. You must give cortisol. (Diagnosis Plasma Cortisol <5 mcg/dl @ 0800.) EXAM

rheumatoid arthritis

Early morning stiffness, sausage joints. Symmetrical involvement. Longer stiffness than OA. Joint space narrowing. Pain, warm, tender, swollen, things. TREAT: NSAIDS, steroids, DMARDS, TNF. Only has BOUCHARDS, SWAN NECK IS DESCRIPTION ON EXAM

Osteoarthritis

Large weight bearing joints. Early morning stiffness with inactivity. Has both nodes. FIRST LINE Acetaminophen. EXERCISE: Isometric exercises for knee OA. Non-weight bearing, like biking, swimming, stationary bike. EXAM

Osteoporosis

OSTEOPOROSIS = WEIGHT BEARING- walking, lifting weights etc. bones are forced against gravity. EXAM

Fibromyalgia polymyalgia

Fibro- 11/18 points. Widespread pain for at least three months. EXAM. polymyalgia Tx prednisone 12 months. Must taper off. RISK for Temporal arteritis, DX BX, elevated ESR

Serotonin syndrome

Acute Serotonin Syndrome- Dilated pupils, high fever, muscular rigidity, mental status changes, hyperreflexes, clonus, uncontrolled shivery. You get this from SSRI, MAOIs, TCA. Could be potentially life threatening. EXAM

Fundal Height

Fundal Height 12 weeks above symphysis pubis. EXAM TOPICS
Fundus 16 weeks between symphysis pubis and umbilicus.
Fundus at 20 weeks is at umbilicus.
2 cm more of less from # of wk gestation is normal if more or less order US.

Psoriasis

Psoriasis- Inherited. Pruritic erythematous plaques, fine silvery-white scales with pitted fingernails. Scalp, elbows, knees, sacrum, intergluteal folds.
(Koebner phenomenon- new psoriatic plaques form over skin trauma)
(Auspitz sign- pinpoint bleeding when plaques are removed).
TREATMENT: Topical steroids, Tar preps (mild). For (severe) do anti-TNF, or immunologic.
D

Dacrocystitis

Darcryotosis lacrimal sac, rub down towards mouth. If think secondary infection abx. EXAM how it presents.

Acne Rosacea

Acne Rosacea- chronic small acne like papules/pustules around nose mouth chin. TREATMENT- Metrogel, Azelex. Low dose tetracycline. EXAM

Parathyroid hormone

PTH is responsible for calcium loss or gain from bones, kidneys, and GI tract. EXAM

Diabetic Retinopathy

Diabetic Retinopathy-Cotton wool spots (moderate retinopathy), micro-aneurysms. ALSO RETINAL HEMORRHAGES ON CENTER OF EYE APPEAR ORANGE RED

HTN Retinopathy

Hypertensive Retinopathy- Copper/silver wire arterioles. AV nicking(mild retinopathy). Retinal Hemorrhages. EXAM

Wilms Tumor

Wilms tumor (Nephroblastoma)- Not painful. Asymptomatic abd mass does NOT cross the midline. 2-3 y. o.d. do not palpate. Do ABD US. PUNT. Think Nephro doesn’t cross. Stays where kidney is. EXAM

Primary Amenorrhea

Primary amenorrhea: NO menarche by 15 y. with or w/o secondary sex characteristics.

Aphthous stomatitis

Cancer sores. Aphthous stomatitis: painful shallow ulcers heal 7-10 days. Magic mouthwash.

Temporal arteritis

Temporal arteritis- one temple indurated cord like gold stand. Biopsy. Abrupt visual changes blindness, inc. ESR. CPR. Most have POLYMYALGIA RHEUMATICA. Treat high dose steroids.

Atopic Dermatitis (eczema)

Inherited. Extremely itchy. On flexural folds, neck, hands. Inc. IgE. “small vesicles that rupture leaving painful, bright-red, weepy lesions” they become lichenified from itching. First line: Topical steroids. Avoid hot water/soaps. PO antihistamines. EXAM

Tinea Corporis

ring like itchy rash, slowly enlarge central clearing”-Treatment: most respond to topical antifungals, if severe do oral Lamisil. EXAM AZOLE ending

Cellulitis

Deep dermis poor demarcated low legs. EXAM/ MULTIPLE QUESTIONS. DVT RISK, DM WITH CELLULITIS WATCH FOR OSTEOMYLITIS.

Erysipelas

Group A strep, painful, Upper dermis, clear demarcated, cheeks, shins.
TREATMENT- Dicloxacillin QID x10d. Cephalexin, Clinda. PCN ALLERGY? Do Azithro x5d.
MRSA TREATMENT: Bactrim, doxy, mino, clinda. If sulfa allergy do not use Bactrim.

Varicella Zoster

“contagious 48 h. before, until all lesions crusted over” low grade fever, generalized lymphadenopathy, intense itching, erythematous macules, papules develop over macules, then vesicles erupt. “initially on trunk, then scalp and face” TREATMENT supportive, antihistamines, acyclovir 20mg/kg 5xd. If given first 24 hours works best. EXAM

Impetigo

Impetigo-Gram positive. Itchy pink-red lesions, evolve into vesiculopustules that rupture. If bullous-large blisters. Severe- Keflex, dicloxacillin. PCN Allergic-Azithro, clinda. If NO BULLAE- Bactroban. EXAM

Scarlet fever- scarlantina

“sandpaper textured-pink rash with sore throat” strawberry tongue, rash starts on head and neck, spreads to trunk. The skin THEN desquamates. EXAM

Lichen planus

LICHEN PLANUS: SMALL FLAT TOPPED, RED TO PURPLE BUMPS THAT MAY HAVE WHITE SCALES/FLAKES.. WHISPY GREY WHITE STREAKS CALLED WICHHAMS STRIAE. INNER WRISTS FOREARMS, AND ANKLES. IF ON SCALP CAUSE HAIR LOSS. Causes hep C, medications, contact with chemicals. EXAM

Spider bite

fever chills, n/v, located arms, upper legs, or the trunk. Biten area becomes swollen, red, and tender, and blisters appear within 24-48 hours. Necrotic in center, which kills the tissue. Ice packs to wound and cold inactivates the toxin, tx like cellulitis of the skin, abx ointment at first, watch etc. Exam

Pityoris rosea

Pityoris rosea itchy, herald patch, xmas tree pattern, rash hands soles/feet think to test for secondary syphilis RPR then VDRL are screening, then dx FTA-ABS. EXAM

Corneal abrasion

Corneal Abrasions- Round/Irregular. Was on EXAM.

Acute Angle Closure Glaucoma

acute/severe halos, cupping optic nerve, cloudy cornea, mid-dilated oval pupil. ER STAT. EXAM

Conductive

Conductive: Lateralization to bad ear. Rinne- BC > AC.
Rinne (1st mastoid, 2 front of ear, time each area).
Weber: Tunning fork midline. CN 8 (acoustic). EXAM

Koplik spots

Koplik Spots- “clusters sm. Size red papules w/ white centers in the buccal mucosa by lower molars”. Rubeolla. Fever, conjunctivitis, coryza, cough (3c). Morbiliform rash. EXAM

Sensorineural

Sensorineural: Lateralization to good ear. Rinne- AC > BC.

OME

Ear pressure, popping, muffled hearing, chronic allergic rhinitis, sterile serious fluid is trapped in the middle ear. TM should NOT BED RED. TM may bulge or retract. TREATMENT: Oral decongestants, steroid nasal spray, treat like allergies. Usually Painless. Weber- Lateralization to affected ear. Rhinne- BC > AC. PRECEDES OR USUALLY FOLLOWS AOM. SUPPORTIVE CARE AND WAIT 3 MOS SOMEX. EXAM

Presbycusis

sensorineural loss without lateralization. Involves the inner ear. Symmetrical progressive. Human speech lost first. AGING ADULT EXAM

OE

Otitis Externa (swimmers ear)- Pseudomonas aeruginosa. (other- S. aureus). External ear pain- d/c itching, hearing loss, tragus, green d/c. TREATMENT: Corticosporin, Cipro EXAM

Sinusitis

TX AMOXICILLIN OR AUGMENTIN ALLERGY MACROLIDE

Meiniers disease

VERTIGO TINNITUS, HEARING LOSS. nystagmas

Mono

test heterophile antibody test. ON EXAM

MR. ASS

(Systolic Murmur) Only systolic murmurs will radiate to a location on the exam.

Mitral Regurg

(Holo/pansystolic)- radiates to axilla. Think Mitral area 5th ics MCL.
Aortic Stenosis (mid systolic ejection) radiates to neck. Think 2ics rsb.

All diastolic murmurs are pathological. Grades Murmurs

I-barely II-audible III- clearly audible. IV- first time thrill V-Steth edge VI-entire steth. EXAM

MVP

MVP- S2 click, followed by systolic murmur. Asymptomatic. MVP with palpitations is treated with BB. LATE SYSTOLIC.

S3- HF,
S4-LVH stiffening,

S3- HF, Kentucky, early diastole. Abn >35. Bell EXAM
S4-LVH stiffening, Tennesse, late diastole. “Atrial kick/gallop” EXAM

Isolated Systolic HTN

CCB

PAD/ PVD

PAD/ PVD (same)- Nocturnal pain relieved by lowering legs, poor pulses, dependent rubor, intermittent claudication, atrophy, shiny, hairless, cold feet. Initial do a pulse check, ABI 0.9 or less is PAD. Ateriography is the most DEFINITIVE test. Try to develop collateral circulation. Otherwise- Trental, Pletal. EXAM

CVI

CVI- Impaired venous return. Achy legs relieved by elevation, edema after prolonged standing, night cramps, brownish discoloration, cold, ulcers. Etc. do support stockings. EXAM

blood pressure

BP – ST 1 (140-159/ 90-99), if you know this you will get the rest!! Normal is <120/80. ELERGLY OVER 60 150/90 IS OK. ISH WILL INCREASE SYSTOLIC NOT DIASTOLIC. ON EXAM.

Thiazide diuretics

no sulfa allergies, hyperuricemia, hypokalemia, hypomagnesia, hyponatremia, hyperglycemia, hypertriglycerides. ON EXAM

Statin

Must check LFT before starting Statin. Know when to start statins and what to check for to decide mod-high dose statins. ON EXAM

Pulses paradoxus

Pulsus paradox Apical pulse can still be heard even though the radial pulse is no longer palpable. Certain issues cause impairment with diastolic filling, 10 or greater drop in the SYSTOLIC pressure. I think her patient had asthma and their pressure dropped by 10 etc. ON EXAM

Emphysema

Emphysema Lungs- Percussion-HYPERENNOSANCE tactile frem + egophony- dec. CXR- flattened diaphragms with hyperinflation. Inc. AP diameter, accessory muscles, pursed-lip breathing, weight loss. ON EXAM

Acute Bacterial Pneumonia- CXR

middle lobe. ON EXAM

OSA

does not include Microglossia which is an absent tongue congenital. EXAM

TB

fatigue, fever, cough. Never do fewer than 3-4 drugs initially if positive, then u can narrow it down. Latent TB usually treated with INH. If u suspect ACTIVE TB order, NAAT, C&S, AFB. The AFB is not diagnostic. SPUTUM FOR C & S if gold standard. Deep morning cough collected for three “consecutive days”. TB is usually upper lobes.

TPO

TPO- this lab is off MEANING ELEVATED in BOTH hyper/hypo thyroidism. TPO is GOLD stand for diagnosis in Hashimotos. But you always want to order a TSH first, THEN ur thyroid panel do not get ahead of yourself. Check ur TSH lab on both in 6-8 weeks but never sooner than 6 weeks that is how long these meds take to work. TOPIC ON EXAM

hyperthyroid

Hyperthyroid- Low TSH, high “FREE” T4/T3. ALWAYS DO FREEs. Graves disease-autoimmune. Lid lag, exophthalmos, everything is hyper (body wise). Treatment: PTU/Tapazole. PTU PREFER IN PREGNANCY
RAIU-no w/ prego. Destroys thyroid, lifelong treatment for hypo then.

A1C > 9

If you are already on TWO oral drugs for diabetes and A1c is 9 or higher, start BASAL insulin. If you cannot tolerate metformin and your A1c is 9 or higher start BASAL insulin. ON EXAM

Parathyroid

For parathyroid- dx blood test. You will have elevated calcium because your parathyroid is releasing too much from bones and this will just cause it to float around and not help ur bones. TX: BIPHOSPHANATES FOR SECONDARY HYPERPARATHY. EXAM

cushings

Central obesity, moon face, purple striae, hairy, hypertension, elevated plasma CORTISOL in AM. “INC BS, SODIUM” Dec K. You must draw cortisol levels in the morning.

Fructosamine test

checks sugar for past 2-4 weeks.

triglycerides

causes pancreatitis >500. If >500 treat with Niacin or Fibrate or Niaspan. If your patient is already on NIACIN you can add a fibrate like (LOPID/TRICOR). Apparently an insulin infusion works also. ON EXAM

Pancreatitis

diagnosed with amylase / lipase draw. Amylase beings 2-12 h. Lipase 4-8 hours. Lipase however is MORE specific and sensitive to alcoholic pancreatitis. ACUTE: Grey Turner/ Cullen sign. Abd pain that rates to midback “boring” epigastric pain. Fever, n/v. EXAM TOPICS

Metformin

Metformin – monitor BUN, Creatinine.
contraindicated renal and liver disease
ARF creatinine up and GFR down.
affects liver enzymes, weight loss, avoid in alcohol drinkers, lactic acidosis, diarrhea flatulence
CT: hold 24 hr before and 48hr after

Polycythemia vera

slow growing blood cancer. blood too thick, clots. bone marrow to many RBC. risk bleed , anemia, CBC

CCB

BLE edema- walk around

Kava Kava

anxiety and insomnia, don’t mix w sedating benzos.

Bipolar med

lithium- monitor TSH, toxicity bind to TH cause hypothyroidism.

Depression med
acute Anxiety/ panic med
GAD med

depression SSRI
panic benzo, Xanax, ativan
GAD SSRI
SNRI- Buspar, Effexor, Cymbalta- Taper OFF

SNRI/ Benzo Discontinuation

Buspar, Effexor, Cymbalta and Benzo Taper OFF

Thiazide GLUT-

Glyceridemia
Lipidemia
Uricacidemia
Triglyceridemia
HYPO-Kalemia

ACE/ARB contraindications

pregnancy
Renal failure
Renal Stenosis

METABOLIC SYNDROME

METABOLIC SYNDROME
NO THIAZIDES,CCB (HF)
NO TZD ACTOS (Pioglitazone)

GIVE Metformin, ACE or ARB- kidney protective, BB- causes hypoglycemia,

Triglyceride < 300 what do we do first?
Triglyceride in 300 plus risk for ___ what do we do?

Lifestyle modification
Pancreatitis, Niacin then add, fenofibrates

Serotonin syndrome ss

hyper rigidity, fever, myoclonus, dilated pupils, AMS, hyper reflexes, from SSRI or mix w MAOI and TCA

NSAID contraindicated in

HF and ARF
increase BP impair renal prostaglandin and sodium retention

strabismus

misalignment of eyes, abnormal after 6 months

macular degeneration

loss of central vision

retinal detachment

floaters, curtain, flashes

Xanthelasmas

sharply demarcated yellowish deposit of cholesterol underneath the eye

Red reflex absent

retinoblastoma (leukorrhea) , cataracts, glaucoma.
Will have white reflex

Mono, return to play and Dx

4-6 weeks when spleenolmegaly resolves
Mono spot/ Heterophile

Viral stomatitis

ulcer on cheek
(Aphthous)- viral canker sore

cholesteatoma

chronic sinusitis or OM. cauliflower, foul-smell, hearing loss.
erodes bones in face affects facial CN 7.
Benign- risk hearing loss refer SURGERY

Acute Closed Angle Glaucoma
Vs
Open Angle

Acute: Sudden pain, halos, cupping, dilated, cloudy , IOP, HA, refer ED

Open (primary) : CN 2, gradual loss peripheral vision first

Papilledema

Swollen, optic disc, increase cup to disc ratio, HA, ICP, HTN,

Acyclovir (cheapest)

200mg 5 x day

Pt on PPI, has osteoporosis has a cough,

refer for EGD r/o Barrett’s.

allergic rhinitis

blue pale turbinate clear drainage. Tx inhaled corticosteroids

hordeolum Vs
chalazion
blepharitis

hordeolum: painful swollen red warm abscess TREAT hot compress erythromycin, dicloxacillin.
chalazion does not hurt
Blepharitis always unilateral, Tx baby shampoo warm compress

Sialolithiaisis

painful lump, calculi or salivary stones.
sub mandibular gland whartons; duct.

Roseola infantum- Sixth disease

viral, young children, high Fever 3-4 days followed by maculopapular rash

Herpetic keratitis

ocular herpes – blurr vision, inflammation of eye; gritty feeling, conjunctivitis, sharp pain, and photophobia- AVOID SUN
REFER OPTHO Tx acyclovir

Cranial nerves responsible for extraoculomotor movements?

a. 2, 3, 6?
b. 3, 4, 6?

Patient with hx of hypertension and stroke, now having memory loss and confusion – indicates what?

a. Alzheimer’s
b. Vascular Dementia?

CAGE acronym

?

Treatment for chronic alcoholism?

?

Frail elderly mammogram breast tissue?

?

Patient has Barrett’s esophagus, insurance no longer covers gastroenterologist who was treating condition, patient at the FNP office, wanting a prescription for medication… What should the FNP do?

a. Refer to oncologist?
b. Refill prescription?

GERD treatment?

a. PPI? Per American College of Gastroenterology, treatment should begin with a PPI.
b. H2? An H2 is inferior to PPI’s.

3 months on Symmetrel, TSH increased, T4 normal, what do you do?

a. Increase medication?
b. Decrease medication?

Lipid level of 1500, increased risk for?

a. pancreatitis?

Frail elder, increased creatinine, indication?

?

Fingernail hematoma treatment?

a. drill hole and drain blood?

Abnormal cells on PAP, what do you do next?

a. Refer for colposcopy?

Red beefy tongue?

a. Pernicious Anemia?
b. Iron Deficiency?
c. Folate Deficiency?

Low Hemoglobin, Low Hematocrit, High MCV – Indicates what?

?

Koplick Spots?

a. Measles?

Signs and Symptoms of Roseola?

Pregnant teacher, with exposure to Fifths’ Disease – what risk is there for the fetus?

6 Month Old with Closed Anterior Fontanel?

3 Month Old infant with Down Syndrome, due to milk intolerance, mom started infant on Goat’s Milk; now has pale conjunctiva but otherwise healthy; with low Hematocrit, what other test would you order?

a. CBC
b. Iron, TIBC

4 Month Old with Strabismus, mom worried, what do you tell her?

a. Normal?

4 Month Old “won’t keep anything down”, what is the main thing you look for?

a. Dehydration?

Man with HTN, CAD, present femoral pulses, but absent pedal pulses?

a. DVT?
b. Venous Insufficiency?
c. Arterial Insufficiency?

Diabetic patient with foot laceration, at risk for what?

a. foot ulcer?
b. acute osteomyelitis

Definition of metabolic syndrome?

Treatment for Gonorrhea?

a. Rocephin IM and Zithromax po
b. Doxycycline po

Diagnose trich?

a. Wet Prep?
b. KOH?

Patient with history of PID, increase risk for?

a. Infertility?

Increased risk for ectopic pregnancy, history of…?

a. Salpingitis

Newborn with foot turned in (“toeing in”), what do you do?

a. Refer immediately to orthopedist?
b. Routine followup?

Osgood Schlatter Disease pain location?

Growth plate fracture (Salter-Harris fx) location and pain?

Rotator Cuff injury presentation?

Causes of tachycardia?

a. Fever?
b. Anemia?
c. Hypotension?

Definitive diagnosis of acute bacterial proctatitis?

a. Urinalysis and Culture
b. Vigorous Massage of Prostate to Release Fluid for Culture?
c. Urethral Culture?

Man with high BPH, perform digital rectal exam – how does prostate feel?

a. Asymmetrical, Nodular, Firm
b. Symmetrical, Boggy
?

Patient present with “bag of worms”, what does this indicate?

a. Varicocele

Legg-Calve-Perthes disease?

Avascular necrosis of the proximal femoral head…

McMurray’s Sign

Epitaxis is most common in the area of the nose known as Kiesselbach’s Triangle, where is this located?

a. anterior septum?
b. middle turbinate?
c. sinus turbinate?

Elderly presents with atrophic vaginitis, small uterus, and palpable 4×5 ovary, what do you do?

a. Pelvic ultrasound?

Grade 3 cells on Pap, treatment?

?

Patient with bleeding after menopause?

a. Endometrial biopsy?

Young adult female, wants birth control, forgets to take pills, doesn’t want to get pregnant for at least 5 years, what do you suggest?

a. Intrauterine Device?

Shingles near eye, patient wants cream and analgesic, what do you do?

a. Order Acyclovir cream and analgesic?
b. Immediate referral to opthamologist?

Patient presents with rash on shoulder, erythematous maculopapular rash with center clearing, and scaling?

a. Tinea Pedis?
b. Tinea Corporis?
c. Psoriasis?

Peripheral vision loss = ?
Central vision loss = ?

Patient with Atopic Dermatitis, look for what other diseases?

a. Asthma?

Patient forgot to start Thanksgiving dinner, and husband states that she has trouble remembering tasks and has trouble with organization… What is this indicative of?

a. Alzheimer’s?
b. Delerium?

Quick assessment of patients fall risk?

a. Timed Get up and Go?

Mitral Regurgitation = sound and heard best at?
Mitral Stenosis = ” ” ” ” ” “?
Mitral Prolapse = ” ” ” ” ” “?
Mitral Sclerosis = ” ” ” ” ” “?
Aortic Stenois = ” ” ” ” ” “?
Aortic Regurgitation = ” ” ” ” ” “?
Aortic Sclerosis = ” ” ” ” ” “?

3 ways to assess cognitive function in patient with signs/symptoms of memory loss – all but which of the following?

a. Mini Mental
b. Depression Screen
c.
d.

Patient voices aching when ambulating? This question did not specify if it was bilateral or unilateral Mentioned something about “rubor” but I can’t remember what it said exactly…

a. DVT?
b. Venous Insufficiency?
c. Arterial Insufficiency?

Patient with HIV took high-potency anti-viral treatments and CD4 is >400, what does this indicate?

a. Patient has full-blown AIDS as defined by the CDC?
?

Patient has following labs: listed serology for Hepatitis, and then had to know if they had an active infection or immune, etc…

how long will a cough last for acute bronchitis?

up to three weeks is completely normal

how do you treat acute bronchitis?

95% cases are viral– tessalon pearls to help with cough. * prednisone is never the answer **

explain the following views of chest xrays: AP, PA, PA and lateral
which one do you want for pneumonia patient?

AP- shows the heart predominantly because its from front to back
PA- shows the lungs predominantly because its back to front
lateral shows fluid line

what is the gold standard for diagnosing community acquired pneumonia

chest xray- PA/lateral, repeat post treatment in 6 weeks (repeat not really done anymore)

treatment guidelines for CAP- healthy adult no comorbidities

MAD LUNG
amoxicillin 1 gram TID (best choice)
doxycycline 100 mg twice daily
macrolide (mycin) in areas with low macrolide resistance so not the best choice

treatment guidelines for CAP- adult with comorbidities

monotherapy: respiratory quinolone (levofloxacin 750 mg daily)
combination therapy: augmentin or cephalosporin PLUS macrolide or doxy

what are common respiratory fluroquinolones?

Think FLOXACIN– moxifloxacin, gemifloxacin, levofloxacin

what are common macrolides for CAP

think “MYCIN” — azithromycin, clarithromycin

what are common tetracyclines for CAP

doxycycline

what are common beta lactams for CAP

amoxicillin, augmentin (amox with clav), cefpodoxime, cefuroxime

how would you treat a pregnant 29 year old with pneumonia?

Amoxicillin 1 g TID– cant give levaquine (flouroquinolones no in pregnancy due to fetal ligament destruction, doxycycline no in pregnancy stains tooth bones in fetus)

how long do you give antibiotics to patient with pneumonia?

5-10 days, you always want to continue antibiotics for 3 more days after clinically stable (no fever)

When do you get a chest xray for pneumonia?

it is the gold standard for diagnosis, you do not need to do resolution chest xray unless things are lingering or you suspect something else.

When do you give pneumococcal vaccine?

adults > 65 years old you give PPSV23 or you can give both PCV 13 and PPSV23 but must be 1 year apart

adults > 65 with immunocompromising condition give both pcv 13 and ppsv23

adults 19-64 at increased risk of pneumococcal disease (asthma, copd, smokers, cv dz) – give PPSV23 only

adults 19-64 with asplenia, cochlear implants, csf leak– give pCV13 NOW then PPSV23 in 8 weeks , then PPSV23 in 5 years. 

What is the diagnostic criterion for COPD

FEV1/FVC ratio of < 0.70

What are characteristics of COPD

midlife onset, symptoms slowly progressive, exposure to lung irritant, DOE is progressive, chronic cough and chronic sputum production

what are characteristics of asthma

onset early in life, symptoms vary widely from day to day, symptoms worse at nighttime/early AM, allergic rhinitis, eczema, family history, obesity

what are characteristics of heart failure (when evaluating diff dx of COPD)

chest xray with dilated heart, pulmonary edema

what are characteristics of tuberculosis when ruling out for COPD patient

onset all ages, chest xray with lung infiltrate, microbiologic confirmation

what are the characteristics of bronchiectasis

LOTS OF PURULENT SPUTUM, chest xray shows bronchial wall thickening and bronchial dilation

What intervention has the greatest influence on slowing progression of a COPD patient?

smoking cessation!!

what are first line smoking cessation drugs? (3)

varenicline (chantix), nicotine patch, buproprion extended release (zyban)

what is second line smoking cessation drug?

sertraline (zoloft)

what smoking cessation drug would you give to someone with a history of a suicide attempt?

nicotine patch– NOT CHANTIX OR ZYBAN

What are the short acting beta agonists

albuterol, levalbuterol

what are the long acting beta agonists

“terols”
Salmeterol
Formoterol

How do inhaled anticholinergic drugs work?

used in COPD, prevent bronchoconstriction

how do beta agonists work?

stimulate beta 1 (tachycardia) and beta 2 (bronchodilation)

what is the suffix for inhaled anti-cholinergic drugs?

“tropium” think ipratropium (atrovent) SHORT ACTING, tiotropium (spiriva) long acting

What is a SAMA?

short acting antimuscarinic (anticholinergic) – Ipratropium

what is a LAMA?

long acting muscarinic antagonist (anticholinergic) Tiotropium (spiriva)

what are the 4 steps of GOLD guidelines

1) SABA or SAMA prn
2) LABA or LAMA PLUS rescue med
3) ICS + LABA/LAMA plus rescue med
4) ICS+ LABA AND LAMA plus rescue med
THEN REFER

How to manage COPD exacerbation?

figure out the cause
mild: sabas with spacers / nebulizer
moderate: SABAs plus AB and or steroid
Severe: hospitalization

right supraventricular nodes signal what malignancies?

lungs, mediastinum, esophagus

left supraventricular nodes signal what malignancies?

abnormal (stomach, GB, liver, pancreas, ovaries, prostate)

us preventive services task force routine lung cancer screening recommends what annual screening for current smokers aged 55-80 with 30 pack year history or have quit within the last 15 years?

annual low dose CT (LDCT)

how do you diagnose asthma > 5 years of age:

presence of asthma features
reversible airway obstruction on spirometry
>12% improvement from baseline or FEV1 > 200 ml after SABA

How do you treat asthma?

saba always as rescue inhaler
Step wise approach (GINA)
Low dose ICS
then ICS + LABA
increase dosage strength

acute bronchitis classic case symptoms

cough that keeps awake at night
dry cough but may be productive
low grade fever or chest pain with cough
wheezing and rhonchi
median duration of cough is 18 days up to 3 weeks
history of a cold before onset of symptoms

objective findings in acute bronchitis

lungs: clear to severe wheezing, rhonchi
percussion: resonant
CXR: normal
afebrile to low grade fever

treatment plan for bronchitis

symptomatic treatment- increas fluids and rest
tessalon perles, expectorant / mucolytic (guaifenesin)
ventolin (albuterol) for wheezing
for severe wheezing consider short term oral steroid

complications of acute bronchitis

exacerbation of asthma
pneumonia from secondary bacterial infection

pertussis “whooping cough”

caused by bordetella pertussis (gram negative)
coughing illness at least 14 days
paroxysmal coughing, inspiratory whooping
neonates / infants at highest risk for death

three stages of pertussis

catarrhal 1-2 weeks, if treated at this stage can shorten disease course
paroxysmal: lasts 2-4 weeks, treatment has little influence but is useful to decrease spread
convalescent: treatment goal is to eradicate carriage state / disease spread

how to diagnose pertussis

nasopharyngeal swab for culture – collect at 0-2 weeks following cough
PCR may provide accurate results up to 4 weeks
Pertussis antibodies by ELISA
CBC with elevated WBCs and marked lymphocytosis
cxr should be negative

how to treat pertussis

administer a course of antibiotics to close contacts iwthin 3 weeks of exposure
first line: macrolides – azithromycin z pack x 5 days
alternative bactrim x 14 days
antitussives, mucolytics, rest, hydration, frequent small meals

complications of pertussis

sinusitis, otitis media, pneumonia, fainting, rib fractures from coughing

signs of pulmonary embolism

new onset dyspnea, hemoptysis, pleuritic chest pain, vital signs with tachycardia, tachypnea, may have signs of DVT

Assessment of the optic disc is a component of the evaluation of cranial nerve:
I.
II.
III.
.IV

Cranial II

You examine a 62-year-old woman with a 20-year history of hypertension. She reports intermittent use of antihypertensive medications, stating, “I feel better without the medications.” Today she presents for a “check-up.”On physical examination, you note the following: PMI with a downward and lateral shift as well as a Gr II/VI holosystolic murmur with radiation to the axilla.

This patient’s health history is most likely to include a report of:
Syncopal episodes.
Episodes of chest pain at rest.
Dyspnea with exertion.
Vertigo.

Ans: dyspnea on exertion

Of the following, which should be performed first in assessing a 48-year-old woman who has a 25-year history of moderate persistent asthma and who now presents with an acute asthma flare?

Oxygen saturation

Arterial blood gas

FEV1 (forced expiratory volume at 1 second)

Chest x-ray

Ans: FEV (force expiratory volume)

You are setting up a healthcare clinic for migrant farm workers. Which of the following sites is most desirable?

Community hall of a local church

Local hospital ambulatory care center

Mobile van

Space in a shopping center

Ans: Mobile van

A 58-year-old woman presents for an initial examination in order to become a patient in your primary care practice. She is a nonsmoker, drinks 1 to 2, 5 oz (0.15 L) glasses of wine per week, and works as an administrative assistant in a law office. She is without complaint and reports that she is generally in good health. Physical exam reveals BMI=34 kg/m2 and BP=144/98 mm Hg bilaterally.

The rest of her examination is unremarkable.

Prescribe a low-dose thiazide diuretic.

Arrange for additional blood pressure measurements within the next four weeks.

Order a serum creatinine, urea nitrogen and urinalysis.

Advise restricting sodium intake and limiting alcohol intake to no more than 1 glass of wine per week.

Ans: Arrange for additional blood pressure measurements within the next four weeks.

The NP demonstrates fulfillment of the advanced practice nursing leadership role by participating in which of the following activities?

Teaching a 56-year-old man with newly-diagnosed type 2 diabetes mellitus about the importance of self-glucose monitoring

Volunteering to teach a class on contraceptive methods to a group of teen mothers

Discussing barriers to achieving blood pressure control with a 65-year-old woman with hypertension who “does not want to take any medicine”

Collaborating with the regional public health department on an initiative to combat obesity through a community-based exercise program

Ans: Collaborating with the regional public health department on an initiative to combat obesity through a community-based exercise program

You are asked to speak to a group of healthcare executives on the NP role. Which of the following provides the strongest support for NP practice?

Number of academic credits earned during the NP program

Evidence of NP practice outcomes

The quantity of clinical hours and type of clinical rotations in the NP program

The professional experience of the person prior to entering the NP program

Ans: Evidence of NP practice outcomes

Rank the following from highest (1) to lowest (4) level of research design.

__Case reports
__Meta-analysis
__Randomized controlled trials
__Expert opinion

1-Case reports 2-Meta-analysis 3-Randomized controlled trials 4-Expert opinion

2-Case reports 1-Meta-analysis 3-Randomized controlled trials 4-Expert opinion

3-Case reports 1-Meta-analysis 2-Randomized controlled trials 4-Expert opinion

4-Case reports 3-Meta-analysis 2-Randomized controlled trials 1-Expert opinion

Ans:
3-Case reports 1-Meta-analysis 2-Randomized controlled trials 4-Expert opinion

In a 46-year-old woman with hypertension and dyslipidemia, choose the correct prevention:

Primary prevention
Secondary prevention
Tertiary prevention

Counseling about reducing risk for sexually transmitted infection __

Skin survey for precancerous lesions __

1-Primary prevention 2-Secondary prevention

1-Secondary prevention 2-Primary prevention

1-Primary prevention 2-Tertiary prevention

1-Tertiary prevention 2-Secondary prevention

Ans: 1-Primary prevention 2-Secondary prevention

In a 66-year-old woman with type 2 diabetes mellitus (T2DM), choose the correct prevention:

Primary prevention
Secondary prevention
Tertiary prevention

1. Administering influenza vaccine __

2. Adjusting therapy to enhance glycemic control __

1-Tertiary prevention 2-Primary prevention

1-Secondary prevention 2-Primary prevention

1-Primary prevention 2-Secondary prevention

1-Primary prevention 2-Tertiary prevention

Ans: 1-Primary prevention 2-Tertiary prevention

In a 25-year-old well woman with a strong family history of T2DM, choose the correct prevention:

Primary prevention
Secondary prevention
Tertiary prevention

1. Checking fasting lipid profile __

2. Teaching the benefits of participating in a consistent program of moderate-intensity physical activity __

1-Primary prevention 2-Secondary prevention

1-Tertiary prevention 2-Secondary prevention

1-Secondary prevention 2-Primary prevention

1-Primary prevention 2-Tertiary prevention

Ans: 1-Secondary prevention 2-Primary prevention

In a 76-year-old man with chronic obstructive pulmonary disease (COPD), choose the correct prevention:

Primary prevention
Secondary prevention
Tertiary prevention

1. Ensuring adequate illumination at home __

2. Screening for physical, emotional, or financial abuse __

1-Primary prevention 2-Secondary prevention

1-Primary prevention 2-Tertiary prevention

1-Tertiary prevention 2-Secondary prevention

1-Secondary prevention 2-Tertiary prevention

Ans: 1-Primary prevention 2-Secondary prevention

Joseph is a 28-year-old man who presents to your practice with a 2 h history of a superficial laceration on his forearm that is approximately 5 cm (2″) that happened when he was replacing a windowpane and was cut by the glass. Physical examination is consistent with history.His record notes a Tdap vaccine given approximately 2 years ago and documentation of a primary tetanus series.

When considering his needs for tetanus immunization today, the NP appreciates that today Joseph should receive:

A Td (tetanus, diphtheria) booster.

A Tdap (tetanus, diphtheria, acellular pertussis) booster.

A dose of TIG (tetanus immune globulin).

No tetanus vaccine.

Ans:

No tetanus vaccine.

In a community undergoing a disaster, the use of select vaccines will be prioritized. Match the type of disaster with the priority vaccine:

Immunization against tetanus
Immunization against influenza
Immunization against Hepatitis A

1. Uninjured adults and children evacuated to a crowded group setting __

2. Adults with multiple deep lacerations from flying debris __

3. A community exposed to unsafe water supply after a hurricane __

1. Immunization against tetanus 2. Immunization against influenza 3. Immunization against Hepatitis A

1. Immunization against influenza 2. Immunization against tetanus 3. Immunization against Hepatitis A

1. Immunization against tetanus 2. Immunization against Hepatitis A 3. Immunization against influenza

1. Immunization against Hepatitis A 2. Immunization against tetanus 3. Immunization against influenza

1. Immunization against influenza 2. Immunization against tetanus 3. Immunization against Hepatitis A

True or False?

1. Immune globulin is a concentrated solution of antibodies derived from pooled donated blood. __

2. Tdap (tetanus, diphtheria, acellular pertussis [Adacel®, Boostrix®]) vaccine should replace a single dose of Td (tetanus, diphtheria) vaccine for most adults who have not previously received a dose of Tdap in order to provide additional protection against pertussis. __

3. Previously unvaccinated adults age 19 through 59 years with diabetes mellitus type 1 or type 2 should be vaccinated against hepatitis B as soon as possible after the diabetes diagnosis is made. __

1. True 2. True 3. False

1. True 2. True 3. True

1. False 2. True 3. False

1. True 2. False 3. False

Ans: 1. True 2. True 3. True

A 67-year-old man with well-controlled hypertension and dyslipidemia who received one dose of PPSV23 (Pneumovax®) 1 year ago asks about a “new pneumonia shot” he heard about in a TV promotion. You advise that:

He is adequately immunized against pneumococcal disease.

That there is another pneumococcal vaccine available but its use is not advised in older adults due to its adverse effect profile.

He is eligible for a dose of PCV13 (Prevnar®) today to be optimally protected against pneumococcal disease.

In 4 more years, he should receive a second dose of PPSV23.

Ans:

He is eligible for a dose of PCV13 (Prevnar®) today to be optimally protected against pneumococcal disease.

Which of the following influenza vaccines is most appropriate for a 67-year-old man with no reported allergic reactions?

Quadrivalent live attenuated influenza vaccine (LAIV)

High-dose trivalent IIV

Intradermal quadrivalent IIV

Cell cultured-based quadrivalent IIV

Ans: High-dose trivalent IIV

While counseling a 32-year-old man in good health about influenza vaccination, he reports that he occasionally experiences hives when consuming eggs (e.g., scrambled eggs), but has no reaction when consuming foods that contain eggs (e.g., cake). The NP considers the best course of action is to:

Note the allergy in his records so that immunization will not be offered in the future.

Offer vaccination with trivalent recombinant influenza vaccine (RIV3).

Offer vaccination with live attenuated influenza vaccine (LAIV).

Refer him to a specialist for allergy testing prior to immunization with high-dose trivalent inactivated influenza vaccine (IIV3).

Ans: Offer vaccination with trivalent recombinant influenza vaccine (RIV3).

When considering influenza vaccination for an adult who claims to experience a hive-form reaction when exposed to eggs without a history of angioedema or respiratory distress, which of the following is most appropriate?

Administer any vaccine appropriate for his age and health status

Only offer the recombinant influenza vaccine (RIV3)

Referral for vaccination at an allergy practice.

Do not administer vaccine as it is contraindicated for this patient

Ans: Administer any vaccine appropriate for his age and health status

A 19-year-old woman returns to receive her third dose of the HPV vaccine series. Her first two doses were with the quadrivalent vaccine, but the practice currently only stocks the 9-valent HPV vaccine. The NP considers the best course of action is to:

Complete her vaccination series with the 9-valent vaccine.

Defer completing the vaccination series until the quadrivalent vaccine is available in the practice.

Refer her to a practice where the quadrivalent vaccine is currently available.

Restart the series with the 9-valent vaccine and schedule her next two doses within 6 months.

Ans: Complete her vaccination series with the 9-valent vaccine.

A person with a latent tuberculosis infection (True/False):

1. Does not exhibit any signs or symptoms of active infection. __
2. Can spread the disease to others. __
3. Will have a positive tuberculin skin test. __
4. Will have >50% lifetime risk of developing active tuberculosis. __

1-False 2-True 3-False 4-False

1-True 2-True 3-False 4-False

1-True 2-False 3-True 4-False

1-False 2-False 3-False 4-False

Ans: 1-True 2-False 3-True 4-False

Identify which of the following patients should undergo testing for tuberculosis (Yes/No).

1. A 22-year-old male with HIV. __
2. A healthy 18-year-old woman who lives with her father who was recently diagnosed with active tuberculosis infection. __
3. 45-year-old man who was recently released from prison after 2 years of incarceration. __
4. 32-year-old woman who recently emigrated from Southeast Asia. __

1-No 2-Yes 3-Yes 4-Yes

1-Yes 2-Yes 3-Yes 4-Yes

1-Yes 2-No 3-No 4-Yes

1-No 2-No 3-No 4-No

Ans: 1-Yes 2-Yes 3-Yes 4-Yes

When evaluating the Mantoux tuberculin skin test (TST), a positive or negative result is determined by:

AThe area of erythema surrounding the injection site.

The diameter of induration surrounding the injection site.

Presence of generalized maculopapular rash.

Presence of fever ≥99.8ᵒC.

The diameter of induration surrounding the injection site.

In a 51-year-old woman at average risk of breast cancer, choose the prevention:

Primary prevention
Secondary prevention
Tertiary prevention

1. Counseling about the hazard of tobacco use. __
2. Obtaining breast cancer screening via mammography. __

1-Secondary prevention 2-Primary prevention

1-Tertiary prevention 2-Primary prevention

1-Primary prevention 2-Secondary prevention

1-Tertiary prevention 2-Secondary prevention

Ans: In a 51-year-old woman at average risk of breast cancer, choose the prevention:

Match each example with the appropriate communication skill.

Broad opening/open-ended question
Clarification
Active listening
Restating

1. “You said that you got angry when your boyfriend came home late.” __
2. Use of eye contact and non-verbal cues. __
3. “What’s on your mind?” __
4. “I don’t think I see the connection. Can you explain that to me again?” __

1-Broad opening/open-ended question 2-Clarification 3-Active listening 4-Restating

1-Broad opening/open-ended question 2-Restating 3-Active listening 4-Clarification

1-Restating 2-Clarification 3-Active listening 4-Broad opening/open-ended question

1-Restating 2-Active listening 3-Broad opening/open-ended question 4-Clarification

Ans: 1-Restating 2-Active listening 3-Broad opening/open-ended question 4-Clarification

A 48-year-old man is recently diagnosed with type 2 diabetes mellitus. All of the following vaccinations are indicated except:

Pneumococcal.

Hepatitis B.

Herpes zoster (shingles).

Influenza.

Herpes zoster (shingles).

Immunization against herpes zoster is recommended by the CDC for all adults:

≥50 years.

≥55 years.

≥60 years.

≥65 years.

Ans:

≥60 years.

A 51-year-old woman of European ancestry who has had no primary care for more than 10 years presents to your practice. She has a 40 pack-year cigarette smoking history, currently smoking 1 PPD and reports drinking about 3-4, 12 oz (0.35 L) beers per month. Her history is otherwise unremarkable as is her physical examination. You order the following screening test:

Hemoglobin electrophoresis.

Mammography.

Chest x-ray.

Fasting serum triglycerides.

Ans: Mammography.

A 65-year-old Native American man presents for a “physical.” He feels well, denies tobacco or alcohol use, and has not seen a healthcare provider in more than 10 years.
The patient states, “I am a really healthy person. I would not come in except my wife and daughter told me I should have a checkup.” As part of today’s visit, he should be screened for:

Pancreatic cancer.

Hemolytic anemia.

Hepatic sclerosis.

Visual defect.

Ans: Visual defect.

A 30-year-old woman of Asian ancestry presents for a routine health visit. She is in good health and reports she is concerned about her personal breast and ovarian cancer risk and asks about BRCA gene mutation testing. You advise the following:

BRCA gene mutation testing should be offered to all women who request this evaluation.

A standardized screening questionnaire for BRCA gene mutation such as FHS-7 should be administered today to determine if BRCA gene mutation testing is warranted.

She is not a candidate for this test due to her ethnicity.

A referral to genetic counseling for guidance on BRCA gene mutation testing should be ordered.

Ans: A standardized screening questionnaire for BRCA gene mutation such as FHS-7 should be administered today to determine if BRCA gene mutation testing is warranted.

Suicide: True or False?

1. Males represent nearly 80% of all completed suicides. __
2. When compared with male suicide attempts, female attempts at suicide are approximately 2-3 times more common. __
3. The highest rate of completed suicide is found in teenage males. __
4. Inquiring about suicidal ideation could precipitate the act. __

1-True 2-True 3-False 4-False

1-True 2-Flase3-False 4-True

1-False 2-True 3-False 4-False

1-False 2-False 3-False 4-False

1-True 2-True 3-False 4-False

Ms. Kane is a 25-year-old woman who presents with finger-shaped ecchymotic areas on her right shoulder that are an incidental finding during a physical examination. She denies abuse or assault. The NP’s most appropriate response is:

“Your bruises look as if they were caused by someone grabbing you.”

“Was this an accident?”

“I notice the bruises are in the shape of a hand.”

“How did you fall?”

Ans: “I notice the bruises are in the shape of a hand.”

Mr. Jacobs is a 65-year-old man with COPD and a 60 pack-year history who is currently smoking 1.5 packs of cigarettes per day. He is reading a pamphlet in your office about smoking cessation.You ask him if he has any questions and he states, “I don’t plan to quit smoking at my age. I am too old to quit now. Why bother?” The NP’s most appropriate response is:

Since you have COPD, you really should quit smoking.

You know your lungs will get more damaged if you continue to smoke.

Tell me what you mean by, “I am too old to quit.”

I can provide medication that will help you to quit smoking.

Ans: Tell me what you mean by, “I am too old to quit.”

According to the Stages of Change Theory, he is most likely in which of the following stages?

Precontemplation

Contemplation

Preparation

Minimization

Ans: Precontemplation

According to current nationally-recognized recommendations, are the following cancer screenings indicated? Yes or No

1. Annual digital rectal exam as colorectal cancer screening in a 63-year-old man __
2. An initial liquid-based Pap test with HPV cotesting in a 19-year-old woman who is one year post-coitarche __
3. Endometrial biopsy in a 52-year-old woman who is two years post-LMP and who denies vaginal bleeding __
4. Annual prostate specific antigen testing in an 81-year-old man who has hypertension and benign prostatic hypertrophy __
5. Lung cancer screening with low-dose CT (LDCT) for a 60-year-old who is generally in good health who has a 35 pack-year cigarette smoking history who quit smoking 5 years ago __

1-Yes 2-Yes 3-No 4-Yes 5-No

1-No 2-No 3-No 4-Yes 5-No

1-No 2-Yes 3-No 4-Yes 5-No

1-No 2-No 3-No 4-No 5-Yes

Ans: 1-No 2-No 3-No 4-No 5-Yes

A person with a latent tuberculosis infection: (True/False)

1. Does not exhibit any signs or symptoms of active infection. __
2. Can spread the disease to others. __
3. Will have a positive tuberculin skin test. __
4. Will have >50% lifetime risk of developing active tuberculosis. __

1-False 2-True 3-True 4-False

1-True 2-True 3-True 4-True

1-False 2-True 3-False 4-False

1-True 2-False 3-True 4-False

1-True 2-False 3-True 4-False

Identify which of the following patients should undergo testing for tuberculosis. (Yes/No)

1. A 22-year-old male with HIV __
2. A healthy 18-year-old woman who lives with her father who was recently diagnosed with active tuberculosis infection __
3. A 45-year-old man who was recently released from prison after 2 years of incarceration __
4. A 32-year-old woman who recently immigrated from Southeast Asia __

1-Yes 2-No 3-Yes 4-No

1-Yes 2-No 3-No 4-No

1-No 2-No 3-Yes 4-No

1-Yes 2-Yes 3-Yes 4-Yes

Ans: 1-Yes 2-Yes 3-Yes 4-Yes

The duration of antimicrobial treatment for active tuberculosis infection is typically:



10 days.

4-6 weeks.

6-9 months.

1-2 years.

Ans:

6-9 months.

According to current nationally recognized recommendations (True/False):

1. Screening for lung cancer with a low-dose CT scan is indicated for a 62-year-old man in good health who is a current smoker and has a 40 pack-year smoking history. __
2. PSA testing is indicated for a 68-year-old man with end-stage heart failure. __
3. PV test and Pap test are recommended for a 37-year-old woman who had negative cervical cytology results 5 years ago. __
A 24-year-old woman who completed the HPV vaccination regimen 2 years ago is not recommended to undergo cervical cancer screening until she reaches 30 years of age. __

1-True 2-False 3-False 4-True

1-True 2-False 3-True 4-True

1-True 2-False 3-True 4-False

1-False 2-False 3-False 4-True

1-True 2-False 3-True 4-False

Match each definition with the appropriate term:

Prevalence
Incidence
False positive
False negative
Specificity
Sensitivity

1. Refers to the ability of a test to correctly identify those with a condition. __
2. Refers to the ability of a test to correctly identify those without a condition. __
3. Proportion or number of individuals with a condition at a given time. __
4. Number of new cases occurring within a period of time. __

1-Prevalence 2-False negative 3-Sensitivity 4-Incidence

1-Sensitivity 2-Specificity 3-Prevalence 4-Incidence

1-False positive 2-False negative 3-Specificity 4-Incidence

Ans: 1-Sensitivity 2-Specificity 3-Prevalence 4-Incidence

You advise a 28-year-old woman who follows a vegan diet to supplement with:

Vitamin A.

Iron.

Vitamin B12.

Folic acid.

Ans: Vitamin B12.

A 78-year-old woman presents with fatigue, spoon-shaped nails and the following laboratory results.
-Hb=9 g/dL (90 g/L)
-Hct=28.1% (0.281 proportion)
-RBC=2.4 million/mm3
-MCV=70 fL
-MCHC=24.2 g/dL (242 g/L)
-RDW=19% (0.19 proportion)

A critical causative diagnosis to consider as origin of her anemia is:

Gastrointestinal blood loss.

Micronutrient malabsorption.

Chronic ileitis.

Folic acid deficiency.

Ans: Gastrointestinal blood loss.

Which of the following represents the optimal advice to a patient who is taking oral iron therapy to maximize the medication’s effectiveness?

Take your medication with an antacid.

Take your medication on an empty stomach.

Take your medication after the largest meal of the day.

Take your medication with a large glass of milk.

Ans: Take your medication on an empty stomach.

A 68-year-old woman presents with a 6-month history of increasingly severe peripheral numbness and oral irritation.

Hemogram results are as follows.
-Hb=6.2 g/dL (62 g/L)
-Hct=20% (0.2 proportion)
-RBC=2.1 million/mm3
-MCV=132 fL
-MCHC=32.4 g/dL (324 g/L)
-RDW=19% (0.19 proportion)
Physical exam reveals pale conjunctiva, a grade 2/6 systolic ejection murmur over the precordium without radiation, and a smooth, red tongue.

In the above-mentioned patient has not been present on previous examination. As a result, you consider that this is likely a _________ murmur and will resolve with anemia treatment.


Pathologic

Hemic

Venous hum

Ans:

Hemic

The most likely cause of this anemia is:


Based on the below scenario:
A 68-year-old woman presents with a 6-month history of increasingly severe peripheral numbness and oral irritation.

Hemogram results are as follows.
-Hb=6.2 g/dL (62 g/L)
-Hct=20% (0.2 proportion)
-RBC=2.1 million/mm3
-MCV=132 fL
-MCHC=32.4 g/dL (324 g/L)
-RDW=19% (0.19 proportion)
Physical exam reveals pale conjunctiva, a grade 2/6 systolic ejection murmur over the precordium without radiation, and a smooth, red tongue.


Vitamin B12 deficiency.

Iron deficiency.

Hemolysis.

Chronic disease.

Ans: Vitamin B12 deficiency.

A 65-year-old woman with rheumatoid arthritis who is on optimized therapy and continues to have significant symptoms presents with the following hemogram.

-Hb=10.1 g/dL (12-14 g/dL)
101 g/L (120-140 g/L)
-Hct=32% (36-42%)
0.32 proportion (0.36-0.42 proportion)
-RBC=3.2 million/mm3 (4.2-5.4 million/mm3)
– MCV=82 fL (80-96 fL)
-RDW=12.8% (11-15%)
0.128 proportion (0.11-0.15 proportion)
-Reticulocytes=0.7% (1-2%)
0.007 proportion (0.01-0.02 proportion)

These findings are most consistent with:


Iron deficiency anemia.

Folate deficiency anemia.

Anemia of chronic disease.

Alpha thalassemia minor.

Ans: Anemia of chronic disease.

You see Maria, a a 32-year-old well woman of Mediterranean ancestry. Hemogram results are as follows:
-Hb=10.6 g/dL (12-14 g/dL)
106 g/L (120-140 g/L)
-Hct=32% (36-42%)
0.32 proportion (0.36-0.42 proportion)
-RBC=5.2 million/mm3 (3.2-4.3 million/mm3)
-MCV=71 fL (80-96 fL)
-MCHC=25.2 g/dL (31-37 g/dL)
252 g/L [310-370 g/L]
-RDW=12% (<15%)
0.12 proportion (<0.15 proportion)

These findings are most consistent with:

Iron deficiency anemia.

Cooley’s anemia.

Beta thalassemia minor.

Acute blood loss.

Ans: Beta thalassemia minor.

Match the best next test to order in an adult with anemia based on hemogram results.

1. Ferritin __
2. Vitamin B12 and folate __
3. Hemoglobin electrophoresis __
MVC ↓ MCHC ↓ RWD ↑
MVC ↓ MCHC ↓ RWD NL
MCV ↑ MCHC NL RWD ↑




1-A 2-B 3-C

1-C 2-B 3-A

1-A 2-C 3-B

1-B 2-A 3-C

Ans:

1-A 2-C 3-B

The use of all of the following nutritional supplements is potentially associated with increased bleeding risk and should be discontinued at least 7‒10 days prior to elective surgical procedure. (True/False)

1. Ginseng __
2. Gingko __
3. Fish oil __
4. Vitamin D __

1-True 2-False 3-True 4-True

1-True 2-True 3-True 4-False

1-False 2-False 3-True 4-True

1-False 2-False 3-True 4-False

Ans: 1-True 2-True 3-True 4-False

Primary care of Sarah, a 27-year-old woman with beta thalassemia minor, should include:

Prescribing a low-dose iron supplement to counteract microcytosis.

An evaluation of hemoglobin electrophoresis every 5 years.

Advising taking a multivitamin with high-dose folate supplementation daily.

Offering genetic counseling prior to pregnancy.

Ans:

Offering genetic counseling prior to pregnancy.

A 37-year-old woman with ulcerative colitis with poor symptom control without rectal bleeding presents with the following hemogram:

-Hg=9.9 g/dL (12-14 g/dL) (99 g/L [120-140 g/dL])
-MCHC=33 g/dL (31-37 g/dL) (330 g/L [310-370 g/L])
-MCV=86 fL (80-96 fL)
-RDW=12% (11%-15%) (0.12 proportion [0.11-0.15 proportion])

These findings are most consistent with:

Iron deficiency.

Beta thalassemia minor.

Anemia of chronic disease.

Pernicious anemia.

Ans: Anemia of chronic disease.

A 67-year-old man taking NSAIDs daily for back pain and reporting fatigue presents with the following hemogram:

-Hg=8.4 g/dL (12-14 g/dL) (84 g/L [120-140 g/dL])
-MCHC=26 g/dL (31-37 g/dL) (260 g/L [310-370 g/L])
-MCV=69 fL (80-96 fL)
-RDW=19% (11%-15%) (0.19 proportion [0.11-0.15 proportion])

These findings are most consistent with:

Iron deficiency anemia.

Beta thalassemia minor.

Vitamin B12 deficiency.

Drug-induced macrocytosis.

Ans: Iron deficiency anemia.

A 54-year-old man of Asian ancestry presents with the following hemogram:

-Hg=11.6 g/dL (12-14 g/dL) (116 g/L [120-140 g/dL])
-Hct=37% (42-48%) (0.37 proportion [0.42-0.48 proportion])
-MCHC=24 g/dL (31-37 g/dL) (260 g/L [310-370 g/L])
-MCV=64 fL (80-96 fL)
-RDW=14% (11%-15%) (0.14 proportion [0.11-0.15 proportion])
-RBC=6.6 million/mm3 (4.7-6.1 million/mm3)

These findings are most consistent with:

Folate deficiency.

Acute blood loss.

Alpha thalassemia minor.

Iron deficiency anemia.

Ans:

Alpha thalassemia minor.

rue or false? (T/F) The U.S. FDA advises that the adverse effects associated with fluoroquinolones generally outweigh the benefits for patients with acute sinusitis, acute bronchitis, and uncomplicated urinary tract infections who have other treatment options.


True

False

Ans: True

You see a 47-year-old man in urgent care who has hypertension, dyslipidemia, and depression. He states, “I am on a big list of medications but I am not sure of all the names.” He also has a history of penicillin allergy with a hive-form reaction. He has not taken a systemic antimicrobial in more than a year and denies recent hospitalization. When developing a treatment plan for acute bacterial rhinosinusitis, you consider prescribing the following course of an oral antimicrobial

A 5-day course of clarithromycin.

A 7-day course of doxycycline.

A 10-day course of amoxicillin- clavulanate.

A 7-day course of moxifloxacin.

Ans: A 7-day course of doxycycline.

Sandra is a 45-year-old well woman diagnosed with a left-sided unilateral acute otitis media 10 days ago and treated with an antimicrobial.

She is seen today with a report of resolution of ear pain, but with persistent sensation of ear fullness and diminished ability to discriminate speech in the affected ear.

Anticipated findings on today’s physical examination include:

Erythema of the ear canal.

Weber test lateralizing to the affected ear

Discomfort on tragus pull.

Anterior cervical lymphadenopathy on the affected side.
Feedback

Ans: Weber test lateralizing to the affected ear

Sandra’s hearing loss is best described as:


Based on the below scenario:
Sandra is a 45-year-old well woman diagnosed with a left-sided unilateral acute otitis media 10 days ago and treated with an antimicrobial.

She is seen today with a report of resolution of ear pain, but with persistent sensation of ear fullness and diminished ability to discriminate speech in the affected ear.

Sensorineural.

Conductive.

Auditory processing.

Mixed origin.

Ans: Conductive.

For Sandra, which of the following represents the best advice for this point?


Based on the below scenario:
Sandra is a 45-year-old well woman diagnosed with a left-sided unilateral acute otitis media 10 days ago and treated with an antimicrobial.
She is seen today with a report of resolution of ear pain, but with persistent sensation of ear fullness and diminished ability to discriminate speech in the affected ear.

She should have a second course of antimicrobial therapy.

A short course of an oral corticosteroid should be prescribed.

The sensation of ear fullness is an anticipated finding.

She should be seen by an otolaryngology specialist.

Ans: The sensation of ear fullness is an anticipated finding.

Hank is a 58-year-old man who presents with a chief complaint of bilateral itchy eyes occurring intermittently throughout the year. Exam reveals 20/30 vision OD (right eye), OS (left eye), OU (both eyes) with corrective lenses bilateral hyperemic bulbar and palpebral conjunctiva, and a small amount of rope-like pale yellow discharge.

These findings are most consistent with:

Bacterial conjunctivitis.

Blepharoconjunctivitis.

Allergic conjunctivitis.

Dry-eye syndrome.

Ans: Allergic conjunctivitis.

The most appropriate treatment option for Hank is the use of:


Based on the below scenario:
Hank is a 58-year-old man who presents with a chief complaint of bilateral itchy eyes occurring intermittently throughout the year. Exam reveals 20/30 vision OD (right eye), OS (left eye), OU (both eyes) with corrective lenses bilateral hyperemic bulbar and palpebral conjunctiva, and a small amount of rope-like pale yellow discharge.

Ocular antimicrobial.

Lubricating eye solution

Ocular antihistamine.

Systemic decongestant.

Ans: Ocular antihistamine.

Appropriate pharmacologic management options for allergic rhinitis in a 29-year-old woman who is a home daycare provider includes all of the following except:

Short-term use of a decongestant nasal spray.

Oral chlorpheniramine.

Flunisolide nasal spray.

Oral loratadine.

Ans: Oral chlorpheniramine.

Edgar is a 75-year-old man with a 60 pack-year history of cigarette smoking and COPD who presents with a chief complaint of a “sore” on the base of his tongue. This lesion has been present for a number of months, remaining relatively stable in size and is not painful. Physical examination reveals a painless ulcerated lesion with indurated margin and is accompanied by a firm, nontender submandibular node. His current medications include inhaled corticosteroids with a long-acting beta2-agonist.

This clinical scenario is most consistent with:

Syphilitic chancre.

Aphthous stomatitis.

Squamous cell carcinoma.

Oral candidiasis.

Ans: Squamous cell carcinoma.

Risk factors for Edgar’s condition include:


Based on the below scenario:
Edgar is a 75-year-old man with a 60 pack-year history of cigarette smoking and COPD who presents with a chief complaint of a “sore” on the base of his tongue. This lesion has been present for a number of months, remaining relatively stable in size and is not painful. Physical examination reveals a painless ulcerated lesion with indurated margin and is accompanied by a firm, nontender submandibular node. His current medications include inhaled corticosteroids with a long-acting beta2-agonist.

Recent high-risk sexual contact.

Current use of an inhaled corticosteroid.

Long-term HPV-16 infection.

Chronic irritation from poorly-fitting dentures.

Ans:

Long-term HPV-16 infection.

The next step in Edgar’s care should include:


Based on the below scenario:
Edgar is a 75-year-old man with a 60 pack-year history of cigarette smoking and COPD who presents with a chief complaint of a “sore” on the base of his tongue. This lesion has been present for a number of months, remaining relatively stable in size and is not painful. Physical examination reveals a painless ulcerated lesion with indurated margin and is accompanied by a firm, nontender submandibular node. His current medications include inhaled corticosteroids with a long-acting beta2-agonist.

An oral antifungal.

Referral for lesion biopsy.

Use of topical anesthetic paste.

Serologic testing to confirm the diagnosis.

Ans: Referral for lesion biopsy.

The function of which of the following cranial nerves is being tested with the following patient requests?

A. CN I
B. CN XI
C. CN III
D. CN VII
E. CN XII

1. Puff out your cheeks. __
2. Do you recognize this scent? __
3. Without moving your head, follow my finger with your eyes. __
4. Shrug your shoulders. __
5. Stick out your tongue. __

1-D 2-A 3-C 4-B 5-E

1-B 2-A 3-D 4-C 5-E

1-A 2-E 3-C 4-D 5-B

1-C 2-D 3-B 4-A 5-E

Ans:

1-D 2-A 3-C 4-B 5-E

Matthew is a 29-year-old man who presents with a 6-hour history of sudden onset of inability to raise his eyebrow or smile on the right side. He also reports decreased lacrimation in the right eye and difficulty closing the right eyelid. The rest of his health history and physical examination is otherwise unremarkable.

This likely represents paralysis of cranial nerve (CN):

III.

VIII.

IV.

VII.

Ans: VII

Which of the following is the most appropriate next step in Matthew’s care?


Based on the below scenario:
Matthew is a 29-year-old man who presents with a 6-hour history of sudden onset of inability to raise his eyebrow or smile on the right side. He also reports decreased lacrimation in the right eye and difficulty closing the right eyelid. The rest of his health history and physical examination is otherwise unremarkable.

Emergent referral for neuroimaging

Initiating a course of oral corticosteroids

Prescribing a short course of high-dose antiviral therapy

Referral to a neurology specialist within the next 24‒48 hours
Feedback

Ans: Initiating a course of oral corticosteroids

You see a 55-year-old woman who presents for a health maintenance visit. She has longstanding myopia and newer-onset presbyopia, both corrected with eyeglasses. She is normotensive and without ocular complaint. You anticipate the following on today’s eye exam:

Retinal arteries wider than veins.

Equal, sluggish pupillary response.

Sharp disc margins.

Lid ectropion.

Ans: Sharp disc margins.

Which of the following is most likely to be found on the funduscopic exam in the person with angle-closure glaucoma?

A deeply-cupped optic disc

Arteriovenous nicking

Papilledema

Hemorrhagic lesions

Ans:

A deeply-cupped optic disc

Match each vision alteration with the most likely etiology.

A. Macular degeneration
B. Untreated open-angle glaucoma
C. Proliferative diabetic retinopathy

1. Peripheral vision loss __
2. Floating spots in visual field Macular degeneration Untreated open-angle glaucoma __
3. Central vision loss __

1-B 2-C 3-A

1-A 2-B 3-C

1-C 2-B 3-A

1-B 2-A 3-C

Ans: 1-B 2-C 3-A

Match the following select ophthalmologic tests with the appropriate indication.

A. Amsler grid test
B. Snellen chart
C. Slit-lamp examination
D. Tonometry

1. Evaluation of anterior eye structures, including cornea, conjunctiva, sclera, and iris __
2. General visual acuity screen __
3. Early detection of macular degeneration __
4. Measurement of intraocular pressure, glaucoma screening test __

1-B 2-C 3-D 4-A

1-C 2-B 3-A 4-D

1-A 2-C 3-B 4-D

1-D 2-C 3-B 4-A

Ans: 1-C 2-B 3-A 4-D

During a clinical encounter with a 78-year-old man who has presbycusis, the NP considers that communication will be enhanced by all of the following except:

Maintaining eye contact with the patient.

Ensuring the patient can see the NP’s face clearly.

Playing soft music in the background.

Providing adequate illumination in the exam room.

Ans: Playing soft music in the background.

nitial treatment for a 23-year-old man with ABRS with no recent antimicrobial therapy (>3 months) and no drug allergy is:

Ampicillin.

Amoxicillin-clavulanate (Augmentin®).

Ciprofloxacin (Cipro®).

Levofloxacin (Levaquin®).

Ans: Amoxicillin-clavulanate (Augmentin®).

Risk factors for ABRS caused by drug-resistant S. pneumoniae (DRSP) include all of the following except:

Recent prior antimicrobial use.

Exposure to a child in large-group daycare.

Living in a rural setting.

Hospitalization within the past 5 days.

Ans: Living in a rural setting.

Macular degeneration would most likely be found in which of the following patients?

A 72-year-old woman who smoked cigarettes up until 10 years ago

A 54-year-old man with a family history of glaucoma

A 64-year-old man with well-controlled hypertension

A 32-year-old woman recently diagnosed with type 2 diabetes mellitus

Ans: A 72-year-old woman who smoked cigarettes up until 10 years ago

Therapy to help avoid nasal symptoms in a 32-year-old man with allergic rhinitis should include:

An oral first-generation antihistamine.

An intranasal corticosteroid.

An oral decongestant.

An intranasal anticholinergic.

Ans: An intranasal corticosteroid.

A first-generation antihistamine can help alleviate all of the following symptoms of allergic rhinitis except:

Sneezing.

Nasal congestion

Rhinorrhea.

Itchy, watery eyes.

Ans: Nasal congestion

Identify the following types of skin lesions with its corresponding description:

A. Macule
B. Papule
C. Plaque
D. Cyst
E. Wheal
F. Purpura

1. Single, uniformly brown-colored, slightly raised, irregular-shaped with defined borders, 6 mm in diameter. Patient states “That mole on my shoulder hasn’t changed in years.” __
2. Single, flat, non-palpable area of discoloration, irregularly-shaped, and 0.5 cm at the widest diameter. Patient states “I’ve had that spot on my lower lip for years.” __
3. Single, firm, smooth, raised, dome-shaped, fluid-filled, flesh-colored encapsulated lesion of 1.5 cm in diameter on back of neck. Patient states “A smelly, liquid leaks out of it sometimes.” __
4. Raised, irregular-shaped with defined borders, different color than surrounding skin, patches of >2 cm in diameter, located over the knees. Patient states “These patches have been here for years. When I pick at one, it bleeds just a drop or two.” __
5. Flat, non-blanchable, confluent, purple-colored irregular-shaped lesions on skin ranging 2?20 mm in size. Patient states “These purple splotches appeared on my arms and legs following a round of chemotherapy.” __
6. Clustered, smooth, slightly-raised, circumscribed, pruritic skin-colored lesions of various sizes up to 2 cm, surrounded by area of erythema. Patient states “This itchy rash appeared all over my body a few days after starting my antibiotic.” __

1-C 2-B 3-A 4-E 5-F 6-D

1-B 2-F 3-A 4-D 5-C 6-D

1-B 2-A 3-D 4-C 5-F 6-E

1-A 2-E 3-D 4-B 5-F 6-C

Ans: 1-B 2-A 3-D 4-C 5-F 6-E

Match each dermatologic term with the description.

A. Annular
B. Scattered
C. Confluent or coalescent
D. Clustered
E. Linear

1. In streaks such as the typical phytodermatitis caused by exposure to plant oil (urushiol) contained in poison ivy, poison oak, poison sumac. __
2. Occurring in a group without pattern, such as the lesions seen in an outbreak of herpes simplex type 1 (HSV-1, “cold sore”). __
3. Generalized over body without a specific pattern or distribution, as seen in a viral exanthem such as rubella or roseola. __
4. Multiple lesions blending together, such as the lesions seen in psoriasis vulgaris. __
5. In a ring, often seen in the characteristic “Bull’s Eye” lesion seen in Lyme disease. __

1-A 2-D 3-E 4-C 5-B

1-B 2-C 3-E 4-D 5-A

1-E 2-D 3-B 4-C 5-A

1-C 2-A 3-B 4-D 5-E

Ans: 1-E 2-D 3-B 4-C 5-A

A 60-year-old otherwise well man presents with a dermatologic condition. Which of the following conditions requires biopsy to confirm the diagnosis?

Scaling flesh-colored lesions in a cluster, ranging in size from 3?10 mm on the dorsal aspect of the hand, present for a number of months, without patient complaint

Well-demarcated round-to-oval erythematous coin-shaped plaques approximately 10 mm in diameter on the anterior aspects of the lower legs described as being intermittently itchy that has been present for a number of months.

Painless ulcerated lesion approximately 1.5 cm over the sternum that has been present for a number of weeks.

Oval plaque that is approximately 5 cm in diameter with a central wrinkled salmon-colored area and a dark red peripheral zone on the anterior trunk that has been present for 5 days without patient complaint.

Ans: Painless ulcerated lesion approximately 1.5 cm over the sternum that has been present for a number of weeks.

Match each treatment option with the relevant condition.

A. Permethrin lotion
B. Medium-potency topical corticosteroid
C. Imiquimod cream
D. Topical ketoconazole
E. Topical metronidazole

1. Psoriasis Vulgaris __
2. Scabies __
3. Verruca Vulgaris __
4. Tinea Pedis __
5. Rosacea __

1-C 2-B 3-A 4-D 5-E

1-D 2-C 3-A 4-B 5-E

1-A 2-B 3-C 4-D 5-E

1-B 2-A 3-C 4-D 5-E

Ans: 1-B 2-A 3-C 4-D 5-E
Medium-dose topical corticosteroid – psoriasis vulgaris
permethrin lotion – scabies
Imiquimod cream – verruca vulgaris
topical ketoconazole – tinea pedis
topical metronidazole – rosacea

dentify each condition with the most likely location.

A. Pityriasis rosea
B. Psoriasis vulgaris
C. Actinic keratosis
D. Scabies
E. Eczema

1. Antecubital fossa __
2. Anterior surface of knees __
3. Sun-exposed areas __
4. Over waistband area __
5. Usually preceded by herald patch on the trunk __

1-A 2-B 3-D 4-E 5-C

1-E 2-B 3-C 4-D 5-A

1-C 2-B 3-D 4-E 5-A

1-D 2-A 3-B 4-E 5-C

Ans: 1-E 2-B 3-C 4-D 5-A
1) eczema 2) psoriasis vulgaris 3) actinic keratosis 4) scabies 5) pityriasis rosea

Ana is a 22-year-old well woman who presents with a four-day history of an intense itch with skin lesions on hands and arms as well as her right cheek. She has used an over-the-counter hydrocortisone cream on the affected area with little effect, and denies any other symptoms or previous history of similar rash. Ana denies recent travel and exposure to new creams, soaps, or medications. She works as a landscaper. When considering a diagnosis of phytodermatitis due to exposure to urushiol (poison ivy, poison oak, poison sumac), you anticipate finding three of the following.

Fever and generalized malaise

Vesicles

Crusts

Fissures

Most of the lesions in a linear pattern

Ans:



Vesicles

Crusts

Fissures

When evaluating Ana, you note that approximately 20% total body surface area is affected. You consider treatment with:

Based on the Evaluation Below:
Ana is a 22-year-old well woman who presents with a four-day history of an intense itch with skin lesions on hands and arms as well as her right cheek. She has used an over-the-counter hydrocortisone cream on the affected area with little effect, and denies any other symptoms or previous history of similar rash. Ana denies recent travel and exposure to new creams, soaps, or medications. She works as a landscaper.

Topical application of a medium-potency corticosteroid cream

An oral antihistamine

A systemic corticosteroid

A topical antihistamine

Ans: A systemic corticosteroid

Across North America, brown recluse spider bites are the most common reason for new-onset ulcerating skin lesion.

True

False

Ans: False

A 28-year-old woman who works in food service presents with a chief complaint of an on-the-job injury caused when her right forearm was accidentally exposed to steam. Approximately 2% body surface area is involved. You assess the injury as a partial thickness (second degree) burn and describe the skin lesion as appearing:

Reddened easily blanched with gentle pressure

Red, moist with peeling borders and scattered bulla

Thickened, hypopigmented tissue

Vesicular with hyperpigmentation

Ans: Red, moist with peeling borders and scattered bulla

You see a 20-year-old college wrestler with purulent cellulitis. He reports that a couple of his teammates were recently diagnosed with MRSA skin infections. Initial treatment options would include all of the following except:

Trimethoprim-sulfamethoxazole (Bactrim®)

Clindamycin (Cleocin®)

Doxycycline (Doryx®)

Levofloxacin (Levaquin®)

Ans: Levofloxacin (Levaquin®)

A first-line treatment for uncomplicated skin infection due to S. aureus in a patient with no identifiable risk factors for MRSA is:

Ciprofloxacin (Cipro®)

Dicloxacillin

Linezolid (Zyvox®)

Daptomycin (Cubicin®)

Ans: Dicloxacillin

Match each description with the most appropriate diagnosis.

A. Psoriasis
B. Viral exanthem
C. Basal cell carcinoma

1. A 64-year-old ex-truck driver who reports a persistent open sore on his left forearm that repeatedly bleeds and then crusts over. __
2. A 36-year-old man who presents with a raised patch of skin on his right elbow covered with silvery-white scaly skin. He reports the area is “very itchy.” __
A 14-month-old unimmunized girl without recent medication use who presents with a diffuse rash and low-grade fever. __
Match each description with the most appropriate diagnosis.

A. Psoriasis
B. Viral exanthem
C. Basal cell carcinoma

1-C 2-A 3-B

Ans: 1-C 2-A 3-B

Untreated hyperthyroidism or hypothyroidism or both?

Hypo
Hyper
Hypo/Hyper

1. Dry Skin __
2. Fine tremor __
3. Hypoactive deep tendon reflexes (DTR) __
4. Mood or mentation change __
5. Menorrhagia __
6. Exophath1-

Hyper 3-Hyper 4-Hypo/Hyper 5-Hypo 6-Hypo/Hyperalmos __

1-Hyper 2-Hyper 3-Hyper 4-Hypo/Hyper 5-Hypo 6-Hypo/Hyper

1-Hypo/Hyper 2-Hypo 3-Hyper 4-Hypo/Hyper 5-Hypo 6-Hypo/Hyper

1-Hypo 2-Hyper 3-Hypo 4-Hypo/Hyper 5-Hypo 6-Hyper

1-Hyper 2-Hypo 3-Hypo/Hyper 4-Hypo 5-Hypo 6-Hypo/Hyper

1-Hypo 2-Hyper 3-Hyper 4-Hypo/Hyper 5-Hypo 6-Hypo/Hyper

Ans: 1-Hypo 2-Hyper 3-Hypo 4-Hypo/Hyper 5-Hypo 6-Hyper

You see a 38-year-old woman with hypothyroidism who is currently taking levothyroxine 75 mcg/d with excellent adherence, stating, “I take the medicine every morning on an empty stomach with a big glass of water.” She is feeling well. Results of today’s laboratory testing includes a TSH=4.5 mIU/mL. The next best step in her care is to:

Continue on the same levothyroxine dose and obtain a repeat TSH in 1 year

Decrease the levothyroxine dose by 25 mcg/d and repeat a TSH in 1 month

Increase the levothyroxine dose by 25 mcg/d and repeat a TSH in 2 months

Provide counseling to take the medication with breakfast

Ans: ncrease the levothyroxine dose by 25 mcg/d and repeat a TSH in 2 months

Mrs. Lange is a 79-year-old woman with a >20-year history of well-controlled hypertension and dyslipidemia, currently taking an ACE inhibitor, low-dose thiazide diuretic, and a statin. She presents today with a chief complaint of a 6-month history of progressive symptoms, including fatigue, difficulty initiating and maintaining sleep, increased difficulty with raising her arms above her head, and a sensation of “my heart not beating right, sometimes I feel like it’s going to hop right out of my chest.” She denies shortness of breath, chest pain, cough, or difficulty breathing when supine and admits to “losing weight without even trying.” Cardiac examination reveals an irregularly irregular cardiac rhythm, without S3, S4, or murmur and no neck vein distention.The result of Mrs. Lange’s electrocardiogram is consistent with:


Sinus tachycardia

Multifocal atrial tachycardia

First-degree heart block

Atrial fibrillation

Ans: Atrial Fibrillation

The remainder of her physical examination reveals flat affect, fine tremor, 3-4+ DTR response, mild proximal muscle weakness, symmetric thyroid enlargement without tenderness or mass and a 10-lb (4.5-kg) weight loss since her last visit 8 months ago. The remainder of Mrs. Lange’s examination is at her baseline. Which of the following is the most likely diagnosis?

Based on the Physical Examination Below:
Mrs. Lange is a 79-year-old woman with a >20-year history of well-controlled hypertension and dyslipidemia, currently taking an ACE inhibitor, low-dose thiazide diuretic, and a statin. She presents today with a chief complaint of a 6-month history of progressive symptoms, including fatigue, difficulty initiating and maintaining sleep, increased difficulty with raising her arms above her head, and a sensation of “my heart not beating right, sometimes I feel like it’s going to hop right out of my chest.” She denies shortness of breath, chest pain, cough, or difficulty breathing when supine and admits to “losing weight without even trying.” Cardiac examination reveals an irregularly irregular cardiac rhythm, without S3, S4, or murmur and no neck vein distention.

Thyrotoxicosis

Statin-induced myopathy

Heart failure

Hypothyroidism

Ans: Thyrotoxicosis

Choose the two most important tests to help support Mrs. Lange’s diagnosis.

Based on the Physical Examination Below:
Mrs. Lange is a 79-year-old woman with a >20-year history of well-controlled hypertension and dyslipidemia, currently taking an ACE inhibitor, low-dose thiazide diuretic, and a statin. She presents today with a chief complaint of a 6-month history of progressive symptoms, including fatigue, difficulty initiating and maintaining sleep, increased difficulty with raising her arms above her head, and a sensation of “my heart not beating right, sometimes I feel like it’s going to hop right out of my chest.” She denies shortness of breath, chest pain, cough, or difficulty breathing when supine and admits to “losing weight without even trying.” Cardiac examination reveals an irregularly irregular cardiac rhythm, without S3, S4, or murmur and no neck vein distention.

Serum creatine kinase

Serum electrolytes

Serum thyroid stimulating hormone

B-type natriuretic peptide

Free thyroxine (FT4)

Ans: Serum thyroid stimulating hormone
Free thyroxine (FT4)

One of the most common causes of asymptomatic hypercalcemia in an otherwise well adult is:

Excessive use of calcium supplements.

Primary hyperparathyroidism.

Renal insufficiency.

Intestinal malabsorption.

Ans: Primary hyperparathyroidism.

When evaluating a patient for a potential thyroid disorder, which of the following is the least informative test to be used during evaluation?

Free T4

Total T4

Free T3

TSH

Ans: Total T4

Examination of a 56-year-old woman identifies a palpable thyroid mass of relatively fixed position. TSH level is within normal limits. Ultrasound reveals a solid mass of approximately 5 cm in size. Which of the following is the most appropriate next course of action?

Watch and wait with rescan in 6 months

Initiate levothyroxine therapy

Refer for fine-needle aspiration biopsy

Refer for radioiodine ablation

Ans: Refer for fine-needle aspiration biopsy

You see a 46-year-old woman with hypothyroidism who is currently being treated with levothyroxine at 100 mcg/d. She insists that she takes her medicine every morning. A laboratory test reveals that her TSH is 0.3 mIU/L (NL=0.4-4.0 mIU/L). You recommend:

Maintaining the levothyroxine dose and reevaluate in 2 months

Increasing the levothyroxine dose and reevaluate in 2 months

Decreasing the levothyroxine dose and reevaluate in 2 months

Discontinuing levothyroxine therapy and reevaluate in 6 months

ans: Decreasing the levothyroxine dose and reevaluate in 2 months

Katie is a 33-year-old woman who reports a 10-year history of unilateral, pulsing headache that lasts about 6-10 hours, occurring 3-4 times per month. The headache is typically preceded by a gradual onset of paresthesia affecting the ipsilateral face and arm, which lasts about 20 minutes.
She reports severe photophobia and phonophobia as well as left-sided cephalgia during the 6-10 h headache duration. She states the headaches appear randomly. As a result of her headaches, Katie typically needs to either call in sick or leave work early at least once a month due to headache.
Katie has used OTC medications with partial relief of pain but continued photo- and phonophobia. She is currently headache-free and neurological exam is within normal limits. Katie’s presentation is consistent with:

Migraine with aura

Tension-type headache

Cluster headache

Intracranial lesion

Ans: Migraine with aura

Katie asks if she needs “any tests to see what causes my headaches.” You respond that she should:

Based on the Scenario Below:
Katie is a 33-year-old woman who reports a 10-year history of unilateral, pulsing headache that lasts about 6-10 hours, occurring 3?4 times per month. The headache is typically preceded by a gradual onset of paresthesia affecting the ipsilateral face and arm, which lasts about 20 minutes.
She reports severe photophobia and phonophobia as well as left-sided cephalgia during the 6-10 h headache duration. She states the headaches appear randomly. As a result of her headaches, Katie typically needs to either call in sick or leave work early at least once a month due to headache.
Katie has used OTC medications with partial relief of pain but continued photo-and phonophobia. She is currently headache-free and neurological exam is within normal limits.

Have head CT conducted

Keep a headache diary for the next month

Be promptly referred to a neurologist

Have head MRI conducted

Ans: Keep a headache diary for the next month

Given Katie’s clinical presentation, you prescribe which of the following? Choose all that apply.

Based on the Scenario Below:
Katie is a 33-year-old woman who reports a 10-year history of unilateral, pulsing headache that lasts about 6-10 hours, occurring 3?4 times per month. The headache is typically preceded by a gradual onset of paresthesia affecting the ipsilateral face and arm, which lasts about 20 minutes.
She reports severe photophobia and phonophobia as well as left-sided cephalgia during the 6-10 h headache duration. She states the headaches appear randomly. As a result of her headaches, Katie typically needs to either call in sick or leave work early at least once a month due to headache.
Katie has used OTC medications with partial relief of pain but continued photo-and phonophobia. She is currently headache-free and neurological exam is within normal limits.

An oral triptan

Migraine prophylactic therapy

An oral NSAID

A short course of a systemic corticosteroid

Ans: An oral triptan,
Migraine prophylactic therapy
An oral NSAID

Katie requests advice on family planning. She has not been pregnant in the past. Given her history, which of the following is the least desirable contraceptive form for Katie?

Based on the Scenario Below:
Katie is a 33-year-old woman who reports a 10-year history of unilateral, pulsing headache that lasts about 6-10 hours, occurring 3?4 times per month. The headache is typically preceded by a gradual onset of paresthesia affecting the ipsilateral face and arm, which lasts about 20 minutes.
She reports severe photophobia and phonophobia as well as left-sided cephalgia during the 6-10 h headache duration. She states the headaches appear randomly. As a result of her headaches, Katie typically needs to either call in sick or leave work early at least once a month due to headache.
Katie has used OTC medications with partial relief of pain but continued photo-and phonophobia. She is currently headache-free and neurological exam is within normal limits.

Levonorgestrel IUD (Skyla®, Lilleta®, Mirena®)

Copper IUD (ParaGard®).

Combined estrogen-progestin oral contraceptive.

A progestin-only implant (Nexplanon®).

Ans: Combined estrogen-progestin oral contraceptive.

The NP considers that which of the following dietary components is the least likely to trigger migraine.

Sourdough bread

Cheddar cheese

Pickled herring

Ricotta cheese

Ans: Ricotta cheese

When considering evidence-based practice (EBP) recommendations for the use of prophylactic migraine treatment, which of the following is the preferred agent?

Propranolol.

Ergotamine.

Rizatriptan.

Verapamil.

Ans: Propranolol.

Which of the following represents the best choice of abortive migraine therapy for a 55-year-old woman with hypertension that is currently not adequately controlled due to poor medication adherence?

Verapamil.

Ergotamine.

Acetaminophen.

Almotriptan.

Ans: Acetaminophen.

According to the EBP recommendations, nutraceutical options for the prevention of recurrent migraine include the use of all of the following except:

Butterbur

Feverfew

Magnesium

Vitamin C

Ans:

Vitamin C

Mrs. Jensen is an 82-year-old woman with generalized osteoarthritis and systolic hypertension who presents with a 3-day history of right-sided headache with accompanying right-sided jaw pain on chewing. OTC analgesics have provided little relief. She states she is eating little due to the pain on chewing but is taking liquids without difficulty. Mrs. Jensen reports “I can hardly wash my hair, my scalp is so sore.” Physical examination reveals: BP=168/88 mm Hg bilateral, P=88 regular, RR=18, alert, appears uncomfortable with poorly-groomed hair on the right only PERLA, cranial nerve function II‒XII intact, remaining neurological exam WNL, carotid upstroke within normal limits without bruit. There is a tender, palpable, pulseless structure in the right temple area. Mrs. Jensen’s clinical presentation is most consistent with:

Postherpetic neuralgia.

Transient ischemic attack.

Giant cell arteritis.

Acute venous occlusion.

Ans: Giant cell arteritis.

Which of the following represents the best choice of initial test to support Mrs. Jensen’s presumptive diagnosis?

Based on the scenario below:
Mrs. Jensen is an 82-year-old woman with generalized osteoarthritis and systolic hypertension who presents with a 3-day history of right-sided headache with accompanying right-sided jaw pain on chewing. OTC analgesics have provided little relief. She states she is eating little due to the pain on chewing but is taking liquids without difficulty. Mrs. Jensen reports “I can hardly wash my hair, my scalp is so sore.” Physical examination reveals: BP=168/88 mm Hg bilateral, P=88 regular, RR=18, alert, appears uncomfortable with poorly-groomed hair on the right only PERLA, cranial nerve function II‒XII intact, remaining neurological exam WNL, carotid upstroke within normal limits without bruit. There is a tender, palpable, pulseless structure in the right temple area.

Erythrocyte sedimentation rate

Enhanced contrast brain MRI

Head CT without contrast

Arterial biopsy

Ans: Erythrocyte sedimentation rate

As you develop a treatment plan for Mrs. Jensen, you consider that likely:

Based on the scenario below:
Mrs. Jensen is an 82-year-old woman with generalized osteoarthritis and systolic hypertension who presents with a 3-day history of right-sided headache with accompanying right-sided jaw pain on chewing. OTC analgesics have provided little relief. She states she is eating little due to the pain on chewing but is taking liquids without difficulty. Mrs. Jensen reports “I can hardly wash my hair, my scalp is so sore.” Physical examination reveals: BP=168/88 mm Hg bilateral, P=88 regular, RR=18, alert, appears uncomfortable with poorly-groomed hair on the right only PERLA, cranial nerve function II‒XII intact, remaining neurological exam WNL, carotid upstroke within normal limits without bruit. There is a tender, palpable, pulseless structure in the right temple area.

NSAIDs will be helpful

Reducing her blood pressure is important

Long-term, high-dose systemic corticosteroid therapy will be needed

Opioid use is contraindicated

Ans: Long-term, high-dose systemic corticosteroid therapy will be needed

Which of the following is a potential serious complication of Mrs. Jensen’s presumptive diagnosis?

Based on the scenario below:
Mrs. Jensen is an 82-year-old woman with generalized osteoarthritis and systolic hypertension who presents with a 3-day history of right-sided headache with accompanying right-sided jaw pain on chewing. OTC analgesics have provided little relief. She states she is eating little due to the pain on chewing but is taking liquids without difficulty. Mrs. Jensen reports “I can hardly wash my hair, my scalp is so sore.” Physical examination reveals: BP=168/88 mm Hg bilateral, P=88 regular, RR=18, alert, appears uncomfortable with poorly-groomed hair on the right only PERLA, cranial nerve function II‒XII intact, remaining neurological exam WNL, carotid upstroke within normal limits without bruit. There is a tender, palpable, pulseless structure in the right temple area.

Hemiparesis

Arthritis

Blindness

Anterior uveitis

Ans: Blindness

When counseling a patient about lifestyle modifications to prevent migraines, the NP recommends:

Using tinted glasses to minimize glare and bright lights

Substituting artificial sweeteners for cane sugar

Avoiding regular exercise for at least one week after each migraine episode

Moving to a high-altitude environment

Ans: Using tinted glasses to minimize glare and bright lights

Which of the following agents can be considered for migraine controller therapy in a 54-year-old with a history of hypertension?

Sumatriptan

Topiramate

NSAIDs

Ergotamine

Ans:
Topiramate

Reasonable goals for using headache prophylactic therapy include all of the following except:

Reduce the frequency of headaches

Reduce the severity of headaches

Eliminate the incidence of headaches

Allow headache medications to work more effectively

Ans: Eliminate the incidence of headaches

Which of the following is unlikely to be reported by Mr. Kane, a 45-year-old man with a BMI=41 kg/m2 who presents with typical GERD symptoms?

Unintended weight loss

Hoarseness.

A recurrent cough

Chronic pharyngitis

Ans:

Unintended weight loss

Anticipated clinical findings in acute appendicitis for Jordan, an otherwise well 24-year-old man with a BMI=32 kg/m2 include all of the following except:

Leukocytosis with neutrophilia and bandemia

Positive obturator and psoas signs

A 2-hour history of sudden onset of vomiting and generalized abdominal pain accompanied by fever

A 12-hour history of epigastric discomfort and anorexia that gradually shifts to nausea and right lower quadrant abdominal pain

Ans: A 2-hour history of sudden onset of vomiting and generalized abdominal pain accompanied by fever

Mr. Lam, a 78-year-old man with longstanding GERD, presents with a 1-month history of “feeling like the food gets stuck way down in my throat.” This sensation occurs with meats and other solid food types and less likely with softer or liquid foods. He denies nausea, vomiting, constipation, diarrhea, aspiration, or melena. His physical examination is unremarkable. A hemogram today reveals a microcytic hypochromic anemia with an elevated RDW. The most likely etiology of this anemia is:

Anemia of chronic disease

Vitamin B12 deficiency

Iron deficiency

Acute blood loss

Ans:

Iron deficiency

All of the following diagnostic studies are available as part of Mr. Lam’s evaluation. Rank from 1 (most helpful) to 3 (least helpful) as these tests pertain to the evaluation of Mr. Lam’s symptoms.

Based on the scenario below:
Mr. Lam, a 78-year-old man with longstanding GERD, presents with a 1-month history of “feeling like the food gets stuck way down in my throat.” This sensation occurs with meats and other solid food types and less likely with softer or liquid foods. He denies nausea, vomiting, constipation, diarrhea, aspiration, or melena. His physical examination is unremarkable. A hemogram today reveals a microcytic hypochromic anemia with an elevated RDW.

A. Barium swallow __
B. Upper endoscopy __
C. Abdominal ultrasound __

A-2 B-1 C-3

A-1 B-2 C-3

A-3 B-1 C-2

A-2 B-3 C-1

Ans: A-2 B-1 C-3

When choosing pharmacologic intervention to prevent recurrence of duodenal ulcer, you prescribe:

A proton pump inhibitor

Timed antacid use

Antimicrobial therapy

A H2-receptor antagonist

Ans: Antimicrobial therapy

The most helpful imaging study in the evaluation of appendicitis for Jordan is an abdominal:

Based on the scenario below:
Jordan, an otherwise well 24-year-old man with a BMI=32 kg/m2 and acute appendicitis.

Flat plate

CT with contrast

MRI

Radionuclide scan

Ans: CT with contrast

Of the following imaging studies, place in rank order from greatest (1) to least (3) amount of ionizing radiation burden to the patient.

A. Abdominal ultrasound __
B. Abdominal CT __
C. Abdominal flat plate __

A-1 B-2 C-3

A-3 B-2 C-1

A-3 B-1 C-2

A-2 B-3 C-1

ans: A-3 B-1 C-2

Risk factors for pancreatic cancer include all of the following except:

Hypertension

History of chronic pancreatitis

Tobacco use

Diabetes mellitus

Ans:

Hypertension

Match each clinical presentation with one of the following conditions.

A. Erosive gastritis
B. Acute pancreatitis
C. Duodenal ulcer
D. Cholecystitis
E. Diverticulitis

1. A 45-year-old man who drinks 8-10 beers/day with a 12-hour history of acute-onset epigastric pain radiating into the back with bloating, nausea, vomiting. Objective: Epigastric tenderness, hypoactive bowel sounds, abdomen distended and hypertympanic. Laboratory evaluation reveals elevated lipase and amylase. __

2. A 64-year-old woman with a 3-day history of intermittent left lower quadrant (LLQ) abdominal pain accompanied by fever, cramping, nausea, and 4-5 loose stools per day. Objective: Abdomen soft, +bowel sounds, tenderness to LLQ abdominal palpation, negative Blumberg’s sign. Laboratory evaluation reveals leukocytosis with neutrophilia. __

3. A 34-year-old man w/3-month history of intermittent upper abdominal pain described as epigastric burning, gnawing pain about 2-3 h PC. Relief with foods, antacids. Awakening at 1-2 AM with symptoms. Objective: Tender at the epigastrium, LUQ, slightly hyperactive bowel sounds. __

4. A 52-year-old woman who was recently laid off from her job, taking 3-4 doses of ibuprofen/day for the past 2-3 months to help with headaches; 1-month history of intermittent nausea, burning, and pain, limited to upper abdomen, often worse with eating. Objective: Tender at the epigastrium, LUQ, slightly hyperactive bowel sounds. __

5. A 54-year-old woman who presents with a 24-h history of significant epigastric and RUQ abdominal pain that is constant with 2‒3-minute periods of increased pain, accompanied by nausea, 2 episodes of vomiting, and intermittent fever. Objective: Tenderness at the epigastrium and abdominal RUQ, positive Murphy’s sign, moderately elevated AST, ALT, and ALP. __

1-B 2-C 3-A 4-E 5-D

1-A 2-D 3-C 4-B 5-E

1-B 2-E 3-C 4-A 5-D

1-C 2-A 3-E 4-B 5-D

Ans: 1-B 2-E 3-C 4-A 5-D

Adults born from 1945 through 1965 should be encouraged to be screened for HCV regardless of HCV risk factors.


True

False

Ans: True

Match the viral hepatitis type with the most common method of acquisition. An option can only be used once.

A. Hepatitis A
B. Hepatitis B
C. Hepatitis C

1. Fecal-contaminated food or water __
2. Injection Drug Use __
3. Sexual Contact __

1-C 2-B 3-A

1-B 2-A 3-C

1-A 2-B 3-C

1-A 2-C 3-B

ans: 1-A 2-C 3-B

Janet, a 47-year-old well woman who is 64″ (163 cm) tall with BMI=25 kg/m2, presents for a periodic health evaluation and to establish care in your practice. She reports drinking approximately 1 to 2 mixed drinks containing 1.5 oz. (44.4 mL) 80-proof liquor per week and takes approximately 1 to 2 doses of acetaminophen per month. Her last visit with a healthcare provider was more than 5 years ago; she was told that her “liver tests were a bit higher than normal.” She felt well and decided not to return for follow-up. She denies history of injection drug use and has no tattoos. Her last vaccines were administered more than 10 years ago. She is without complaint. Abdominal examination is within normal limits. Janet’s laboratory results are as follows.

HBsAg=positive (evidence HBV on board, Ag=Always growing, HBV persists, in acute or chronic hepatitis B) Anti-HAV=positive (immune to HAV, include HAV IgG, G=Gone) Anti-HCV=negative (no evidence of past or present HCV infection) AST=56 unit/L (0-40) (modest elevation, with ALT>AST) ALT=98 unit/L (0-40) (modest elevation, with ALT>AST)
These findings are most consistent with which of the following? Choose two that apply:

Chronic hepatitis B

Evidence of hepatitis B infection immunity

Evidence of hepatitis A immunity

Acute hepatitis B infection

Evidence of hepatitis C infection

Ans: Chronic hepatitis B,

Evidence of hepatitis A immunity

Choosing all that apply, you offer Janet the following immunizations against:

Based on the scenario below:
Janet, a 47-year-old well woman who is 64″ (163 cm) tall with BMI=25 kg/m2, presents for a periodic health evaluation and to establish care in your practice. She reports drinking approximately 1 to 2 mixed drinks containing 1.5 oz. (44.4 mL) 80-proof liquor per week and takes approximately 1 to 2 doses of acetaminophen per month. Her last visit with a healthcare provider was more than 5 years ago; she was told that her “liver tests were a bit higher than normal.” She felt well and decided not to return for follow-up. She denies history of injection drug use and has no tattoos. Her last vaccines were administered more than 10 years ago. She is without complaint. Abdominal examination is within normal limits. Janet’s laboratory results are as follows.

HBsAg=positive (evidence HBV on board, Ag=Always growing, HBV persists, in acute or chronic hepatitis B) Anti-HAV=positive (immune to HAV, include HAV IgG, G=Gone) Anti-HCV=negative (no evidence of past or present HCV infection) AST=56 unit/L (0-40) (modest elevation, with ALT>AST) ALT=98 unit/L (0-40) (modest elevation, with ALT>AST).

Hepatitis B

Hepatitis A

Seasonal influenza

Tetanus, diphtheria, and acellular pertussis

Ans: Seasonal influenza

Tetanus, diphtheria, and acellular pertussis

Thomas is a 45-year-old man who has a past history of injection drug use, currently with 5 years of sobriety. He is unsure about his vaccine history but believes he had “some when I was a child.” Results of recent laboratory tests reveal the following. HCV RNA=positive (evidence of current HCV infection) Anti-HBs/HBSab=positive (evidence of HBV immunity, HBSab= B=”Bye” as HBV no longer on board) Anti-HAV=negative (no evidence of HAV infection or immunity, negative= “Never had HAV”) AST=45 unit/L (0-40) (modest elevation, with ALT>AST)ALT=72 unit/L (0-40) (modest elevation, with ALT>AST)
You offer the following immunization(s) against:

Hepatitis B

Hepatitis A

Hepatitis A and B

Seasonal influenza, tetanus, diphtheria, and acellular pertussis

Ans: Hepatitis A

Which of the following WBC response would be most consistent in a patient with acute appendicitis?

WBC 5000/mm3, Neutrophils 32%, Bands 4%

WBC 17,500/mm3, Neutrophils 72%, Bands 10%

WBC 14,000/mm3, Neutrophils 24%, Bands 3%

WBC <1000/mm3, Neutrophils 52%, Bands 6%

Ans: WBC 17,500/mm3, Neutrophils 72%, Bands 10%

ndicate whether each characteristic is better associated with irritable bowel syndrome (IBS) or inflammatory bowel disease (IBD).

1. Intestinal ulceration present.
2. Absence of rectal bleeding.
3. CRP and ESR levels are elevated.
4. Surgical intervention often needed.

1-IBD 2-IBS 3-IBS 4-IBD

1-IBS 2-IBS 3-IBS 4-IBD

1-IBD 2-IBS 3-IBD 4-IBD

1-IBS 2-IBD 3-IBS 4-IBD

Ans: 1-IBD 2-IBS 3-IBD 4-IBD

Appropriate outpatient treatment for a 62-year-old woman with mild diverticulitis who is without vomiting and suitable for outpatient therapy is:

Oral or IV linezolid (Zyvox®)

Oral metronidazole (Flagyl®)

Oral levofloxacin (Levaquin®) plus oral metronidazole (Flagyl®)

IV daptomycin (Cubicin®) plus oral clindamycin (Cleocin®)

Ans: Oral levofloxacin (Levaquin®) plus oral metronidazole (Flagyl®)

You anticipate finding the following on physical examination during an acute asthma flare with a markedly reduced FEV1?

Intercostal retraction

Inspiratory crackles

Increased tactile fremitus

Hyperresonance

Ans:

Hyperresonance

Likely causative organisms in community-acquired pneumonia include:

S. pneumoniae and select respiratory viruses

H. influenzae and S. aureus

M. catarrhalis and atypical pathogens

K. pneumoniae and Legionella species

Ans:

S. pneumoniae and select respiratory viruses

Mr. Spaulding is a 70-year-old man with a 50 pack-year cigarette smoking history, chronic obstructive pulmonary disease, and hypertension, who presents with a 24-hour history of increasing dyspnea and productive cough with white-yellow sputum. He is alert, oriented, and answers questions with ease. Physical examination reveals the following:
Alert, breathing slightly labored at rest, BP=130/78 mm Hg, T=99.8°F (37.7°C), HR=96, RR=22, dullness to percussion over the left base with increased tactile fremitus and tubular breath sounds as well as crackles in the right base. Cardiac examination reveals no S3, no S4, no murmur, with nondistended neck veins.
His physical examination findings are suggestive of:

A left lower lobe consolidation

Diffuse hyperinflation

Heart failure

Compromised pulmonary vascular perfusion

Ans:

A left lower lobe consolidation

Teresa, a 38-year-old woman with no chronic health problems, presents with a chief complaint of “A cold I cannot shake for the past three weeks. She also reports an intermittent frontal headache and has taken acetaminophen with relief, as well as general malaise and a dry cough that is particularly problematic at night. She denies nausea, vomiting, chills, fever, or dyspnea. She has not taken an antimicrobial within the past year, underwent a bilateral tubal ligation approximately 10 years ago, and is allergic to penicillin with a hive-form reaction.
Examination reveals the following: SaO2=97%, BP=114/70 mm Hg, T=98°F (36.7°C), HR=88, RR=20, bilateral coarse late inspiratory crackles without wheeze. She is in no acute distress. Chest x-ray demonstrates bilateral interstitial infiltrates.
Which of the following describes the recommended additional diagnostic testing for Teresa? Choose all that apply.

Blood urea nitrogen (BUN) and creatinine (Cr)

CBC with WBC differential

Blood culture

Sputum culture

Ans: Blood urea nitrogen (BUN) and creatinine (Cr), CBC with WBC differential

The results of her laboratory testing do not show alarm findings. In treating Teresa, you prescribe the following.

Based on the scenario below:
Teresa, a 38-year-old woman with no chronic health problems, presents with a chief complaint of “A cold I cannot shake for the past three weeks. She also reports an intermittent frontal headache and has taken acetaminophen with relief, as well as general malaise and a dry cough that is particularly problematic at night. She denies nausea, vomiting, chills, fever, or dyspnea. She has not taken an antimicrobial within the past year, underwent a bilateral tubal ligation approximately 10 years ago, and is allergic to penicillin with a hive-form reaction.
Examination reveals the following: SaO2=97%, BP=114/70 mm Hg, T=98°F (36.7°C), HR=88, RR=20, bilateral coarse late inspiratory crackles without wheeze. She is in no acute distress. Chest x-ray demonstrates bilateral interstitial infiltrates.

A 7-day course of oral doxycycline

A 10-day course of oral moxifloxacin

A 3-day course of parenteral vancomycin

Timed inhaled short-acting beta2-agonist use

Ans: A 7-day course of oral doxycycline

Teresa’s CURB-65 score __



Based on the scenario below:
Teresa, a 38-year-old woman with no chronic health problems, presents with a chief complaint of “A cold I cannot shake for the past three weeks. She also reports an intermittent frontal headache and has taken acetaminophen with relief, as well as general malaise and a dry cough that is particularly problematic at night. She denies nausea, vomiting, chills, fever, or dyspnea. She has not taken an antimicrobial within the past year, underwent a bilateral tubal ligation approximately 10 years ago, and is allergic to penicillin with a hive-form reaction.
Examination reveals the following: SaO2=97%, BP=114/70 mm Hg, T=98°F (36.7°C), HR=88, RR=20, bilateral coarse late inspiratory crackles without wheeze. She is in no acute distress. Chest x-ray demonstrates bilateral interstitial infiltrates.

0-1

1-2

2-3

3-4

Ans: 0-1

Teresa’s location of treatment should be __

Based on the scenario below:
Teresa, a 38-year-old woman with no chronic health problems, presents with a chief complaint of “A cold I cannot shake for the past three weeks. She also reports an intermittent frontal headache and has taken acetaminophen with relief, as well as general malaise and a dry cough that is particularly problematic at night. She denies nausea, vomiting, chills, fever, or dyspnea. She has not taken an antimicrobial within the past year, underwent a bilateral tubal ligation approximately 10 years ago, and is allergic to penicillin with a hive-form reaction.
Examination reveals the following: SaO2=97%, BP=114/70 mm Hg, T=98°F (36.7°C), HR=88, RR=20, bilateral coarse late inspiratory crackles without wheeze. She is in no acute distress. Chest x-ray demonstrates bilateral interstitial infiltrates.

Inpatient

Outpatient

Ans: Outpatient

Mr. Spaulding returns in 2 days. He states that he is feeling somewhat better, with less shortness of breath but with continued fatigue and production of small amounts of white-yellow sputum. He states he has taken his antimicrobial therapy as advised without difficulty. Physical examination reveals the following: Alert, BP=130/78 mm Hg, T=97.8°F (36.6°C), HR=88, RR=18, dullness to percussion over the left base with increased tactile fremitus and tubular breath sounds. Cardiac examination reveals no S3, no S4, no murmur, with nondistended neck veins. At this point, you consider the following two best options:

Based on the scenario below:
Mr. Spaulding is a 70-year-old man with a 50 pack-year cigarette smoking history, chronic obstructive pulmonary disease, and hypertension, who presents with a 24-hour history of increasing dyspnea and productive cough with white-yellow sputum. He is alert, oriented, and answers questions with ease. Physical examination reveals the following:
Alert, breathing slightly labored at rest, BP=130/78 mm Hg, T=99.8°F (37.7°C), HR=96, RR=22, dullness to percussion over the left base with increased tactile fremitus and tubular breath sounds as well as crackles in the right base. Cardiac examination reveals no S3, no S4, no murmur, with nondistended neck veins.


A repeat chest x-ray should be obtained today

His antimicrobial needs to be changed to an agent with wider spectrum of activity

Mr. Spaulding should be advised to complete his current course of therapy

Pneumococcal and seasonal influenza vaccines should be updated today as needed

Ans: Mr. Spaulding should be advised to complete his current course of therapy

Pneumococcal and seasonal influenza vaccines should be updated today as needed

Which of the numbered areas on the chest x-ray best correlates with Mr. Spaulding’s physical examination findings?

Based on the scenario below:
Mr. Spaulding is a 70-year-old man with a 50 pack-year cigarette smoking history, chronic obstructive pulmonary disease, and hypertension, who presents with a 24-hour history of increasing dyspnea and productive cough with white-yellow sputum. He is alert, oriented, and answers questions with ease. Physical examination reveals the following:
Alert, breathing slightly labored at rest, BP=130/78 mm Hg, T=99.8°F (37.7°C), HR=96, RR=22, dullness to percussion over the left base with increased tactile fremitus and tubular breath sounds as well as crackles in the right base. Cardiac examination reveals no S3, no S4, no murmur, with nondistended neck veins.

Ans: left lower lobe with consolidation (picture in module) look for Xray film images online.

Mr. Spaulding’s current medications include an inhaled corticosteroid, a long-acting beta2-agonist, an ACE inhibitor with a thiazide diuretic, a statin, and low-dose aspirin, as well as a short-acting beta2-agonist as needed. Laboratory testing reveals BUN and hematocrit=WNL.
The preferred choice of antimicrobial therapy for Mr. Spaulding is a:

Based on the scenario below:
Mr. Spaulding is a 70-year-old man with a 50 pack-year cigarette smoking history, chronic obstructive pulmonary disease, and hypertension, who presents with a 24-hour history of increasing dyspnea and productive cough with white-yellow sputum. He is alert, oriented, and answers questions with ease. Physical examination reveals the following:
Alert, breathing slightly labored at rest, BP=130/78 mm Hg, T=99.8°F (37.7°C), HR=96, RR=22, dullness to percussion over the left base with increased tactile fremitus and tubular breath sounds as well as crackles in the right base. Cardiac examination reveals no S3, no S4, no murmur, with nondistended neck veins.

7-day course of oral clarithromycin

5-day course of oral levofloxacin

10-day course of oral amoxicillin-clavulanate

3-day course of injectable ceftriaxone

Ans: 5-day course of oral levofloxacin

Mr. Spaulding’s CURB-65 score. __

Based on the scenario below:
Mr. Spaulding is a 70-year-old man with a 50 pack-year cigarette smoking history, chronic obstructive pulmonary disease, and hypertension, who presents with a 24-hour history of increasing dyspnea and productive cough with white-yellow sputum. He is alert, oriented, and answers questions with ease. Physical examination reveals the following:
Alert, breathing slightly labored at rest, BP=130/78 mm Hg, T=99.8°F (37.7°C), HR=96, RR=22, dullness to percussion over the left base with increased tactile fremitus and tubular breath sounds as well as crackles in the right base. Cardiac examination reveals no S3, no S4, no murmur, with nondistended neck veins.

0

1

2

3

4

Ans:

1

Mr. Spaulding’s location of treatment should be __.


Based on the scenario below:
Mr. Spaulding is a 70-year-old man with a 50 pack-year cigarette smoking history, chronic obstructive pulmonary disease, and hypertension, who presents with a 24-hour history of increasing dyspnea and productive cough with white-yellow sputum. He is alert, oriented, and answers questions with ease. Physical examination reveals the following:
Alert, breathing slightly labored at rest, BP=130/78 mm Hg, T=99.8°F (37.7°C), HR=96, RR=22, dullness to percussion over the left base with increased tactile fremitus and tubular breath sounds as well as crackles in the right base. Cardiac examination reveals no S3, no S4, no murmur, with nondistended neck veins.

Inpatient

Outpatient

Ans: Outpatient

Which of the following is the most common pathogen implicated in acute bronchitis?

S. pneumoniae

H. influenzae

M. pneumoniae

Respiratory virus

Ans: Respiratory virus

A 63-year-old man who resides at the homeless shelter is brought to urgent care with a chief complaint of chronic cough and chest pain. His temperature is 102.3°F (39.1°C), respiratory rate of 45 bpm, and blood pressure of 135/85 mm Hg. Which of the following findings would best support a diagnosis of active tuberculosis infection?

Erythematous plaques on the extremities

Hemoptysis

Dry cough with bronchospasm

Unexplained weight gain

Ans: Hemoptysis

A chest x-ray is ordered on a 47-year-old man with suspected active tuberculosis. The radiograph supports this diagnosis with findings of a/an: (Picture 2 Ch 10 Tuberculosis)


Infiltrate in the left upper lobe

Infiltrate in the right upper lobe

Pleural effusion in the left upper lobe

Pleural effusion in the right upper lobe

Ans: nfiltrate in the right upper lobe ( picture in the module or online)

Which of the following is consistent with the diagnosis of all stages of chronic obstructive pulmonary disease?

FEV1: FVC ratio <0.70 post-bronchodilator

Dyspnea on exertion

Hypoxemia

Orthopnea

Ans:

FEV1: FVC ratio <0.70 post-bronchodilator

You see Michelle, a 38-year-old woman with moderate persistent asthma who is using medium-dose inhaled fluticasone daily and albuterol via MDI as needed. In a typical month, she uses albuterol 2 puffs “about 2 times, when I feel my chest getting a little tight, and it works right away.” In an average month, she has no episodes of nocturnal awakening with cough or wheeze, typically has excellent activity tolerance. Which of the following is the most important additional clinical parameter to obtain today in evaluating Michelle’s baseline asthma control?

SaO2

Peak expiratory flow

Auscultation of breath sounds

Resting respiratory rate

Ans:

Peak expiratory flow

Michelle returns for an urgent care visit three weeks later with a 2-day history of URI symptoms. She has a dry cough and reports, “The albuterol is not working as well as usual. I was up all night coughing. When I cough, a little bit of white phlegm sometimes comes up.” She denies fever, nausea, or vomiting. Which of the following is the most important clinical parameter in assessing Michelle’s asthma flare?
Based on the scenario below:
You see Michelle, a 38-year-old woman with moderate persistent asthma who is using medium-dose inhaled fluticasone daily and albuterol via MDI as needed. In a typical month, she uses albuterol 2 puffs “about 2 times, when I feel my chest getting a little tight, and it works right away.” In an average month, she has no episodes of nocturnal awakening with cough or wheeze, typically has excellent activity tolerance.

SaO2=97%.

Peak expiratory flow=55% of personal best

Presence of bilateral expiratory wheezes

Patient report of reduced response to beta2-agonist use

Ans: Peak expiratory flow=55% of personal best

Michelle’s physical examination is consistent with an asthma flare triggered by viral RTI. Her medication regimen should be adjusted to include which of the following? Choose all that apply.
Based on the scenario below:
You see Michelle, a 38-year-old woman with moderate persistent asthma who is using medium-dose inhaled fluticasone daily and albuterol via MDI as needed. In a typical month, she uses albuterol 2 puffs “about 2 times, when I feel my chest getting a little tight, and it works right away.” In an average month, she has no episodes of nocturnal awakening with cough or wheeze, typically has excellent activity tolerance.

An oral systemic bronchodilator titrated to therapeutic level

A short course of oral corticosteroid

An oral macrolide antimicrobial

A single dose of injectable corticosteroid

Ans: A short course of oral corticosteroid

A 76-year-old man with COPD, who uses an inhaled LAMA daily on a set schedule and SABA via MDI as needed for symptom relief, presents with a COPD exacerbation. Which of the following describes the role of imaging in the evaluation of COPD exacerbation?

Chest x-ray is a routine part of the evaluation of a person with COPD exacerbation

In COPD exacerbation, chest x-ray should not be obtained due to radiation risk

A chest x-ray should be ordered in COPD exacerbation in the patient with fever and/or low SaO2 to help rule out concomitant pneumonia

A thoracic CT is the preferred imaging study to order in a COPD exacerbation

Ans: A chest x-ray should be ordered in COPD exacerbation in the patient with fever and/or low SaO2 to help rule out concomitant pneumonia

All of the following are characteristics of long-acting muscarinic antagonists (LAMA) when used in the management of COPD except:

Provide relief of acute bronchospasm

Offer a protracted duration of bronchodilation

Minimize the risk of COPD exacerbation

Are usually used on a daily set schedule

Ans: Provide relief of acute bronchospasm

Matthews is a 78-year-old woman with severe COPD, who is currently using tiotropium bromide via inhaler and medium-dose inhaled fluticasone with salmeterol via MDI, and who presents for an acute care visit. Her current medications include enalapril, hydrochlorothiazide, lovastatin, and low-dose aspirin. She reports increasing dyspnea and a productive cough with small amounts of yellow-green sputum for the past 24 hours. She states, “I hardly slept at all last night. I kept waking up coughing.” She denies nausea, vomiting, or fever. Physical examination reveals bilateral expiratory wheezes and rhonchi with hyperresonance to percussion without increased tactile fremitus or dullness to percussion. SaO2=98%, T=97.6°F (36.4°C), BP=136/84 mm Hg, P=92, regular, RR=20.
When considering pharmacologic therapy to treat Ms. Matthews, the NP prescribes:

Oral cefpodoxime and a single dose of an injectable, sustained-release corticosteroid

Oral clarithromycin with an increase of her inhaled corticosteroid/LABA via MDI by 2 puffs per day

IM ceftriaxone and an opioid-containing cough suppressant

Oral doxycycline with a short-course of an oral corticosteroid

Ans: Oral doxycycline with a short-course of an oral corticosteroid

Which of the following classes of agents used in the management of COPD best provides a protracted duration of bronchodilation?

Short-acting beta2-agonists (SABA)

Inhaled corticosteroids

PDE-4 inhibitor

Long-acting beta2-agonists (LABA)

Ans: Long-acting beta2-agonists (LABA)

Which of the following agents/drug classes used in the management of COPD provides one of the first-line drug therapies used to minimize the risk of exacerbation?

Oral xanthine (e.g., theophylline [Theolair®])

Inhaled long-acting muscarinic antagonists (LAMA) (e.g., tiotropium bromide [Spiriva®])

Inhaled short-acting beta2-agonist (SABA) (e.g., albuterol [ProAir®])

Oral PDE-4 inhibitor (e.g., roflumilast [Daliresp®])

Ans: Inhaled long-acting muscarinic antagonists (LAMA) (e.g., tiotropium bromide [Spiriva®])

The NP is counseling a 67-year-old patient with severe COPD who is considering the use of long-term oxygen therapy. The patient is currently taking an inhaled corticosteroid (ICS) with a long-acting beta2-agonist (LABA). The NP advises that:

Oxygen therapy can be used in place of the ICS

Effective treatment will require at least 4‒6 hours of therapy per day

Effective treatment will require at least 15 hours of therapy per day

Consistent use of oxygen therapy can improve FEV1 to near normal values

Ans: Effective treatment will require at least 15 hours of therapy per day

Juan is a 30-year-old man with a 10-year history of moderate persistent asthma who presents for a routine visit, requesting refills of his medium-dose budesonide per DPI and albuterol per MDI. He states, “I am doing great, no complaints here.” His blood pressure, heart rate, temperature, and respiratory rate are within normal limits, he is in no distress, and his physical exam reveals no evidence of wheezing or crackles. You consider that:

His current treatment regimen should be continued

Due to patient report and today’s objective findings, his asthma appears to be well-controlled

His asthma therapy should be intensified

More information is needed to determine his level of asthma control

Ans:

More information is needed to determine his level of asthma control

You see Kevin, a 25-year-old man with a BMI=38 kg/m2 and hyperpigmented plaques with a velvet-like appearance at the nape of the neck and axillary region. He states his skin has had this appearance since he was approximately age 13 and it has not changed significantly over time. He denies itch or pain in these areas. This most likely represents:

Acanthosis nigricans.

Lichens planus.

Actinic keratosis.

Erythema migrans.

Ans: Acanthosis nigricans.

Ms. Leonardo is a 68-year-old woman of Italian ancestry who presents with a new diagnosis of primary hypertension. Recent lab analysis reveals the
following: A1C=5.3% (0.053 proportion), calculated GFR=98 mL/min/1.73 m2.
Physical examination findings include the following: BMI=26.4 kg/m2, no S3, S4, or murmur, PMI at 5th ICS, MCL. Funduscopic examination is within normal limits. In the past 6 months, she has increased physical activity by walking for at least 40 minutes 5-6 days per week and has cut back on dietary sodium. Which of the following represent(s) the best advice? Choose all that apply.

Advise that beta-blocker therapy should be started

Initiate therapy with a thiazide diuretic

Encourage weight reduction

Advise Ms. Leonardo that drug therapy will be initiated when there is evidence of target organ damage

Ans: Initiate therapy with a thiazide diuretic, Encourage weight reduction

As part of Kevin’s healthcare, consideration should be given for obtaining which of the following laboratory tests?

Based on the below scenario:
You see Kevin, a 25-year-old man with a BMI=38 kg/m2 and hyperpigmented plaques with a velvet-like appearance at the nape of the neck and axillary region. He states his skin has had this appearance since he was approximately age 13 and it has not changed significantly over time. He denies itch or pain in these areas.

Hemoglobin A1C

Serum transaminases

Rapid plasma reagent

Erythrocyte sedimentation rate
Feedback

Ans:

Hemoglobin A1C

With Kevin’s skin condition, lesions are most often found in all of the following areas except:

Based on the below scenario:
You see Kevin, a 25-year-old man with a BMI=38 kg/m2 and hyperpigmented plaques with a velvet-like appearance at the nape of the neck and axillary region. He states his skin has had this appearance since he was approximately age 13 and it has not changed significantly over time. He denies itch or pain in these areas.

Groin folds.

Over the knuckles.

Elbows.

Plantar surface of the feet.

Ans: Plantar surface of the feet.

You see an 81-year-old woman with type 2 DM. Today she presents for follow-up care. Results of recent laboratory assessment include the following: GFR=45 mL/min/1.73 m2, A1C=7.2% (0.072 proportion), K+=4.8 mEq/L (4.8 mmol/L). Her current medications include metformin, glyburide, lisinopril, and simvastatin. She is generally feeling well, is self-sufficient in ADL, and states, “I can get around in my home with my cane. I am a little less certain on my feet if I do not use the cane.”


Due to age and impaired renal function, her risk of adverse effects from metformin is increased.

Glipizide is preferred over glyburide as a sulfonylurea in the older adult.

With poor hydration, she is at risk for hyperkalemia.

She should be treated to an A1C goal of <7% (0.07 proportion).

Ans: She should be treated to an A1C goal of <7% (0.07 proportion).

You see a 55-year-old woman with type 2 diabetes mellitus, hypertension, and dyslipidemia. Evaluation today reveals a BMI=36 kg/m2. She states, “I just do not know where to start in trying to lose some weight.” Which of the following is the most appropriate response to this statement?


“How much weight do you want to lose?”

“How do you feel about your weight?”

“What barriers do you see to losing weight?”

“Your blood sugar control will likely improve if you lose some weight.”

Ans: “What barriers do you see to losing weight?”

With an 8:00 AM dose of the following insulins, followed by inadequate dietary intake/excessive energy utilization, at what time would hypoglycemia occur?

Select the time of onset of hypoglycemia Medication:

A. 8:30AM-10:30AM
B. 10:00AM-11:00AM
C. 2:00 PM-10:00 PM

1. Lispro (Humalog®) __
2. Regular insulin (Humulin R®) __
3. NPH (Novolin N®, Humulin N®) __

1-A 2-B 3-C

1-C 2-A 3-B

1-B 2-A 3-C

1-A 2-C 3-B

Ans: 1-A (onset: 30 mins, peak: 1-2 hr) 2-B ( onset: 1hr peak: 1-3 hrs 3-C (onset: 1-2 hrs; peak: 3-14hrs)

Mr. Jones, a 52-year-old African-American man, presents for treatment of hypertension. He is currently taking a calcium channel blocker for the past 2 months. He is feeling well with no complaint and physical examination is within normal limits. His blood pressure today is 160/94 mm Hg bilaterally. The next best step is to:

Add an ACEI and have the patient follow-up in 2 weeks

Discontinue the calcium channel blocker and start an aldosterone antagonist

Advise Mr. Jones that his blood pressure is in an acceptable range and he should follow-up in approximately 2 months

Prescribe a thiazide diuretic and advise a 1-month follow-up

Ans: Prescribe a thiazide diuretic and advise a 1-month follow-up

Mr. Kelly is a 66-year-old man with type 2 DM and HTN who presents with the following lipid profile:

HDL=35 mg/dL (0.9 mmol/L)
LDL=150 mg/dL (3.9 mmol/L)
Triglycerides=210 mg/dL (2.4 mmol/L)

Which of the following represents the best choice of dyslipidemia therapy for Mr. Kelly?

Optimized oral selective cholesterol absorption inhibitor therapy

Moderate-intensity oral statin therapy

Injectable PCSK9 therapy

High-intensity oral omega-3 fatty acid therapy

Ans: Moderate-intensity oral statin therapy

Which of the following medications should be avoided in a 45-year-old man with poorly-controlled hypertension who asks, “What medications can I take when I have a cold?”

Dextromethorphan

Chlorpheniramine

Pseudoephedrine

Guaifenesin

Ans: Pseudoephedrine

A 45-year-old woman with hypertension and dyslipidemia is currently taking moderate-intensity statin therapy and she is at LDL-reduction goal. She is feeling well. As part of her ongoing healthcare, the NP considers that:


A. Serum transaminases should be checked periodically.

Evaluation of serum creatine kinase is needed.

A CBC with WBC and platelet count is advised.

In the absence of symptoms or concerns, no particular laboratory monitoring is recommended

Ans: In the absence of symptoms or concerns, no particular laboratory monitoring is recommended.

When initiating treatment with an antihypertensive medication for a 72-year-old male with mild renal impairment, you consider all of the following except:

Sodium depletion can result with the use of a thiazide diuretic.

An increased risk of hyperkalemia is associated with ARB use.

The risk of hyperkalemia with the use of an aldosterone antagonist is increased when combined with an ACE inhibitor.

Calcium channel blockers are appropriate to use in the presence of heart failure.

Ans: Calcium channel blockers are appropriate to use in the presence of heart failure.

The typical international normalized ratio (INR) goal during warfarin therapy for a 65-year-old woman with atrial fibrillation is:

1.0‒2.0.

1.5‒2.5.

2.0‒3.0.

2.5‒3.5.

ans: 2.0‒3.0.

Which of the following drug classes will yield the greatest decrease in blood pressure for a middle-aged African American male?

Calcium channel blocker

Beta-blocker

ACE inhibitor

Angiotensin receptor blocker

ans: Calcium channel blocker

You see a 56-year-old male who is newly diagnosed with dyslipidemia. Prior to initiating statin treatment, his hepatic enzyme levels are measured to establish baseline values. In the absence of symptoms of liver injury, which of the following is the recommended schedule for hepatic enzyme monitoring during statin use?

Every 3 months

Every 6 months

Every 12 months

No additional hepatic enzyme monitoring is needed

Ans: No additional hepatic enzyme monitoring is needed

You start a patient on spironolactone who is also on an angiotensin-converting enzyme inhibitor. You advise the patient to return in 4 weeks to check which of the following laboratory parameters?

Sodium

Calcium

Potassium

Chloride

Ans: Potassium

Match each medication used to manage dyslipidemia with its major anticipated change in the lipid profile. An answer can be used more than once.

A. Decreases triglycerides
B. Decreases triglycerides, increases HDL-C
C. Decreases LDL-C

1. Statins __
2. Niacin __
3. Fibrates __
4. Fish oil __
5. PCSK9 inhibitor __

1-C 2-A 3-B 4-B 5-A

1-C 2-B 3-B 4-A 5-C

1-B 2-C 3-A 4-B 5-A

1-A 2-B 3-C 4-C 5-A

Ans: 1-C (↓ triglycerides) 2-B ↓triglycerides, ↑ HDL 3-B (↓ triglycerides, ↑HDL) 4-A ( ↓trigylcerides ) 5-C (↓LDL)

All of the following are potential causes of secondary hypertriglyceridemia except:

Untreated or undertreated hypothyroidism.

High-dose ACE inhibitor use.

Poorly-controlled DM.

Excessive alcohol use.

Ans: High-dose ACE inhibitor use.

The routine use of diuretics can be considered for patients with heart failure beginning at:

Stage A.

Stage B.

Stage C.

Stage D.

Ans:

Stage C.

Genital Candida albicans infection in men typically presents with all of the following except:

Penile discharge.

Balanitis.

Groin-fold involvement.

Scrotal excoriation.

Ans: Penile discharge.

Which of the following medications can be considered for patients at risk for heart failure but without structural heart disease (Stage A)?

Beta-blockers

Thiazide diuretics

ARB or ACEI

Digitalis

ans: ARB or ACEI

Choose from the following:

A. Bacterial vaginosis
B. Candida vulvovaginitis
C. Nongonococcal cervicitis/vaginitis

1. Clue cells with alkaline pH __
2. Pseudohyphae __
3. Abundant WBCs __

1-A 2-B 3-C

1-B 2-A 3-C

1-C 2-B 3-A

1-A 2-C 3-B

Ans: 1-A (clue cells with alkaline pH) 2-B (pseudohyphae) 3-C (WBC’s)

Choose from the following (an option can be used more than once):

A. Candida vulvovaginitis
B. Trichomoniasis
C. Bacterial vaginosis
D. Trichomoniasis, Bacterial vaginosis

1. Clotrimazole cream __
2. Oral metronidazole __
3. Metronidazole gel __
4. Clindamycin cream __

1-A 2-D 3-C 4-C

1-B 2-C 3-D 4-A

1-D 2-D 3-C 4-A

1-A 2-D 3-B 4-C

Ans: 1-A ( Candida vulvovaginitis – use Clotrimazole) 2-D (BV and Trichomoniasis uses oral Metridonazole ) 3-C (BV uses metronidazole cream) 4-C 9 (metronidazole cream treats trichomoniasis)

Treatment for which of the following STIs? An option can be used more than once.

A. Ceftriaxone
B. Injectable penicillin
C. Imiquimod
D. Trichloroacetic acid (TCA)

1. External genital warts in a 25-year-old man __
2. Gonococcal urethritis in a 22-year-old man __
3. Syphilis in a 45-year-old man __
4. External genital warts in a 28-year-old pregnant woman __

1-C 2-A 3-B 4-D

1-D 2-A 3-B 4-C

1-A 2-B 3-C 4-D

1-B 2-D 3-A 4-C

Ans: 1-C (use Imiquimod cream to treats external genital warts ) 2-A (Ceftriaxone 2 gr IM treats gonococcal urethritis ) 3-B (Injectable Penicillin 2 Mill VK treats Syphillis) 4-D (Trichloacetic acid uses to treat external genital warts for 28yr pregnant woman)

considerations in caring for a 68-year-old man with a BMI=38 kg/m2 who is otherwise well and presents with genital candidiasis includes which of two most helpful measures?

Advice on the use of antibacterial soap to the region.

Obtain an in-office blood glucose.

Prescribing topical miconazole.

Order a medium-potency topical corticosteroid to the affected region to help with symptom control.

Ans:

Obtain an in-office blood glucose.
Prescribing topical miconazole.

Which of the following is inconsistent with benign prostatic hyperplasia?

Obliterated median sulcus

Size ≥2.5 cm × 3 cm

Symptoms improved with use of an alpha-1 receptor blockade such as tamsulosin.

Surgical intervention should be offered early in the disease process.

Ans: Surgical intervention should be offered early in the disease process.

Which of the following is least likely to be noted in the 55-year-old man who presents with bladder cancer?

Textile worker for the past 25 years

60 pack-year cigarette smoking history

Report of intermittent painless gross hematuria

Report of recent-onset intermittent acute urinary retention

Ans:

Report of recent-onset intermittent acute urinary retention

Which of the following is a worrisome finding noted during pelvic examination on a 62-year-old woman?

Flattening of the vaginal rugae

Vaginal pH=5.6

Ovary palpable on bimanual examination

Scant white vaginal discharge

Ans: Ovary palpable on bimanual examination

A 84-year-old woman recently started a medication to help with overactive bladder symptoms. Since taking the medication, she has developed dry mouth and worsening constipation.

Oxybutynin (Ditropan ®).

Tamsulosin (Flomax®).

Mirabegron (Myrbetriq®).

Finasteride (Proscar®).

Ans: Oxybutynin (Ditropan ®).

Match the following urinary incontinence types:

A. Urge incontinence
B. Stress incontinence
C. Functional incontinence
D. Transient incontinence

1. Associated with lifting __
2. Occurs during an acute illness __
3. Reports of strong sensation of needing to void __
4. Often occurs in presence of mobility problems __

1-D 2-A 3-B 4-C

1-B 2-A 3-D 4-C

1-C 2-D 3-B 4-A

1-B 2-D 3-A 4-C

ans:

1-B (stress incontinence/associated with lifting) 2-D (transient incontinence occurs during in an acute illness) 3-A(urge incontinence/reports strong sensation of needing to void) 4-C (functional incontinence/often occurs in presence of mobility problems)

Which of the following patients should have screening for HPV-associated cancer?

An annual Pap test for cervical cancer screening in a 21-year-old woman

A cervical Pap test every 3 years for a 40-year-old woman who has undergone a total hysterectomy for uterine fibroids

An anal Pap in a 45-year-old man with a history of anal and genital warts

An anal Pap in a 21-year-old woman who has received HPV-9 vaccine

Ans: An anal Pap in a 45-year-old man with a history of anal and genital warts

Samantha, a healthy 32-year-old woman who is taking combined oral contraceptives, is here to review the results of her recent liquid-based Pap screening that revealed atypical squamous cells of unknown significance (ASCUS) and high-risk HPV positive.
She has no history of previous abnormal cervical cytology, with her last screening obtained approximately 2 years ago. After explaining the significance of these findings, the most appropriate next step is to:

Advise that she return in 6-12 months for a repeat Pap with HPV cotesting.

Obtain screening tests for N. gonorrhoeae and C. trachomatis infection.

Referral for colposcopy.

Counsel that the usual cervical cancer screening guidelines should be followed.

Ans: Referral for colposcopy.

Which of the following is demonstrated to provide the most symptom relief in treating vasomotor symptoms?

Clonidine

Paroxetine

Conjugated estrogen

Venlafaxine

Ans: conjugated estrogen

Match the following cancer types:

A. Cervical cancer
B. Endometrial cancer
C. Ovarian cancer
D. Cervical cancer, Endometrial cancer
E. Endometrial cancer, Ovarian cancer
F. All

1. Risk factors include obesity and personal history of PCOS __
2. Risk factors include long-term infection with HPV 16 and/or 18 __
3. Risk factors include BRCA1 and BRCA2 gene mutation __
4. Clinical presentation includes unexplained abdominal bloating and constipation __
5. Clinical presentation includes abnormal vaginal bleeding __
6. Often without significant clinical signs and symptoms until later disease __
7. Screening includes Pap test __
8. No specific screening tests available or recommended __

1-D 2-F 3-A 4-C 5-E 6-B 7-A 8-C

1-B 2-A 3-C 4-C 5-D 6-F 7-A 8-E

1-E 2-F 3-A 4-C 5-E 6-B 7-A 8-C

1-A 2-E 3-B 4-D 5-B 6-A 7-F 8-D

Ans: 1-B ( PCOS and obesity increases risks of endometrial cancer )
2-A risks of longterm infection with HPV 16/18 increases risks of cervical cancer
3-C (BRCA1 and BRCA2 gene mutation increase risks of ovarian cancer)
4-C (ovarian cancer has clinical presentation include unexplained abdominal bloating and constipation)
5-D (cervical and endometrial cancers often have abnormal vaginal bleeding)
6-F ( ovarian, endometrial and cervical are often without significant clinical s/s until later disease
7-A Screening for cervical cancer recommends only
8-E (No recommended screening for endometrial and ovarian at present time)

You see a 24-year-old male with small flesh-colored verruca-form lesions on the shaft and tip of the penis. He reports some itching and discomfort in the genital area, but no voiding symptoms. The most likely diagnosis for this patient is:

Syphilis

Trichomoniasis

Gonorrhea

Genital warts

ans: genital warts

Match the following:

A. Hydrocele
B. Varicocele
C. Testicular torsion
D. Phimosis
E. Paraphimosis
F. Cryptorchidism

1. Retracted foreskin that cannot be brought forward to cover the glans __
2. A palpable “nest of worms” scrotal mass that is only evident in standing position __
3. Collection of serous fluid that causes painless scrotal swelling, easily recognized by transillumination __
4. With this, the foreskin cannot be pulled back to expose the glans __
5. Characterized by scrotal pain and loss of the cremasteric reflex __
6. Testicle located in inguinal canal or abdomen __

1-B 2-C 3-A 4-D 5-E 6-F

1-D 2-A 3-B 4-C 5-F 6-E

1-E 2-B 3-A 4-D 5-C 6-F

1-A 2-F 3-E 4-C 5-D 6-B

Ans: 1-E (paraphimosis) 2-B (varicocele) 3- (hydrocele) A 4- phimosis D 5- testicular torsion C 6- cryptochidism

Match each type of urinary incontinence with its most appropriate management option [Each condition can have multiple answers].

A. Availability of an assistant to help with toileting activities
B. Identify and treat the underlying condition
C. Antimuscarinic therapy
D. Pelvic floor exercises
E. Antimuscarinic therapy or pelvic floor exercises

1. Urge incontinence __
2. Stress incontinence __
3. Functional incontinence __
4. Transient incontinence __

1-B 2-C 3-D 4-A

1-A 2-D 3-C 4-B

1-E 2-D 3-A 4-B

1-E 2-A 3-C 4-B

Ans: 1-E urge incontinence (antimuscarinic therapy or pelvic floor exercises)
2-D stress incontinence – pelvic floor exercises (Kegel)
3- A functional incontinence – availabilty of an assistant to help with toileting activities
4-B transient incontinence -usually associated with acute illness/ identify and treat underlying condition.

A 32-year-old woman is diagnosed with a urinary tract infection. She reports never having a UTI previously. Her records indicate that she is allergic to sulfa medications. You recommend treatment with:

Amoxicillin (Amoxil®)

Nitrofurantoin (Macrobid®)

Moxifloxacin (Avelox®)

Ceftriaxone (Rocephin®)

ans: Nitrofurantoin (Macrobid)

Treatment for a 45-year-old woman with pyelonephritis who is otherwise healthy and has no sulfa allergy is:

Nitrofurantoin (Macrobid®)

Amoxicillin (Amoxil®)

Levofloxacin (Levaquin®)

Fosfomycin (Monurol®)

ans: Levofloxacin (Levaquin)

You see a 28-year-old woman with HIV and being treated with antiretroviral therapy. At this visit, she mentions that she is considering pregnancy. In counseling the patient, you mention that antiretroviral therapy:

Should be discontinued prior to attempting pregnancy

Should be discontinued once pregnancy is confirmed

Will likely continue throughout the pregnancy

Should be discontinued only during the first trimester of pregnancy

ans: Will likely continue throughout the pregnancy

For the patient described above, the most appropriate treatment would be:

Based on the below scenario:
You see a 24-year-old male with small flesh-colored verruca-form lesions on the shaft and tip of the penis. He reports some itching and discomfort in the genital area, but no voiding symptoms.

IM ceftriaxone (Rocephin®)

Topical imiquimod (Aldara®)

Oral doxycycline (Doryx®)

Topical acyclovir (Zovirax®)

ans: topical imiquimod cream

In evaluating an 18-year-old man with testicular torsion, the NP considers the most appropriate course of action is:

Watch and wait with re-evaluation in 2 days as it may resolve spontaneously

Attempt manual detorsion

Analgesics plus an anti-inflammatory agent

Prompt referral to an urologist for surgical evaluation

ans: Prompt referral to an urologist for surgical evaluation

Match the following. An item can be used more than once.

A. Paroxetine
B. Fluoxetine
C. Citalopram
D. Escitalopram

1. Most energizing SSRI __
2. Most sedating SSRI __
3. Dose limitation due to potential QT prolongation __
4. Least drug interaction potential __
5. Longest T ½ __

1-B 2-A 3-C 4-D 5-B

1-D 2-B 3-C 4-A 5-B

1-B 2-A 3-D 4-A 5-C

1-A 2-C 3-B 4-D 5-A

Ans: 1-B (Flouxetine- most energizing) 2-A (Paroxitine- most sedating) 3-C Citalopram (increase QT prolongation) 4-D (Escitalopram- least interactions with other meds) 5-B (Flouxetine- longest half-life 84 hours)

Which of the following is likely to cause cardiac dysrhythmia and seizures when taken in an intentional ingestion equivalent to a typical adult therapeutic dose?


A 4-week supply of fluoxetine (SSRI)

A 2-week supply of nortriptyline (TCA)

A 3-week supply of venlafaxine (SNRI)

A 3-day supply of diazepam (BENZO)

Ans: A 2-week supply of nortriptyline

Of the following in need of an antidepressant, who is the best candidate for fluoxetine (Prozac®) therapy?

An 80-year-old woman who is taking multiple medications and who presents with depressed mood and agitation

A 45-year-old man with anorgasmia who is an occasional marijuana user

A 28-year-old woman who occasionally “skips a dose” of her prescribed medication and is using a progestin implant (Nexplanon®) for contraception

A 44-year-old woman with decreased appetite who is on hydrochlorothiazide for the treatment of hypertension

Ans: A 28-year-old woman who occasionally “skips a dose” of her prescribed medication and is using a progestin implant (Nexplanon®) for contraception

ou see a 45-year-old woman with major depressive disorder (MDD) who started taking standard dose sertraline one week ago. She returns today with a chief complaint of “not really feeling any better.” “In fact, I think I might feel worse. I have this on-and-off headache right over my eyes since I started the medication.” Results of physical examination reveal the following: Well-groomed and appropriately dressed for the occasion, PERLA, fundi WNL, CN 2-12 intact, clear, fluid speech. Today, you provide the following advice, choosing two of the following responses.


The sertraline dose is likely inadequate and needs to be increased.

She should be switched to a SNRI such as venlafaxine.

Due to her headache report, the sertraline should be immediately discontinued.

A lag of a number of weeks in the onset of SSRI therapeutic effect is expected.

Frontal headache is a common short-term problem with early SSRI use.

Ans: A lag of a number of weeks in the onset of SSRI therapeutic effect is expected.
Frontal headache is a common short-term problem with early SSRI use.

Ms. Yancy is a 56-year-old woman with a history of depression and has been taking citalopram for about 6 months. Four days ago, she decided to discontinue the citalopram as she states her depressive symptoms have been much improved.
Today, she presents with “not feeling well,” with a 2-day history of persistent frontal headache. When assessing Ms. Yancy for additional findings of antidepressant discontinuation syndrome, the NP anticipates that Ms.Yancy will report all of the following except:

Nausea.

Fever.

Flu-like symptoms.

Anxiety.

Ans: Fever.

Which of the following two tests should be periodically checked while on olanzapine (Zyprexa®)?

BUN and creatinine

Creatine kinase and potassium

Complete blood count and platelet count

Blood sugar and lipid profile

Ans: Blood sugar and lipid profile

Which of the following provides the most accurate information on assessing for alcohol abuse in a 55-year-old man who states, “I drink 5‒6 beers every night but I get to work every day.”

Elevated ALT/AST to ≥6 times upper limit of normal

Positive response to two items on the CAGE questionnaire

Modest RBC macrocytosis

Elevated serum triglycerides

Ans: Positive response to two items on the CAGE questionnaire

The use of opioid analgesia is most appropriate in which of the following clinical situations?

A 40-year-old man with acute low back pain × 2 days

A 28-year-old woman with a 24-hour history of migraine headache

A 48-year-old woman who is 1 day postoperative after abdominal hysterectomy

A 54-year-old woman with 5-year history of fibromyalgia

Ans: A 48-year-old woman who is 1 day postoperative after abdominal hysterectomy

Select components of substance-abuse disease include all of the following except:

Craving or a strong desire to use the problematic substance.

Over time, diminished effect with the same amount of the substance.

Set of characteristic withdrawal symptoms.

Social or occupational function remains intact.

Ans: Social or occupational function remains intact.

You see a 26-year-old college student diagnosed with major depressive disorder. He has been treated with a SSRI for the past 6 months with minimal clinical effect. You recommend as the next course of action:

Adding a benzodiazepine.

Adding a second-generation antipsychotic (SGA).

Switching to a SNRI.

Switching to a tricyclic antidepressant (TCA).

Ans: Switching to a SNRI.

A 56-year-old woman who is being treated for COPD, hypertension, and dyslipidemia is recently diagnosed with moderate depression and requires antidepressant therapy. Which of the following agents is the least favorable choice due to concerns about potential drug-drug interactions?

Citalopram (Celexa®)

Escitalopram (Lexapro®)

Bupropion (Wellbutrin®)

Fluoxetine (Prozac®)

Ans: Fluoxetine (Prozac®)

Treatment with a second-generation antipsychotic (SGA) is being initiated for a 37-year-old woman with schizophrenia who is otherwise healthy. Following initiation of the SGA, the NP understands that regular monitoring should be performed for which of the following?

AST/ALT

Fasting blood glucose

Hct

TSH

Ans: Fasting blood glucose due to weight gain

According to the recommendations found in the Beers Criteria, the use of certain medications and the other members of its class should be avoided in the older adult. Match the medication with the rationale for avoiding or using with caution in the elder.

A. Zolpidem (Ambien®)
B. Amitriptyline (Elavil®)
C. Naproxen sodium (Aleve®, Anaprox®)
D. Sertraline (Zoloft®)

1. Significant risk of orthostatic hypotension __
2. Increase in fall and fracture risk __
3. Potential to promote fluid retention and minimize effect of many anti-HTN medications __
4. Increased risk for hyponatremia, especially when used with diuretic __

1-C 2-B 3-A 4-D

1-B 2-D 3-C 4-A

1-B 2-A 3-C 4-D

1-D 2-C 3-B 4-A

Ans: 1-B (Amtriptiline) 2-A (Ambien) 3-C (Naproxen sodium) 4-D (Sertraline-Zoloft)

Match the patient presentations with the following terms:

A. Syncope
B. Dizziness
C. Vertigo

1.”All of a sudden, I passed out and woke up on the floor.” __
2. “I feel lightheaded.” __
3. “The room is spinning.” __

1-B 2-C 3-A

1-A 2-B 3-C

1-C 2-B 3-A

1-A 2-C 3-B

Ans: 1-A (Sycope) 2-B (Dizziness) 3-C (the room is spinning)

Mrs. Little is a 78-year-old woman with recently-diagnosed Alzheimer-type dementia (AD) who is here today for an office visit with her 55-year-old daughter. According to her daughter, Mrs. Little struggles with word-finding and has difficulty following directions. She appears “not to care about what is going on around her,” while other times is engaged in family activities. According to her daughter, Mrs. Little will have an angry verbal outburst that is triggered by a minor problem. Her daughter states, “This is not like my mother. Usually she is very patient.” When evaluating Mrs. Little, the NP considers that irritability in a person with early-stage dementia is often indicative of a:


Mood disorder.

Thought disorder.

Normal pressure hydrocephalus.

Hyperparathyroidism.

ans: Mood disorder.

Mrs. Little currently resides with her daughter’s family and they voice a desire for this to continue “as long as possible and safe.” When evaluating Mrs. Little’s healthcare needs, choose the two options that do not apply.

Based on the below scenario:
Mrs. Little is a 78-year-old woman with recently-diagnosed Alzheimer-type dementia (AD) who is here today for an office visit with her 55-year-old daughter. According to her daughter, Mrs. Little struggles with word-finding and has difficulty following directions. She appears “not to care about what is going on around her,” while other times is engaged in family activities. According to her daughter, Mrs. Little will have an angry verbal outburst that is triggered by a minor problem. Her daughter states, “This is not like my mother. Usually she is very patient.”


A home safety evaluation should be conducted and appropriate modifications carried out.

If Mrs. Little has a sudden change in mental status, her healthcare provider should be contacted as soon as possible.

Behavioral difficulties often arise in individuals with AD if their usual routine is disrupted.

The use of a cholinesterase inhibitor will likely improve her mental status to a point that is nearly equivalent to her predementia baseline.

The most common adverse effects from cholinesterase inhibitor use include nausea and diarrhea.

A second-generation antipsychotic such as risperidone should be started to help minimize the angry outburst risk.

Ans: The use of a cholinesterase inhibitor will likely improve her mental status to a point that is nearly equivalent to her predementia baseline.



A second-generation antipsychotic such as risperidone should be started to help minimize the angry outburst risk.

Ms. Hopkins is a 78-year-old woman with a history of hypertension, dyslipidemia, and stable angina pectoris who presents for follow-up care. She is a former smoker who quit 25 years ago with a 40 pack-year history. She denies chest pain or shortness of breath. Her current medications include a beta-adrenergic antagonist, sustained-release nitrate, angiotensin-converting enzyme inhibitor, statin, low-dose aspirin, and a thiazide diuretic. She complains of feeling a “catch or cramp” in her lower posterior legs bilaterally when she walks for extended periods. This is promptly relieved by rest. Which of the following do you anticipate finding on examination of her lower extremities?


Hyperpigmentation with bilateral ankle edema

Diminished bilateral pedal pulses with thinning of the skin

Extensive dry skin with evidence of lichenification on the plantar aspects of both feet

Diminished sensory perceptions and abnormal monofilament examination

Ans: Diminished bilateral pedal pulses with thinning of the skin

An appropriate diagnostic test to perform next for Ms. Hopkins is:


Based on the below scenario:
Ms. Hopkins is a 78-year-old woman with a history of hypertension, dyslipidemia, and stable angina pectoris who presents for follow-up care. She is a former smoker who quit 25 years ago with a 40 pack-year history. She denies chest pain or shortness of breath. Her current medications include a beta-adrenergic antagonist, sustained-release nitrate, angiotensin-converting enzyme inhibitor, statin, low-dose aspirin, and a thiazide diuretic. She complains of feeling a “catch or cramp” in her lower posterior legs bilaterally when she walks for extended periods. This is promptly relieved by rest.

Venography of lower legs.

Ankle-brachial index (ABI).

ECG.

Serum B-type natriuretic peptide (BNP).

Ans:

Ankle-brachial index (ABI).

In peripheral arterial disease, all of the following are recommended except:

Referral to vascular surgery.

Sclerotherapy.

Oral cilostazol (Plental®) to reduce symptoms.

Management of cardiovascular risk factors

Ans: Sclerotherapy.

Match the diagnosis with its most appropriate treatment option.

A. Tricyclic antidepressant
B. Cilostazol (Pletal®)
C. Compression stockings

1. Peripheral artery disease __
2. Venous insufficiency __
3. Peripheral neuropathy __

1-B 2-C 3-A

1-A 2-C 3-B

1-B 2-A 3-C

1-C 2-B 3-A

Ans: 1-B (Cilostazol) 2-C (compression stockings) 3-A (TCA-Neuropathy)

Indicate whether each event is likely due to normal age-related mental changes (N) or is a possible warning sign of Alzheimer-type dementia (W).

1. Occasionally adding the wrong amount of an ingredient when following a recipe. __
2. Getting lost while driving to usual house of worship. __
3. Placing the house keys in the freezer. __
4. Forgetting a son’s birthday until late in the day. __
5. Incorrectly identifies the current US President. __
6. Cannot immediately remember what they had for dinner the night before but remembers this later. __

1-N 2-N 3-W 4-N 5-W 6-W

1-W 2-W 3-N 4-N 5-W 6-W

1-N 2-W 3-W 4-N 5-W 6-N

1-N 2-W 3-N 4-N 5-W 6-W

Ans: 1-N (Age-related change- occasionally adding wrong ingredient…)
2-W ( Dementia related – getting lost while driving to familiar house…)
3-W (placing the house keys in the freezer)
4-N (forgetting son’s birthday until later in the day)
5-W (incorrectly identifies the current US president)
6-N (cannot remember what they had for dinner last night but remembers it now)

Which of the following do you not expect to find in the examination of a full-term healthy newborn?

Holding the baby about 16-20 inches (41-51 cm) away from the caregiver’s face takes advantage of the newborn’s visual range.

Bluish scleral tint is noted regardless of ethnicity or eye color.

The newborn’s eyes are quite light and glare sensitive.

If an object goes towards the newborn’s eye, the baby will likely react with a defensive blink reflex.

Ans: Holding the baby about 16-20 inches (41-51 cm) away from the caregiver’s face takes advantage of the newborn’s visual range.

During a well neonate visit for Christopher, a healthy 2-week-old boy born at 41 weeks’ gestation, you anticipate that the baby will have:

A visual preference for the human face.

Hear low-pitched voices best.

Will not react to the cry of other neonates.

A poorly-developed sense of smell.

Ans: A visual preference for the human face.

A full-term pregnancy is one where the birth occurs between:

37 weeks and 38 weeks of gestation.

37 weeks and 38 weeks plus 6 days of gestation.

39 weeks and 40 weeks plus 6 days of gestation.

41 weeks and 41 weeks plus 6 days of gestation.

Ans: 39 weeks and 40 weeks plus 6 days of gestation.

When counseling the parents of Joshua, a healthy term newborn, about sleeping safety, the NP advises the following:

Position the baby on the side with a positioning wedge or back, as he is most comfortable.

Place the baby in a face-up position for sleep.

Place a soft bumper in the crib to minimize the risk of injury.

Even when a caregiver is in attendance, the tummy-down position is not safe in the first two months of life

Ans:

Chapter 16-A- Review of Questions- Primary Care of the Well and Sick Infant, Child, and Teen

n considering jaundice in the neonate, the NP considers that:

Neonatal jaundice usually starts first in the trunk and extremities and then progresses to the face.

Using visual evaluation of bilirubin levels to estimate the degree of jaundice is an acceptable way to evaluate the newborn.

The onset of physiologic jaundice is usually within the first 12 hours of life.

Encouraging feedings at minimum every 2?3 hours at the breast per day while avoiding dextrose and water feedings will help minimize the newborn’s risk of hyperbilirubinemia.

Ans: Encouraging feedings at minimum every 2?3 hours at the breast per day while avoiding dextrose and water feedings will help minimize the newborn’s risk of hyperbilirubinemia.

You are providing counseling for the parents and caregivers of a healthy full-term newborn who is being breastfed. The counseling should include information on all of the following except:


The baby should make at least 6 wet diapers a day.

Newborns often lose up to 10% of birth weight in the first week of life.

A breastfed baby usually has 4 or more bowel movements per day.

The baby should be back up to birth weight by age 3-4 weeks.

Ans: The baby should be back up to birth weight by age 3-4 weeks.

You see Alexandra, born at 40.5 weeks’ gestation, who is now 14 days old. According to her mother, she is a vigorous eater and is both breast- and formula-fed. On examination of the neonate, you note bilateral breast engorgement with physiologic galactorrhea on the left. You appreciate all of the following are correct concerning this condition except:


Its onset is usually at approximately day 3-4 of life.

That maternal hormonal influences are likely the cause.

This breast engorgement will resolve without intervention within the first two months of life.

Further evaluation is required to confirm this assessment.

Further evaluation is required to confirm this assessment.

A 12-day-old infant who is otherwise well presents with a 2-day history of irritation of both eyes. He was born at a local birth center and received standard newborn care including ocular chemoprophylaxis. Examination reveals bilateral lid swelling, chemosis, and mucoid eye discharge. The most likely cause of this condition is:

Chemical irritation from neonatal ocular chemoprophylaxis.

Chlamydial (inclusion) conjunctivitis.

Gonococcal conjunctivitis.

Neonatal adenovirus infection.

Ans: Chlamydial (inclusion) conjunctivitis.

You are rounding in the nursery and see the neonate of a mother who is HBsAg-positive. Your most appropriate action is to:

Check the baby for HBsAb.

Inform the mother that she should not breastfeed.

Administer hepatitis B immunization to mother and infant.

Give hepatitis B immunization and hepatitis B immune globulin to the newborn.

Ans: Give hepatitis B immunization and hepatitis B immune globulin to the newborn.

Indicate the appropriate neonatal/-infant reflex for each description.

A. Moro reflex
B. Palmar grasp
C. Babinski reflex
D. Stepping reflex
E. Rooting reflex

1. Walking motion made with legs and feet when held upright and feet touching the ground. Appears for first 3-4 months, then reappears at 12-24 months. __
2. Turning of head and sucking when cheek is stroked. No longer seen by 6-12 months. __
3. Throwing out arms and legs followed by pulling them back to the body following a sudden movement or loud noise. No longer seen by 16 weeks. __
4. Stroking the sole of the foot elicits fanning of the toes. No longer seen by 6 months. __

1-D 2-E 3-A 4-C

1-A 2-B 3-D 4-E

1-D 2-E 3-B 4-A

1-C 2-D 3-B 4-A

Ans: 1-D (stepping reflex) 2-E (rooting reflex) 3-A (moro reflex) 4-C (babinski reflex)

Which of the following is the most important time to screen for hearing defects?

In the first days of life

During the time of most intense speech formation

Before the child enters school

Once antimicrobial therapy for AOM is completed

Ans: in the first days of life

n teaching a family about “time out,” this method is appropriate to introduce the concept at about the age of _____________.

12-18 months.

18-24 months.

24-30 months.

30-36 months.

Ans: 18-24 months.

The child remains in “time out” for what period of time?

30 seconds for each year of life

1 minute for each year of life

2 minutes for each year of life

3 minutes for each year of life

Ans: 1 minute for each year of life

In a healthy 3½-year-old, what percentage of speech should be intelligible by people who are not in daily contact with the child?

About 25%

About 50%

About 75%

Nearly 100%

Ans: Nearly 100%

You see a well child who resists being placed in a supine position on the exam table, cries loudly when her parent is out of view, and has visibly erupting lower central incisors. This child is approximately what age?


6-8 weeks

3-4 months

7-8 months

2-3 years

Ans: 7-8 months

As part of pediatric primary care, parents and caregivers should be counseled to schedule the child’s first dental visit:

When the child has erupting molars.

With eruption of the first tooth.

At age 3 years.

When the child has been weaned from breast or bottle.

Ans: With eruption of the first tooth.

Which of the following is most consistent with a normal developmental exam for a thriving 6-month-old infant born at 32 weeks’ gestation?


Responds to own name and sits without support

Reaches for toy with one hand and recognizes familiar people and objects at a distance

Babbles mamama, bababa and transfers objects hand-to-hand without difficulty

Vocalizes “ah” and “oh” sounds, and is able to lift head briefly when positioned on the tummy and turn it from side-to-side

Ans: Reaches for toy with one hand and recognizes familiar people and objects at a distance

Thomas is a neonate who was born at 32 weeks’ gestation. The adjusted age calculation should be used to assess his development until age:


6 months.

12 months.

18 months.

24 months.

Ans: 24 months.

You examine a healthy 2-month-old boy and note that his foreskin cannot be retracted. You consider that:

The foreskin should be forcibly retracted to facilitate cleaning.

In most instances, the foreskin is not easily retractable until the child is about 3 years old.

The risk for hypospadias or epispadias is increased in the presence of this finding.

Persistent maternal hormonal influences contribute to this problem.

Ans: n most instances, the foreskin is not easily retractable until the child is about 3 years old.

You examine a thriving 4-week-old boy who was born at 39 weeks’ gestation and note a painless, tense, non-reducible, relatively symmetric scrotal enlargement that brightly and evenly transilluminates. The parents report that the scrotum “always looks like this,” without change in size during the course of the day. Bilateral testes, approximately 1 cm in length, are palpable and held within the scrotum. The penis is approximately 4 cm in length. You consider these findings are most consistent with:


A normal examination.

Bilateral inguinal hernias.

Micropenis.

Noncommunicating hydrocele.

Ans: Noncommunicating hydrocele.

Which of the following is most consistent with pyloric stenosis (upper GI obstruction [PS]) or intussusception (lower GI obstruction [I]) or both?

A. Pyloric stenosis
B. Intussusception
C. Both

1. Significantly more common in males. __
2. Sudden onset, colicky, severe, and intermittent abdominal pain, often with knees drawn to chest during most intense discomfort. __
3. Accompanied by loose stools that are often described as currant jelly appearance (mixture of blood and sloughed mucous). __
4. Most common time for symptom onset is= Approximately age 3 weeks. __
5. Post-fed projectile vomiting is present, with the baby eager to eat again immediately post emesis. __
6. Accompanied by a sausage-shaped abdominal mass. __
7. Olive-shaped RUQ abdominal mass occasionally noted. __
8. Usually occurs between ages 6-12 months. __
9. Ultrasonography is usually first-line diagnostic study. __

1-A 2-B 3-A 4-A 5-B 6-C 7-C 8-B 9-A

1-C 2-B 3-B 4-A 5-A 6-B 7-A 8-B 9-C

1-C 2-A 3-B 4-A 5-B 6-B 7-A 8-B 9-A

1-B 2-B 3-C 4-C 5-C 6-A 7-B 8-A 9-B

Ans: 1-C (both) 2-B (Intussusception) 3-B (Intussusception) 4-A (pyloric stenosis) 5-A (pyloric stenosis) 6-B (intussusception) 7-A (pyloric stenosis) 8-B (intussusception) 9-C (both)

Pediatric immunizations: True or false?

1. MMR should not be given to a 12-month-old whose mother is pregnant. __
2. A 6-month-old who is taking amoxicillin for acute otitis media (AOM) should have immunizations delayed until the antimicrobial course is completed. __
3. Preterm infants are usually immunized at the schedule that corresponds with their birth or extrauterine age. __
4. One of the best ways to protect infants younger than 6 months of age from influenza is to make sure members of their household and their caregivers are vaccinated against the disease. __
5. Pregnant women should receive inactivated influenza vaccine to protect both mother and unborn child. __
6. The risk of autism can be reduced through the use of an early childhood vaccination schedule that minimizes the number of immunizations given at a single visit. __
7. In order to avoid post-vaccine discomfort, younger children should be given a weight- and age-appropriate dose of an antipyretic, such as acetaminophen or ibuprofen, prior to receiving immunizations. __
8. Children 6-11 months of age who are traveling outside the United States should receive 1 dose of MMR. __

1-True 2-True 3-False 4-True 5-False 6-True 7-False 8-False

1-False 2-True 3-True 4-True 5-False 6-True 7-False 8-True

1-False 2-False 3-True 4-True 5-True 6-False 7-False 8-True

1-False 2-True 3-False 4-True 5-False 6-True 7-False 8-True

Ans: 1-False 2-False 3-True 4-True 5-True 6-False 7-False 8-True

Of the following, who should receive Tdap (tetanus, diphtheria, acellular pertussis) vaccine? (Yes or no)

1. A 28-year-old woman who is 29-weeks pregnant and received Tdap approximately 3 years ago __
2. The spouse of a woman in the 2nd trimester of pregnancy who provides documentation of receiving Tdap 2 years ago __
3. A 70-year-old man who received a Td about 8 years ago, slated to be one of his son’s newborn’s caregivers __

1-Yes 2-No 3-Yes

1-Yes 2-Yes 3-Yes

1-No 2-No 3-Yes

1-No 2-Yes 3-Yes

Ans: 1-Yes 2-No 3-Yes

Match each image with the correct diagnosis.

A. Metatarsus adductus
B. Club foot
C. Polydactyly
D. Syndactyly

Ans: Metatarsus adductus (forefoot turns inward, high arch and wide gap between big toes and second). Club foot (foot is turned inward, often with bottom of foot facing sideways). Polydactyly (presence of extra digits). Syndactyly (fusion of 2 or more digits or webbing of the skin between digits).

The American Academy of Pediatrics recommends screening for autism at which of the following times in early childhood?

6 and 12 months

12 and 18 months

18 and 24 months

24 and 30 months

Ans: 18 and 24 months

A 2½-year-old boy is brought by his mother who reports that, approximately 10 minutes ago, the child was injured when he pulled a pot of boiling water off the stove. The child is alert and crying, and age-appropriately resists the examination. Skin survey reveals approximately 18% body surface area of moist, red skin with peeling borders, largely involving the posterior thighs, buttocks, and scrotum. No other injury is noted. Identify the two most important considerations in his care.

The injury should be débrided as soon as possible.

The affected area should be promptly washed with an antiseptic solution.

Specialty burn care should be promptly sought.

Child protective services should be immediately notified.

A course of oral antibiotics should be initiated.

Ans: Specialty burn care should be promptly sought. Child protective services should be immediately notified.

You see Sharon for a well-child visit. She is a 12-year-old who is at Tanner stage 2-3 and states unhappily, “I am the shortest girl in my class.”When reviewing her growth chart, you notice she has been consistently between the 10th and 15th percentile for height and weight during her childhood. The rest of her examination is within normal limits. You advise that:

She should have an evaluation by a pediatric endocrinology specialist.

Her growth spurt will start soon.

Due to her age, she is likely near her adult height.

X-ray determination of bone age should be obtained.

Ans: Her growth spurt will start soon.

Physiologic gynecomastia is usually found in which of the following?

A 14-year-old male who is at Tanner stage 3

A 12-year-old male who is at Tanner stage 2

A 17-year-old male who is at Tanner stage 5

A 10-year-old male who is at Tanner stage 1

Ans: A 14-year-old male who is at Tanner stage 3

You see a Tanner stage 4 14-year-old male who you suspect has Fragile X syndrome because of the notation of all of the following except:

Macroorchidism.

Large body habitus.

History of learning differences.

Hip and breast enlargement.

Ans: Hip and breast enlargement.

Fragile X-syndrome: In males, large forehead, ears, prominent jaw, tendency to avoid eye contacts, large testicles (macrochidism), noted after beginning after puberty, large body habitus, behavioral and learning differences common.
In females, less prominent findings, occurs in all racial ethnic groups, common cause for autism in both genders.

Klinefelter syndrome (XXY) only in males, with specific physical habitus, low testicular volume, hip and breast enlargement, infertility most commonly is language development

Turner syndrome: (XO) : usually is short stature <5ft, evident by age 5, wide webbed neck broad shield shaped-chest absent menses, infertility, most noted often at birth with high-arched palate, low-set ears, edema of hands and feet,

A 12-year-old boy presents with his mother for a well-child visit. What is the most helpful approach to this visit?

Interview and examine the child in the absence of the mother.

Interview the child with the mother, asking her to leave for the examination.

Ask the child if he wishes his mother to be there for the interview and examination.

Ask the mother if she wishes to be included in the interview and examination

Ans: Ask the child if he wishes his mother to be there for the interview and examination.

Match the following.

A. Benzoyl peroxide
B. Isotretinoin (Accutane®)
C. Tretinoin (Retin-A®)
D. Combined oral contraceptive

1. Indicated for treatment of cystic acne __
2. Most cost-effective topical antibacterial in mild acne __
3. Used as a keratolytic in acne treatment __
4. Use results in reduction of androgen levels __

1-A 2-B 3-C 4-D

1-B 2-A 3-C 4-D

1-C 2-A 3-D 4-B

1-B 2-C 3-A 4-D

Ans: 1-B (Accutane) 2-A (Benzoyl peroxide) 3-C (Tretinoin Retin-A) 4-D(Combined oral contraceptives)

In the USA, which of the following is the most common cause of adolescent death?

Suicide

Homicide

Accidental injury

Malignancy

Ans: Accidental Injury

Adolescent issues: True or false?

1. Although adolescents tend to drink alcohol less frequently than adults, they drink considerably more alcohol per occasion of drinking. __
2. USPSTF recommends depression screening using a validated questionnaire in adolescents (ages 12-18 years). __
3. The majority of states require either parental consent or notification for teenagers younger than 18 to have a pregnancy termination. __

1-True 2-True 3-True

1-False 2-True 3-True

1-False 2-True 3-False

1-True 2-False 3-False

Ans: 1-True 2-True 3-True

James is a 15-year-old who arrives for a well-teen visit with his mother.Prior to the beginning of the visit, his mother pulls you aside and states, “I want him checked for all drugs, but he said he is not using anything and does not want to be tested.” Which of the following is your most appropriate response?


“What drugs do you think James is taking?”

“I cannot force James to take a drug test.”

“Let’s discuss your concerns with James.”

“Since you are concerned, I can order the test without James’ consent.”

Ans:

“Let’s discuss your concerns with James.”

The most common contraceptive method used by teens is:

Male condom.

Combined oral contraceptives.

DMPA injection (Depo-Provera®).

Withdrawal.

Ans: male condoms

In which of the following scenarios is parental consent for care required?

An 18-year-old female who is seeking a pregnancy termination

A 16-year-old female requesting a prescription for oral contraceptives

A 15-year-old male requesting testing for sexually transmitted infection

A 17-year-old male who requests treatment for contact dermatitis

Ans:

A 17-year-old male who requests treatment for contact dermatitis

True or false? When approaching the provision of primary care for LGBT youth, the NP considers that:

1. The healthcare provider should ask the adolescent how he/she self-identifies. __
2. Therapy which attempts to change one’s sexual orientation or gender identity is inconsistent with current standards of medical care. __
3. Because victimized LGBT youth are at increased risk of depression and suicidality, providers should screen for these mental health issues and intervene as appropriate. __
4. In adolescents, sexual orientation and gender identity are relatively fixed constructs. __

1-True 2-True 3-True 4-True

1-False 2-True 3-True 4-True

1-True 2-True 3-True 4-False

1-True 2-False 3-True 4-False

Ans: 1-True 2-True 3-True 4-False

You are seeing 17-year-old Cynthia. As part of the visit, you consider her risk factors for type 2 diabetes mellitus would likely include all of the following except:

Obesity.

Pacific Islander ancestry.

Family history of type 1 diabetes mellitus.

Personal history of polycystic ovary syndrome (PCOS).

Ans: Family history of type 1 diabetes mellitus.

Sam is a 15-year-old with a BMI=40 kg/m2 who presents with a lipid profile that reveals low HDL, elevated triglycerides, and an acceptable A1C.Which of the following is the recommended treatment option?


Oral niacin

Oral fibrate therapy

Weight loss

Oral statin therapy

Ans: Weight loss

Tina is an otherwise well 15-year-old who presents with her mother. They report that Tina has had a one-day history of “sore throat and swollen glands” as well as a low-grade fever and rash. Examination reveals a diffuse maculopapular rash, mildly tender posterior cervical and postauricular lymphadenopathy, and pharyngeal erythema without exudate. The remainder of her history and review of systems is unremarkable. Per her mother’s report, Tina has not received any immunizations since age 6 months.
The most likely diagnosis is:

Scarlet fever.

Roseola.

Rubella.

Rubeola.

Ans: Rubella.

Jannetta is a 16-year-old who presents with a 3-day history of pharyngitis and fatigue. Findings include exudative pharyngitis, minimally tender anterior and posterior cervical lymphadenopathy, and right and left upper quadrant abdominal tenderness. Per Jannetta’s record, she is up-to-date with all recommended vaccinations.
This is most consistent with:

S. pyogenes pharyngitis.

Infectious mononucleosis.

Hodgkin disease.

Gonococcal pharyngitis.

Ans: Infectious mononucleosis.

Jared is a 17-year-old with no known medication allergy who has suspected infectious mononucleosis. He is febrile and complains of acute otalgia on the left for the past three days. Physical examination reveals a left tympanic membrane that is red and bulging. When considering therapy for Jared, which of the following should not be prescribed?


Acetaminophen

Ibuprofen

Amoxicillin

Azithromycin

Ans: Amoxicillin (secondary reaction due to Mononucleosis infection)

Timmy is a 4-year-old boy who presents with his Mom today for a sick visit. For the past 8 days, he has had intermittent fever as high as 104.5ºF (40.3ºC) and has complained of a sore throat and increased throat pain with swallowing, but without difficulty taking fluids. He has little appetite, but his mother denies nausea, vomiting, diarrhea, or constipation. On examination, you note he is alert, appears ill without acute distress, and has extensive cervical lymphadenopathy, injected conjunctivaoral erythema, and a peeling rash on his hands. You consider a diagnosis of:

Infectious mononucleosis.

Fifth’s disease.

Hand, foot, and mouth disease.

Kawasaki disease.

Ans: Kawasaki disease.

Identify the two children that are at greatest risk for iron deficiency anemia.

3-month-old who takes about 24 oz (0.71 L) of iron-fortified formula per day

11-month-old, breastfed about 7 times a day, taking iron-fortified cereal, fruit, and vegetables three times per day

16-month-old who drinks about 1 qt (0.95 L) whole milk per day and a few solids

4-year-old who eats small amounts of meat, fruit, and cheese

6-month-old who was born at 30 weeks’ gestation and is exclusively breastfed without additional supplements

Ans: 16-month-old who drinks about 1 qt (0.95 L) whole milk per day and a few solids

6-month-old who was born at 30 weeks’ gestation and is exclusively breastfed without additional supplements

Jackson is a 13-month-old who is in for a well-child visit. Height and weight are at approximately 40th percentile and he is on target developmentally. His diet consists of approximately 18 oz (0.53 L) of whole cow’s milk per day and a variety of vegetables, fruits, lean meats, and grains. You advise that Jackson:

Is eating a well-balanced diet and no nutritional supplements are needed.

Should receive iron supplement equivalent to 1 mg/kg/d.

Receive vitamin D 400 IU as a daily oral supplement.

Should be taking in more calcium via increased cow’s milk intake to ≥28 fl oz (0.83 L) per day.

Ans: Receive vitamin D 400 IU as a daily oral supplement.

You are seeing an 8-year-old healthy boy who is brought in by his mother who states, “He is in trouble in school and failing two classes. His teacher thinks he is too hyper and will not sit still and wants him to be evaluated. I do not understand this. He is fine at home.”
When considering the diagnosis of attention deficit/hyperactivity disorder in this child, you understand that:

In the majority of children diagnosed with ADHD, the symptoms and behaviors will resolve by early adulthood.

Psychostimulant use in the child with ADHD can lead to substance abuse disorders in adolescence.

To confirm the diagnosis of ADHD, assessment of the child’s behavior at home and school should be conducted using standardized scales.

When psychostimulants are prescribed with the diagnosis of ADHD, concomitant behavioral therapy is usually not needed.

Ans: To confirm the diagnosis of ADHD, assessment of the child’s behavior at home and school should be conducted using standardized scales.

Adam, a 7-year-old boy with a prior diagnosis of moderate persistent asthma, presents with his parents for a well-child visit. He is new to your practice and has not had a healthcare visit in the past year. Adam’s mom mentions that he took a “pill to control his breathing, but we ran out.” “Right now, he uses the albuterol pump once or twice a day. This keeps his cough under pretty good control. The inhaler works quickly most of the time.” You advise the following:

Add a twice-a-day long-acting beta2-agonist as needed to ensure Adam has better cough control.

An inhaled corticosteroid should be added to Adam’s treatment regimen.

A leukotriene modifier is an acceptable first-line controller medication for Adam.

No additional medication is needed as Adam has adequate symptom control with the current albuterol dose and frequency.

Ans: An inhaled corticosteroid should be added to Adam’s treatment regimen.

dentify the following children with AOM as a candidate for initial antimicrobial therapy or watchful waiting therapy.

A. Antimicrobial therapy
B. Watchful waiting therapy

1. A 5-month-old with unilateral AOM, T=101.2°F (38.4°C),. and 48 -h history of crankiness __
2. A 3-year-old with bilateral AOM and, T=102.4°F (39.1°C), and a 8- h history of otalgia complaint __
3. A 4-year-old with unilateral AOM, T= 100.6°F (38.1°C), and a 12 -h history of mild ear discomfort __
4. A 2 ½-year-old with bilateral AOM, T=101.2°F (38.4°C), and a 3-day history of otalgia complaint __

1-A 2-B 3-A 4-B

1-B 2-A 3-B 4-B

1-A 2-A 3-B 4-A

1-B 2-A 3-A 4-A

Ans: 1-A 2-A 3-B 4-A

Taylor is a 2½-year-old otherwise well child who presents today for a sick visit. According to his father, Taylor has vomited approximately 10 times in the past 18 hours, with the last episode about 4 hours ago. He has retained sips of water during the last hour. He also developed watery stools approximately 8 hours ago with 4 episodes in total. The last stooling episode was about 1 hour ago. Fever is reported to be elevated to 38.4ºC (101.2ºF) on a number of occasions during the past day. On assessment, he is lying in his father’s arms, is alert and nods his head when asked if he is thirsty. His lips are dry and slightly cracked and there is a small amount of saliva in the oral cavity. His heart rate is 110 BPM and capillary refill is <2 seconds. Dad tells you Taylor’s diaper was dry when he awoke 4 hours ago but due to watery stools, he is unsure if Taylor has voided this morning. Currently, Taylor is wearing a diaper that is slightly wet in the front. Based on these findings, you consider that Taylor appears to be:

Mildly dehydrated.

Moderately dehydrated.

Severely dehydrated.

Adequately hydrated.

Ans: Moderately dehydrated.

You provide the following information to Taylor’s father.


Based on the below scenario:
Taylor is a 2½-year-old otherwise well child who presents today for a sick visit. According to his father, Taylor has vomited approximately 10 times in the past 18 hours, with the last episode about 4 hours ago. He has retained sips of water during the last hour. He also developed watery stools approximately 8 hours ago with 4 episodes in total. The last stooling episode was about 1 hour ago. Fever is reported to be elevated to 38.4ºC (101.2ºF) on a number of occasions during the past day. On assessment, he is lying in his father’s arms, is alert and nods his head when asked if he is thirsty. His lips are dry and slightly cracked and there is a small amount of saliva in the oral cavity. His heart rate is 110 BPM and capillary refill is <2 seconds. Dad tells you Taylor’s diaper was dry when he awoke 4 hours ago but due to watery stools, he is unsure if Taylor has voided this morning. Currently, Taylor is wearing a diaper that is slightly wet in the front.

Taylor can go home now on rehydration therapy with an appropriate oral rehydration solution and clear liquids.

Taylor should be started on rehydration therapy with an appropriate oral rehydration solution in the office now with a goal of demonstrating ability to tolerate oral fluids.

Given Taylor’s hydration status, he should be hospitalized for parenteral fluid replacement.

Taylor is able to go home on a diet of dry toast, mashed bananas, applesauce and white rice along with sips of clear liquids.

Ans: Taylor should be started on rehydration therapy with an appropriate oral rehydration solution in the office now with a goal of demonstrating ability to tolerate oral fluids.

A father presents his 2-year-old daughter with suspected gastroenteritis. She has had two episodes of vomiting and several episodes of diarrhea beginning 24 hours ago. Her last diarrheal episode was 2 hours ago and her diaper is wet. She has been able to consume small amounts of fluid but shows signs of mild dehydration. An appropriate treatment option to prevent further dehydration is a single oral dose of:

An antidiarrheal agent (e.g., bismuth salicylate [Pepto Bismol®]).

An antimotility agent (e.g., loperamide [Imodium®]).

A 5-HT3 antagonist (e.g., ondansetron [Zofran®]).

Antiparasitic antimicrobial (e.g. metronidazole [Flagyl®]).

Ans: A 5-HT3 antagonist (e.g., ondansetron [Zofran®]).

True or False?

1. In a febrile child, the degree of temperature reduction in response to antipyretic therapy is not predictive of presence or absence of bacteremia. __
2. Response to antipyretic medication does not change the likelihood of a febrile child having a serious bacterial infection and should not be used for clinical decision-making. __
3. The absence of tachypnea is the most useful clinical finding for ruling out pneumonia in children. __

1-True 2-True 3-False

1-False 2-True 3-False

1-True 2-True 3-True

1-False 2-False 3-False

Ans:

1-True 2-True 3-True

You are seeing Benjamin, an 18-month-old, who presents with his mother for a sick visit. His last well-child visit was at age 5 months, when he was up-to-date for recommended immunizations. Mom states that Benjamin has not been seen by another healthcare provider nor received vaccines since his last visit at your practice.
He now presents with a 2-day history of crankiness and fever. Benjamin has had a poor appetite for the past 2 days but has not vomited and has been taking small amount of fluids. His last wet diaper was approximately 2 hours ago. Exam reveals T=39.6ºC (103.4ºF), P=150 BPM, RR=45/min. Additional findings include slightly dry mucous membranes, capillary refill of <2 seconds, oropharyngeal redness, bilateral, red, immobile TMs, and a clear chest. The child has a high-pitched cry, is difficult to console, and does not regard his mother’s face. Your next best action is to:

Start the child on high-dose PO amoxicillin and oral analgesia with ibuprofen.

Give Benjamin an age- and weight-appropriate dose of an oral antipyretic with plans to reassess after 1 hour.

Initiate an evaluation for sepsis and consider for inpatient admission.

Administer a single dose of IM ceftriaxone and arrange for revisit tomorrow.

Ans: Initiate an evaluation for sepsis and consider for inpatient admission.

A mother presents her 5-year-old son who has a 3-day history of a reticular-form pink-red rash originating on his face that is spreading to his trunk and extremities. The mother reports that he had a mild fever and headache as well but is without GI symptoms other than slightly reduced appetite. A number of other children in his preschool have similar signs and symptoms. The most likely diagnosis for this patient is:

Croup.

Measles.

Roseola.

Fifth’s disease.

Ans: Fifth’s disease.

dentify at which age each of the following characteristics would indicate a potential problem with a child’s development.

A. By 9 months
B. By 12 months
C. By 16 months

1. Fails to return a smile or other facial expression. __
2. Lack of any spoken words. __
3. Shows no response to his/her name”. __

1-A 2-C 3-B

1-B 2-A 3-C

1-A 2-B 3-C

1-C 2-A 3-B

Ans: 1-A 2-C 3-B

A 6-year-old boy returns to the clinic with his mother 2 months following an episode of acute otitis media that was treated with a regimen of amoxicillin (Amoxil®). Examination reveals persistent effusion and mild bulging of the tympanic membrane, but there is no observed redness. The patient reports little-to-no pain in the affected ear and he is without fever. The most appropriate management option for this patient is:


Watchful waiting.

A course of an oral decongestant.

1 week therapy with amoxicillin-clavulanate (Augmentin®).

Cefpodoxime (Vantin®) at a therapeutic dose for 5 days.

Ans: Watchful waiting.

A 14-month-old girl is brought in for evaluation by her mother. She reports that her daughter has been fussy for the past 3 days and tugging at her left ear. Evaluation reveals moderate bulging of the tympanic membrane and her temperature is 102.6ºF (39.2ºC). The child has no reported medication allergies and has not received any antimicrobials in the past 3 months. You recommend:


Watchful waiting and follow-up in 3 days.

Amoxicillin (Amoxil®).

Clarithromycin (Biaxin®).

Levofloxacin (Levaquin®).

Ans: Amoxicillin (Amoxil®).

A woman brings in her 23 month-old grandson who has been fussy for the past two days with frequent holding of both ears and crying for the past 2 days. Evaluation reveals bulging and erythema of both tympanic membranes and a temperature of 100.4ºF (38.0ºC). The boy has no reported allergy to penicillin and is otherwise healthy. The most appropriate course of action is:


Analgesics and watchful waiting.

Amoxicillin.

Cefpodoxime.

Azithromycin.

Ans: Amoxicillin.

A father brings in his 7-year-old daughter for evaluation. She has experienced moderate pain in both ears for the past 2 days and has a fever (103.1ºF [39.5ºC]). Examination reveals bilateral moderate bulging tympanic membranes. She has a history of penicillin allergy. You prescribe therapy with:

Analgesics only and follow-up in 2 days.

Amoxicillin-clavulanate (Augmentin®).

Cefuroxime axetil (Ceftin®).

Ciprofloxacin (Cipro®).

Ans: Cefuroxime axetil (Ceftin®).

While counseling the mother of a healthy full-term newborn who is being formula-fed, she asks how much and how often will the baby eat. The most appropriate response is:


1.5 to 3 ounces (45-90 mL) every 2-3 hours.

4 to 5 ounces (120-150 mL) every 2-3 hours.

About 6 ounces (180 mL) every 3 hours.

About 2 ounces (60 mL) every 6-8 hours.

Ans:

1.5 to 3 ounces (45-90 mL) every 2-3 hours.

When counseling the parents of a healthy 2-week-old boy about supervised tummy time, you recommend slowly building up to a total of _______ per day.

15 minutes

30 minutes

1 hour

2 hours

Ans: 30 minutes

Which of the following is not a component of the CRAFFT questionnaire when evaluating an adolescent for substance abuse?”


Do you ever have trouble sleeping because you crave alcohol or drugs?

Have you ever gotten in trouble while you were using alcohol or drugs?

Do you ever use alcohol or drugs when you are alone?

Do you ever feel the need to use alcohol or drugs to relax or feel better about yourself?

ans: Do you ever have trouble sleeping because you crave alcohol or drugs?

You see a 10-year-old boy who is obese and has elevated triglycerides. You recommend:

Initiating treatment with a statin.

Initiating treatment with a fibrate.

Initiating treatment with fish oil.

Encouraging weight loss.

Ans: Encouraging weight loss.

breastfeeding mother brings in her 5-month-old son experiencing an episode of gastroenteritis. She reports that he has had 3 loose stools in the past 24 hours and vomited twice. Evaluation reveals that blood pressure, pulse, and skin turgor are all within normal limits. The NP recommends:”

Temporarily switching to soy-based formula.

Initiating therapy with oral rehydration solution.

Supplementing with 50% diluted fruit drink or sports drink.

Continuing breastfeeding.

Ans:

Continuing breastfeeding.

Which of the following patients with acne is the best candidate for oral isotretinoin (Accutane®)?

A 15-year-old with 10-15 comedones and has not responded adequately to benzoyl peroxide.

A 17-year-old with type 1 diabetes and a total lesion count of 80-100 and has a sulfa allergy

An 18-year-old with 5-10 nodules and ~60 inflammatory lesions who has not responded to multiple prior therapies

A 19-year-old woman with severe acne (>125 total lesions) who does not wish to initiate contraceptive therapy

Ans: An 18-year-old with 5-10 nodules and ~60 inflammatory lesions who has not responded to multiple prior therapies

First-line therapy for the treatment of community-acquired pneumonia in a 5-year-old child with a presumed atypical pathogen is:

Amoxicillin (Amoxil®).

Azithromycin (Zithromax®).

Oseltamivir (Tamiflu®).

Cefpodoxime (Vantin®).

Ans: Azithromycin (Zithromax®).

When examining a 1-day-old boy who was born at 39-weeks’ gestation, the NP notes a flat facial profile, hypotonia, hyperflexible joints, a single palmar crease in both hands, and Brushfield spots. This is most suggestive of:

Cerebral palsy.

Down syndrome (Trisomy 21).

Edwards syndrome (Trisomy 18).

Fragile X syndrome.

Ans: Down syndrome (Trisomy 21).

Adolescent issues: By high school senior year (Match the following).

20%
40%
50%
70%

A. Tried alcohol. __
B. Used an illegal/restricted drug including marijuana. __
C. Smoked a cigarette. __
D. Used a prescription drug for a nonmedical purpose. __

A-50% B-70% C-40% D-20%

A-70% B-40% C-50% D-20%

A-40% B-50% C-20% D-70%

A-70% B-50% C-40% D-20%

Ans: A-70% B-50% C-40% D-20%

Match the correct car safety device for each of the following children. An answer may be used more than once.

A. Rear-facing car seat
B. Forward-facing seat with harness
C. Belt-positioning booster seat
D. Seat belt

1. An 18-month-old born at 38-weeks’ gestation. __
2. A 9-year-old who is 4 feet 10 inches tall. __
3. A 1-year-old who is at 50th percentile in height and weight. __
4. A 7-year-old who is at 75th percentile in height and weight. __

1-C 2-B 3-D 4-A

1-A 2-D 3-A 4-C

1-B 2-B 3-A 4-D

1-A 2-C 3-C 4-B

Ans:

1-A 2-D 3-A 4-C

As part of a well-child visit, you advise the parents of a well 6-month-old that a mild fever of 1-2 days in duration is most likely to occur after the baby receives:


Inactivated polio virus (IPV) vaccine.

Haemophilus influenzae type B (Hib) vaccine.

Pneumococcal conjugate 13-valent vaccine (PCV13).

Injectable influenza vaccine.

Ans: PCV 13

You see a 13-year-old boy with no immunizations documented. In updating this child’s immunizations, which of the following should be omitted?


Hepatitis B

Haemophilus influenzae type B (Hib)

Tetanus, diphtheria, acellular pertussis (Tdap)

Measles, mumps, rubella (MMR)

Ans: Haemophilus influenzae type B (Hib)

A 78 Year-old woman with a 30-year history of hypertension and dyslipidemia and a 10-year history of type 2 diabetes presents with suspected acute coronary syndrome. She is most likely to report a recent onset of:

Retrosternal chest pain with radiation to the left arm.

Syncope.

Unusual fatigue.

Nausea and vomiting.

Ans: Unusual fatigue

Jack is a 16 year-old varsity basketball player who presents for a sports clearance physical examination. His health history is unremarkable and reports excellent activity tolerance. Which of the following would be a finding on Jack’s cardiac examination that would warrant immediate evaluation?

A split second heart sound that increases on inspiration and closes on expiration accompanied by a point of maximum impulse (PMI) at the 5th intercostal space (ICS) midclavicular line (MCL)

A grade 2/6 midsystolic murmur that increases in intensity with position change from supine to standing accompanied by a loud S4

PMI with palpable single impulse at the 5th intercostal space accompanied by S1 louder than S2 at the apex

A grade 2/6 systolic murmur noted over the precordium without radiation that disappears with position change from supine to standing accompanied by S2 louder than S1 at the base

Ans: A grade 2/6 midsystolic murmur that increases in intensity with position change from supine to standing accompanied by a loud S4

Jane is a 27 year-old woman who presents for a “pap test.” She is new to your practice, has not significant health history and reports excellent exercise tolerance, running up to 5 miles 5-7 days a week and recently was first place in her age division in a 6K road race. Jane reports taking a low-dose combined oral contraceptive and a daily multivitamin. Physical examination reveals a BMI=22.1 kg/m2, mild pectus excavatum, PMI WNL with both heart sounds intact. There is a midsystolic click with late systolic murmur and the murmur moves forward with position change from supine to stand. The rest of her physical examination is unremarkable. The next most appropriate step in her care is to:

Prescribe an antibiotic that she should take prior to dental work.

Inform her that she should curtail her running program.

Advise Jane that she should discontinue the use of a combined oral contraceptive immediately.

Inform Jane that obtaining an echocardiogram is a prudent next step.

Ans: Inform Jane that obtaining an echocardiogram is a prudent next step.

Mrs. Rivera is an 82 year-old woman with infrequent contact with her HCP. She presents for sick visit with the following chief complaint, “I get really dizzy when I walk up a flight of stairs.” She denies chest pain, agrees that she feels “a bit winded” when physically active; this symptom resolves quickly with cessation of trigger activity. Physical examination reveals the following: BP=110/90, gr 2/6 harsh systolic murmur with radiation to the neck, loudest along USB, softer towards axilla, both heart sounds preserved, no S3 or S4, no neck vein distention, no carotid bruit, delayed carotid upstroke. Her clinical presentation is most consistent with the presence of:

Mitral stenosis.

Carotid artery dissection.

Systolic heart disease.

Calcific aortic stenosis.

Ans:

Calcific aortic stenosis.

MR-ASS or MS-ARD
Mitral regurgitation -Aortic stenosis = Systolic murmurs (PMI at 5th ICS)
Mitral stenosis- Aortic stenosis = Diastolic murmurs (2nd USB radiating to right side of the neck)

Raymond is an 18 year-old high school basketball player. He is in for a sports clearance physical examination and denies activity intolerance, chest pain, and syncope. Today’s Assessment Reveals the Following: 74″ (188 cm), 220 lb (99.8 kg), Tanner Stage 5. Both heart sounds intact, a grade 2/6 harsh systolic murmur with radiation to the neck, loudest along USB, softer towards axilla, somewhat softer with position change from supine to standing, no delay in carotid upstroke, other pulses intact. Raymond’s findings are most likely the result of:

Aortic stenosis.

Aortic regurgitation.

Mitral valve prolapse.

Physiologic murmur.

Ans: Aortic stenosis.

Richard is a 62 year-old man w/ HTN heart disease and HF. He presents today for follow up and is without new symptoms. Physical examination reveals the following: PMI @ 5th ICS AAL with a sustained impulse, Gr 3/6 blowing holosystolic murmur with radiation to axilla. The murmur accentuated by rolling patient onto left side, softens when going from supine to standing position, louder with hand grip, 2d heart sound not preserved, and full carotid upstroke bilaterally is noted. These findings are most consistent with:

Mitral valve prolapse.

Aortic stenosis.

Pulmonic stenosis.

Mitral regurgitation.

Ans: Mitral regurgitation

MR-ASS or MS-ARD
Mitral regurgitation -Aortic stenosis = Systolic murmurs (PMI at 5th ICS)
Mitral stenosis- Aortic stenosis = Diastolic murmurs (2nd USB radiating to right side of the neck)

Joe is a 54-year-old man with COPD who presents with an 18-hour history of sudden-onset monoarticular pain consistent with acute gouty arthritis. He denies trauma to the area and has taken acetaminophen 1 g × 2 doses with little effect. The most likely presentation of this condition is:

Swelling at the third distal interphalangeal joint.

Redness at the first metatarsophalangeal.

Firm, white 4 mm nodular auricular lesion.

Effusion in the right knee.

Ans:

Redness at the first metatarsophalangeal.

Which of the following medications would be ineffective in treating Joe’s episode of acute gouty arthritis?
Based on the below scenario:
Joe is a 54-year-old man with COPD who presents with an 18-hour history of sudden-onset monoarticular pain consistent with acute gouty arthritis. He denies trauma to the area and has taken acetaminophen 1 g × 2 doses with little effect.

Intraarticular corticosteroid injection

Naproxen sodium (Aleve®, Anaprox®)

Febuxostat (Uloric®)

Colchicine (Colcrys®)

Ans: Febuxostat (Uloric®)

Which of the following is not a potential acute gouty arthritis trigger?

Use of a thiazide diuretic

Consumption of organ meats

Alcohol consumption

Report of recently increased intake of acidic foods

ans: Report of recently increased intake of acidic foods

Match each test or sign with the relevant condition below:
A. Drop arm test
B. Finkelstein test
C. McMurray test
D. Lachman test
E. Spurling test
F. Straight-leg raising test
G. Talar tilt
H. Tinel’s Sign
Conditions:
1. Meniscal tear __
2. Ankle instability __
3. Carpal tunnel syndrome __
4. Anterior cruciate ligament tear __
5. Lumbar nerve root compression __
6. Cervical nerve root compression __
7. Rotator cuff evaluation __
8. De Quervain’s tenosynovitis __

Ans: 1-C (McMurray Test- Meniscal tear of the knee)
2-G (Talar Tilt test for ankle instability)
3-H (Tinel’s sign for Carpal tunnel syndrome – two hand place against with hyperflexion of the wrists, if numbness or tingling sensation is elicited test is +)
4-D (Lachman Test for ACL injury of the knee where tendon is twisted causing injury; patient cannot squat, very painful)
5-F (straigth leg raising test_ leg is raised 45 degrees, ask pt to hyperextend against the examiner’s hand if pt is experienced pain, test is +)
6-E (Spruling test – test for range of motion (flexion.. of neck)
7-A (Drop-arm test for rotator cuff evaluation)
8-B (Finkelstein test)

Ms. Hannigan is a 70-year-old woman with well-controlled hypertension, dyslipidemia, and hypothyroidism. Her current medications include a statin, ACE inhibitor, a thiazide diuretic, and levothyroxine replacement. She presents today with a chief complaint of a 2-month history of fatigue and aching sensation accompanied by morning stiffness in her hips and shoulders. During this time, she has lost 8 lb (3.6 kg) “without even trying” and reports struggling to get dressed, especially when putting on a shirt or pants. Physical exam reveals full limb strength, decreased active range of motion in the hips and shoulders, cool, smooth joints, and no excessive muscle tenderness. Laboratory test results include:
Hb=10.8 g/dL (108 g/L)
Hct=32% (0.32 proportion)
MCV=86 fL
RDW=12.2% (0.122 proportion)
ESR=112 mm/h (ULN=20 mm/h)
The result of her hemogram reveals:

Iron deficiency anemia.

Pernicious anemia.

Anemia of chronic disease.

Age-relative normative findings.

Ans: Anemia of chronic disease.

Ms. Hannigan’s clinical presentation is most consistent with:
Based on the below scenario:
Ms. Hannigan is a 70-year-old woman with well-controlled hypertension, dyslipidemia, and hypothyroidism. Her current medications include a statin, ACE inhibitor, a thiazide diuretic, and levothyroxine replacement. She presents today with a chief complaint of a 2-month history of fatigue and aching sensation accompanied by morning stiffness in her hips and shoulders. During this time, she has lost 8 lb (3.6 kg) “without even trying” and reports struggling to get dressed, especially when putting on a shirt or pants. Physical exam reveals full limb strength, decreased active range of motion in the hips and shoulders, cool, smooth joints, and no excessive muscle tenderness. Laboratory test results include:
Hb=10.8 g/dL (108 g/L)
Hct=32% (0.32 proportion)
MCV=86 fL
RDW=12.2% (0.122 proportion)
ESR=112 mm/h (ULN=20 mm/h)

Rheumatoid arthritis.

Fibromyalgia.

Osteoarthritis.

Polymyalgia rheumatica.

Ans: Polymyalgia Rheumatica

The most appropriate next step in Ms. Hannigan’s plan of care is:
Based on the below scenario:
Ms. Hannigan is a 70-year-old woman with well-controlled hypertension, dyslipidemia, and hypothyroidism. Her current medications include a statin, ACE inhibitor, a thiazide diuretic, and levothyroxine replacement. She presents today with a chief complaint of a 2-month history of fatigue and aching sensation accompanied by morning stiffness in her hips and shoulders. During this time, she has lost 8 lb (3.6 kg) “without even trying” and reports struggling to get dressed, especially when putting on a shirt or pants. Physical exam reveals full limb strength, decreased active range of motion in the hips and shoulders, cool, smooth joints, and no excessive muscle tenderness. Laboratory test results include:
Hb=10.8 g/dL (108 g/L)
Hct=32% (0.32 proportion)
MCV=86 fL
RDW=12.2% (0.122 proportion)
ESR=112 mm/h (ULN=20 mm/h)

Referral to physical therapy.

Rest and ice to the affected areas.

Initiating a course of systemic corticosteroids.

Prescribing oral low-dose bisphosphonate therapy.

Ans: Initiating a course of systemic corticosteroids

Which of the following is unlikely to be noted in Tom, a 62-year-old man with lumbar spinal stenosis?

Report of back pain worsening with standing

Bilateral leg numbness

Pain improvement with flexed forward position

Absent pedal pulses

Ans: Absent pedal pulses

Richard is a 28-year-old man who presents with a chief complaint of left knee pain and swelling for the past month, as well as redness and tearing in the left eye for the past week. He also has had intermittent dysuria and 2-3 loose stools per day for the past 2 weeks. He denies weight loss, skin rash, or fever. Physical exam reveals a smooth, swollen, red, warm left knee with decreased range of motion, pupils equal and reactive to light with marked unilateral conjunctival redness. The urinary meatus is reddened. Richard’s clinical presentation is most consistent with:

Systemic lupus erythematosus.

Polymyalgia rheumatica.

Reactive arthritis (Reiter’s syndrome).

Psoriatic arthritis.

ans: Reactive arthritis (Reiter’s syndrome).

n evaluating Richard, the next most appropriate test to obtain is:
Based on the below scenario:
Richard is a 28-year-old man who presents with a chief complaint of left knee pain and swelling for the past month, as well as redness and tearing in the left eye for the past week. He also has had intermittent dysuria and 2-3 loose stools per day for the past 2 weeks. He denies weight loss, skin rash, or fever. Physical exam reveals a smooth, swollen, red, warm left knee with decreased range of motion, pupils equal and reactive to light with marked unilateral conjunctival redness. The urinary meatus is reddened.

Serum antinuclear antibodies.

Serum rheumatoid factor.

Urinary PCR testing for N. gonorrhoeae and C. trachomatis.

Stool for ova and parasites.

ans: Urinary PCR testing for N. gonorrhoeae and C. trachomatis.

Josh is a 14-year-old basketball player who presents with anterior knee pain that has occurred intermittently over the past 3 months. The pain worsens with squatting and walking up or down stairs, and is better with rest. He denies fever, weight loss, joint redness, or skin rash. Physical examination reveals a Tanner 3 male in no acute distress with a tender, swollen tibial tuberosity in the affected knee. Pain can be reproduced with resisted active extension and passive hyperflexion of the knee. No effusion is present. Josh’s presentation is most consistent with:

Osgood-Schlatter disease.

Prepatellar bursitis.

Meniscal tear.

Reactive arthritis.

Ans: Osgood-Schlatter disease.

Intervention for Josh should include information about:
Based on the below scenario:
Josh is a 14-year-old basketball player who presents with anterior knee pain that has occurred intermittently over the past 3 months. The pain worsens with squatting and walking up or down stairs, and is better with rest. He denies fever, weight loss, joint redness, or skin rash. Physical examination reveals a Tanner 3 male in no acute distress with a tender, swollen tibial tuberosity in the affected knee. Pain can be reproduced with resisted active extension and passive hyperflexion of the knee. No effusion is present.

Curtailing his participation in sporting activities.

Avoiding sports that involve heavy quadriceps loading or deep knee bending.

The benefit of intraarticular corticosteroid injection for the control of discomfort.

The likely need for surgical correction of the defect.

Ans: Avoiding sports that involve heavy quadriceps loading or deep knee bending.

Which of the following is an acceptable team sport for a college-age young adult with a bleeding disorder to participate in?

Lacrosse

Soccer

Swimming

No team sport is acceptable.

Ans: Swimming

You are evaluating Karen, a 48-year-old African American woman who is a nonsmoker and drinks 2 glasses of wine per week. She has a longstanding intermittent lumbosacral strain that she attributes in part to her work, which requires her to be physically active. Today she presents with a 2-week history of shooting pain down the right leg and occasional “dragging” of the right foot with walking. Karen denies any recent trauma or precipitating event. Examination reveals abnormal straight-leg raising, diminished right patellar reflex, and difficulty performing heel walking. She relates, “I am really uncomfortable. This is different than my usual back pain.” Her presentation is most consistent with:

Lumbar radiculopathy.

Exacerbation of chronic lumbar-sacral strain.

Vertebral compression fracture.

Spondyloarthropathy.

Ans: Lumbar radiculopathy.

Which of the following represents the most appropriate next step in Karen’s plan of care?
Based on the below scenario:
You are evaluating Karen, a 48-year-old African American woman who is a nonsmoker and drinks 2 glasses of wine per week. She has a longstanding intermittent lumbosacral strain that she attributes in part to her work, which requires her to be physically active. Today she presents with a 2-week history of shooting pain down the right leg and occasional “dragging” of the right foot with walking. Karen denies any recent trauma or precipitating event. Examination reveals abnormal straight-leg raising, diminished right patellar reflex, and difficulty performing heel walking. She relates, “I am really uncomfortable. This is different than my usual back pain.”

Advise a 3-day course of bedrest

Refer to physical therapy

Order a lumbosacral MRI

Obtain a standing lumbosacral x-ray

Ans: Refer to physical therapy

You are counseling a 56-year-old woman who asks about increasing her dietary calcium intake with nondairy foods. You counsel about adding all of the following foods that are calcium-rich except for:

Spinach.

Tofu.

Almonds.

Red grapes.

ans: Red grapes.

Match each of the following findings with the associated disease state. An item can be used more than once.
A. Rheumatoid arthritis
B. Systemic lupus erythematosus
C. Osteoarthritis

1. Anemia of chronic disease
2. Elevated C-reactive protein
3. Joint-space narrowing on x-ray
4. Positive antinuclear antibody titer

1-A & B 2-A & B 3-C 4-A & B

1-A 2-C 3-B & A 4- A & B

1-A & B 2-A 3-A 4-A & C

1-C 2-A 3-C 4-A & B

Ans: 1-A & B ( Anemia of chronic disease -autoimmune- RA and Lupus)
2-A & B (RA and Lupus CRP elevated )3-C (Osteoarthritis presents with joint-space is narrowing on Xray due to thinning of the cartilage) 4-A & B (ANA titers elevated in RA and Lupus as well)

Mrs. Garcia is a 78-year-old woman who presents with a chief complaint of progressively increasing aches, limited to her hands and fingers, particularly after doing yard work. Otherwise, she denies musculoskeletal problems. Physical examination reveals bilateral Heberden’s and Bouchard’s nodes.
1. Identify the location of the Heberden’s nodes. _____
2. Identify the location of the Bouchard’s nodes. _____


1-A 2-B

ans: Bouchar’s node is proximal interphalengeal joint of the hands in RA
Heberden’s node is distal interphalengeal joint of the hands and fingers

Mrs. Garcia’s most likely diagnosis is:

Systemic lupus erythematosus.

Rheumatoid arthritis.

Osteoarthritis.

Reactive arthritis.

Ans: Osteoarthritis.

Jane is a 56-year-old woman who undergoes DXA testing of the spine and reveals a T-score of ‒2.9. Based on recommendations by the American College of Physicians, the NP suggests:

No treatment needed at this time.

Initiating treatment with risedronate.

Initiating treatment with estrogen therapy.

Initiating treatment with raloxifene.

Ans: Initiating treatment with risedronate.

For a woman initiating bisphosphonate therapy for osteoporosis, treatment should continue:

Until BMD improves by 25%.

For 2 years.

For 5 years.

Indefinitely.

ans: for 5 years

When initiating pharmacologic treatment for osteoporosis in women, the American College of Physicians recommends bone mineral density monitoring:

Annually.

At 6 months following initiation of treatment, then annually thereafter.

Every 2 years.

At the completion of therapy.

Ans: At the completion of therapy

All of the following are characteristics of knee osteoarthritis except:

Limited range of motion.

Morning stiffness.

Erythema surrounding the joint.

Crepitus.

Ans: Erythema surrounding the joint.

The preferred technique for initial diagnosis of osteoarthritis of the knee is:

X-ray.

MRI.

CT scan.

Arthroscopy.

Ans: Xray

All of the following are recommended by the American Academy of Orthopaedic Surgeons (AAOS) for the management of OA of the knee except:

Weight loss if BMI ≥25 kg/m2.

Use of hyaluronic acid injection.

Low-impact aerobic exercise.

Strengthening exercises.

Ans: use of hyaluronic acid injection

A 20-year-old woman requests information about emergency contraception using high-dose progestin after having unprotected sexual intercourse approximately 18 hours ago. Today is day 12 of her normally 27-29 day menstrual cycle. You respond that:

The use of this type of emergency contraception reduces the risk of pregnancy by approximately 33% if taken at this point.

All forms of emergency contraception must be used within 18 hours of unprotected intercourse to be effective.

The likelihood of pregnancy is minimal and emergency contraception is not indicated.

Progestin-based emergency contraceptive is likely helpful in reducing pregnancy risk when used as long as 120 hours post intercourse.

Ans: Progestin-based emergency contraceptive is likely helpful in reducing pregnancy risk when used as long as 120 hours post intercourse.

According to the US medical eligibility criteria for contraceptive use which of the following clinical scenarios describes a Category 3 (exercise caution) situation for the use of a combined oral contraceptive?

Presence of factor V Leiden mutation

Report of ectopic pregnancy 2 years ago

Treatment 3 years ago for high grade squamous intraepithelial neoplasia (HGSIL) with loop electrical excision procedure (LEEP)

History of gastric bypass surgery

Ans:

History of gastric bypass surgery
Individual with hx of gastric bypass often lack of intrinsic factors due to the resection of gastric portion therefore; malabsorption is an issue for oral contraceptive effects is reduced, yielding high risk of pregnancy.

The reduction in free androgens noted in the woman taking combined oral contraceptives can yield an improvement in:

Menstrual irregularity.

Acne vulgaris.

Breast tenderness.

Rheumatoid arthritis symptoms.

Ans: Acne vulgaris

The use of which of the following antibiotics is most likely to reduce combined oral contraceptive effectiveness?

Amoxicillin

Ciprofloxacin

Doxycycline

Rifampin

Ans: Rifampin is CYP450 3A4 Isoenzymers to break down the substrates of oral contraceptive therefore OCC is ineffective.

Osteopenia is defined as a spine, hip, or forearm bone mass density as determined by DXA devices that is ____ standard deviation (SD) below that of a young adult norm.

<1

1

1-2.5

≥2.5

Ans: -1 – -2.5

Who is a Cu-IUD (ParaGard®) or LNG-IUS (Mirena®) candidate (category 1 or 2)?
Yes or No

1. 45-year-old nulliparous woman
2. 33-year-old who smokes 2 PPD
3. 25-year-old with seizure disorder
4. 33-year-old woman with HIV (excludes AIDS-defining illness)

1-Yes 2-Yes 3-No 4-No

1-Yes 2-Yes 3-Yes 4-Yes

1-No 2-Yes 3-No 4-Yes

1-No 2-No 3-Yes 4-No

Ans: 1-Yes 2-Yes 3-Yes 4-Yes Cu_IUD (ParaGard) and LNG -IUS (levonogestrol – progestin-based only, no estrogen intrauterine device) therefore, no contraindications for all these women.

A 23-year-old- woman is breastfeeding her healthy newborn. She wishes to use hormonal contraception. Which of the following represents an unacceptable choice?

Combined oral contraception initiated at 2 weeks

Progestin-only oral contraception initiated at 3 weeks

Medroxyprogesterone acetate (Depo-Provera®) given day 1 postpartum

CU-containing IUD (ParaGard®)

Ans: Combined oral contraceptives initiated at 2 weeks.

Age-related changes in older adults can result in a decreased drug effect for all of the following except:

Inhaled muscarinic antagonists

Warfarin

DHP calcium channel blockers

Beta2-agonists

Ans:

Warfarin

For a patient who is >60 years old, with hepatic impairment, or taking concomitant cimetidine, the recommended daily dose of citalopram should not exceed:

10 mg

20 mg

40 mg

60 mg

Ans: 20 mg

Who is a combined oral contraceptive (estrogen/progestin) candidate?
A. Category 1
B. Category 2
C. Category 3
D. Category 4
E. Category 1 for Initiation & Category 2 for continuation
F. Category 3 for Initiation & Category 4 for continuation

1. 22-year-old who smokes 1 PPD__
2. 29-year-old with pelvic inflammatory disease __
3. 45-year-old with recurrent tension-type headache: Non-migrainous headache __
4. Age ›40 years __
5. 36-year-old woman with well controlled primary hypertension who is taking ARB and HCTZ: HTN with adequate control __
6. HTN with poor control __

1-A 2-B 3-E 4-C 5-C 6-D

1-B 2-A 3-E 4-B 5-C 6-D

1-C 2-A 3-D 4-F 5-B 6-A

1-F 2-D 3-A 4-A 5-A 6-C

Ans: 1-B (no contraindications – smoking put pt in Cat 2)
2-A (No contraindications)
3-E (Age >35yrs, hz of non-migrainous headache; if this individual has migrain headache then she is in Cat 4 -absolute risk of stroke or blood clots)
4-B (Age > 35yrs but no other contraindications)
5-C (caution use with hx of HTN but her HTN is controlled; increased caution of OCC use due to tendency to increase blood pressure)
6-D (Cat 4 Absolute contraindications for OCC use regardless of age)

Which of the following psychotropic medications would be preferred in the elderly due to its low anticholinergic and sedation effect?

Amitriptyline

Nortriptyline

Sertraline

Paroxetine

Ans: Sertraline

Accordi The A1C goal for older adults who are frail or with limited anticipated life expectancy of ≤5 years is:

≤6.5%

≤7.0%

≤8.0%

≤9.0%

Ans:

≤8.0%

Age-related changes in the elder include all of the following except:

Decrease in relative kidney weight

Body weight as fat increases for both men and women

Increase in serum albumin

Decrease in percent body weight as water

Ans: Increase in serum albumin

Potential adverse effects from medications with significant anticholinergic effect include all of the following except:

Hypothermia

Dry mouth, skin

Blurred vision

Urinary retention

Ans:

Hypothermia

Absorption of which of the following is least affected by long-term PPI use?

Iron

Calcium citrate

Vitamin B12

Magnesium

Ans: Calcium citrate

PPIs such as lansoprazole, omeprazole and esomeprazole are all inhibitors of CYP450:

3A4

2C19

1A2

2D6

Ans:

2C19

TMP-SMX should be avoided in elders who are being concomitantly treated with:

Moderate- to high-intensity statin therapy

ARBs or ACEIs

2nd-generation antihistamine

SSRIs

Ans: ARBs orACIs due to increase risks of hypotension and cardiac events

For an 87-year-old man with a history of cardiovascular disease and requires an LDL-C reduction of approximately 33%, the NP recommends:

Initiating atorvastatin 20 mg daily

Initiating rosuvastatin 20 mg daily

Initiating simvastatin 10 mg daily

No statin therapy as there is likely no significant benefit at his age

Ans: Initiating atorvastatin 20 mg daily (moderate intensity statin therapy reduction of <33% of LDL level)

The NP certification exam will contain a number of questions about my state’s NP practice act.

True

False

Ans: False

Once I have achieved national certification, I can practice as a nurse practitioner.

True

False

Ans: False

Since each state has distinct requirements for NP scope of practice, no questions specific to a certain state’s practice act will appear on national certification exams.

True

False

ans: True

The charge of the State Board of Nursing is to ensure public safety.

True

False

Ans: True

State level authorization dictates the ability of the NP to obtain a Federal DEA number.

True

False

ans: True

Which of the following describes the ethical principle of beneficence?

The right of the competent person to choose a personal plan of life and action.

The obligation of the healthcare provider to help people in need.

The duty of healthcare provider to do no harm.

The responsibility of the healthcare provider to treat all in the same fair manner.

Ans: The obligation of the healthcare provider to help people in need.

The majority of NPs are eligible to obtain a Federal DEA number.

True

False

ans: True

Which of the following describes the ethical principle of utilitarianism?

Healthcare resources are allocated so that the best is done for the greatest number of people.

The healthcare provider must be truthful and avoid deception.

The healthcare provider has an obligation to be faithful to commitments made to self and others.

The responsibility of the healthcare provider is to treat all people in the same fair manner.

Ans: Healthcare resources are allocated so that the best is done for the greatest number of people.

Which of the following describes the ethical principle of veracity?

Healthcare resources are allocated so that the best is done for the greatest number of people.

The healthcare provider must be truthful and avoid deception.

The healthcare provider has an obligation to be faithful to commitments made to self and others.

The responsibility of the healthcare provider is to treat all people in the same fair manner.

Ans: The healthcare provider must be truthful and avoid deception.

An organization has donated 1000 pediatric vaccines to a community clinic. Ensuring that the distribution of the vaccines is done in a fair and unbiased manner is an example of:

Utilitarianism

Justice

Beneficence

Ans: Justice

While walking in the parking lot of a grocery store, an NP notices an elderly man fall and hit his head on the ground. The NP quickly rushes to provide aid to the man. This is an example of:

Beneficence

Autonomy

Veracity

Utilitarianism

Ans: Beneficence

A mother presents her 3-year-old son with a 2-day history of cough and runny nose without fever. The NP explains that this is likely a viral infection and treatment should focus on symptom management. However, the mother is insistent on getting a prescription for antibiotics despite the NP’s explanation that antibiotics will have limited effectiveness and may cause adverse effects. In this scenario, which of the following best demonstrates the ethical principle of nonmaleficence?

Providing a prescription for an antibiotic regimen.

Providing a 2-day sample of antibiotics with follow-up at 3 days to determine if additional antibiotics are needed.

Denying a prescription for antibiotics.

Getting approval from a physician prior to giving the antibiotic prescription

Ans: Denying a prescription for antibiotics.

Which of the following best demonstrates the intent of informed consent?

Discussing the risks of surgery with the parent of a 20-year-old

Advising a 48-year-old woman about the likely course of illness if no treatment is rendered

Conferring with the husband of a 30-year-old woman who has pneumonia about the benefits of antimicrobial therapy

Recommending a course of treatment to a 70-year-old man with prostate cancer

Ans: Advising a 48-year-old woman about the likely course of illness if no treatment is rendered

A patient who underwent CABG following a MI develops a surgical site infection. Upon reviewing the patient’s records, the NP notices that standard antimicrobial prophylaxis was not provided prior to surgery. The ethical principle of ________ requires the NP to report this finding.

Nonmaleficence

Justice

Veracity

Utilitarianism

Ans: Veracity

A mentally competent patient with advanced cancer is explained 2 treatment options. He decides to take the less invasive technique, despite counseling him that his choice has a lower chance for success. Respecting the patient’s choice demonstrates the ethical principle of:

Justice

Veracity

Paternalism

Autonomy

Ans :

Autonomy

A 90-year-old woman with moderate dementia, heart failure, and chronic renal insufficiency presents with a new onset of weakness. Further evaluation reveals marked hyperkalemia with a eGFR=22 mL/min/1.73 m2. Nephrology recommends hemodialysis. When discussing this option with the patient, she declines this treatment. She also inaccurately reports the current year, location, and name of the USA president. When asked why she does not want dialysis, she states, “I do not want to be on a machine. I am quite old and my health is not good.” When asked what she believes will happen if she does not have dialysis, she states, “I guess I will die. That is OK.” In reflecting on this patient, the NP appreciates that:

She demonstrates competency in this situation and her wishes can be followed.

Due to her inability to report accurate date, location, or current president, she is not competent to make healthcare decisions.

A patient with a documented diagnosis of dementia is considered incompetent to make all healthcare decisions.

Court-appointed guardianship should be sought.

Ans:

She demonstrates competency in this situation and her wishes can be followed.

Mr. Nelson is a 75-year-old man in need of surgery. Mr. Nelson has been treated unsuccessfully with conservative therapy for a bowel obstruction. Documented on his record is his consent to surgery that was obtained by the chief surgical resident. During your preoperative visit with Mr. Nelson, he is alert and relatively comfortable. His wife mentions that her husband had a dose of promethazine (Phenergan®) to manage his nausea around the time the surgical consent was obtained. Mrs. Nelson asks, “Is the consent OK since my husband clearly does not feel well?” You respond:

Since the chief surgical resident obtained the surgical consent, this should be considered valid.

Legal counsel should be obtained prior to proceeding.

That surgical consent should be obtained from Mrs. Nelson as she is Mr. Nelson’s de facto healthcare proxy.

You will call the chief surgical resident to clarify the issue of consent.

Ans: You will call the chief surgical resident to clarify the issue of consent.

For an individual to demonstrate the capability to make an informed healthcare decision, all of the following must be present except:

The patient has the ability to understand the nature of the clinical condition.

The patient can interpret the information presented.

The patient is able give a reason for the choice if asked.

The patient does not need to be able to communicate what the care preference is.

Ans: The patient does not need to be able to communicate what the care preference is.

The adult daughter of an 88-year-old man telephones the nurse practitioner to inquire about her father’s medications. The daughter reports that “my father relies upon me to explain everything to him.” The nurse practitioner’s initial response is to:

Ask the daughter to provide a copy of the patient’s advance directive document.

Assure the daughter that the NP can share the requested information.

Inform the daughter that she must come to the clinic to discuss her father’s care.

Tell the daughter that the NP can discuss the information only with the patient.

Ans: Tell the daughter that the NP can discuss the information only with the patient.

A 50-year-old man of Croatian ancestry has a follow-up appointment after cardiac bypass surgery. The patient brings his father with him into the examination room. They are both fluent English speakers. The NP provides culturally sensitive care by the following action.

Ask the patient’s father if he has any questions regarding his son’s care.

Request the patient’s father to leave the room due to confidentiality issues.

Acknowledge the father’s presence and complete the visit while reporting all findings to the patient.

Perform the clinical evaluation, then report to the patient’s father the examination findings.

Ans: Ask the patient’s father if he has any questions regarding his son’s care.

You see a 62-year-old man, Mr. Tran, who is from Southeast Asia. He is accompanied in the exam room by his 39-year-old son. Both are fluent English speakers. Mr. Tran is seen in follow-up on type 2 diabetes and hypertension. As you speak with Mr. Tran, he smiles, nods his head in response to your questions. He frequently looks down at the floor. You consider that:

Mr. Tran is conveying that he understands your questions.

His downcast gaze should trigger an assessment for depression or other mood disorder.

Directing your questions to Mr. Tran’s son is likely a more appropriate approach to the visit.

Mr. Tran is showing respect for the healthcare provider’s authority.

Ans:

Mr. Tran is showing respect for the healthcare provider’s authority.

Mr. Miller is a 60-year-old man with hypertension. On today’s office visit, his blood pressure is noted to be 122/78. The rest of his history and physical examination are unremarkable. He mentions that, in addition to his antihypertensive medication you have prescribed, he uses a teaspoon of hot pepper sauce in a glass of warm water daily to help “clean my blood and lower the pressure. I feel much better when I take it.” Your most appropriate response is to:

Advise Mr. Miller to stop using the hot pepper sauce.

Explore what Mr. Miller means by “clean my blood.”

Inform Mr. Miller that this practice can result in serious gastrointestinal complications.

Ask Mr. Miller to stop the hot pepper sauce use until you can examine the contents of the product he uses.

Ans: Explore what Mr. Miller means by “clean my blood.”

Mr. Santorini is a 71-year-old man with a 10-year history of hypertension. He is currently taking a fixed-dose lisinopril/hydrochlorothiazide tablet daily and reports excellent adherence. On today’s visit, he is without complaint and physical examination is within normal limits, with BP=138/84. He mentions that, for the past three months, he has been sprinkling a “pinch” of cumin and coriander mixture on his food once a day to “help control my blood pressure” and reports “feeling like I have more energy” since starting this. You consider that:

This mixture will likely interact with his antihypertensive medications.

Using cumin or coriander could raise his blood pressure.

Additional medications can be prescribed if his blood pressure control worsens.

He is advocating for his health and the practice should be supported.

Ans:

He is advocating for his health and the practice should be supported.

An APRN prescribes clarithromycin for the treatment of sinusitis in a woman taking simvastatin. While on the antibiotic, the patient develops rhabdomyolysis. Can you see a correlation between the above events?

Yes

No

Ans:

Yes

Medicaid is the largest payer for nursing facility care.

True

False

Ans: True

Impoverishment is the primary requirement for Medicaid eligibility.

True

False

Ans: True

A 52-year-old woman who is Muslim arrives for an office visit. Her last primary healthcare visit was more than 10 years ago. She mentions that she does not want to disrobe or remove her head cover for a physical exam. You consider that:

Her healthcare visit cannot proceed until the patient is able to disrobe for the physical exam.

A mammogram should be ordered without prior breast examination.

The option of having a modified physical examination with minimal disrobing should be discussed with the patient.

The health history can be completed today and the physical examination deferred until a future office visit.

ans: The option of having a modified physical examination with minimal disrobing should be discussed with the patient.

You see a 54-year-old woman, Senora Sanchez, who is not fluent in speaking English, who comes into the emergency department of a 300-bed hospital with a chief complaint of abdominal pain and is accompanied by Tomas, her 10-year-old nephew. Tomas states, “I am here to help my aunt since she does not speak English.” Which of the following is the most appropriate way in conducting this clinical encounter?

Direct questions to Tomas, asking him to translate the information to his aunt.

Direct questions to Senora Sanchez, asking Tomas to translate.

Advise that Senora Sanchez return with an adult interpreter.

Call the hospital’s interpreter services department for assistance.

Ans: Call the hospital’s interpreter services department for assistance.

You see a 24-year-old mother who presents with her healthy 18-month-old son. He is being seen for a well child visit. An interactive child who is quite playful, his examination is within normal limits. When you ask if the mom has any concerns, she states, “I do not like it when people stare at my baby. He is going to get sick.” You respond:

“What type of illness do you think your son will get when people look at him?”

“You are concerned he will get sick if people look at him?”

“Please make sure you come back for a sick visit with your baby whenever you have concerns about your son.”

“Your baby is just delightful. I can imagine that people often look at him.”

Ans:

“You are concerned he will get sick if people look at him?”

The Medicare program pays for any services that are covered by Medicare before any payments are made by the Medicaid program, since Medicaid is always the payer of last resort.

True

False

Ans: True

Your neighbor asks you to refill her high blood pressure medicine as she could not make her last office visit to her provider due to a family emergency. You are aware that she is going through a rough time personally and agree to call in the prescription. Is she now your patient?

Yes

No

Ans:

Yes

A malpractice suit concerning a delayed cancer diagnosis is filed naming a NP in a primary care practice. Will the APRN be held to standard of care of an APRN with the same level of education and practice in primary care or in an oncology practice?

Primary Care

Oncology Practice

Ans: Primary Care

The APRN writes a prescription for trimethoprim-sulfamethoxazole for a patient who has a skin and soft tissue infection and known sulfa allergy. The patient takes two doses of the medication, has no reaction, then realizes this might be one of the problematic antibiotics. The patient notifies the practice and the APRN changes the medication. He makes a full recovery.
Yes or No

1. Is there negligence? __
2. Is there damage? __

1-Yes 2-Yes

1-No 2-Yes

1-Yes 2-No

1-No 2-No

Ans:1-Yes 2-No

Dental care for Medicaid enrollees under the age of 21 years is a basic requirement of the plan.

True

False

Ans: True

The services rendered under Medicaid that is provided by one state can differ considerably in amount, duration, or scope from services provided in similar or neighboring states.

True

False

Ans: True

What type of professional liability insurance policy covers you only if the injury occurs within the policy period and the claim is filed during the period the policy is in effect?

Claims-made

Occurrence

Interval coverage

Incident petition

Ans: Claims-made

Medicaid benefits are paid to the enrollee, then the enrollee is responsible for paying the healthcare bill.

True

False

Ans:

False

A clinical trial is studying whether a vaccine prevents disease. Among those who receive the vaccine, 10% get the disease compared to 40% who received placebo. In this case, the ARR is:

10%

30%

75%

300%

Ans:

30% (40%-10%= 30%) ARR is absolute relative risk reduction or risk difference. If there are 40% who received placebo and only 10% receives the vaccine, then risk difference is remaining 30% of those who do not receive the vaccine or placebo)

The relationship between obesity and type 2 diabetes is best described as:

Causal

Correlational

Indirect

Qualitative

Ans: Causal

A researcher wishes to evaluate the effectiveness of Antimicrobial Z for complicated skin infections by analyzing patient charts over the past 3 years. This is generally considered what type of study?

Double-blind

Single-blind

Prospective

Retrospective

Ans: Retrospective

A person who is Medicaid eligible in one state might be deemed to be ineligible in another state.

True

False

Ans: True

Billing and coding are basically the same thing.

True

False

Ans: False

True/False In keeping with HIPAA’s requirements:

1. You cannot announce a patient’s name in a waiting room.__
2. If interrupted during a patient visit, prior to leaving the exam room, the healthcare provider should lock or otherwise disable the electronic health record (EHR) to minimize the risk of unauthorized access to information.__
3. A healthcare provider can leave a patient a message on his or her answering machine unless specifically asked not to do so but the amount of information disclosed should be limited.__
4. Unless asked not to do so by the patient, a healthcare provider is allowed to leave a message with a family member or other person who answers the phone when the patient is not home.__
5. If interrupted during a patient visit, prior to leaving the exam room, the healthcare provider should take the paper healthcare record with him/her, in order to minimize the risk of unauthorized access to information.__
6. Unless under the control of the healthcare provider or staff, paper records should be kept in a secure location, such as a locked desk, locked filing cabinet or office with appropriate staff.__
7. An employee of a healthcare facility is only able to access patient records for legitimate, job-related purposes.__
8. If a patient requests a paper copy of his/ her record, the healthcare facility can charge a reasonable fee for this service.__

1-False 2-True 3-True 4-True 5-True 6-True 7-True 8-True

1-False 2-False 3-True 4-False 5-True 6-True 7-True 8-True

1-True 2-True 3-True 4-False 5-False 6-True 7-True 8-False

Ans: 1-False 2-True 3-True 4-True 5-True 6-True 7-True 8-True

CPT stands for:

Clinical procedures and techniques

Current procedural terminology

Coding for practice tasks

Codes for price tracking

Ans: Current procedural terminology

A 45-year-old woman calls up your practice and speaks to the triage nurse, stating, “I am going to sue you all. That NP gave me amoxicillin and now I have a have yeast infection.” Does this meet the “damages” standard?

Yes

No

Ans: No

Which information concerning the use of email messages is included in the Health Insurance Portability and Accountability Act?

Email messages are not an acceptable form of communication under any circumstances.

Email messages are suitable only if this communication contains medical record numbers, instead of patient names.

Email messages are acceptable between healthcare providers, but not between healthcare providers and patients.

Email messages containing private patient information are acceptable if the information is encrypted.

Ans: Email messages containing private patient information are acceptable if the information is encrypted.

Which of the following represents a valid ICD 10 code?

370.21

99202

K25.1

AZ5.32

Ans: K25.1

In the preceding question, the dependent variable is the:

iPhone® application.

10% weight loss.

Gender.

Sample Size.

Ans: 10%

diagnostic test that is used to identify patients with colon cancer is determined to have a high sensitivity. Which of the following statements about the test is most accurate?

A negative test result is not a reliable indicator for the absence of colon cancer.

The test reliably provides a negative result for those without colon cancer.

A positive test result reliably identifies someone with colon cancer.

A positive test result is not a reliable indicator for the presence of colon cancer.

Ans: A positive test result reliably identifies someone with colon cancer.

The NP is counseling a 51-year-old male patient who is considering undergoing a PSA test to check for prostate cancer. In explaining the possibility of a false negative result of the test, the NP explains that this can occur when:

A patient without cancer gets a negative test result.

A patient without cancer gets a positive test result.

A patient with cancer gets a negative test result.

A patient with cancer gets a positive result.

Ans: A patient with cancer gets a negative test result.

Evaluation of a diagnostic test for the presence of rheumatoid arthritis is shown to very reliably provide negative results for those without the disease. The test is said to have:

High specificity.

High sensitivity.

Low specificity, high sensitivity.

Low rate of false positives.

Ans: High specificity

A clinical trial is studying whether a vaccine prevents disease. Among those who receive the vaccine, 10% get the disease compared to 40% who received placebo. In this case, the RRR is:

10%

30%

75%

300%

Ans: RRR is the relative risk (10/40) x 100 = 25%; 100-25% = 75%

Of 40% who received placebo, only 10% received the vaccine which is proportionally equated to 0.25% who have prevented from diseases; remaining 75% who do not receive the vaccine can be infected with diseases.

When RRR is > 1, risk of good or bad outcome of the intervention or treatment is increased. When RRR< 1 the risk of good or bad outcome of the intervention is decreased; when RRR = 1 no change.

In the above example, the annual incidence of PAD in this population is approximately:

1%.

10%.

500.

1/2000 persons.

Ans: 1/2000 persons

if there 500 new cases out of 10,000 people with PAD are diagnosed each year. Calculation : 1,000,000 / 500 = 2000 persons

Felicia is 18-weeks pregnant with her first child. Which of the following is most consistent with her gestational age?

Uterine fundus palpable through the abdominal wall

Nausea and breast tenderness

Report of quickening

Fetal heart tones detectable by abdominal Doppler

Ans: Report of quickening

Treatment options for uncomplicated Chlamydiatrachomatis infection in pregnancy include:

Clarithromycin.

Doxycycline.

Azithromycin.

Ofloxacin.

Ans: Azithromycin

When a pregnant woman takes a teratogenic drug, the fetal effects are usually seen in:

Certain target organs in a predictable manner.

Random body systems without a predictable pattern.

Select organ systems with random outcomes.

A pattern of systemic injury.

Ans: Certain target organs in a predictable manner.

Treatment of atopic dermatitis present on the cheeks of an otherwise healthy 4-month-old infant can include all of the following except:

Moisturizing cream to hydrate the skin.

Antihistamines to reduce itching.

Alcohol-based lotion to prevent infection.

Topical corticosteroid to control flares.

Ans: Alcohol-based lotion to prevent infection.

A new family nurse practitioner is beginning a practice in weight management. While reviewing the literature on techniques to promote weight loss, he finds a research report of 30 adult women who recorded their daily caloric intake on a new iPhone® application while on a weight loss plan. The women lost an average of 10% starting weight during the study. In the discussion section of the paper, the author suggested that based on these data, adult women who used this iPhone® application while on a weight loss plan would lose 10% of starting weight. The new FNP questions whether or not the iPhone® application is the cause of the weight loss.

This suggests questionable:

Reliability.

Power.

Confidence level.

External validity.

Ans:

External validity.

In this situation, 30 adult women used Iphone appl. to log in their daily caloric intake has helped these women to lose 10% starting weight. The Iphone appl. is considered an external inference which can be generalised to other situations or outside of the studied population. FNP questions if ” the Iphone appl. is the cause of the weight loss for other people as well?”

The most likely diagnosis is:

(Please reference the previous question).

Hemangioma.

Port wine lesion.

Melanoma.

Erythema toxicum neonatorum.

Ans:

Hemangioma.

For the newborn described above, you advise:

A referral to a dermatologist who specializes in pulse dye laser therapy.

These lesions will likely lighten over time and disappear during childhood.

Evaluation at 6-month intervals due to increased malignancy potential.

Initiating a course of corticosteroid cream for the affected area.

Ans: These lesions will likely lighten over time and disappear during childhood.

In a population of 1 million people, 10,000 are diagnosed with peripheral arterial disease (PAD) and 500 new cases are diagnosed each year. The prevalence of PAD in this population is:

1%.

10%.

500.

1/2000 persons.

ans: 1%
10,000/1,000,000= 0.001 x 100% = 1%

A 21 year old woman with normal BMI prior to pregnancy is now 38-weeks pregnant. Her weight gain up to 28 weeks was 12 lbs (5.45 kg) and she has gained 2 lbs (0.9 kg) from 29 weeks to 38 weeks. Her weight gain pattern is most likely to affect her baby’s:

Birth weight.

Head circumference.

Birth length.

Apgar scores.

Ans: Birth weight

A 2-week-old boy is presented by his mother for evaluation. The boy was born at 39 weeks’ gestation and appears healthy. The mother expresses concern about an area of abnormal growth under his left ear. She states that he was born with a pinkish “blotch” in the area but has noticed a significant change over the past few days. Upon examination, you note a red slightly-protruding spongy mass approximately 0.5 cm in diameter.

For the infant described above, an appropriate initial treatment option would be:

Corticosteroid injection.

Oral propranolol.

Mohs surgery.

Laser therapy.

Ans: Oral propranolol.

Parents of Asian ancestry brings in her healthy 1-month-old daughter born at 40-weeks’ gestation for a well child evaluation. Upon examination, you notice multiple non-tender blue-black macular lesions on the lower back and buttocks.

The most likely diagnosis is:

Port wine lesion.

Milia.

Keratosis pilaris.

Mongolian spot.

Ans: Mongolian spot

When counseling a patient with hypertension and/or dyslipidemia on recommended physical activity, all of the following are appropriate goals to strive for except:

Aerobic physical activity (e.g., brisk walking) of at least 40 minutes at a time.

Exercise of at least 3‒4 times per week.

Achieving a peak heart rate of 150 BPM or greater.

No more than 48 hours without exercise.

Ans: Achieving a peak heart rate of 150 BPM or greater.

A 32-year-old landscaper with a history of seasonal allergic rhinitis presents with a variety of symptoms. Match each symptom with the appropriate medication.

A. Oral antihistamine
B. Decongestant nasal spray
C. Corticosteroid nasal spray

1. Acute relief of nasal congestion __
2. Relieve pharyngeal itch __
3. Prevent allergy symptoms __

1-A 2-B 3-C

1-C 2-B 3-A

1-B 2-A 3-C

1-A 2-C 3-B

Ans: 1-B (decongestant nasal spray) 2-A (oral histamine) 3-C ( prevent allergy symptoms)

Match each of the following patients with the most appropriate diagnosis.
A. Increased intracranial pressure
B. Migraine with aura
C. Tension-type headache
D. Cluster headache
1. A 33-year-old man who reports a 3-week history of recurrent headaches that awaken him during the night. Pain lasts up to 2 hours and is focused primarily behind the right eye. __
2. A 29-year-old woman with a 2-year history of recurrent, unilateral, pulsating headaches that are often accompanied by nausea/vomiting and photophobia. She reports seeing “squiggles before my eyes” about 10 minutes before a headache occurs. __
3. A 54-year-old man reporting occasional bilateral, pressing, nonpulsatile headaches of moderate intensity without nausea, photophobia, or phonophobia. __
4. A 45-year-old man who reports a pressing, nonpulsatile headache that is greatest in severity upon awakening each morning and lessens as the day progresses. __

1-A 2-C 3-B 4-D

1-D 2-B 3-C 4-A

1-B 2-A 3-D 4-C

Ans: 1-cluster head ( 4. A 45-year-old man who reports a pressing, nonpulsatile headache that is greatest in severity upon awakening each morning and lessens as the day progresses)

2- Migraine w aura (A 29-year-old woman with a year history of recurrent, unilateral, pulsating headaches that are often accompanied by nausea/vomiting and photophobia. She reports seeing “squiggles before my eyes” about 10 minutes before a headache occurs.

3-tension-type h/a (A 33-year-old man who reports a 3-week history of recurrent headaches that awaken him during the night. Pain lasts up to 2 hours and is focused primarily behind the right eye)

4- Increased intracranial pressure ( A 45-year-old man who reports a pressing, nonpulsatile headache that is greatest in severity upon awakening each morning and lessens as the day progresses).

Type 2 diabetes mellitus is best described as:

A genetically-base autoimmune process.

A consequence of obesity.

A condition largely dictated by environmental factors.

A disorder of insulin resistance with eventual insulin deficiency.

Ans: A disorder of insulin resistance with eventual insulin deficiency.

When evaluating a patient with low back pain without radiculopathy, the nurse practitioner will most likely identify:

Discomfort to direct palpation of paraspinal muscles.

A positive straight leg raise.

Abnormal lower extremity DTR.

Pain relief with lumbar flexion.

Ans: When evaluating a patient with low back pain without radiculopathy, the nurse practitioner will most likely identify:

When a patient is seen by a nurse practitioner, the visit can be charged to Medicare using “incident-to” billing. If the criteria for this type of visit are fulfilled, then anticipated reimbursement is at what percentage of the physician rate?

75

80

90

Ans: 100%

Which of the following is a potential adverse effect of chronic use of higher-potency topical corticosteroids?

Atrophy in intertriginous areas

Epidermal sloughing

Cutaneous fungal infection

Acne rosacea

ans:

Atrophy in intertriginous areas

An appropriate treatment option for community-acquired pneumonia in a 45-year-old woman with no significant comorbidities, no recent history of antimicrobial use (>3 months), and no known drug allergies is:

Ampicillin.

Doxycycline (Doryx®).

Cefuroxime (Ceftin®).

Moxifloxacin (Avelox®).

Ans: Doxycycline (Doryx®).

While evaluating a patient with psoriasis vulgaris, the nurse practitioner expects to find:

Lichenification in the post-auricular region.

Maculopapular lesions over the upper thorax.

Scaling lesions within the nasolabial folds.

Well-demarcated plaques and coalescing papules on the knees.

Ans: Well-demarcated plaques and coalescing papules on the knees.

When initiating pharmacologic therapy for the patient with first diagnosis of major depression disorder, the nurse practitioner advises the patient that treatment should:

Generally be given for about 4-6 months before improvement is noted.

Continue for at least 6 months after remission is achieved.

Be continued indefinitely after therapeutic goals are met.

Be started at a higher dose then titrated to a lower dose once symptom relief is achieved.

Ans: Continue for at least 6 months after remission is achieved.

our neighbor asks you to refill her high blood pressure medicine as she could not make her last office visit to her provider due to a family emergency. You are aware that she is going through a difficult time personally and agree to call in the prescription. As a result:

A healthcare provider-patient relationship has been formed.

The Good Samaritan Act will protect you from liability since you volunteered to help her out.

You should charge your neighbor for a standard office visit in order to keep your relationship with her on a professional level.

You realize this action is acceptable as her antihypertensive medication is not a controlled substance.

Ans: A healthcare provider-patient relationship has been formed.

Gary is a 26-year-old graduate student who is diagnosed with major depressive disorder (MDD) without psychotic features. His medical history is unremarkable and he states that he does not wish to undergo psychotherapy. An appropriate first-line medication is:

Sertraline (Zoloft®).

Bupropion (Wellbutrin®).

Amitriptyline (Elavil®).

Trazodone (Oleptro®).

Ans: Sertraline (Zoloft®).

Which of the following is a typical physical finding in a patient with carpal tunnel syndrome?

Decreased vibratory sense of distal phalanges

Hyperactive radial reflex

Reproduction of symptoms with forced flexion of the wrists

Positive Allen’s test

Ans:

Reproduction of symptoms with forced flexion of the wrists

The nurse practitioner is evaluating laboratory studies in a patient with newly-diagnosed type 2 diabetes mellitus. Which lipid abnormality is most likely?

Low total cholesterol

High triglycerides

Low LDL-C

High HDL-C

Ans: High triglycerides

You ask a patient to follow your finger, while not moving her head, through the positions of gaze. This is in part an evaluation of cranial nerves:

II, III, VI.

I, II, III.

V, VI, VII.

III, IV, VI.

Ans: III, IV, VI.
I – oldfactory nerve -smell – SENSORY
II – optic (vision) -MOTOR- test for accommodation, visual field, pupillary reaction
III- oculomotor nerve – eye move up or down; outward or inward; drooping lid-reflag. MOTOR
IV- trochlear nerve- eye movement down and inward – MOTOR
V- trigeminal nerve – corneal blink reflex, sensation (wisp of cotton, light vs. dull touch) BOTH
VI- Aducens nerve- eye movements (ability to move eyes bilaterally) MOTOR
VII- Facial nerve – facial expression, clench teeth, puff cheeks out, production of saliva and tears, raise eyebrown, close eyes against resistance- MOTOR
Usually V and VII are tested together.
VIII- Acoustic nerve- hearing -Wiber test (lateralization) Rinne test (air vs bone conduction) SENSORY
IX- glossaphareangeal – test gag reflex (say Ahh)- BOTH
X- Vagus nerve- swallowing or gag reflex, voice – BOTH
XI- Spinal accessory – test shoulder movements like shrugging, push up against resistance – MOTOR
XII- Hypoglossal – tongue movements (sticks tongue out, move tongue side-to-side) MOTOR

A 37-year-old female presents with flank pain, fever, and vomiting. Physical examination is significant for costovertebral angle tenderness. A urinalysis is likely to reveal:

Urobilinogen.

Specific gravity <1.010.

White blood cell (WBC) casts.

Ketones.

Ans: White blood cell (WBC) casts.

The nurse practitioner is seeing a patient who is starting on an HMG-CoA reductase inhibitor. Laboratory assessment should include an initial measurement of:

LDH.

ALT.

ALP.

GGT.

Ans: ALT.

The most important option for the primary prevention of shingles is the use of:

Varicella zoster immune globulin.

Zoster vaccine.

High-dose acyclovir at the onset of an outbreak.

A tricyclic antidepressant post zoster episode.

ans: Zoster vaccine.

Mr. Woods is a 63-year-old man who presents for initial evaluation at your practice. He has a 45 pack-year smoking history and COPD that is currently being treated with daily oral theophylline and albuterol via MDI prn. He complains of shortness of breath and chronic cough with sputum production. He states, “I can barely walk to my car without getting out of breath.” Spirometric assessment reveals an FEV1 of 45% predicted and a FEV1:FVC of 62%. His health history reveals 3 exacerbations in the past year. The best course of action for Mr. Woods to minimize COPD exacerbation risk is to:

Continue on theophylline therapy.

Advise using albuterol on a set schedule rather than prn.

Switch to a long-acting beta2-agonist as needed for shortness of breath.

Add therapy with a long-acting muscarinic antagonist uses on a set schedule.

Ans: Add therapy with a long-acting muscarinic antagonist uses on a set schedule.

The nurse practitioner is evaluating a 34-year-old male who reports a 3-month history of epigastric pain that he describes as intermittent, burning, and improves with eating then returns 2-3 hours after meals. The leading diagnosis is:

Cholecystitis.

Gastritis.

Esophageal reflux.

Duodenal ulcer.

Ans: Duodenal ulcer.

The nurse practitioner is evaluating a patient with recurrent heartburn. When considering gastroesophageal reflux as a differential diagnosis, all of the following symptoms are anticipated except:

Eructation.

Cough.

Dysphonia.

Hematemesis.

Ans: Hematemesis.

While counseling a patient about the adverse effects of first-generation antihistamines, the nurse practitioner advises that all of the following usually occur except:

Sedation.

Dry mouth.

Pruritus.

Urinary retention.

Ans: Pruritus.

A 20-year-old male complains of being fatigued for the past week. The physical examination is significant for anterior and posterior cervical lymphadenopathy and right and left upper quadrant abdominal tenderness. A white blood cell differential is significant for lymphocytosis with reactive forms. Which of the following additional findings does the nurse practitioner anticipate?

Pharyngitis with exudate

Supraclavicular lymphadenopathy

Diffuse maculopapular rash

Marked sinus tenderness

Ans: Pharyngitis with exudate

A 33-year-old woman comes to your office and reports that she is “having a stroke.” She has no medical history and no significant risk factors for cerebrovascular disease, but today she has sudden onset of inability to tightly close her eyelid and to frown or smile on the right side of her face. Her physical examination is otherwise unremarkable. The nurse practitioner recognizes that this likely represents paralysis of cranial nerve (CN):

IV.

V.

VI.

VII.

Ans:

VII.

Laboratory assessment of a patient with Hashimoto’s thyroiditis typically includes the presence of:

Polycythemia.

TPO antibodies.

Leukocytosis.

Hypokalemia.

Ans: TPO antibodies. (thyroid protease antibodies)

The nurse practitioner knows that all patients with an acute eye complaint should have:

Dilated pupil examination.

Assessment of visual acuity.

Everted lid examination.

Fluorescein staining.

Ans: Assessment of visual acuity.

One of the preferred pharmacologic options of persistent cough in an otherwise well 25-year-old woman with acute uncomplicated bronchitis would most likely include:

Antibiotic therapy to cover atypical pathogens.

An inhaled short-acting muscarinic antagonist.

An inhaled corticosteroid.

A dextromethorphan-based cough suppressant.

Ans: An inhaled short-acting muscarinic antagonist.

The primary complaint of a patient with bacterial vaginosis is usually:

Copious discharge.

Painful intercourse.

Intense pruritus.

Profound vaginal odor.

ans: Profound vaginal odor.

The Federal Drug Enforcement Administration (DEA) provides registration numbers for prescription of controlled substances to which of the following groups?

NPs with federal prescription authority

Physicians, veterinarians, dentists, and NPs who practice in oncology or palliative care

NPs with state-determined controlled substance prescriptive authority

Nationally-certified APRNs with state licensure

Ans:

NPs with state-determined controlled substance prescriptive authority

In initial prescription of thyroxine therapy, which of the following statements is true?

TSH should be checked about 2 days after dose adjustment.

The anticipated thyroxine dose is usually calculated using an age- and weight-based formula.

TSH should be suppressed to undetectable measures when seeking therapeutic effect.

The therapeutic dose of thyroxine needed by the elderly is approximately the same as that needed by the younger adult and child.

Ans: The anticipated thyroxine dose is usually calculated using an age- and weight-based formula.

A 38-year-old woman presents complaining of weight loss, agitation, and new-onset tremor. The nurse practitioner suspects hyperthyroidism. Laboratory assessment will most likely include:

TSH=8.9 mIU/L (0.4-4.0 mIU/L), free T4 =15 pmol/L (10-27 pmol/L).

TSH<0.15 mIU/L (0.4-4.0 mIU/L), free T4 =79 pmol/L (10-27 pmol/L).

TSH=24 mIU/L (0.4-4.0 mIU/L), free T4 =3 pmol/L (10-27 pmol/L).

TSH=1.9 mIU/L (0.4-4.0 mIU/L), free T4 =22 pmol/L (10-27 pmol/L).

Ans: TSH<0.15 mIU/L (0.4-4.0 mIU/L), free T4 =79 pmol/L (10-27 pmol/L).

Clinical evaluation of a 43-year-old man with a BMI=42 kg/m2 who presents with recurrent genital Candida albicans infection should include all of the following except:

Treatment with oral or topical antifungals.

Offering testing for HIV antibodies.

Avoidance of soap on the genitals.

Checking A1C.

Ans: Avoidance of soap on the genitals.

Primary care of a person with alpha thalassemia minor should include:

Routine iron supplementation.

Offering genetic counseling prior to pregnancy.

Periodic evaluation of hemoglobin electrophoresis.

Folate supplementation.

Ans:

Offering genetic counseling prior to pregnancy.

Physical examination of a patient with symptomatic Graves’ disease will likely reveal:

Generalized hypopigmentation.

Tremor.

Blunted pupil response.

Thinning hair.

Ans: Tremor.

First-line therapy to add for a patient with Stage 3 heart failure and preserved left ventricular ejection fraction is:

A statin.

An alpha blocker.

Digitalis.

A diuretic.

ans: A diuretic.

Which of the following are potential candidates to receive the PCV13 pneumococcal vaccine? (YES or NO)

1. A 67-year-old man who received the PPSV23 vaccine 1 year ago __
2. A healthy 58-year-old woman who is receiving the seasonal influenza vaccine today __
3. A 48-year-old man who uses smokeless tobacco __
4. A 36-year-old woman with HIV infection who received the PPSV23 vaccine 3 years ago __

1-Yes 2-No 3-Yes 4-No

1-No 2-No 3-Yes 4-No

1-Yes 2-No 3-No 4-No

1-Yes 2-No 3-No 4-Yes

Ans: 1-Yes 2-No 3-No 4-Yes

he preferred therapeutic option to resolve the underlying pathology in Graves’ disease is:

Non-cardioselective beta blockade.

Surgical resection of hyperfunctioning nodule.

Radioactive ablation of overactive thyroid tissue.

Levothyroxine suppression therapy.

Ans: Radioactive ablation of overactive thyroid tissue.

Thyroid stimulating hormone (TSH) levels are suppressed in all of the following except:

Graves’ disease.

Autonomous toxic nodule.

Acute viral thyroiditis.

Subacute hypothyroidism.

ans: Subacute hypothyroidism.

A 54-year-old man with COPD is experiencing an exacerbation characterized by increased cough and shortness of breath for the past 48 hours, as well as mild sore throat, clear nasal discharge and body aches. He is currently being managed with a long-acting beta2-agonist and long-acting muscarinic antagonist . An appropriate treatment for this patient would include:

Oral amoxicillin/clavulanate.

Oral levofloxacin.

Oral corticosteroid.

Theophylline.

Ans:

Oral corticosteroid.

Which of the following is the best choice of antimicrobial therapy for a skin and soft tissue infection caused by community-acquired methicillin-resistant Staphylococcus aureus (MRSA)?

Dicloxacillin

Cefadroxil

Trimethoprim-sulfamethoxazole

Amoxicillin-clavulanate

ans: Trimethoprim-sulfamethoxazole

The mechanism of action of saxagliptin includes:

Increase in hepatic glucose utilization.

Enhanced insulin release.

Facilitation of renal glucose excretion.

Enhancement of insulin receptor site activity.

Ans: Enhanced insulin release.

Metformin – Biguanide – 1st line therapy- insulin sensitizer; neutral weight loss. Complication: can cause GI symptoms (diarrhea) or renal acidosis; monitor renal function.
Glipizide, glipizide..- Sulfonuleas class- constant insulin release- causes weight gain, high risk of hypoglycemia, monitor in adult patients.
Sitagliptin- DDP-4 (-gliptins) – insulin release post glucose rise in the body- neutral weight control- rare side effects
Pioglitazones (TZDs)- insulin sensitizers- modest gain weight, avoid use in CHF causing edema, increase risk of fractures.
Exenatide (SQ Injection)- insulin release post glucose rise- GI symptoms
Canagliflozin – SGLT2- glucose excretion (through urine) post glucose rise- modest weight loss, increase chance of urethral/vaginal yeast infection, dehydration or fractures, monitor renal function.

Who among the following patients is at the greatest risk for suicide?

A 95-year-old female whose only child died last week

An 85-year-old male who lives alone

A 15-year-old female who has been using marijuana

A 12-year-old male who is questioning his gender identity

Ans: An 85-year-old male who lives alone

In a healthy person, approximately what percentage of the body’s total daily physiologic insulin secretion is released basally?

10%

25%

50%

75%

Ans:

50 %

You see a 29-year-old woman with a BMI=38 kg/m2 and hyperpigmented plaques with velvet-like appearance at the nape of the neck and axilla (see image).
Laboratory assessment should include which of the following tests?

Hemoglobin A1C

Serum electrolytes

HIV antibody screening

Hemogram

Ans: Hbg A1C

Which of the following is most consistent with the diagnosis of depression?

Recurrent diarrhea and cramping

Difficulty initiating sleep

Diminished cognitive ability

Consistent early morning wakening

Ans :

Consistent early morning wakening

A 52-year-old woman presents to your office as a new patient. She reports a long history of high blood pressure, and admits she does not regularly take her medication. Which of the following is an anticipated funduscopic finding?

Narrowing of arterioles

An increased cup:disk ratio

Macular degeneration

Cotton-wool spots

Ans: Narrowing of arterioles

Performing Weber and Rinne tests is part of the evaluation of cranial nerve (CN):

V.

VI.

VII.

VIII.

Ans: VIII.

All of the following are examples of environmental control for a patient with allergic rhinitis except:

Keeping pets out of the bedroom.

Laundering sheets in hot water.

Encasing pillows in plastic covers.

Keeping bedroom windows open in the early morning.

ans: Keeping bedroom windows open in the early morning.

A 22-year-old man presents for hepatitis screening. He is without symptoms but needs the testing for a job in the food service industry.

Laboratory results are as follows:
-Anti-HCV with HCV RNA present
-HBsAg=positive
-Anti-HAV=negative

You advise the patient that he:

Has chronic hepatitis A, B, and C.

Is immune to hepatitis A and B but has hepatitis C.

Has chronic hepatitis B and C and needs immunization against hepatitis A.

Had hepatitis A in the past, is immune to hepatitis B, and has chronic hepatitis C.

Ans: Has chronic hepatitis B and C and needs immunization against hepatitis A.

A 41-year-old woman was recently diagnosed with systemic lupus erythematous. She complains of feeling generally fatigued and sometimes palpitations.

Her hemogram results are as follows:
-Hb=9.5 g/dL (95 g/L)
-Hct=28% (.28 proportion)
-MCV=86 fL
-RDW= 12.5%
-Reticulocytes=0.9% (.09 proportion)
These findings are most consistent with:

Iron deficiency anemia.

Anemia of chronic disease.

Pernicious anemia.

Alpha thalassemia minor.

Ans: Anemia of chronic disease.

The process of absorption, distribution, metabolism, and elimination of a drug is known as:

Pharmacokinetics.

Therapeutic transformation.

Pharmacodynamics.

Drug interactions study.

ans:

Pharmacokinetics.

Which of the following is true concerning acute bacterial prostatitis?

Gram-positive organisms are the most common cause of infection.

Length of antibiotic therapy is usually 1 week.

Perineal pain with defecation is a common complaint.

Cephalosporins are first-line therapy.

Ans:

Perineal pain with defecation is a common complaint.

A 22-year-old man presents with new-onset pain and swelling in the feet and ankles as well as conjunctivitis, oral lesions, and dysuria. The most important test to obtain is:

Rheumatoid factor.

Sedimentation rate.

Urethral cultures.

Antinuclear antibody.

Ans: Urethral cultures.

Laboratory assessment of a patient with subclinical hypothyroidism is likely to reveal:

TSH=0.1 mIU/L (0.4-4.0 mIU/L); total T4=7.2 mcg/dL (4.5-12.0 mcg/dL).

TSH=2.6 mIU/L (0.4-4.0 mIU/L); total T3=310 ng/dL (95-190 ng/dL).

TSH=7.7 mIU/L (0.4-4.0 mIU/L); free T4=22 pmol/L (10-27 pmol/L).

TSH=0.2 mIU/L (0.4-4.0 mIU/L); free T3=1.7 ng/dL (0.2-0.5 ng/dL).

Ans: TSH=7.7 mIU/L (0.4-4.0 mIU/L); free T4=22 pmol/L (10-27 pmol/L).

A 45-year-old man complains of bilateral, intermittent itchy eyes, often accompanied by rope-like discharge and without vision change or eye pain. The nurse practitioner expects that the history will include a patient report of:

Trauma to the eye.

Sexually transmitted infection.

Seasonal allergies

Decreased visual acuity.

Ans:

Seasonal allergies

Which of the following is most consistent with the diagnosis of herpes zoster?

Papules in a dermatomal distribution

Low-grade fever and malaise prior to eruption

Presence of pustules and honey-crusted lesions

A 1-2-day prodrome of pain prior to lesions erupting

ans: A 1-2-day prodrome of pain prior to lesions erupting

One week into sertraline (Zoloft®) therapy, a 34-year-old woman complains of a recurrent dull frontal headache that is relieved with acetaminophen. You advise her that:

This is a common, transient adverse effect of SSRI therapy.

Bupropion (Wellbutrin®) should be substituted.

Desipramine (Norpramin®) should be added.

She should discontinue the medication.

Ans: This is a common, transient adverse effect of SSRI therapy.

When prescribing an SSRI, the nurse practitioner considers that:

There will be full therapeutic effect within 2-3 days of the first dose.

These drugs are potentially photosensitizing.

These medications cannot be used in patients with cardiovascular disease.

Mild GI upset is often reported in first weeks of use.

Ans: Mild GI upset is often reported in first weeks of use.

A 52-year-old man presents for evaluation of fatigue. His physical examination is significant only for intention tremor and asymptomatic pharyngeal redness. He reports a history of recurrent gastritis.

Laboratory examination results are as follows:
-Hb=14.3 g/dL (143 g/L)
-Hct=48% (.48 proportion)
-MCV=109 fL
-HDL-C=58 mg/dL (1.5 mmol/L)
-LDL-C=118 mg/dL (3.06 mmol/L)
-Triglycerides=318 mg/dL (3.59 mmol/L)
These findings are most consistent with:

Pernicious anemia.

Iron deficiency anemia.

Alcohol abuse.

Normal findings.

Ans: Alcohol abuse.

Which of the following is an absolute contraindication to post-menopausal hormone therapy?

Unexplained vaginal bleeding

Seizure disorder

Endometriosis

Cigarette smoking

Ans: Unexplained vaginal bleeding

A 27-year-old woman presents for care. She reports a 3-day history of intense vaginal itch, burning with urination, and white, clumping discharge. She denies a foul odor but says her genitals “smell a little musty.” Which of the following history findings supports your diagnosis?

New sexual partner

Swimming in a local lake

Recent antibiotic use

Recent insertion of levonorgestrel-releasing IUD

Ans: Recent antibiotic use

A 32-year-old woman presents with a recent onset of copious, green-yellow vaginal discharge. Physical examination reveals “strawberry spots” on the cervix. The NP anticipates finding the following on microscopic examination.

ans: Trichomoniasis (pics on the desk top)
flagella tail bacteria

A 36-year-old man presents with a 12-h history of anorexia, nausea, and right lower quadrant abdominal pain.

A white blood cell count with differential demonstrates:
-TWBC=16,500 cells/mm3
-Neutrophils=66%
-Bands=8%
-Lymphocytes=22%
Expected physical examination findings include:

Murphy’s sign.

A palpable left lower quadrant mass.

Periumbilical ecchymosis.

A positive obturator sign.

Ans:

A positive obturator sign (appendicitis)

When counseling patients to promote hepatitis C prevention, the nurse practitioner realizes the most important measure from the list below is:

Safe sex practices.

Using single-use injection drug paraphernalia.

Strict handwashing.

Avoidance of raw seafood ingestion.

Ans: Using single-use injection drug paraphernalia.

36-year-old man with asthma has been treating his asthma with albuterol once or twice a week for the last few years. He reports that for the last month or so he has had to use it much more often—”not every day, but almost.” The nurse practitioner should consider all of the following except:


Administering the Asthma Control Test questionnaire.

Spirometric assessment.

A complete symptom assessment.

Adding an inhaled long-acting beta2-agonist.

Ans: Adding an inhaled long-acting beta2-agonist.

A 40-year-old man complains of a recurrent dry cough. He is not a smoker, has no airway disease, and his history is significant only for recently diagnosed hypertension. He admits having just started taking medication for his high blood pressure, but doesn’t know what it is called. The nurse practitioner suspects that he is most likely taking a(n):

Angiotensin-converting enzyme (ACE) inhibitor.

Calcium channel antagonist.

Angiotensin receptor blocker.

Beta-adrenergic antagonist.

Ans: Angiotensin-converting enzyme (ACE) inhibitor.

A characteristic feature of erythema migrans is:

Blanching to pressure.

Central clearing.

Rapid response to topical corticosteroids.

Intense pruritus.

Ans: Central clearing.

A patient with hypothyroidism is being started on thyroid hormone replacement therapy. The nurse practitioner advises that the medication should be taken:

With 8 oz (236.6 mL) of milk.

On an empty stomach.

At bedtime.

Before a meal.

Ans: On an empty stomach.

A 47-year-old man of Northern European ancestry presents for evaluation of erythematous, papular lesions on his forehead and chin. Which of the following differential diagnoses is most likely?

Acne rosacea

Tinea facialis

Hidradenitis suppurativa

Atopic dermatitis

Ans: Acne rosacea

A 51-year-old woman presents with a 1-day history of fever, abdominal pain, and vomiting. The diagnosis of acute cholecystitis would be confirmed by:

Isolated lymphocytosis.

Elevated amylase and lipase.

Right upper quadrant abdominal ultrasound.

Abdominal plain films.

Ans: Right upper quadrant abdominal ultrasound.

Actinic keratoses can be described as:

Hyperpigmented macules on sun-exposed areas.

Vesicular lesions along a dermatomal distribution.

Ulcerating lesions in groin folds.

Slightly rough, pink or flesh-colored lesions on the face.

Ans: Slightly rough, pink or flesh-colored lesions on the face.

A 52-year-old female of European ancestry with a BMI=24 kg/m2 is diagnosed with new-onset hypertension. She has no significant medical history, no concomitant health problems, and takes no medications. Along with therapeutic lifestyle changes, therapy should begin with which of the following classes of drug?

Aldosterone antagonist

Beta-adrenergic antagonist

Centrally-acting agent

Thiazide diuretic

Ans:

Thiazide diuretic

A 53-year-old woman complains of vaginal dryness and discomfort during sexual activity . She feels like she might have a urinary tract infection, but the urinalysis is negative. Pelvic examination reveals a lack of vaginal rugae. The nurse practitioner considers prescribing:

Antimicrobial therapy.

Oral metronidazole.

Vaginal antifungal.

Topical estrogen.

Ans:

Topical estrogen.

A 54-year-old woman presents for evaluation of a “cold sore” on her tongue. She has never had one in the past. She denies any pain, drainage, foul taste, or any other symptoms—just has an ulcer that has been there “for a while.” The history is significant for a 62 pack-year smoking history. Physical examination reveals an ulcerated lesion with firm, palpable, indurated margins. The most likely diagnosis is:

Aphthous stomatitis.

Foreign body.

Herpangina.

Squamous cell carcinoma.

Ans:

Squamous cell carcinoma.

Gastroparesis is a contraindication to the use of which of the following medications in type 2 diabetes mellitus?

Exenatide (Byetta®)

Pioglitazone (Actos®)

Sitagliptin (Januvia®)

Glyburide (Glynase®)

Ans: Exenatide (Byetta®)

A new 27-year-old female patient presents for a health maintenance examination as required by her employer. She tells you that she has had a heart murmur “all of her life” and has been told that it is an “innocent” murmur. Anticipated findings include:

A harsh late systolic snap.

A soft systolic murmur that disappears when in a standing position.

A diastolic murmur best heard at the base of the heart.

A high-pitched murmur that is synchronous with the carotid pulse.

A soft systolic murmur that disappears when in a standing position.

Which of the following findings is most consistent with a diagnosis of acute bacterial rhinosinusitis?

Fever >100.7ᵒF (38.2ᵒC)

Upper respiratory tract infection symptoms persisting for ≥10 days

Concomitant presence of facial pressure

Copious amounts of nasal discharge

Ans: Upper respiratory tract infection symptoms persisting for ≥10 days

A patient presenting with Graves’ disease will most likely demonstrate which laboratory finding?

Free T4 =3 pmol/L (NL=10-27 pmol/L)

TSH=0.04 mIU/L (NL=0.15-4.0 mIU/L)

ESR=42 mm/h (NL <15 mm/h)

TWBC=12,400 cells/mm3 with 70% neutrophils

Ans: TSH=0.04 mIU/L (NL=0.15-4.0 mIU/L)

The most important aspect of skin care for a person with eczema is:

Use of occlusive dressings.

Frequent bathing with antibacterial soap.

Aggressive towel drying after bathing.

Frequent application of lubricants.

Ans: Frequent application of lubricants.

A patient with newly-diagnosed type 2 diabetes mellitus who has an A1C=8.2% will most likely begin therapy with a medication from which of the following drug classes?

GLP-1 agonist such as exenatide

DPP-4 inhibitor such as saxagliptin

Thiazolidinedione such as pioglitazone

Biguanide such as metformin

Ans: Biguanide such as metformin

You are examining a 21-year-old man who is seeking a sports clearance physical examination. He is generally healthy with an unremarkable medical history. During cardiovascular examination, you identify a physiologic split S2 and realize that this:

Is usually first noticed in early adulthood.

Can progress to a pathologic condition during adulthood.

Will be more pronounced with inspiration.

Is associated with S4 heart sound.

Ans: Will be more pronounced with inspiration.

A prostate cancer lesion detectable by digital rectal examination will likely be described as:

A hard, immobile induration.

A bilateral enlargement.

A painful flocculent t lesion.

A rubbery gland with central depression.

Ans: A hard, immobile induration.

The nurse practitioner has diagnosed acute bacterial rhinosinusitis (ABRS) and is considering antibiotic choices. He recognizes that the most common causative pathogen is:

H. influenzae.

S. aureus.

S. pneumoniae.

M. catarrhalis.

Ans: S. pneumoniae.

Actinic keratosis can be a precursor to:

Malignant melanoma.

Acanthosis nigricans.

Basal cell carcinoma.

Squamous cell carcinoma.

Ans: Squamous cell carcinoma.

An 18-year-old male presents with profound scrotal pain. Which of the following findings supports a diagnosis of testicular torsion?

Positive Prehn’s sign

High fever

Profound neutrophilia

Loss of the cremasteric reflex

Ans: Loss of the cremasteric reflex

The nurse practitioner is beginning pharmacologic management of dyslipidemia in a 47-year-old male patient whose lipid panel is as follows:

-Total cholesterol=249 mg/dL (6.45 mmol/L)
-HDL-C=39 mg/dL (1.01 mmol/L)
-TG=279 mg/dL (3.15 mmol/L)
-LDL-C=191 mg/dL (4.94 mmol/L)

The appropriate class of agent is a(n):

Omega-3 fatty acid such as fish oil.

Fibric acid derivative such as fenofibrate.

Cholesterol absorption inhibitor such as ezetimibe.

HMG-CoA reductase inhibitor such as simvastatin.

Ans: HMG-CoA reductase inhibitor such as simvastatin.

In keeping with HIPPA’s requirements, all of the following are true except:

The healthcare provider or designate cannot announce a patient’s name in a waiting room.

If interrupted during a patient visit, prior to leaving the examination room, the healthcare provider should lock or otherwise disable the electronic health record (EHR) to minimize the risk of unauthorized access to information.

An employee of a healthcare facility is only able to access patient records for legitimate, job-related purposes.

If a patient requests a paper copy of his/her record, the healthcare facility can charge a reasonable fee for this service.

ans: The healthcare provider or designate cannot announce a patient’s name in a waiting room.

Managed care is best defined as a:

Collaborative approach to provide coordination of the healthcare services to a defined population.

System for determining what healthcare services are reimbursable.

Process of reviewing the appropriateness of select healthcare interventions.

Method of review to ensure adherence to evidence-based practice.

Ans: Collaborative approach to provide coordination of the healthcare services to a defined population.

Meeting the eligibility requirements for national NP certification signifies that the nurse practitioner:

Attained the required academic and clinical skills for specialty practice.

Graduated from an accredited nurse practitioner educational program.

Has achieved select prerequisites determined by a nongovernmental professional organization.

Has permission to practice as a nurse practitioner.

Ans: Has achieved select prerequisites determined by a nongovernmental professional organization.

Metformin is contraindicated in the patient with a GFR <45 mL/min/1.73 m2 because of the risk of:

Hypokalemia.

Worsening renal function.

Hypocalcemia.

Lactic acidosis.

Ans: Lactic acidosis.

Post-menopausal hormone replacement therapy is indicated for:

Prevention of cerebrovascular events.

Prevention of cardiovascular disease.

Reduction in frequency and severity of vasomotor symptoms.

Reduction of cardiovascular events.

Ans: Reduction in frequency and severity of vasomotor symptoms.

A 38-year-old woman reports having 3-5 migraine episodes each month that can last 1-3 days. Her medical history is otherwise unremarkable other than bilateral tubal ligation (BTL) 6 years ago. She asks if there is any medication that can help to prevent these migraines. An appropriate prophylactic treatment options to offer this patient is:

Ergotamine (Ergomar®).

Sumatriptan (Imitrex®).

Topiramate (Topamax®).

Verapamil (Verelan).

ans: Topiramate (Topamax®

Radiographic assessment of a patient with early osteoarthritis will most likely reveal:

Soft tissue swelling.

Involvement of the metacarpals.

Osteophyte formation.

Joint space narrowing on X-ray.

Ans: Joint space narrowing on X-ray.

Risk factors for the development of type 2 diabetes in women include:

History of early menarche.

Cigarette smoking.

Personal history of polycystic ovary syndrome.

Caucasian race.

Ans: Personal history of polycystic ovary syndrome.

Sulfonylureas are characterized by their impact on:

Insulin release.

Insulin receptor site activity.

Hepatic glucose production.

Renal glucose excretion.

Ans:

Insulin release.

The difference between a macule and a patch is:

Location.

Disease state.

Size.

Configuration.

Ans:

Size.

he mechanism of action of a GLP-1 agonist includes:

Attenuation of insulin resistance.

Promotion of renal glucose excretion.

Increase in insulin release in response to rising glucose.

Decrease in gastric emptying.

Ans: Increase in insulin release in response to rising glucose.

The typical international normalized ratio (INR) goal during warfarin therapy for a 65-year-old patient with atrial fibrillation is:

1.0‒2.0.

1.5‒2.5.

2.0‒3.0.

2.5‒3.5.

Ans: 2.0‒3.0.

The murmur of mitral valve insufficiency is best described as:

Mid-late systole with a “snap.”

Diastolic and generalized across the precordium.

Diastolic with radiation to the neck.

Systolic with radiation to the axillae.

Ans: Systolic with radiation to the axillae.

In a middle-aged adult, which of the following results would not indicate a category at increased risk for diabetes mellitus?

A fasting glucose of 109 mg/dL (6.05 mmol/L)

A 1-hour post-prandial glucose of 98 mg/dL (5.44 mmol/L)

A hemoglobin A1C=5.9%

A 2-hour post-prandial glucose of 152 mg/dL (8.44 mmol/L)

Ans:

A 1-hour post-prandial glucose of 98 mg/dL (5.44 mmol/L)

A 64-year-old man presents complaining of difficulty initiating urination, a weak urine stream, and inability to completely empty the bladder. Suspecting benign prostatic hyperplasia, you prescribe tamulosin and advise the patient that this will:

Decrease the size of the prostate.

Relax the bladder neck muscles to ease urination.

Eradicate bacteria that cause inflammation of the prostate.

Reverse hormonal changes that cause prostate enlargement.

Ans: Relax the bladder neck muscles to ease urination.

A 68-year-old woman presents for evaluation. She reports a recent history of fatigue and says she can barely go up the steps without becoming short of breath.
Hemogram results are as follows:
-Hb=8.7 g/dL (87 g/L)
-Hct=27% (.27 proportion)
-MCV=70 fL
-RDW=18% (.18 proportion)

A critical diagnosis to consider is:

Vitamin B12 deficiency.

Folic acid deficiency.

Renal insufficiency.

Gastrointestinal blood loss.

Ans: Gastrointestinal blood loss.

The nurse practitioner is counseling a patient who is new to insulin therapy. The patient is advised that when injecting a short-acting, rapid-onset formulation such as insulin aspart, the greatest risk time for hypoglycemia is:

15-60 minutes after injection.

1-3 hours after injection.

3-4 hours after injection.

4-5 hours after injection.

Ans: 1-3 hours after injection.

Insulin Analog — Aspart and Lispro (short-acting)-onset 15mins, peak 1hr, duration 3-4hrs
Humilin R and Novolin R (regular)- 30mins, peak 2-3hrs, duration 3-6 hrs
NPH (intermediate), Humilin N and Novolin N- 1-2 hrs, peak 6-14hrs, duration 16-24hrs
Long-acting Lantus or Detemir- 1-2 hrs, peak and duration – none

A 66-year-old man presents for a “check-up.” He has no complaints and admits he’s been smoking 2 packs a day for 40 years. Which additional finding would be consistent with a diagnosis of chronic obstructive pulmonary disease?

Markedly increased chest AP diameter

Pleuritic chest pain

FEV1:FVC ratio <0.70 post SABA use

FEV1 improving by 50% with use of SAMA

Ans:

FEV1:FVC ratio <0.70 post SABA use

A 63-year-old patient presents for evaluation of fever. She reports that 3 days ago she went to the urgent care and was diagnosed with acute bacterial rhinosinusitis (ABRS). She was given a prescription for antibiotics which she is taking as ordered, but is concerned because she has a fever today. Which of the following findings suggests a serious complication of ABRS?

Tender preauricular lymph nodes

Foul-tasting post nasal drip

Facial tenderness

Eyelid erythema and edema

Ans: Eyelid erythema and edema

A 61-year-old retired schoolteacher presents with moderate persistent allergic rhinitis that is impacting his ability to sleep most nights. He has a history of hypertension and suffered an NSTEMI 3 years ago. Which of the following should be avoided for this patient?

Antihistamine nasal spray

Leukotriene modifier

Oral decongestant

Intranasal corticosteroid

Ans: Oral decongestant

The preferred technique for initial diagnosis of osteoarthritis of the knee is:

X-ray.

MRI.

CT scan.

Arthroscopy

Ans: X-ray.

Upon examining a 67-year-old woman, the NP detects a palpable thyroid nodule that is 1.5 cm in diameter. The most appropriate next step for this patient is:

Schedule follow-up in 3 months to determine if the size changes.

Obtain TSH measurement and thyroid ultrasound.

Schedule MRI imaging of the head and neck.

Referral to an oncologist.

Ans: Obtain TSH measurement and thyroid ultrasound.

When communicating healthcare decisions through a healthcare proxy, the NP recognizes:

The proxy has the same rights to request or refuse treatment that the patient would have if he or she were capable of making and communicating decisions.

Only management decisions that are considered life-saving are required to be communicated to the healthcare proxy.

The proxy has the right to refuse treatment except in situations when treatment is necessary to preserve life.

The closest relative to the patient is assumed to take the proxy responsibilities.

Ans: The proxy has the same rights to request or refuse treatment that the patient would have if he or she were capable of making and communicating decisions.

A 61-year-old woman sees you in follow-up after being discharged from the hospital. She recently had an inferior wall myocardial infarction and is asking about supervised cardiac rehabilitation. She is shocked that she had a heart attack, and says she never even knew that her cholesterol was so high. She hasn’t seen a healthcare provider since having her last child 27 years ago. Tertiary prevention strategy for this patient will include:

Referral for mammography.

Lipid-lowering therapy with an HMG-CoA reductase inhibitor (statin).

Influenza and pneumococcal vaccination.

Screening for diabetes.

Ans: Lipid-lowering therapy with an HMG-CoA reductase inhibitor (statin).

Non-contraceptive benefits of combined oral contraceptive use include a decrease in all of the following except:

Ovarian cancer with protracted use.

Iron deficiency anemia.

C. trachomatis cervicitis.

Pelvic inflammatory disease.

Ans:

C. trachomatis cervicitis.

A common complaint of patients who have been using medroxyprogesterone depot (Depo-Provera®) for more than 1 year is:

Hypermenorrhea.

Depression.

Acne.

Weight gain.

Ans:

Weight gain.

All of the following are appropriate candidates for tuberculosis (TB) testing except:

A 28-year-old with HIV infection.

A 55-year-old man who recently emigrated from a country where TB is endemic.

A 33-year-old woman who works at a child care center.

A 56-year-old man who was recently released from prison.

Ans: A 33-year-old woman who works at a child care center.

Indicate whether each of the suggested long-acting reversible contraceptives (LARCs) for the following patients is (yes) or is not (no) appropriate.

1. A 19-year-old woman to receive the levonorgestrel-releasing IUS (Mirena®) __
2. A 30-year-old woman who smokes 1 PPD to receive the etonogestrel implant (Nexplanon®) __
3. A 38-year-old woman with a history of large uterine fibroids and heavy menses to receive the copper-containing IUD (ParaGard®) __
4. A 28-year-old woman who is currently taking antiepileptic medications to receive the etonogestrel implant (Nexplanon®) __

1-No 2-Yes 3-No 4-Yes

1-No 2-No 3-No 4-Yes

1-Yes 2-Yes 3-No 4-Yes

1-Yes 2-No 3-No 4-Yes

Ans: 1-Yes 2-Yes 3-No 4-Yes

Match each of the following signs and symptoms with the appropriate dehydration status for a younger child.

A. Mild dehydration
B. Moderate dehydration
C. Severe dehydration

1. Reduced blood pressure, Fontanels depressed, lethargic __
2. Normal blood pressure, capillary refill of <1.5 seconds, slightly dry lips and thick saliva __
3. Slightly sunken eyes, normal blood pressure, capillary refill of ~2 seconds __

1-B 2-A 3-C

1-A 2-B 3-C

1-C 2-B 3-A

1-C 2-A 3-B

Ans: 1-C severe dyhration (reduced BP, fontanels depressed and lethargic)
2-A mild dehydration (normal BP, cap refill<1.5seconds, slightly dry lips and thick saliva) 3-B moderate dehydration ( slightly sunken eyes, normal BP, cap refill < 2 sec)

The most appropriate next diagnostic step for Caleb is:

Based on the below scenario:
Caleb, a 9-month-old boy, is presented for evaluation after sudden onset of intermittent severe abdominal pain. He often draws his knees to his abdomen when he appears to be in greatest pain. The mother reports that he has had episodes of diarrhea that contain a mixture of blood and mucous. He has not vomited during this time, but also has a lack of appetite.

Ultrasound of the abdomen

Flat plate of the abdomen

Stool culture with susceptibility testing

Stool sample to check for ova and parasites

Ans: Ultrasound of the abdomen

The most appropriate treatment option for Jackson is:

Based on the below scenario:
A mother brings in Jackson, her 4-year-old son, for evaluation after
reporting that she stayed up all night with him because he was
experiencing somewhat labored breathing and had a loud “barking-like”
cough. The child has a hoarse voice, is without acute respiratory
distress, and a temperature of 101.6ᵒF (38.7ᵒC).

Short-term systemic corticosteroid

Nebulized beta2-agonist

Inhaled corticosteroid

Guaifenesin

Ans:

Short-term systemic corticosteroid

You are evaluating a 19-year-old man with acne vulgaris and determine he has approximately 20 closed comedones. The most cost-effective first-line treatment for this patient is:

Oral doxycycline.

Benzoyl peroxide lotion with salicylic acid facial wash.

Tretinoin gel.

Oral isotretinoin.

Ans: Benzoyl peroxide lotion with salicylic acid facial wash.

A 19-year-old female comes to your office to discuss birth control. She has just become sexually active and wants to be responsible. She has heard a lot from her friends about “the pill” (combined oral contraception {COC}) and is asking you for advice. You tell her that:

Premenstrual syndrome symptoms are often improved with COC use.

Long-term use is discouraged as the body needs a COC “rest” from time to time.

The menstrual flow volume is typically increased with COC use.

There is an increase in the rate of uterine cancer after protracted COC use.

Ans: Premenstrual syndrome symptoms are often improved with COC use.

In advising a woman with menstruation-related migraine without aura and combined oral contraceptive use, the nurse practitioner considers that:

A high-dose estrogen pill should be prescribed.

Uninterrupted use can help minimize headache frequency and severity.

Headache severity is likely to increase.

Cardiovascular risk is markedly increased.

Ans:

Uninterrupted use can help minimize headache frequency and severity.

One of the first-line therapy options for moderate acne is:

Topical tretinoin.

Topical corticosteroids.

Oral isotretinoin.

Systemic antibiotics.

Ans:

Topical tretinoin.

According to the World Health Organization Medical Eligibility for Contraceptive Use, which of the following is a Category 4 (use represents unacceptable health risk) for the use of a combined oral contraceptive?

Controlled hypertension

Migraine with focal neurologic symptoms

Age >35 years and smoking <10 cigarettes per day.

BMI ≥30 kg/m2

Ans: Migraine with focal neurologic symptoms

Unless no other contraceptive form is acceptable, depot medroxyprogesterone acetate (DMPA; Depo-Provera®) use should be limited to ≤2 years due to the potential for:

Endometrial hyperplasia.

Acne.

Atrophy at the injection site.

Bone mineral density loss.

Ans:

Bone mineral density loss.

A father presents his 3-year-old with viral gastroenteritis. He reports that she has been experiencing multiple episodes of diarrhea for the past 24 hours, with the most recent episode about 1 hour ago. Evaluation of the child suggests moderate dehydration. The NP recommends:

Initiating oral rehydration therapy with oral rehydration solution.

Referral for parenteral IV fluid.

Offering child ginger ale in small sips.

Offering child sports drink (e.g., Gatorade) diluted 1:1 with water in small sips.

Ans:

Initiating oral rehydration therapy with oral rehydration solution.

our patient has been on combined oral contraceptives for 4 years and is now ready to start a family. She asks how long she should wait after stopping her pills. You advise that it is safe to conceive:

Immediately.

After 1-2 months.

After 3-4 months.

After 2 normal menstrual cycles.

Ans:

Immediately.

You are seeing your 27-year-old female patient for contraceptive care. A levonorgestrel-IUD (Mirena®) was inserted 10 months ago. You anticipate that the patient will report:

Dyspareunia.

Vaginal dryness.

Oligomenorrhea.

Breast tenderness.

Ans: Oligomenorrhea.

n a woman taking a combined oral contraceptive, the reduction in free androgens can yield an improvement in:

Cycle control.

Acne vulgaris.

Rheumatoid arthritis.

Breast tenderness.

Ans: Acne vulgaris.

At which age is a young child at greatest risk of developing iron deficiency anemia?

6 months

18 months

36 months

4 years.

Ans:

18 months

A 16-year-old young woman presents to you the day after a “condom break.” She is concerned that she might become pregnant and is asking about emergency contraception. Appropriate counseling about the use of hormonal emergency contraception (EC) includes all of the following except:

An established pregnancy will not be interrupted.

There is about a 50% reduction in pregnancy risk with properly-timed use.

There is no increased risk of birth defect if pregnancy occurs.

Ulipristal (ella®) is more effective than levonorgestrel in days 3-5 following unprotected intercourse.

Ans: There is about a 50% reduction in pregnancy risk with properly-timed use.

A mother brings in her 4-year-old son for a well child visit. The mother explains that he is recovering from a cold, but he is in generally good health with no complaints. Upon examination, the NP notices a reddened tympanic membrane but an absence of any bulge. The NP explains:

A follow-up visit is needed in 2 days to determine if the infection clears.

Antimicrobial therapy with amoxicillin should be initiated.

Antimicrobial therapy with azithromycin should be initiated.

The benefits of antimicrobial therapy do not outweigh potential risks of adverse effects and resistance development.

Ans: The benefits of antimicrobial therapy do not outweigh potential risks of adverse effects and resistance development.

When counseling a mother on how to assess if a newborn is nursing adequately, the NP advises that the newborn should:

Be able to sleep 2-3 hours between feedings.

Have at least 1-2 bowel movements per day.

Have 6-8 wet diapers per day.

Cease wanting to nurse after about 10 minutes of feeding.

Ans: Have 6-8 wet diapers per day.

Major indications that a child could have attention-deficit/hyperactivity disorder (ADHD) include all of the following except:

Does not readily respond to his/her name on a regular basis.

Symptoms are present before the age of 7 years.

Exhibits problematic behavior in 2 different settings.

Struggling in school academically.

Ans:

Does not readily respond to his/her name on a regular basis.

An appropriate time to educate parents about floor safety (i.e., potential dangers of what a child can find on the floor) is:

Prior to the time when a child learns to walk.

Prior to the time when the child can roll from tummy-to-back and back-to-tummy.

Around the time the baby can pull to stand.

Prior to child being able to sit solo.

Ans: Prior to the time when the child can roll from tummy-to-back and back-to-tummy.

All of the following would support initial testing for type 2 diabetes mellitus in a 12-year-old overweight child except:

Family history of type 2 diabetes mellitus.

African ethnicity.

Absence of signs for start of puberty.

Presence of acanthosis nigricans.

Ans: Absence of signs for start of puberty.

The mother of a 12-month-old son is expected to bring him to clinic for vaccinations later that day. She asks if she can give him some ibuprofen prior to the visit to minimize pain and fever from the vaccinations. The NP responds:

Acetaminophen is preferred over ibuprofen.

Ibuprofen should be administered about 1 hour prior to the visit.

Half a dose can be given before the visit and half immediately following the visit.

Antipyretic use is not recommended prior to vaccinations.

Ans: Antipyretic use is not recommended prior to vaccinations.

When counseling parents on ‘tummy time’ for a newborn, the NP mentions that tummy time should be encouraged until the infant:

Is able to sit solo.

Can pull up to a standing position.

Can roll from tummy-to-back and back-to-tummy with ease.

Can crawl for at least 5 feet.

Ans: Can roll from tummy-to-back and back-to-tummy with ease.

A 3-year-old girl presents with gastroenteritis and who has vomited 4 times in the past 12 hours. The child has signs of mild-to-moderate dehydration. The most appropriate treatment for this child is:

Parenteral dexamethasone (Decadron®).

Oral metoclopramide (Reglan®).

Oral disintegrating tablets of ondansetron (Zofran®).

Rectally-administered trimethobenzamide (Tigan®).

Ans:

Oral disintegrating tablets of ondansetron (Zofran®).

Caleb, a 9-month-old boy, is presented for evaluation after sudden onset of intermittent severe abdominal pain. He often draws his knees to his abdomen when he appears to be in greatest pain. The mother reports that he has had episodes of diarrhea that contain a mixture of blood and mucous. He has not vomited during this time, but also has a lack of appetite.

The most likely diagnosis for this child is:

Pyloric stenosis.

Small bowel obstruction.

Giardiasis.

Intussusception.

Ans:

Intussusception.

A mother brings in Jackson, her 4-year-old son, for evaluation after reporting that she stayed up all night with him because he was experiencing somewhat labored breathing and had a loud “barking-like” cough. The child has a hoarse voice, is without acute respiratory distress, and a temperature of 101.6ᵒF (38.7ᵒC).

The most likely diagnosis is:

Upper airway foreign body.

Croup.

Asthma.

Bronchiolitis.

Ans: Croup.

A mother who is breastfeeding her 2-day-old daughter expresses concern because the child makes a “clicking sound” when nursing. On physical examination of the baby, the NP notices that she is able to thrust her tongue beyond the lip margins with ease. The NP explains that the noise is most likely due to:

An incomplete latch.

Immature motor control.

Ankyloglossia.

An occult cleft palate.

Ans: An incomplete latch.

During a well child visit of a 14-year-old boy, the mother expresses concern that he is the shortest boy on the soccer team. Physical examination reveals a small amount of coarse, pigmented hair at the base of the penis and the scrotal sac is slightly reddened and rugated. You advise:

Testing for serum human growth hormone level.

That he is likely near his adult height.

That his growth spurt will start soon.

Performing a genetic test to detect a chromosomal abnormality.

Ans: That his growth spurt will start soon.

A 15-week-old infant presents with a fever of 100.5ᵒF (38.1ᵒC) and bilateral erythematous tympanic membranes. The infant is alert with excellent skin turgor, no evidence of difficulty breathing, acknowledges her mother’s face, and is wearing a wet diaper. Her parents report that she is vigorously nursing every 3 hours without vomiting or excessive stooling. The appropriate management would be to:

Counsel the mother to observe for 72 hours and return to clinic if there is no improvement.

Start topical therapy with otic antibiotic drops.

Initiate a sepsis workup.

Begin a systemic antibiotic regimen.

Ans: Begin a systemic antibiotic regimen.

A 17-year-old woman presents complaining of left lower abdominal and groin pain. She admits that she has had a “light period everyday” for nearly 3 weeks. The presumptive diagnosis of ectopic pregnancy is supported by which of the following?

Positive urine hCG

Negative pelvic ultrasound

Abdominal rebound tenderness

Breast pain

Ans: Positive urine hCG

A first-time father brings in his 1-month-old infant for evaluation. He is concerned that she “throws up all of the time.” The child has an appropriate weight gain since the last office visit at age 1 week and appears well today. The nurse practitioner knows that the most common cause of frequent spitting and vomiting in a young infant is:

GI tract immaturity allowing reflux.

Overfeeding.

Pyloric stenosis.

Allergy or intolerance to a component of infant formula.

ANs: GI tract immaturity allowing reflux.

Developmental assessment of a healthy 11-month-old born at 32-weeks’ gestation is expected to reveal an infant who:

Follows a two-step command.

Plays “peek-a-boo.”

Walks short distances solo.

Says “mama, dada” specifically.

Ans: Plays “peek-a-boo.”

During a 12-month well-child checkup, the nurse practitioner notices that the child is able to pull to stand, haltingly cruises, but is not walking independently. The mother is very concerned and insists that her other three children were all walking by the age of 1 year. The most appropriate action would be to:

Tell the mother that while on average children are walking by 1 year, many children do so later; schedule a follow-up in 1-2 months to assess progress.

Reassure the mother that children with older siblings often do not start walking until later; schedule the next routine well-child checkup in 3 months.

Counsel the mother that while there is no need to panic, this does represent a developmental delay and the child should see a pediatric orthopedic specialist.

Request a radiographic assessment of the spinal cord, pelvis, and femurs to rule out structural bony anomalies that frequently delay walking.

Ans: Tell the mother that while on average children are walking by 1 year, many children do so later; schedule a follow-up in 1-2 months to assess progress.

Mrs. Jansen is a 34-year-old healthy female patient who is nursing her first child. She has just been diagnosed with mastitis and wants to know what to do about breastfeeding. The nurse practitioner advises her to:

Continue to nurse only on the unaffected side.

Pump the breasts and discard the milk.

Continue to nurse the baby as the mother tolerates.

Discontinue breastfeeding.

Ans:

Continue to nurse the baby as the mother tolerates.

Mrs. Perry presents her 19-month-old for evaluation of a 2-day history of crankiness and fever. Physical examination is significant for a sustained fever (up to 103.4ᵒF or 39.7ᵒC). The tympanic membranes are erythematous around the edges but otherwise normal. The chest is clear to auscultation and her respiratory rate is within normal limits. The child is alert and age-appropriately resists examination. The remainder of the physical examination is normal. Initial assessment should include:

A chest radiograph.

A lumbar puncture.

A sterile urine specimen for culture and sensitivity.

Stool for ova, parasites, and culture.

Ans: A sterile urine specimen for culture and sensitivity.

One site for atopic dermatitis usually only noted during infancy is the:

Face.

Antecubital fossa.

Buttocks.

Hands.

Ans: Face.

A father brings in his 20-month-old daughter who presents with a diffuse rosy-pink macular rash and fever of 102.3ᵒF (39.1ᵒC). He reports that he first noticed the rash this morning but that she has had a fever for the past 3-4 days. Today, the child is without fever. . The most likely diagnosis is:

Roseola

Rubeola.

Scarlet fever.

Rubella.

Ans:

Roseola – study childhood rash and fevers condition.

Scarlet fever: sand-paper like rash with exudative pharyngitis fever, headache, tender localized anterior cervical lymphadenopathy. Rash usually erupts on day 2 of pharyngitis, and often peels a few days later.
Treatment: caused by Strep, oral Amoxicillin as 1st line, Penicillin Po or IM, oral Macrolide (Azithromycin, Clarithromycin, Erythromycin) only if pts has PCN allergy.

Roseola (Agent Herpes virus) HHV 6: dicrete pink-rose macular rash or maculopapillar rash lasting hours to 3-7 days often with high fevers. 90% cases found in children < 2 yrs. Treatment: supportive therapy

Rubella (virus): mild symptoms sore throat, malaise, nasal discharge, diffuse maculopapillar rash lasting about 3 days; posterior cervical and postauricular lymphadenopathy begining 5-10 days. Prior to rash eruption, 10% most common in women c/o myalgia.
Incubation period: 10-14 days, most infectious 1 -2 wk prior to onset of rash rash, generally self-limiting, greatest risks to unborn child especially during 1st trimester (infant can expose to congenital rubella syndrome). It is a reportable disease to HHS department, laboratory confirmation by presence of serum Rubella IgM.
MMR virus- usually acute on presentation with fever, nasal discharge, cough, generalized lymphadenopathy, conjuctivitis (copius clear discharge), Koplik spots appears 2 days prior with blue rings held within red spots on oral mucosa, pharyngitis w/o exudative, maculopapular rash onset 3-4 days after onset of symptoms, may coalesce to generalized erythema. Complications: CNS and respiratory tract are common, permanent neurological impairment or death possible, is a reportable disease to HHS department, Vaccine-preventable disease, laboratory confirmatory by presence of serum rubeola IgM.
Infectious Mononucleosis: caused by Epstein-Barr virus- Maculopapular rash or petechiae rare, fever, purple white pharyngeal exudate, malaise, marked diffuse lymphadenopathy, hepatic and splenic tenderness, splenic enlargement.
Dx: Mono spot, lymphocytosis, atypical lymphocytes presence.
Incubation period: 20-50 days.
>90% will develop rash if given Amoxicillin or Ampixicillin during the illness. Potential for resp. distress due to enlarged tonsils and lymphoid tissues impinged on the airway, corticosteroids maybe helpful, splenomegaly often noted between day 6-21 after onset of illness. Avoid contact sports > 1 mo due to risk of splenic rupture.

When evaluating a patient at 12-weeks’ gestation, the nurse practitioner anticipates which physical finding?

Fetal heart tones undetectable by Doppler

Clear nipple discharge

Negative Chadwick’s sign

Uterus fundus palpable directly above symphysis pubis

Ans: Uterus fundus palpable directly above symphysis pubis

While reviewing the records of a 3-year-old patient who is new to your practice, you learn that she has a Still’s murmur. As a result, you anticipate which of the following physical findings?

An audible systolic murmur with a buzzing quality

Height and weight in the bottom 25% of the growth chart

A hyperdynamic point of maximal impulse

Joint laxity and hyperextension

ans: An audible systolic murmur with a buzzing quality

All of the following are risk factors for child mistreatment except:

Child 4-5 years of age.

Child with special needs that can increase caregiver burden.

Parental history of child maltreatment.

Residing in a community with significant baseline violence.

Ans: Child 4-5 years of age.

Maria is a G1P0AB0 woman with is 28-weeks pregnant. Her pregnancy to date has been uneventful and she has no chronic health problems. Maria has not received any immunizations in the past year.

1. What should she receive today? Choose all that apply.

A. A Tdap

B. Injected influenza immunization

C. Live attenuated influenza vaccine (LAIV)

D.Pneumococcal vaccine.

Ans:

A. A Tdap
B. Injected influenza immunization

Which of the following vaccines should be offered to him today? Choose all that apply.

Based on the below scenario:
Maria is a G1P0AB0 woman with is 28-weeks pregnant. Her pregnancy to date has been uneventful and she has no chronic health problems. Maria has not received any immunizations in the past year.

Maria’s 28-year-old spouse, who is generally in good health, also presents today.


A. Tdap

B. Injectable influenza vaccine

C. Pneumococcal conjugate vaccine

D. Zoster vaccine

Ans: A. Tdap

B. Injectable influenza vaccine

ADA screening for DM in Children

-symptomatic children (polyuria, polydipsia, polyphagia, blurred vision) regardless of risk factors
-asymptomatic children after puberty or 10 years of age or older if overweight or obese (>85th percentile). Plus 1 of the following:
*T2DM in 1st or 2nd degree relative
*high risk racial/ethnic group
*signs of insulin resistance (HTN, dyslipidemia, acanthosis nigricans, PCOS, SGA)
*maternal hx of DM or GDM during the child’s gestation

Statistics

– leading causes of death: Heart disease, cancer, lung disease
– leading cause of cancer death: lung
– leading cause of death in adolescents: accidents
– most common cancer: skin.
– in males: prostate. in females: breast

suicide: males more successful, women more attempts. highest rate is older white males.

Osgood-Schlatter:

knee pain in young adults, overuse. Repetitive stress pain, tenderness, swelling at the tendon’s insertion site. The tibial tuberosity. Rule out avulsion fracture if there is an acute onset and order a lateral xray. RICE. Usually stops when the growth stops.

Image: Osgood-Schlatter:

If patient has right sided weakness, etc. the CVA occurred where

left side

initial evaluation of symptoms of acute prostatitis

Urinalysis and urine culture

A 65-year-old woman presents for a follow-up examination after a new patient visit. She has not seen a healthcare provider for several years. She is a smoker and her hypertension is now adequately controlled with medication. Her mother died at age 40 from a heart attack. The fasting lipid profile shows cholesterol = 240 mg/dL, HDL = 30, and LDL = 200. In addition to starting Therapeutic Lifestyle Changes, the nurse practitioner should start the patient on:

1. bile acid sequestrant.

2. a statin drug.

3. a cholesterol absorption inhibitor.

4. low-dose aspirin.

A statin drug

Ortolani’s Click

a click is heard or felt as dislocation is reduced (developmental dysplasia of hip) (good until one year)

Which of the following laboratory tests should a nurse practitioner order when the suspected diagnosis is temporal arteritis?

Erythrocyte sedimentation rate (ESR)

What are narrow therapeutic index drugs?

1. Warfarin sodium (Coumadin): monitor INR
2. Digoxin (lanoxin): monitor digoxin level, EKG, electrolytes(potassium, magnesium, calcium)
3. Theophylline: monitor blood levels
4. Carbamazepime (Tegretol) and Phenytoin (Dilantin): Monitor blood levels
5. Levothyroxine: Monitor TSH
6. Lithium: Monitor blood levels, TSH (risk of hypothyroidism)

Otitis Externa tx

Fluoroquinolone & Polymyxin B cortisporin drops

An elderly male patient complains of a new-onset, left-sided temporal headache accompanied by scalp tenderness and indurated temporal artery. The NP suspects temporal arteritis. What screening test would you order to assist with diagnosis?

sedimentation rate (expect to be very elevated)

Basal Cell Carcinoma

Pearly domed nodule with overlaying telangiectatic vessels. Could be plaque, papule, possible central ulceration and crusting. Dx: Biopsy Tx:

Normal, healthy woman of reproductive age

white, clear, flocculent(physiologic leukorrhea), no complaints, pH 3.8-4.2 (toward acidic), no odor, microscopic shows lactobacilli (gram+bacteria)

Multiple infections from bacteria and fungus?

Screen for HIV

Screening Tests

– sensitivity: detect those WITH the disease. higher the sensitivity is higher the false positives
– Specificty: detect those who DONT have the disease.

erythromycin for chlamydia eye infection in infants

to assess pts ability to think abstractly a nurse pract could ask the patient

the meaning of a common proverb

The most commonly prescribed medication for mild systemic lupus erythematosus (SLE) is:

1.
azathioprine (AZA).
2.
belimumab (Benlysta).
3.
ibuprofen (Advil).
4.
cyclophosphamide (Cytoxan).

ibuprofen (advil)

A 17-year-old female is suspected of having polycystic ovary syndrome. In addition to testosterone, the most appropriate diagnostic tests to order would be:

follicle-stimulating hormone (FSH), luteinizing hormone (LH), prolactin, and thyroid-stimulating hormone (TSH).

Barlow’s Maneuver

Feeling of a slip as the femoral head slips away from the acetabulum (toward the butt) (good until 6 mo)

Candida vulvovaginitis

etiology: candida albican (80-90%)
white, curdy, “cottage-cheese” like, sometimes increased, itching/burning discharge, pH <4.5, odor is usually absent, microscopic shows mycelia, budding yeast, pseudohyphae w/KOH prep. Treatment: oral diflucan or vaginal miconazole or terconazole

PSEUDOHYPHAE, CLOTRIMAZOLE CREAM

Proton Inhibitors

Increased risk of fractures(postmenopausal women),
Pneumonia, Clostridium difficile infection, hypomagnesemia, B12 and iron malabsorption, atrophic gastritis, and kidney disease

Bacterial Conjunctivitis tx

Eye drops or ointment: Polytrim, trimethoprim, polymyxin, macrolide

A patient with an elevated WBC (>11k) accompanied by neutrophilia (>70%) and the presence of bands is what kind of shift and prognosis?

-Shift to the left
-Serious bacterial infection

Actinic Keratosis

Rough flat, dry crusty, erythematous papules or plaques. Scaly patch of red brown skin caused by years of SUN exposure. Precursor to squamous cell carcinoma. Dx. Biopsy. Tx: topical 5 fluroracil 5-FU, cryotherapy.

A patient with macular degeneration has deficit vision in?

Central vision

Dacyrocystitis

Typical symptoms of acute tear duct infection include:
Pain, redness and swelling of the lower eyelid at the inner corner of the eye
Excessive tearing
Pus or discharge from the eye
Fever
Tx:
lacrimal sac massage- rub down towards mouth.
oral clindamycin (topical tobramycin or moxifloxacin if mild-purulent drainage, no redness)

Image: Dacyrocystitis

Pre-DM in children values

A1C: 5.7% to 6.4%
Fasting: 100-125
2 hour GTT: 140-199

Primary Prevention

– individual actions: eating nutritious diet, exercise, seatbelts, gun safety. IMMUNIZATIONS

squamous epithelial cells with stippling appearance, no lactobacilli and many WBCs is

wet mount that shows BV

trigeminal neuralgia manifests

electric shock facial pain

The most common sign of cervical cancer is:

1.
postcoital bleeding.
2.
strong odor from vaginal discharge.
3.
itching in the vaginal area.
4.
molluscum contagiosum.

postcoital bleeding

To assess a patient’s ability to think abstractly, a nurse practitioner could ask the patient:

the meaning of a common proverb.

Auspitz sign

droplets of blood when scales removed = psoriasis

Bacterial vaginosis

etiology: unclear, likely polymicrobial, associated with G. vaginalis, M. hominid, others.
thin, homogeneous, white, gray, adherent, often increased, discharge is foul odor(fishy), itching is occasionally present, pH 5-7(alkaline- no active bacilli in vagina), “fishy” smell, microscopic > 20 clue cells/HPF, few or no WBCs. Treatment: metronidazole topical, oral Flagyl, clindamycin vaginal cream, oral tinidazole (Tindamax)

CLUE CELLS, METRONIDAZOLE GEL OR ORAL, CLINDAMYCIN CREAM

Image: Bacterial vaginosis

Omeprazole (Prilosec)

interacts with with Warfarin (Coumadin),
diazepam (Valium),
Carbamazepine (Tegretol),
Pheytoin (Dilantin),
ketocanazole (Nizoral)

Viral Conjunctivitis tx

Antihistamine, decongestant drops (Trigluridine in herpes conjunctivitis)

What is the common presentation of a navicular fracture?

Tenderness at the “N spot,” which is defined as the proximal dorsal portion of the navicular (see the image below). This is the most important physical finding.

Melanoma

ABCDE: asymmetry, border irregularity, color variation, diameter greater than .6 mm, elevation above skin level.

Type 2 DM in children values

A1C: >6.5%
Fasting: >126
2 hour GTT: >200
Random: >200

Central clearing lesion after camping trip flu like symptoms with muscle aches for several days is? and treated by?

Lyme disease; doxycycline

Wilms tumor

(Nephroblastoma)- Not painful. Asymptomatic abd mass does NOT cross the midline. 2-3 y. o.d. do not palpate. Do ABD US. PUNT. Think Nephro doesn’t cross. Stays where kidney is.

Image: Wilms tumor

the headache of an intracranial tumor

focal neurological signs and pain worse in supine position

socioeconomic status

not important during employment physical with 21 yo with bruising on breasts

The nurse practitioner prescribes amitriptyline (Elavil) for a patient with neuropathic pain secondary to diabetes mellitus. On follow-up, the patient complains of urine retention and dry mouth. The practitioner would:

1.
discontinue amitriptyline and begin ibuprofen (Motrin).
2.
refer to physical therapy.
3.
start methocarbamol (Robaxin).
4.
discontinue amitriptyline and begin gabapentin (Neurontin).

discontinue amitriptyline and begin gabapentin (neurontin)

To assess spinal function at the S1 level, which deep tendon reflex should be tested?

Achilles

Herald patch + christmas tree

pityriasis rosea

Secondary Prevention

– screening tests (pap, mammogram, CBC).

Strep Pharyngitis tx

PCN, amoxicillin, macrolide, cephalosporin

Vitamin K Agonist Warfarin (Coumadin)

Interactw with “G” hers:
Garlic
Ginger
Gingko
Ginseng

Other herbs/supplemts:
Feverfew
green tea
fish oil

**Discontinue 7 days before surgery

What is the gold standard test for sickle cell anemia, glucose-6-phosphage dehydrogenase (G6PD) anemia, and alpha or beta thalassemia??

Hemoglobin electrophoresis

Postherpetic neuralgia

Tx: Prophylaxsis TCA-Elavil

Atrophic vaginitis (genitourinary syndrome of menopause GSM)

etiology: estrogen deficiency (after menopause). D/C scant, white-clear dryness as well sometimes urinary incontinence, itching/burning, discharge but often w/o symptoms, pH >5 (little to no lactic bacilli), odor is absent, microscopic few or absent lactobacilli.
Treatment: topical and/or vaginal estrogen if symptomatic and/or recurrent UTI. (Oral estrogen as solo intervention likely inadequate)

Diverticulitis: Treatment s/s:

Antibiotics and clear liquids and increased fiber (some say no nuts or seeds). (7 to 10 days ABT)
Ciprofloxacin (500 mg PO twice daily) plus metronidazole (500 mg PO three times daily). Amoxicillin-clavulanate (875/125 mg twice daily) is an acceptable alternative.

The criteria for patients with acute uncomplicated diverticulitis to be treated in the outpatient setting include:

●Reliability to return for medical reevaluation if condition worsens

●Compliance with outpatient treatment plan

●Abdominal pain is not severe

●No higher than a low-grade fever

●Can tolerate oral intake

●No or minimal comorbid illnesses

●Available support system

Approximately six weeks following the resolution of symptoms of acute diverticulitis, patients who have not had a recent colonoscopy should undergo one to exclude other possible diagnoses (such as colonic neoplasia) and to evaluate the extent of the diverticulosis.
Recomendations for surgery: Patients in whom elective surgery has been recommended following a single attack of diverticulitis include younger patients (variously defined in the literature as less than 40 or 50 years of age) and those who are immunosuppressed.

Image: Diverticulitis: Treatment s/s:

Neuroblastoma-

painful abd mass fixed first irregular, crosses midline. Most common side is adrenal glands. Weight loss fever. HORNERS syndrome. RACOON eyes, bone pain, HYPERTENSION. 1-4 year olds. Dx ultrasound PUNT to nephro. NEURO think brain in middle crosses midline. **Urine catecholamines and anemia

Image: Neuroblastoma-

ADHD

hyperactivity, impulsive and/or inattention.

DSM-5 Criteria:
*sx present before 12 years of age
*sx for at least 6 months
*sx in 2 different settings

Medications considered 1st line if over 6 years of age

TZD
Pioglitazone (Actos)

Black Box warning: cause or exacerbate congestive heart failure in some patients; do not. use if New York health association Class III or IV Heart failure

STOP if causes dyspnea, weight gain, cough (heart failure)

in order to improve longevity of patient with COPD, tx of choice is

oxygen

current social relationships,
history of present or past traumas
mental health status

all important on the pe of 21 yo, for employment with bruising on breasts

A 17-year-old male with rheumatoid arthritis is being treated with an NSAID and omeprazole (Prilosec). The patient complains of headache, abdominal pain, and gas. These symptoms are most likely:

1.
associated with the omeprazole.
2.
related to the underlying condition.
3.
the result of the NSAID.
4.
caused by viral gastroenteritis.

associated with the omeprazole

Koplick Spots

white with red ring inside cheek from rubeola or mumps

Tertiary Prevention

– AA, support groups, education for those with the disease, rehab, exercise programs for obese

Which of the following signs/symptoms are often associated with headaches due to an intracranial tumor?

Pain worse in supine position; focal neurological signs

Acute rhinosinusitis tx

Wait 10 days then Amoxicillin or Augmentin (If allergy, use fluouroquinolones/tetracyclines)

MRSA

Tx:

Autism

Impairment in social communication and social interaction. Restricted, repetitive patterns of behavior, interests and activities.

Red flags: delayed language/communication milestones, regression in social and language skills, sibling with autism

screen at 9, 18, 24 and 30 months or when concerns are raised by parents

Genital Herpes

causative organism: human herpes virus 2
clinical findings: painful, ulcerated lesions, marked lymphadenopathy with initial lesions. Women=thin vaginal discharge if lesion at vagina or intoitus
with recurrence symptoms vary- asymptomatic transmission common
treatment: oral acyclovir (Zovirax), famciclovir (Famvir), valacyclovir(Valtrex), dose and length of treatment depends on the medication choice and the clinical presentation.

Diverticulosis: Treatment s/s:

Diverticu-lO-sis has nO inflamation: Both diverticulitis and *osis are similar to umbilical hernia in that a vein running through the muscle of the bowel muscle weakens the area and pressure causes a hernia “out pouching”.

Image: Diverticulosis: Treatment s/s:

RETINOBLASTOMA

leukocoria: Hallmark sign white spots in eye. Cancer. Red light reflex negative

Image: RETINOBLASTOMA

What are first and second line antibiotics for acute otitis media?

1. Amoxicillin
2. Amoxicillin-Clavulanate (Augmentin)

Screening Tests:

– Breast Cancer: 50yrs q2years until 75yo. (40-49 is individualized)
– Cervical Cancer: 21yrs q3yrs until 65y. no screening after hysterectomy
– Colorectal Cancer: 50yrs until 75. FOBT x3 yearly. Flex sig q5yrs OR colonoscopy q 10yrs
– Lipids: men >35y and women >45y. unless increased risk for heart disease
– Prostate: don’t screen
– Skin Cancer counseling: 10-24yo with fair skin

which of the following criteria differentiates a TIA from a CVA

absence of residual symptoms – TIA sx occur rapidly and then resolve, CVA sx are residual – TIA is temporary block and usually no residual sx

CD4 count and viral load

most widely accepted indicators of HIV infection

The medication of choice for the initial treatment of juvenile rheumatoid arthritis is:

1.
acetaminophen.
2.
prednisone.
3.
aspirin.
4.
ibuprofen.

ibuprofen

Which of the following laboratory tests are most widely accepted as indicators of the progression of HIV infection?

CD4 count and viral load

slapped cheek and lacy exanthem

Erythema Infectiosum or Fifths disease

Intertrigo tx

Nystatin

Atypical antipsychotics:
Risperidone (Risperdal)
Olanzapine (Zyprexa)
Quetiapine (Seroquel)

High risk for weight gain
metabolic syndrome
Type 2 DM

** high mortality in elderly patients

Monitor: weight and blood sugar
TSH, L lipids, weight body mass index

If a patient is allergic to penicillin and has a gram+ infection what are the alternative antibiotic choices?

1. macrolides (azithromycin, clarithromycin)
2. clindamycin
3. quinolones with gram+ activity (levofloxacin or moxifloxacin)

Xerosis

Dry skin. Use petroleum based product.

Nongonococcal urethritis and cervicitis

causative organism: chlamydia trachoma’s, ureaplasma urealyticum, mycoplasma genitalium (obligate intracellular pathogen)
clinical findings: Friable cervix=bleeds easily, may have no symptoms, irritative voiding symptoms, occasional mucopurulent discharge, microscopic shows large number of WBCs. Treatment: azithromycin 1 g PO 1 time dose

ABUNDANT WBCs

Bacterial vaginosis: s/s and treatment

Clue cells such as: stipling of squamous epithelial cells with indistinct borders, no lactobacillus rods, many white blood cells

Nonpregnant women
Drugs — Metronidazole or clindamycin administered either orally or intravaginally results in a high rate of clinical cure (70 to 80 percent at four weeks of follow-up) (table 4) [88-91]. Oral medication is more convenient, but associated with a higher rate of systemic side effects than vaginal administration.

Side effects of metronidazole (oral or vaginal) include a metallic taste, nausea (in 10 percent of patients), transient neutropenia (7.5 percent), a disulfiram-like effect with alcohol, prolongation of International Normalized Ratio in patients taking vitamin K antagonists (eg, warfarin), and peripheral neuropathy.

Image: Bacterial vaginosis: s/s and treatment

Erythema infectiosum

(5th disease)- “slapped cheeks” 5-14 y.o. LACY, spreads to upper arms lgs trunks dorsum of hands and feet. Rash can last up to 40 days. Fever, rash, runny nose, headache. EXAM humanparovirus19, no labs for it.

Image: Erythema infectiosum

Most common bacterial pathogen causing pneumonia

strep pneumoniae, but most pathogens 6 months to 5 years are viral

Biphosphonates:

Alendronate (Fosamax)
Risedronate (Actonel)

Erosive esophagitis, abdominal pain,
Stop immediately if symptoms:
esophagitis (chest pain, difficulty swallowing, burning (mid back)
jaw pain (osteonecrosis)

Contraindicated: Active GI disease (GERD, PUD), CKD, esophageal stricture/varices

prednisone

management of polymalgia rheumatica

A 12-year-old with sickle cell anemia has recently experienced a sickle cell crisis and presents for a follow-up examination after a recent hospitalization. It is most important to continue monitoring growth, development, and:

1.
white blood cell levels.
2.
fecal occult blood test.
3.
hemoglobin levels.
4.
urine dipsticks.

hemoglobin levels

A 3-year-old patient presents at an inner-city clinic with fever, cough, malaise, and loss of appetite. The patient lives with several relatives, including a grandmother who also has a cough. Which of the following diagnostic tests would be most appropriate for the patient?

Sputum culture

Thumb sign

swelling of the epiglottis, which may be visible on a lateral radiograph in patients with Epiglottitis

Lung Cancer Screening

– 55-74yo with >30y pack smoking history and who quit <15y ago. LDCT annually

med of choice for polymyalgia rheumatica

prednisone

Urticaria tx

Benadryl or Zyrtec

Shingles

Prodrome: itching burning photophobia fever headache malaise. Acute phase: Dermatomal rash 3-4 days, unilateral, macupapular rash progresses to vesicles then pustules 3-4 days. Convalescent phase: 2-3 week rash resolves. Dx PCR. Tx: acyclovir, zostrix, gaba, amitriptyline. (one dermatone)

Gonococcal urethritis and vaginitis
1 million cases daily WW- abx resistance prevalent

causative organism: neisseria gonorrhoeae (gram – bacteria)
clinical findings: irritative voiding symptoms, occasional purulent discharge, often w/o symptoms in either gender
Microscopic exam: large number of WBCs
STI most likely to give penile DC also called the “drip”
Treatment: ceftriaxone 250 mg IM as a one time dose plus azithromycin 1 g PO x 1 dose

If you find Gonorrhea ALWAYS tx for chlamydia

CEFTRIAXONE

10 month old child with runny nose, rash, cough, with tiny white papules with red areola in mouth what does this suggest?

Measles

ADHD

hyperactive, impulsive, inattentive.
Present prior to 12 years.
Symptoms last > 6 months,
should be evident in at least 2 different settings. Treated with schedule II – Ritalin, Adderal, vyvanse, streterra etc

most sensitive sign of pneumonia in children

increased respirations. fever is inconsistent.

A patient with COPD is prescribed ipratropium bromide (Atrovent) for dyspnea. If no relief at follow up visit what is the next step?

1. Albuterol inhaler (Ventolin)
2. OR a combination inhaler

Statins
Atrovastatin (Lipitor)
Lovastatin (Mevacor)
Rosuvastatin (Crestor)
Simvastain (Zocor)

Do not mix with grapefruit juice
Drug induced hepatitis
Rhabdomylsis higher if mixed with azole antifungals

High dose zocor (80mg) has highest risk of rhabdomyolis (muscle pain/tenderness)

Chenese Descnet: higher risk myopathy or rhabdomyoliss when taking simvastatin 40mg/day with niacin
Creatine Kinase level goes up

polymyalgia rheumatica

Risk factors include female gender and age >50
Pain stiffness of the shoulder/pelvic girdle typically seen in >50 year olds with the “hallmark” difficulty combing/brushing hair responding well to low dose steroids

A 90-year-old female is brought to the clinic by her neighbor. She states that everything is fine, but the nurse practitioner notes that she has poor hygiene and bruises on her trunk. The neighbor is concerned that the patient often has no money to buy food, despite income from social security and a coal miner’s pension. The nurse practitioner suspects abuse. Which of the following is the nurse practitioner obligated to do next?
1.
Report the case to the proper authorities.
2.
Tell the neighbor to check on the woman daily and report back.
3.
Document the data and report the information to risk management.
4.
Call the patient’s family and inquire about the concerns.

report the case to proper authorities

Which of the following best describes psoriatic lesions in an elderly patient?

Red, sharply defined plaques with silvery scales

Steeple Sign

tapering of the upper trachea on a frontal chest radiograph reminiscent of a church steeple suggestive of Croup

Active Immunity

– via immunization or a person who was exposed to agent

helical CT pulmonary angiography

test for PE

Actinic Keratosis tx

Topical 5 fluoracil 5-FU cryotherapy

What is the classic triad of symptoms for mononucleosis?

1. sore throat
2. prolonged fatigue
3. enlarged cervical nodes
(usually a teen)

Dog bite

Treat analgesia (tyenol, Nsaids, Demerol) Ab: Augmentin/doxycycline/Bactrim, wound cleaning with soap and water, betadine, lidocaine. Wound debridement, facial bites closed with sutures. Tetanus

Trichomoniasis

causative organism: trichomonas vaginalis (protozoan pathogen)
clinical findings: dysuria, itching, vulvovaginal irritation, yellow-green vaginal discharge, occasionally frothy (30%), cervical petechial hemorrhages (“strawberry spots”) in about 30%, often w/o symptoms in either gender, microscopic exam: motile organisms and large number of WBCs, pH is alkaline
Treatment: metronidazole 2 g (No alcohol) PO or tinidazole 2 g PO as a 1 time dose (parasitic infection) no etoh -GI upset

ORAL METRONIDAZOLE

Trunk like rash:

Roseola: measles

MASTITIS

red firm tender area fever chills, flu like symptoms. Dicloxacillin, or Keflex. If you suspect MRSA, do Bactrim or clinda.

Image: MASTITIS

treatment of bacterial pneumonia in children:

High dose amoxicillin, augmentin or 3rd generation cephalosporin (cefdinir).

if type 1 reaction to PCN use clindamycin or azithromycin

electric shock unilateral facial pain

trigeminal neuralgia

serum ferritin level

distinguish iron deficiency anemia from other anemias

In most cases, the first manifestation of Alzheimer’s disease is:

1.
impaired judgment.
2.
decrease in short-term memory.
3.
disorientation in time and place.
4.
decrease in long-term memory.

decrease in short-term memory

A 25-year-old presents with the chief complaint of decreased mobility and pain of the right shoulder exacerbated by movement. The patient reports that he participated in extensive house painting 24 hours prior to the onset of pain. He denies any trauma. Passive ROM is intact. No redness or ecchymosis is present. What is the next step that should be taken in order to make a diagnosis?

Palpate structures around the shoulder.

Galeazzi’s Sign (aka Allis sign)

uneven knee heights – Developmental Displasia of the Hip

Passive Immunity

– immunoglobulin or through breast feeding/from mother

Atopic Dermatitis (eczema) tx

Topical steroids, emollients

Lincosamides
Clindamycin (cleocin)

Higher risk of CDAD
Metronidazole (flagyl) PO TID x 10-14 days
Probiotics daily-BID x few weeks

Alpha thalassemia is more common with what ethnicity?

Southeast Asians (Filipinos)

Lupus

Multisystem autoimmune disease, ch by remission and exacerbations. Butterfly rash, avoid sun exposure, Tx: Refer rheumatology, topical steroids and oral steroids.

Syphilis

causative agent: treponema pallidum (spirochete bacterium)
clinical findings:
Primary stage: chancre, firm, round, painless genital and/or anal ulcers with clean bas and indurated margins, localized lymphadenopathy, aprox 3 weeks duration, resolves w/o treatment. Secondary stage: nonpruritic skin rash, *palms and soles, as well as mucous membrane lesions, fever, lymphadenopathy, sore throat, patchy hair loss, headaches, weight loss, muscle aches, and fatigue. Resolution w/o treatment is possible.
Latent stage: presentation variable w/ decrease in cognitive function, occurs when primary and secondary symptoms have resolved
Treatment: antimicrobial therapy, with dosage and length of therapy usually dictated by disease stage. Options include injectable PCN or PO doxycycline
IM Bi-cillin- used as well

INJECTABLE PENICILLIN OR IF SEVERE PENICILLIN THEN GIVE DOXYCYCLINE

Other Childlike rashes: Key characteristics:

INSERT HERE

UTI in pregnancy

10 (3) wbc is considered positive in prego with symptoms. Normal people its 10 (5). MEDS: Macrobid (not for 3 trimester) Augmentin, Amoxicillin, Cephalexin, Fosfomycin.

bronchiolitis

Lower resp. tract illness that occurs when an infecting agent causes inflammation and obstruction of the small airways (bronchioles).
Common under 2 years of age.
Management: supportive. No bronchodilators or steroids

Anaphylaxis

– Pruritus/uticaria or angioedema AND EITHER respiratory compromise OR hypotension/end organ dysfucnction
– give Epi pen then send to ED
– refer to allergy if cause is unknown

sudden vision loss in which person feels like a curtain came down over his eye

retinal detachment

tx with clear liquids and oral antibiotics

pt with diverticulosis, temp 100, localized LLQ discomfort, palpable mass, leukocytosis, and CAN TOLERATE FLUIDS

The optimal treatment for latent tuberculosis is:

1.
rifampin (Rifadin) for 5 months.
2.
isoniazid (Nydrazid) for 9 months.
3.
pyrazinamide for 6 months.
4.
ethambutol for 6 months.

osioniazid (nydrazid) for 9 months

A patient who is 28 weeks pregnant reports a single episode of vaginal bleeding. History indicates normal prenatal progress to date, and the patient denies pain, vaginal itching, or discharge. Which of the following is the most appropriate intervention to aid in the diagnosis of this case?

Ultrasound

Gower’s Maneuver

patient that has to use their hands and arms to “walk” up their own body from a squatting position due to lack of hip and thigh muscle strength = muscular dystrophy

Anthrax

Tx: doxycycline/fluoroquinolones

Seborrheic Dermatitis tx

Rotation of prescription/non-prescription antifungal shampoo (Ketoconazole/metronidazole), Capitrol shampoo, selenium sulfide, selsun blue (adults / children) ciclopirox shampoo, topical stroid gel (hydrocortizone face, ears hydrocortizone cream), eyelids – baby shampoo

cystic fibrosis

*Effects lungs, GI and sweat glands.
*Autosomal recessive
*Routine screening in all states
*Presents with recurrent lower respiratory infections and persistent productive cough
*Weight loss and greasy BM common
*Mucous blocks ducts of the pancreas

Genital warts (Condyloma acuminata)

Causative organisms: HPV (commonly HPV-6 & 11)
Clinical findings: verruca-form lesions can be subclinical or unrecognized
Treatment: prevent w/immunization, topical podofilox, liquid nitrogen, cryoprobe, trichloroacetic acid, bichloracetic acid, surgical removal, or topical imiquimod (only indicated for external warts treatment).
trichloroacetic acid use acceptable in pregnancy

DO NOT USE IF PREGNANT: podofilox, podophyllin, sinecatechins and imiquimod

IMIQUIMOD

Erythema toxicum

A rash of small yellow or white bumps surrounded by red skin. Can appear anywhere on the body. Disappears on its own in about two weeks. Common in newborns, usually showing up two to five days after birth.

Alpha fetal protein test

AT 16 WEEKS TEST FOR AFP
Low- Downs
High-Neural tube deficits

Image: Alpha fetal protein test

Beta thalassemia is more common with what ethnicity?

Mediterranean people

Live Vaccines

– MMR: 1 yr and 4-6y (if out of country give 1 dose)
– Varicella: 1 yr and 4-6yr
– FluMist
– Zoster
– Rotavirus

*give same day or separate doses by 1 mo.

decreased gastric production with aging

tests for polycystic ovarian syndrome

testosterone, follicle stimulating hormone, luteinizing hormone, prolactin, thyroid stimulating hormone

Unilateral spontaneous serous or serosanguineous discharge from a single duct of a breast is most often caused by:

1.
intraductal papilloma.
2.
mucinous breast lesions.
3.
Paget’s disease.
4.
ductal carcinoma in situ.

intraductal papilloma

A 29-year-old male with noncomplicated Chlamydia infection may exhibit:

no remarkable clinical symptoms.

Fat pad/Sail sign

Elbow fracture (in kids, a posterior fat pad sign suggests a condylar fracture of the humerus. In adults it suggests a radial head fracture)

Moderate acne tx

oral abx + topical retinoid +/- benzoyl peroxide (tetracycline + tazarotene +/- Benz Pero ….
Retin topical, oral tetracycline then Accutane (isotretinoin)

What do pica and spoon shaped nails indicate?

Iron deficiency anemia

Subungual hematoma

collection of blood underneath a toenail or finger nail. Tx: Trephination, drill hole and drain the blood.

Which HPV viruses most likely to cause malignancies

16, 18, 31, 33, 45, 52,& 58. Gardasil effective against all of these plus 6 & 11

Fifth disease

Starts with a slight fever, achiness, and cold symptoms, followed a few days later by bright red cheeks and a lacy, red, sometimes itchy rash on the trunk and feet. Also called slapped cheeks disease or erythema infectiosum. Most common in preschool and school-age children.

Condyloma Acuminata

genital warts types 6/11 HPV. Treatment: trichloracetic acid….Condylox, aldara, veregne. In pregnancy: TCA or ablation

Image: Condyloma Acuminata

Lead toxicity

*Sx: fatigue. stomach ache, irritability.
*IDA
*Cognitive and behavioral changes cannot be reversed

Hep B Vaccine

– 3 doses.
– don’t ever restart series
– test pregnant women for HBsAg

* if received series and then exposed to Hep B if no HbsAb test done give another dose to be safe

MED FOR MYCOPLASMA PNEUMO

AZITHROMYCIN

primary amennorhea presentation

17 yo Tanner Stage III- never had menses, normal weight and height

A young child with asthma presents for follow-up evaluation. After numerous changes in medications and doses, the parents report that the child continues to have difficulty with coughing, especially during the night. Which of the following conditions would be the most likely cause of the continued asthma symptoms?

1.
Vocal cord dysfunction
2.
Cystic fibrosis
3.
Gastroesophageal reflux
4.
Allergic rhinitis

gastroesophageal reflux

An 88-year-old patient presents with right-sided weakness after being unable to rise unassisted following a fall to the bathroom floor. History includes aphasia and noncompliance with a hypertension medication regimen. What is the most likely diagnosis?

Left-sided stroke

Adam’s sign

Forward bend test for scoliosis

Leukemia

*Most common in children ALL
*Pancytopenia
*Long bone pain
*Hepatosplenomegaly
*Lymphoblasts

Hidradentitis suppurativa tx

Doxycycline, topical atb

What is a common presentation for systemic lupus erythematosus (SLE)?

malar rash (butterfly rash) and should minimize sunlight exposure

Peripheral arterial disease

Impedance of arterial blood flow in lower extremity ankle brachial index <0.9, plaque develops in vessel d/t atherosclerosis, pain with exercise, relief w rest, lack of hair growth on LE, gangrene toes Tx: check pedal pulses, ABI test, exercise by walking, lifestyle Modifications – smoking cessation, antiplatelet

Pelvic inflammatory disease (PID)

causative organisms: N. gonorrhoeae C. trachomatis, bactericides, enterobacteriaceae, streptococci, gram – anarobes
clinical findings: irritative voiding symptoms, fever, abdominal pain, cervical motion tenderness, vaginal discharge. *Increased risk of ectopic pregnancy and/or infertility with each outbreak.
treatment– Ceftriaxone 250mg IM as a single dose plus doxy 100mg po bid x14days with or without metronidazole 500mg PO BID 14days

Folliculitis

Pimples or pustules form around hair follicles and may crust over. Typically occurs on the neck and in the underarm or groin area. Uncommon before age 2.

Syphillis

Syphillis- PAINLESS GENITAL CHANCRE- Condyloma lata. First test do RPR, VDRL SCREENING if reactive then confirm with FTA ABS.

Image: Syphillis

Td/ TdaP

– q10yrs. or if dirty wound give booster if last dose >5yrs ago.
– Tdap is only once (older than 7yo), q10is Td. and during EVERY PREGNANCY
– C/I if Guillain- Barre syndrome

-Dtap 3 doses in children or unimmunized (adults get 2 Dtap and 1 tdap)

SSRIs can cause delayed ejaculation

secondary amennorhea

started but now stopped. Causes are: pregnancy, weight loss, obesity, manipulation. Treatment is hormone replacement, ovulation stimulation (CLOMID) periodic progesteronal

A pregnant woman with known HIV infection can reduce the risk of perinatal transmission through zidovudine (Retrovir) therapy. Based on current research, optimal therapy is to start daily dosing:

1.
post amniocentesis.
2.
after 14 weeks of gestation.
3.
if premature rupture of membranes occurs.
4.
if maternal viral loads are greater than 10,000.

after 14 weeks gestation

A patient with macular degeneration has difficulty seeing objects:

in the center of the visual field.

Kernig’s Sign

Flex hips 90 degrees pain with extension of leg = meningitis or subarachnoid hemorrhage

Headache red flags

*awakens child
*thunderclap
*neuro findings – n/v, AMS
*<3 years old
*absent family hx of migraines

Postherpetic Neuralgia PHN Tx

Prophylaxis: TCA (Elavil)

What is the presentation of polymyalgia rheumatica (PMR) and first-line treatment?

1. Pain, severe stiffness in shoulders and hip girdle (risk for temporal arteritis)
2. Long-term steroids

chronic venous insufficiency

Varicose veins results from venous incompetence secondary to valvular dysfunction. Symptoms LE edema, skin discoloration, ulceration, DVT/PE are complication. WARM TO TOUCH. Tx: light exercise, stockings, wt loss, elevate legs

Vulvovaginitis or STI?
1. Clue cells with alkaline pH
2. Pseudohyphae
3. Abundant WBCs

1. Clue cells with alkaline pH = Bacterial vaginosis
2. Pseudohyphae= Candida vulvovaginitis
3. Abundant WBCs= Nongonococcal cervicitis/vaginitis

Hand, foot, and mouth disease

Fever, loss of appetite, and a sore throat, followed by painful, blisterlike sores in the mouth. Rash on the palms of the hands, the soles of the feet, and sometimes the buttocks. The rash starts as small flat red dots that may turn into bumps or blisters. Most common in preschoolers but can occur at any age.

chanCROID

PAINFUL, purulent lesion (NOT chancre, which is painless and associated with syphillis) oral dose (1 gram) of azithromycin, a single IM dose of 250 mg ceftriaxone, oral 500 mg of erythromycin q.i.d for seven days, or 500 mg of Ciprofloxacin b.i.d for three days.

Image: chanCROID

best laboratory test to distinguish IDA anemia from other anemias is

serum ferritin

whether the pt has reversible airway obstruction

office spirometry using an albuterol nebulizer can confirm asthma, because it can indicate

T-wave inversion with a normal ST segment on a 12-lead EKG may represent:

1.
acute coronary ischemia.
2.
right ventricular hypertrophy.
3.
atrial hypertrophy.
4.
hyperkalemia.

acute coronary ischemia

Which of the following is the leading cause of cancer-related deaths in the majority of women?

Lung Cancer

Brudzinski’s Sign

Involuntary flexion of legs when neck is passively flexed = meningitis

Flu vaccine

– 6mo or older. allergy to egg is no longer a contraindication
– flu mist not to pregnant women

– children 6mo-8yo need 2 doses for first flu season

Cellulitis tx

Pcn, Macrolide

What is the gold standard exam for temporal arteritis?

1. biopsy of the temporal artery
2. Refer to ophthalmology

Bacterial endocarditis

Fever, chills, anorexia, weight loss. malaise, headache, myalgia, arthralgia, night sweats, abdominal pain, dyspnea, cough, pleuritic pain. Symptoms janeway lesions, red spots on the soles of the feet or palms. Osler’s nodes red, tender spots under the skin, whites of your eyes, or inside your mouth.

Splinter hemorrhages on nails, Janeway lesion (red macules palms/soles not painful), Osler’s nodes (painful violaceous nodes found mostly on pads of the fingers and toes).

Intervention in vulvovaginitis
1. Clotrimazole cream
2. Metronidazole gel
3. Oral metronidazole
4. Clindamycin cream
A. Candida vulvoginitis
B. Trichomoniasis
C. Bacterial vaginosis

1. Clotrimazole cream. A. Candida vulvoginitis

2. Metronidazole gel. C. Bacterial vaginosis

3. Oral metronidazole.
C. Bacterial vaginosis
B. Trichomoniasis
.
4. Clindamycin cream. C. Bacterial vaginosis

Impetigo

Small red bumps that may be itchy. Often develops around the nose and mouth but can easily spread to other parts of the body. Bumps become pus-filled blisters that may burst and develop a soft yellowish-brown crust. Child may have a fever and swollen lymph glands in the neck. Most common in children between 2 and 6 years old.

PID

cervical motion tenderness indicates PID.
results in infertility
Treat symptomatic PID even if chlamydia and gonorrhea are negative. plus doxycycline (100 mg orally twice daily for 14 days)

ceftriaxone 250 mg IM x1 + Doxy 100 mg Q12 x 14d Follow up with bimanual exam in 2-3 days

Image: PID

child headaches

*c/o abdominal pain, n/v
*auras common

bump on testicle can indicate cancer of testicle

CBC and hemoglobin electrophoresis

12 yo, family hx of thalassemia and anemia, with Tanner Stage II, HCT 35 should order this

The 16-year-old mother of a 2-month-old presents the infant, reporting that the child is very irritable and does not feed well. During physical examination, the child’s head drops back and the child exhibits sudden flexing of the extremities. As the flexing stops, the child cries uncontrollably. Funduscopic examination reveals retinal hemorrhages. Which of the following diagnostic tests should be ordered?

1.
Skull X-rays
2.
MRI
3.
CT scan
4.
Pet scan

CT scan

Which of the following would be most appropriate to perform in the initial evaluation of a patient with symptoms of acute prostatitis?

Urinalysis and urine culture

Café au lait spots

neurofibromatosis (tumors and seizures)

Varicella

– LIVE
– 2 doses one month apart.
– give if no history of chicken pox

Erysipelas tx

PCN or macrolide

What does a positive Finkelstein test indicate?

de Quervain’s tenosynovitis

Treatment for which of the following STIs?
1. Ceftriaxone
2. Injectable pcn
3. Imiquimod
4. Trichloroacetic acid (TCA)
A. External genital warts in a 25yo man
B. Gonococcal urethritis in a 22 yo man
C. Syphilis in 45 yo man
D. External genital warts in a 28 yo pregnant woman

1. Ceftriaxone-B. Gonococcal urethritis in a 22 yo man
2. Injectable pcn-C. Syphilis in 45 yo man
3. Imiquimod-A. External genital warts in a 25yo man
4. Trichloroacetic acid (TCA)-A. External genital warts in a 25yo man. *D. External genital warts in a 28 yo pregnant woman

Measles

Starts with fever; a runny nose; red, watery eyes; and a cough. Red bumps with tiny white dots appear a few days later on the inside of the cheeks. Next, a rash appears on the face and progresses down the back and trunk to the arms and hands and finally to the legs and feet. The rash starts as flat, red patches but eventually develops bumps and may be itchy. It lasts about five days, then fades to a brownish color, leaving skin dry and flaky. Rare due to vaccinations; most likely to occur in unvaccinated children.

Gonorrhea

Rocephin 250 mg IM and Azithromycin 1 gm po x1, or doxy 100 mg BID x7d. Green colored vaginal discharge, friable cervix

hyperbilirubinemia

*Infants >35 weeks w/ total bilirubin >95th percentile.
*Total bilirubin >25 to 32 = risk for neurologic dysfunction.
*Physiologic – 2 to 3 days after birth (full-term). Peaks day 5 to 7 in preterm infants.
*Pathologic – 1st 24 hours after birth.

PPD >5mm

Induration of 5 or more millimeters HIV-infected persons
-A recent contact of a person with TB disease
-Persons with fibrotic changes on chest radiograph consistent with prior TB
-Patients with organ transplants
-Persons who are immunosuppressed for other reasons

Shingles/Zoster

– 60yo
– regardless of history of shingles

for diabetic patients taking beta blockers, best indication of hypoglycemic episode will be

sweating (WILL NOT SEE TACHY AND PALPITATIONS DUE TO BETA BLOCKER USE)

absence of residual symptoms

differentiates TIA from stroke

The most important diagnostic factor in evaluating angina pectoris is the patient’s:

1.
King of Heart’s monitor.
2.
physical examination.
3.
history.
4.
echocardiogram.

history

Which of the following gastrointestinal changes is associated with normal aging?

Decreased production of gastric acid

Howell-Jolly Bodies and target cells

Sickle Cell Anemia

MRSA tx

Bactrim or tetracyclines

What does a positive Anterior drawer or Lachman maneuver indicate?

Positive indicated anterior cruciate ligament (ACL) is damaged

PPD: 10mm positive for

An induration of 10 or more millimeters
-Recent immigrants (< 5 years) from high-prevalence countries
-Injection drug users
-Residents and employees of high-risk congregate settings
-Mycobacteriology laboratory personnel
-Persons with clinical conditions that place them at high risk
-Children < 4 years of age
– Infants, children, and adolescents exposed to adults in high-risk categories

Genital Candida albicans infection in men typically presents

balanitis, groin-fold involvement, and scrotal excoriation (no penile discharge. Jock itch does not affect the scrotum.)

Milia

Tiny white or yellow pearly bumps on the nose, chin, and cheeks. Common in newborns. They go away without treatment in a few weeks.

mammogram frequency

50-74 q2years unless family history or other concerns (Women with a parent, sibling, or child with breast cancer are at higher risk for breast cancer and thus may benefit more than average-risk women from beginning screening in their 40s.)

Pyloric Stenosis

*Narrowed pyloric sphincter due to hypertrophy
*Sx mostly likely to occur at 3-6 weeks
*Projectile, non-bilious vomit after eating.
*Olive like mass
*Dx with US
*Refer for surgery

Meningococcal MCV4

1-2 doses: 11-12yo and booster at 16yo

lung ca

leading cause of cancer deaths in men and women.

There are three reasons for latex allergies. Which of the following does NOT cause the problem?

1.
Immediate hypersensitivity
2.
Irritant contact of dermatitis
3.
Cytotoxic hypersensitivity
4.
Immune complex reaction

cytotoxic hypersensitivity

Which of the following is NOT an indication of preeclampsia?

Gluosuria

Burtonian Lines

Thin, blue-black discoloration of gingival border = lead poisoning

S1 spinal function is assessed with what DTR

achilles

Xerosis tx

Petroleum based product (not lotions)

PPD: 15mm

An induration of 15 or more millimeters is considered positive in any person, including persons with no known risk factors for TB.

Consideration in caring for a 68 yo m w/a BMI of 38, otherwise healthy, who is now presenting with genital candidiasis includes which of the two most helpful measures?
A. Order a medium potency topical steroid to the affected region to help with symptom control
B. prescribe topical miconazole
C. obtain an in office blood glucose
D. advice on the use of antimicrobial soap to the region

B. prescribe topical miconazole
C. obtain an in office blood glucose

Molloscum contagiosum

Flesh-colored, dome-shaped lesions that can be pearly in appearance and have a dimpled center. May be itchy. Uncommon before the first birthday.

Postmenopausal bleeding f/u testing

– ENDOMETRIAL BX

GER

*Frequent regurgitation
*GERD = pathology
*Immature LES until 9-12 months old
*Red Flags: choking with eating, coughing with eating, forceful vomiting, bilious vomit, blood in stool, poor weight gain, refusal to eat, constipation or diarrhea, abdominal tenderness, fever.
*Avoid smoke, small frequent feedings (thickened), trial of PPI.

What can a positive McMurray’s sign indicate?

Meniscus injury of the knee.

Pneumonia Pneumovax- PPSV-23

– one dose 65yo in well adults
– if vaccinated before 65y need booster in 5 yrs and then again at >65yo

Paxil, SSRI

delayed ejaculation

A patient on warfarin (Coumadin) therapy for recurrent deep vein thrombosis (DVT) is about to have lumbar spinal fusion surgery. The patient’s warfarin is put on hold starting 5 days prior to the surgery and subcutaneous Lovenox has been ordered for DVT prophylaxis until the resumption of the warfarin. The nurse practitioner knows that the patient’s postoperative warfarin dose should be restarted based on the:

1.
value of her morning Prothrombin time.
2.
loading dose of 10 mg, plus the previous warfarin dose.
3.
baseline PT and INR values.
4.
target INR of 2.

caseline PT and INR values

Assessment findings that would support a diagnosis of mitral regurgitation include:

loud, high-pitched pansystolic murmur.

Goodwell’s Sign

Softening of the cervix (pregnancy indication)

children over 8 years old can use doxy

Psoriasis tx

Topical Steroids

Lateral X-ray of the neck

Epiglottitis

Acute, uncomplicated UTI in non pregnant women

Pathogen: E. coli (gram-, most common), Klebsielle spp. (gram-), S. saprophyticus (gram+)
Treatment: E. coli: TMP/SMX-DS PO BID x 3 days (always order double strength). If E. coli resistant or sulfa allergy then nitrofurantoin (Macrobid) 100 mg BID x 5 days or fosfomycin (Monurol) 3 g PO x 1 dose. Add phenazopyridine (Pyridium) PO to assist with symptom control.

Roseola

Usually starts with a sudden relatively high fever, often over 103 degrees Fahrenheit, that typically lasts three to five days. A pink rash on torso and neck follows and may spread to the arms, legs, and face. Child may be irritable and may have diarrhea or vomiting. Most common in children between 6 months and 3 years old.

Atrophic vaginitis cause and tx

lack a estrogen, apply topical estrogens

Intussusception

*Intestinal obstruction
*Sudden onset. Intermittent. Crampy abdominal pain.
*Cries and pulls legs up to chest
*Currant jelly BM
*Dx: US
*Non-operative: enema under fluoroscopy or surgery

On eye exam, what does neovascularization, cotton wool spots, and micro-aneurysms indicate?

diabetic retinopathy

PCV13

– children receive
– give first
– only ever need 1 dose

in doing initial eval of patient with suspected acute prostatitis, what test to do

urinalysis and culture

Somogyi effect

Type 1 diabetes with increased early am levels with correct insulin dose

An 87-year-old patient presents with round, pruritic plaques and small vesicles on the lower legs. The most likely diagnosis is:

1.
allergic contact dermatitis.
2.
plaque psoriasis.
3.
cutaneous T-cell lymphoma.
4.
nummular eczema

nummular eczema

An 18-year-old college student presents for an athletic physical. When asked about current medications, she mentions that she takes “some herb” she bought at a health food store for migraines and menstrual cramps. Which of the following herbal remedies has been commonly used for these conditions?

Feverfew

Chadwick’s Sign

bluish discoloration of the cervix, vagina, and labia resulting from increased blood flow observed 6-8 weeks after conception

On eye exam, what can atrioventricular nicking, silver and/or copper wire arterioles indicate?

Hypertensive retinopathy

Shingles Dx & Tx

Dx: Viral Culture, polymerase chain reaction (PCR)
Tx: Acyclovir, Zostrix cream, Gabapentin, amitriptyline

Croup (laryngotracheobronchitis) Parainfluenza (*)

cold symptoms, low fever, stridor, barking cough, and hoarseness. No drooling, nasal flaring, sore throat, resp distress, ab breathing. Steeple sign frontal radiograph of neck. Mild: Outpt supportive care Mod: Hosp resp support Racemic epi, short course corticosteroids

Which of the following is inconsistent with BPH?
A. obliterated median sulcus
B. size >= 2.5 cm x 3 cm
C. symptoms improved w/use of an alpha-1 receptor blockade such as tamsulosin (Flomax)
D. surgical intervention should be offered early in the disease process

D. surgical intervention should be offered early in the disease process

Ringworm

Rash of one or several red rings, ranging from dime- to quarter-size. Rings are usually crusty or scaly on the outside and smooth in the center and may get larger over time. May also appear as dandruff or bald spots on scalp. Most common in children 2 and older.

WET PREP for which gu infections

BV, YEAST, TRICH

Encopresis

*Involuntary soiling in child 4 years or older
*Underlying issue: constipation
*Loss of urge to defecate
*Management: PEG, Miralax. Behavior change, fiber, fluids, reward system.

Hegar’s Sign

Softening of the cervicouterine junction

Koplik spots

measles

dawn phenomenon

Characterized by hyperglycemia that is present on awakening in the morning due to the release of counterregulatory hormones in the predawn hours.

A 3-year-old presents with a 2-day history of acute diarrhea with a total of 8 watery stools without blood or mucus, and 2 episodes of vomiting in the past 48 hours. Assessment reveals no current antibiotic therapy, dehydration <5%, soft abdomen with hyperactive bowel sounds, no masses or organomegaly; other physical findings are normal. Which of the following is the most appropriate management plan?

1.
Stool culture; Immodium AD liquid.
2.
No laboratory workup; instruct parents on signs and symptoms of dehydration; diet of clear liquids, advancing to bananas, rice cereal, Jell-O, and soup.
3.
Start diet of clear liquids, advancing to bananas, rice cereal, Jell-O, and soup over next 72 hours; Imodium AD liquid, 1 mg/sml
4.
Stool for ova and parasites; advise parents of likely sources of Giardia; prescribe furazolidone (Furoxone)

No laboratory workup; instruct parents on signs and symptoms of dehydration; diet of clear liquids, advancing to bananas, rice cereal, Jell-O, and soup.

An 88-year-old patient has had a gradual onset of hearing loss in the left ear. Examination shows a large accumulation of cerumen in the external auditory canal. Assuming there is no neural loss, the nurse practitioner would expect the Weber test to:

lateralize to the left

BCG

– LIVE for TB
– not in US.

Spider bite tx

Atb on wound, cold packs, NSAIDS

When checking deep tendon reflexes (DTRs) in a patient with severe sciatica or diabetic peripheral neuropathy, what can you expect the Achilles reflex to show?

The achilles reflex may be absent or hypoactive (0-1+)

Epiglottis (*)

Hemophilus influenza, Streptococci, Pneumococci. (6-10) Rapid onset, acute bacterial infection, high fever, stridor, choling sensation, resp distress, drooling, head tilted back to breath (hyperextension). Thumb sign on radiograph. ER!! Cephalosporin

Epididymoorchitis (upper reproductive e tract infection with inflammation of epididymis/testis)

Age ⬇️35
Causative organisms: N. gonorrhoeae, C. trachomatis
Presentation: irritative voiding symptoms, fever, painful swelling of epididymis and scrotum. Infertility is possible post infection due to scarring of the vas deferins
Treatment: Ceftriaxone 250 mg IM x 1 dose plus doxycycline 100 mg po BID x 10 day.
Scrotal elevation to help with symptom relief. (Prehn’s sign=relief of discomfort with scrotal elevation)

Rubella (German measles)

Pink-red rash that appears first on the face, then spreads over the body and lasts two to three days. Child may have a mild fever, swollen lymph nodes behind the ears, a runny or stuffy nose, a headache, and a sore throat. Rare due to vaccinations; most likely to occur in unvaccinated children.

Trichomonas s/s, dx, tx

Dysuria, itchy, strawberry cervix, +/- discharge: grayish bubbly, wet prep: mobile unicellular organisms with flagella tx Flagyl 2g x 1 OR 500 mg q12h x 7d

Image: Trichomonas s/s, dx, tx

Viral Gastroenteritis

*Fecal oral route common
*Measure for dehydration: weight loss, capillary refill, loss of skin turgor, increased and deep RR.
*Oral rehydration for mild to moderate dehydration over 3-4 hours
*50 mL/kg (mild) to 100 mL/kg (moderate) over 4 hours

Dog bite tx

Analgesics: Tylenol, nsaids, Demerol
ATB: Augmentin/doxycycline/Bactrim,

macrolide antibiotic

smoker with acute bronchitis with cough with thick, yellow-green foul smelling sputum

A mother presents her 12-month-old child with concern because the child does not yet say “mama” or “dada.” The mother reports that in the first months of life, the child loved listening to music and being talked to. The past medical history is negative for ear infections. The nurse practitioner should:

1.
evaluate for hearing loss.
2.
refer to speech therapy.
3.
re-evaluate in 3 months.
4.
check for cerumen impaction.

evaluate for hearing loss

During an employment physical examination of a 21-year-old female, bruising around the areolae on the breasts is noted. An appropriate health history for these findings should include all of the following EXCEPT:

socioeconomic status

Chvostek sign

tap the facial nerve in front of the tragus and the ipsilateral muscles of face contract – indicative of hypocalcemia

pernicious anemia is B12 deficiency

True or false, a benign S4 heart sound may be auscultated in some elderly patients

True

Dacrocystitis

inflammation of the lacrimal sac. Symptoms includes thick eye discharge, pain, redness/swelling/warmth of lower eyelid. Tx includes lacrimal sac massage downward toward mouth 2-3 times daily and systemic antibiotics if needed.

Epididymoorchitis usually e-coli

Age ⬆️ 35 or insertive partner in anal intercourse
Causative organisms: Enterobacteriaceae (coliforms)
Presentation: irritative voiding symptoms, fever, painful swelling of epididymis and scrotum. Infertility is possible post infection.
Treatment: Levofloxacin 500 mg PO QD or ofloxacin 300 mg PO BID x 10 days

Scabies

Severely itchy rash of scattered red bumps, usually between the fingers, around the wrists, in the armpits and diaper area, and around the elbows. May also show up on the kneecaps, palms, soles, scalp, or face. May leave curvy white or thin red lines or little blisters on nearby skin. Itching is most intense after a hot bath or at night and may keep the child awake. May occur at any age.

VVC

Candidia Vaginitis- wet smear= pseudohyphae / spores w/ lrg wbc.
Cheese curd like pruritis, itching, swelling, redness. TREAT: Diflucan 100 mgx1. Or OTC= Monistat, clotrimazole

Hydrocele

Common in newborns and is a collection of fluid in the scrotum. Usually resolves before 12 months of age.

most appropriate dx tests for PCOS are

testosterone, LH, prolactin, TSH

The most common side effect of the oral ribavirin used in the treatment of hepatitis C is:

1.
hemolytic anemia.
2.
weight loss.
3.
depression.
4.
hypothyroidism.

hemolytic anemia

Which of the following findings is typically a sign of acute appendicitis?

Rebound tenderness at McBurney’s point

Dix-Hallpike or Nylen-Barany maneuver

+ = Vertigo & nystagmus when quickly move pt to supine position with head 30 degrees lower than table and to turned to the side at 45 degree

What is a rare but life threatening reaction to ace-inhibitors? What is a common side effect?

Angioedema
-cough

Lyme Disease tx

Doxycycline, amoxicillin, or azithromycin

Erysipeals

Skin infection involving upper dermis and superficial lympahtics. Strep A and has clear demarcated boarder, tx includes PCN such as dicloxacillin qid x 10 days, Cephalexin, clindamycin for PCN allergic do Azithromycin.

Acute bacterial prostatitis

Age ⬇️ 35
Causative organisms: N. gonorrhoeae, C. trachomatis
“it hurts when my bottom hits the chair.”
Presentation: irritative voiding symptoms, suprapubic, perineal pain, fever, a tender, boggy prostate, leukocytosis increased white count
Treatment: ceftriaxone 250 mg IM as 1 x dose w/doxycycline 100 mg PO BID x 10 days

Scarlet fever

The rash begins as a mass of tiny red bumps in the armpits, neck, chest, and groin and rapidly spreads over the entire body. The bumps feel like fine sandpaper and may itch. Child may have a fever and a red sore throat. Early in the infection, the tongue may have a white or yellowish coating that later turns red. The bumps on the tongue may appear larger than normal, a condition called strawberry tongue. Tonsils may be swollen and red. As the rash fades, the skin may peel, especially on the hands and feet and in the groin. Uncommon before age 2.

BV : s/s, microscopy, test, tx,

fishy, smelly vaginal discharge, wet smear reveals epithelial cells with bacterial coating, KOH to cotton swab – whiff test, flagyl cream x 7 days, clindamycin cream, flagyl PO

UTI

*Causes: Bowel and bladder dysfunction (withholding, incontinence, constipation), anatomic abnormalities, vesicoureteral reflux.
*Treatment: 2nd or 3rd generation cephalosporin (cefixime, cefdinir, ceftibuten) for 3-5 days if afebrile, 10 days if febrile.
*Renal and bladder US for all infants 2 to 24 months with 1st febrile UTI.
*Voiding cystourethrogram for vesicoureteral reflux.
*Get US in any child with recurrent febrile UTIs and UTI with family hx of renal or urologic disease, poor growth or HTN

Lupus tx

Topical and oral steroids

weight loss, shoulder and pelvic pain, pain with ROM, no weakness, elevated ESR

polymyalgia rheumatica!!!!

An 86-year-old patient presents with pinguecula, a yellow triangular degenerative tissue thickening of the bulbar conjunctiva. This condition is:

1.
cured by eye drops.
2.
a genetic process.
3.
a malignant growth.
4.
a solar-induced lesion

a solar-induced lesion

The best laboratory test to distinguish iron deficiency anemia from other anemias is:

serum ferritin level

McBurney’s Point

RLQ of abdomen that is one-third of the distance from the anterior superior iliac spine to the umbilicus (location of appendix)

Acute bacterial prostatitis (uncomplicated disease in men with low risk for STI)

Causative organisms: Enterobacteriaceae (coliforms)
Presentation: irritative voiding symptoms, suprapubic, perineal pain, fever, a tender, boggy prostate, leukocytosis
Treatment: ciprofloxacin 500 mg PO BID or ofloxacin 200 mg PO daily x 14 days

What class of drug is Ipratropium (Atrovent)

Anticholinergic

Normal HR at birth

120-160

Warts

A small grainy bump or cluster of bumps, usually on a hand but can occur anywhere on the body. Warts are usually skin-toned but may be lighter or darker in color and can contain black dots. Flat warts, which are smaller and smoother, can also appear anywhere on the body, but in children they most often show up on the face. Plantar warts show up on the soles of the feet. Most warts disappear on their own in a few months to a few years. Uncommon before age 2.

LSIL in 21-24 y/o. 25-29 year old. >30 year old

21-24y.o.: Repeat PAP in 12 mo
25-29 yo: refer for colpososcopy/biopsy.
>30 if HPV (-) repeat PAP in 12 mo. If HPV (+) then colp / biopsy

Cellulitis

Skin infection involving deep dermis. Diffuse, acute infection of the skin marked by local heat, redness, pain, and swelling. Tx:. MRSA tx: Bactrim, Clinda, or Doxy. Non MRSA infection: Keflex

Anthrax tx

Doxycycline/Fluoroquinolones (Cipro)

pansystolic murmur

mitral regurgitation

During a well-child examination of an 18-month-old, premature tooth decay and inflamed gums, particularly of the maxillary incisors, are noted. The approach to this problem includes:

1.
referring the child to a dentist, assessing for bottle feeding and fluoride in local water.
2.
teaching the mother to brush the child’s teeth, encouraging vitamin supplements.
3.
recommending fluoride mouthwash and toothpaste, suggesting celery sticks for snacks.
4.
providing fluoride treatment, teaching dental hygiene, and eating foods high in calcium.

referring the child to a dentist, assessing for bottle feeding and fluoride in local water

A patient is referred with a diagnosis of diabetes mellitus, hypertension, and coronary artery disease. The patient is on both insulin and a beta blocker. Assuming that the patient will continue the beta blocker, it will be important to educate the patient on the recognition of hypoglycemia. Which symptom would be most indicative of hypoglycemia in this patient?

sweating

McMurray’s Test

audible/palpable click when knee is raised with tibia externally rotated and then straightened- one hand on knee other on foot

What drug class are erythomycin, azithromycin (Z-pack), clarithromycin (Biaxin)?

Macrolides

Cutaneous anthrax

Normal prostate

firm, smooth, nontender
*about as firm as pressing in on the tip of your nose*

What antibiotic is used for corneal abrasion:

Gentimicin ophthalmic solution

Young woman with breast mass. Preferred imaging

Ultrasound in young (less dense tissue, better image). Can tell if mass is fluid filled cyst or solid

HR of 3 year old

80-120

Raynaud’s Phenomenon tx

CCB

tx corneal abrasion with

gentamicin

Which of the following is typically observed in the GI system upon stimulation of the parasympathetic nervous system?

1.
No overall effect
2.
Increased sphincter tone
3.
Increased peristalsis
4.
Decreased secretions

increased peristalsis

An 88-year-old male presents with concerns about memory loss. He feels good, takes an aspirin daily, and has no chronic diseases. He lives alone, drives his own car, and manages his financial affairs. To evaluate his memory, which of the following tests should the nurse practitioner choose?

Folstein Mini-Mental State Examination

Lachman’s Test

drawer test for acl or pcl test knee at 20-30 degrees one hand on thigh other on proximal tibia, pull tibia forward to assess anterior motion of tibia to femur (intact ACL should prevent anterior movement)

HR of 6 year old

70-110

name first generation cephalosporins

Keflex

Contraindication for beta blockers?

Asthma, COPD, chronic bronchitis, Emphysema, bradycardia and AV block. Inhibits of bronchodilator response to beta agonists

Acute prostate

tender, boggy, indurated
*about as firm as pressing in over your puffed out cheek*

S/s of CHF:

s3, edema, dyspnea, JVD, x-ray reveals cardiomegaly, long standing hypertension, systolic murmur, tachycardia INSERT MORE HERE

Risk factors/findings for ovarian ca

Family history.
Should not ever be able to palpate an ovary, r/o US ovarian CA.
Risks: >50, early menarche, late menopause, obesity, family history, 1st prego after 35, or not ever prego.

Chronic Venous Insufficiency tx

Light exercise, stockings, Wt loss, elevate legs

feverfew is used for

migraines and menstrual cramps

Type 2 diabetes mellitus is characterized by which of the following?

1.
Decreased production of exogenous glucagon-like peptide
2.
Autoimmune beta-cell destruction
3.
Relative deficiency of dipeptidyl peptidase-4
4.
Decreased reabsorption of sodium-glucose transporters

decreased reabsorption of sodium-glucose transporters

Which of the following wet-mount results confirms a preliminary diagnosis of bacterial vaginosis?

Squamous epithelial cells with stippling appearance and indistinct borders, no lactobacillus rods, and many white blood cells

Apley’s Grind test

flex knee 90 degrees with patient prone put pressure on heel hands and rotates leg laterally and medially, repeat while pulling up on leg (+ = limited ROM or pain)

Prostate cancer

nodular, firm, nontender
*usually malignant lesions not palpable until disease is advanced*

Name second and third generation cephalosporins

Second- Cefaclor, Ceftin, Cefzil
Third- Rocephin, Suprax, Omnicef

When to start screening BP

3 years old – age, gender and height

Levothyroxin may worsen?

Osteoporosis

Natural estrogen-

Isoflavones

Adverse effects of beta blockers

Bronchospasm, bradycardia, depression, fatigue, ED, HF, hypoglycemia

Infective Endocarditis tx

Amoxicillin or macrolide

most common 2 pathogens for older adults residing in community

strep pneumo, H. flu

In pulmonary function testing, forced vital capacity represents the:

1.
maximum volume of air that can be forcefully exhaled after maximum inspiration.
2.
total volume of air that is exhaled after normal inspiration.
3.
total volume of air that the lungs can hold, minus the expiratory reserve volume.
4.
volume of air that can be forcefully exhaled in 1 second.

maximum volume of air that can be forcefully exhaled after maximum inspiration

Office spirometry performed with an albuterol nebulizer treatment can confirm a diagnosis of asthma because it indicates:

whether a patient has reversible airway obstruction.

Straight leg raise

lie supine, leg elevated off table produces radiating or sciatic pain – herniated disc

Name a quinolone antibiotic

Ciprofloxacin, ofloxacin

TB

Gold std Culture and stain

Which of the following is least likely to be noted in a 55 yo M who presents with bladder cancer?
A. textile worker for 25 years
B. 60 pk yr smoking history
C. Report of intermittent painless gross hematuria
D. report of recent-onset intermittent acute urinary retention

D. report of recent-onset intermittent acute urinary retention

Testicular torsion s/s?

Absent cremaster reflex, testicular pain, difficulty walking, nausea, MORE?

Ectopic pregnancy s/s, physical exam findings, increased risk for

Light to scant bleeding in 6-7 weeks/lower abd pain/pelvic pain. Intermittent cramping, if radiating to right shoulder think rupture.

Pain is worsen with SUPINE or with JARRING. Previous ectopic pregnancy, tubal ligation, PID. Anything that is causing scaring

what age should dyslipidemia risk assessment begin?

2 years old. 2, 4, 6, 8, 10 then annually

Ventricular Septal Defect tx

Lasix & Lisinopril

what class of anti-HTN meds is absolutely CONTRAINDICATED in bilateral renal artery stenosis and has been associated with acute renal failure

ACEs – With stenosed renal arteries, afferent flow cannot be increased, angiotensin II can actually become the only mechanism by which the kidney can increase filtration, and with severe stenosis, efferent arteriole constriction is crucial for maintaining some degree of filtration. ACE inhibitors would prevent conversion of Ang. I (which is inactive) to Ang. II, and therefore make everything worse by removing the kidney’s only remaining regulatory mechanism as well as cutting the perfusion pressure and eliminating what little renal function remains. With tubular cells that are already less effective due to damage from chronic ischemia, the resulting decrease in GFR would effectively accelerate kidney failure .

An adult male presents as first patient of the day after awaking about 5:00 AM with indigestion and chest pressure. On checking his pulse, he found no change from previous measurements. He states that, while waiting for the nurse practitioner to arrive, he began to feel a little clammy. A 12-lead EKG records the following patterns. On the basis of these EKG readings, the diagnosis is:

1.
anterior-septal acute myocardial infarction.
2.
lateral acute myocardial infarction.
3.
inferior acute myocardial infarction.
4.
anterior acute myocardial infarction.

inferior acute myocardial infarction

A 32-year-old patient reports a 6-month history of intermittent symmetrical swelling of the wrists and daily morning stiffness lasting 1 hour or more in and around other joints. What is the most likely diagnosis?

Rheumatoid arthritis

Pelvic Rock test

Screens for sacroiliac joint dysfunction. Place hands on bilateral anterior superior illac spines and attempt to rotate SI joint, or side lying apply pressure to ilium (+ = pain at SI joint)

CHD is the most common what in newborns?

congenital anomaly

Name a quinolone with Gram + coverage

Levaquin, moxifloxacin, gatifloxacin

ectopic pregnancy

Light scant bleeding in 6-7 weeks, lower abd pain, intermittent cramping, pain radiating to right shoulder and pain is worsen with supine position

Which of the following is a worrisome finding noted during pelvic exam on a 62 yo F?
A. flattening of the vaginal rugae
B. vaginal pH=5.6
C. ovary palpable on bimanual exam
D. scant white vaginal discharge

C. ovary palpable on bimanual exam

Alternate ABT for erythromycin for mycoplasma pneumonia?

macrolide such as azithromycin.

BPH physical findings, tx, when to take meds, evaluating PSA post tx,

BPH- Symmetrical rubbery and enlarged. All of BPH meds, take at bedtime. Hytrin is good for ppl with HTN and BP; If on Proscar must times PSA X2.

HCTZ should not be taken with digoxin

may increase risk of digoxin toxicity

During the past 24 hours, a 62-year-old has experienced abdominal pain that radiates to the back. The patient also reports several episodes of nausea and vomiting, a low-grade temperature, and a history of excessive drinking. Physical examination reveals a distended abdomen. Laboratory serum values indicate elevated alkaline phosphatase, amylase, and serum lipase. The most likely diagnosis is:

1.
alcoholic liver disease.
2.
acute mesenteric ischemia.
3.
viral hepatitis.
4.
acute pancreatitis.

acute pancreatitis

A 47-year-old patient presents with complaints of upper abdominal discomfort with nausea and burning after eating. The patient does not currently take any medications. The most likely differential diagnoses would include:

gastritis and peptic ulcer disease.

Prehn’s sign

lifting the scrotum relieves pain of epididymitis

Cholesterol <200
HDL >40
LDL <100
Triglycerides <150

Lifestyle modification first line
Niacin, fenofibrate

Name two sulfa drugs

1. Trimethoprim-sulfamethoxazole (Bactrim, Septra)
2. Nitrofurantoin (macrobid)

Herniated disc and sciatica

better with standing and worse with sitting

Alternate ABT for common conditions such as Pneumonia, Atypical pneumonia, MRSA, Impetigo, Bladder infection, Otitis media etc

INSERT HERE

Acute bacterial prostatitis: S/s, physical exam findings, labs, tx based on age

High fever, chills, suprapubic, perineal pain, radiates to back or rectum, s/sx of uti.
Prostate is warm and boggy.
UA/ Culture is definitive. CBC shift to left (band cells) UA, pyuria, hematuria.
Older than 35= cipro, levaquin 4-6 wk. other Bactrim.
<35-Rocephin 250mg IM and doxy 100 mg BIDx10d.

Innocent murmur clues

*Grade <2
*Softer intensity when sitting compared to supine
*Not holosystolic
*Minimal radiation
*Musical or vibratory quality

Urge incontinence

reports of strong sensation of needing to void that cannot be supressed, often has involuntary loss of urine
most common in older adults
Treatment: behavioral therapy, pharmacotherapy (antimuscarinics such as: Detrol,Ditropan, VESIcare, Enablex, Toviaz. Adverse effects = dry mouth, sedation, mental status change, particularly in higher doses.

Addison’s Disease tx

Steroid Cortisol

Erythema multiforme

a milder form of Stevens-Johnson syndrome that produces pink-to-red targetlike lesions, wheals, and blisters, with no mucosal involvement. The clinician should look for a history of antibiotic and other drug treatment such as sulfa drugs, penicillins, and other drugs. Hypersensitivity reaction from meds, allergy, or infection.

An otherwise healthy adult female presents complaining of vaginal burning and itching during urination. She has a social history of two sexual partners. She denies fever or vaginal discharge, but has noted a lesion on one labium. The nurse practitioner should order all of the following EXCEPT:

1.
wet mount and potassium hydroxide.
2.
dark-field microscopy of fluid from the lesion.
3.
nucleic acid amplification testing for gonorrhea.
4.
serology for Haemophilus ducreyi.

dark-field microscopy of fluid from lesion

A 2-month-old infant is presented for examination and immunizations. History includes an uncomplicated full-term delivery and hepatitis B virus immunization shortly after birth. Examination is unremarkable except for a diffusely erythematous (non-beet-red) macular rash in the diaper area, sparing the inguinal folds. No satellite lesions are noted. The infant’s diaper rash is most likely caused by:

contact dermatitis.

Romberg Test

Evals proprioception and cerebellar fxn; stand with feet together, arms at side, close eyes. + = loss of balance

Name two tetracycline drugs

1. doxycycline
2. minocycline (Minocin)

Lumbar stenosis

is aggravated by long periods of standing and walking, relieved by sitting and rest.

Stress incontinence

associated with lifting, coughing, sneezing, exercise, anything that causes intraabdominal pressure
more common in F, rare in M
Treatment: vaginal tampon, urethral stents, periurethral bulking agent injections, pessary use. Kegel exercise in younger, premenopausal patients. Pelvic floor rehab w/biofeedback, electrical stimulation and bladder training.

Side effects and uses of ortho-novum and other antibiotics:

INSERT HERE: spotting? amenorrhea etc

Generalized anxiety disorder

SSRI, SNRI, Wellbutrin. May do benzo for short time (for acute anxiety attack)

Pathologic murmur clues

*Grade >3
*Holosystolic
*Max intensity at LUSB
*Harsh or blowing quality
*Systolic clicks
*Diastolic murmurs
*Increased intensity in upright position
*Gallop
*friction rub

Meniere’s Disease tx

Meclizine (Antivert), diuretics, diazepam

how is MS dx

lumbar puncture

A 3-year-old female has been diagnosed with bacterial meningitis. She attends preschool daily at a local church day care program. The day before her diagnosis, she ate lunch with her mother at a local restaurant. For which contact(s) would chemoprophylaxis with rifampin be recommended?

1.
Preschool contacts in the past 7 days
2.
A friend of the patient’s sister who visited after school yesterday
3.
The checkout employee at the grocery store yesterday
4.
Diners at the restaurant during lunch the day the patient ate there

preschool contacts in past 7 days

A nurse practitioner is evaluating a 40-year-old patient suspected of having a pulmonary embolus. The patient complains of anxiety and cough. A stat chest X-ray is normal. Which of the following tests should the nurse practitioner perform next?

Helical CT pulmonary angiography

Finger to Nose test

Tests cerebellar function – patient alternately points from their nose to examiner’s finger

Barlow-Ortolani maneuver

– To detect hip dislocation in newborn

What category of drugs are allowed for pregnant or lactating women? (A, B, C, D)

Category B
ex: pick acetaminophen over NSAID. Avoid nitrofurantoin and sulfa drugs during third trimester (hyperbilirubinemia)

sentinel nodes (virchow’s nodes)

Left superaclavicular area. They are the first lymph nodes that a cancer lesion will drain into. Therefore, when cancer is diagnosed, these nodes are biopsied to see whether the cancer has spread into the lymph system.

Functional incontinence

often occurs I presence of mobility problems, associated w/the inability to get to the toilet or the lack of knowing that there is a need to void.
Usually found in those with limited mobility or altered cognition. Worsens when there is limited availability of an assistant for toileting.
Treatment: Ameliorated by having an assistant who is aware of voiding cue available to help with toileting activities.

Orthonovum (estrogen based contraception) causes spotting? T/F

True

PTSD, major and minor depression, first choice meds?

SSRI Paxil (sex dysfunc causes). Zoloft, Celexa (Good for older few drug interactions) Lexapro. Gradually wean paxil.

Hep B tx

Peg-IFN, Interferon, ETV, TDF

diabetes can contribute to pregnancy induced HTN

Congenital dysplasia of the hip:

1.
more commonly affects both hips.
2.
occurs more commonly in females.
3.
is correctable at any age.
4.
typically self-corrects by 12 weeks of age.

more commonly affects both hips

A 65-year-old male presents to a clinic complaining of increasing fatigue, dyspnea on exertion, and ankle edema during the day. He has a history of mild hypertension, for which he saw his physician years ago. The physician advised the patient to decrease his salt intake. On physical examination, the patient is tachycardic, positive for jugular venous distention, and positive for S3, with a systolic murmur. Chest X-ray reveals cardiomegaly with vascular redistribution. The nurse practitioner’s diagnosis should be:

heat failure

Heel to shin test

Tests cerebellar function – runs heel of one foot along the shin of the opposite leg fairly quickly

What HTN medication can cause acute renal failure and is contraindicated in bilateral renal stenosis?

ACE inhibitors. How to monitor? Test renal function.

What is the preferred treatment for cutaneous anthrax and how do you get it?

1. Ciprofloxacin 500mg BID for 60 days (8 weeks)
2. Doxycycline 100mg BID if cipro allergy
3. Comes from touching fur or animal skins that are contaminated with anthrax spores (not contagious)

Transient incontinence

occurs during an acute illness such as UTI, delirium, medication use, restricted activity
Treatment: discontinuation of offending medication.

Where should the examiners 2nd-5th fingers be placed to perform Ortolani’s

the greater trochanter

Acute serotonin syndrome, s/s , causes

s/s Dilated pupils, high fever, muscular rigidity, mental status changes, hyperreflexia/clonus, shivery.
causes: SSRI, MAOIs, TCA. Could be potentially life threatening

syncope

Blood glucose. Checking the blood glucose is indicated for patients with syncopal and near-syncopal episodes. The nurse practitioner should also perform a thorough history of the incident. Possible causes of syncope are cardiac arrhythmia, vasovagal, hypoglycemia, orthostatic hypotension, seizure, accidental fall, and others.

Hep C tx

Antivirals and PEG-IFN

Anticholinergic agents such as ipratropium (Atrovent) and tiotropium (Spiriva) are used in COPD primarily to:

1.
induce bronchodilation.
2.
decrease airway inflammation.
3.
expand the lung fields.
4.
treat hypoxemia.

induce bronchodilation

Trigeminal neuralgia manifests itself primarily with:

electric-shock-like unilateral facial pain.

Cullen’s sign

bruising or bluish discoloration and/or edema around the umbilicus = pancreatitis, ruptured ectopic pregnancy, aortic rupture, abdominal trauma

hold assistive device (like cane) on opposite side

Optic disk

normal examination has sharp margins, a yellowish orange to a creamy pink color, and round or oval shape. The ratio of veins to arteries is 3:2

Treatable causes of urinary incontinence

(DIAPPERS)
Delirium
Infection (urinary)
Atrophic urethritis and vaginitis
Pharmaceuticals (diuretics, others)
Psychologic disorders (depression)
Excessive urine output (heart failure, hyperglycemia due to undetected or poorly controlled DM)
Restricted mobility
Stool impaction

Acute bacterial sinusitis is treated with?

amoxicillin

Lumbar stenosis

Aggravated by long periods of standing and walking. Relieved by forward flexion, sitting and rest.

Club foot AKA

talipes equinovarus

sickle cell, thalassemia and leukemia can all cause

hepatosplenomegaly – but IDA wil not

The clinical presentation of placenta previa that develops during the third trimester includes:

1.
intermittent dark-red spotting.
2.
occult bleeding with abdominal or back pain.
3.
sudden onset of painless and profuse bleeding.
4.
painless occult bleeding that becomes visible upon onset of labor.

sudden onset of painless and profuse bleeding

Which of the following criteria differentiates a TIA from a stroke?

Absence of residual symptoms

Grey Turner’s Sign

bruising or bluish discoloration of the flanks indicative of retroperitoneal hemorrhage = pancreatitis

arcus senilis

gray-white arc or circle around the limbus of the iris that is common with aging. caused by lipid deposits deep in the edge of the cornea and is quite commonly present in elderly. However, it can also appear early in life as a results of hypercholesterolemia

Idiopathic Thrombocytopenia purpura tx

Initial: Glucocorticosteroids (prednisone)

Overactive bladder medication with highest anticholinergic effect and lowest

Highest is oxybutynin (Ditropan)
Lowest is mirabegron (Myrbetriq)
Not used to treat OAB: tamulosin (Flomax) or finsasteride (Proscar)

S/s of acute bacterial sinusitis versus viral?

Insert here. How long before treat? Alternative to amox allergy?

sciatica

sciatica is a form of radiculopathy and one of the most common causes is a herniated disc. So both are aggravated by long periods of sitting.

metatarus adductus

“c” shaped foot or “kidney bean shaped” foot without ankle deviation. in-toeing

Myerson’s Sign / Glabellar tap sign

tapping between eyebrows, above bridge of nose (glabella) produces a sustained blink response = Parkinson’s Disease

VSD sx

cyanosis and poor feeding – murmur is heard in tricuspid area

A 27-year-old male patient, who works as a janitor, presents with a 6-month history of an intermittent rash on his hands. History reveals itching and occasional burning. Examination reveals irregularly-distributed scaly maculopapular erythematous patches extending from the dorsum of the hand several inches up the forearms, and dry palms with no nail involvement. The most likely diagnosis is:

1.
contact dermatitis.
2.
scabies infestation.
3.
psoriasis.
4.
eczema.

contact dermatitis

A 20-year-old male presents with a 1-month history of a “bump on my testicle.” He reports being sexually active, with frequent episodes of unprotected intercourse with multiple partners. Which condition should be suspected?

Cancer of the testicles

Ishihara chart

A chart used to test for color blindness. (7 yr old)

Contraindications for Metformin

Significant renal & hepatic disease, Sepsis, Alcoholism, Hypoxia causing conditions (cardiac/pulmonary), dehydration, & advanced age.

Healthy 32 yo F who is taking combined oral contraceptives, is here to review the results of her recent liquid-based Pap screening that revealed atypical squamous cells of unknown significance(ASCUS) and high risk HPV +. No h/o previous abnormal cervial cytology, with her last screening aprox 2 yr ago. After discussing the significance of these findings, the most appropriate next step is to:
A. Advise that she return in 6-12 months for a repeat Pap w/HPV cotesting
B. obtain screening tests for N. gonorrhoeae and C. trachomatis infection.
C. referral for colposcopy
D. counsel that the usual cervical cancer screening guidelines​ should be followed

C. referral for colposcopy

Delayed ejaculation is caused by a common medication:

Paxil- SSRI

Treatment to prevent fracture in patient with low vitamin D high TSH low HCT

– VITAMIN D 600-800, CALCIUM 1000-1200.

Nursemaid’s elbow

Annular ligament displacement over the radial head

Macrocytic Folate Deficiency tx

Folate, leafy green veggies, beans, liver, pasta, cereals

Anorexia nervosa occurs most commonly in which of the following?
1.
Painters
2.
Only children
3.
High-level athletes
4.
Individuals from large families

high level athletes

A 65-year-old female presents with shoulder and pelvic girdle pain for the past 6 months. She reports recent unintentional weight loss. On physical examination, there is pain on ROM, with no weakness noted. Laboratory studies show a low hemoglobin and an elevated sedimentation rate. Which of the following is the most likely diagnosis?

Polymyalgia rheumatica

Murphy’s sign

Deep pain on inspiration while fingers are placed under right rib cage = cholecystitis

Down’s syndrome patients can have

congenital heart defects, seizures, eye issues, hearing loss, obesity – NOT failure to thrive

Straight leg raise

This test is performed by laying on your back with both legs straight and the physician raises one of your legs upward, keeping the knee straight. This is done to help find the reason for low back and leg pain. (sciatic nerve/herniated disk)

Which of the following is demonstrated to provide the most symptom relief in treating vasomotor symptoms? (hot flashes)
A. clonidine
B. paroxetine
C. conjugated estrogen
D. venlafaxine

C. conjugated estrogen

Uncomplicated chlymydia: may present with?

No clinical s/s:
Difference between complicated and uncomplicated chlamydia?

Morton’s neuroma

Morton’s Neuroma- do mulder test. “pebble, burning, numbness” ¾ metatarsals. PUNT TO pod

Mulder sign or foot squeeze test – Sharp pain b/w 3rd & 4th toe when foot is squeezed in pt w/ MORTON NEUROMA

Scoliosis

>10 degree curve. Use Adam’s forward bend test

hyperinflation of lungs can be seen on chest X-ray in patients with asthma

A frail elderly patient presents with constipation. Which of the following normal physiologic changes seen with aging is the most likely cause?

1.
Decreased bowel muscle tone
2.
Increased bile secretion
3.
Increased absorption of calcium
4.
Decreased pancreatic secretions

decreased bowel muscle tone

An 85-year-old patient relates that on the way to an annual physical examination, the patient suffered a sudden loss of vision in the right eye characterized by “a bunch of lights” and a feeling that “a curtain came down.” The most likely diagnosis is:

retinal detachment.

Psoas sign

RLQ pain with passive right leg elevation off table or with right thigh extension side lying = appendicitis

Lachman’s Test

ACL instability of the knee

What is Galaezzi sign

one femur appears shorter when infant is supine

Risk Factors and Cancer:
Obesity and history of PCOS: endometrial cancer
HPV 16 and/or 18: Cervical cancer
BRCA 1 and BRCA2 gene mutation: Ovarian cancer
Unexplained abdominal bloating and constipation: Ovarian cancer

Cancer:
Abnormal vaginal bleeding: Cervical and Endometrial cancer
Often without significant signs or symptoms: All Cervical, Endometrial and Ovarian cancer
Pap test: Cervical cancer
No specific screening tests available or recommended: Ovarian and Endometrial cancer

Treatment for chlamydia?

INSERT HERE: azithromycin. Alternative tx?

Tibial overuse stress syndrome

“inner border” painful on palpation.

DO bone scan or MRI cuz a plain X-ray won’t show a stress fracture.

FOLLOW RICE.

Osgood-Schlatter disease

*Anterior knee pain
*Osteochondritis of the tibial tubercle
*Common during growth spurt
*Pain reproduced with extending the knee against resistance
*Straight leg raise is painless
*RICE and quadricep strengthening

What is Acute Dacryostenosis? How to dx?

redness warmth tenderness and swelling of one lacrimal duct, complication of orbital cellulitis
Dx: Lacrimal sac massage & culture of discharge then treat with atb.

A patient reports to your office, having been struck on the right cheek with a baseball bat. On examination, you notice tenderness and swelling over the cheek. The patient is unable to gaze laterally with his left eye, and he complains of double vision when he attempts to do so. This is most likely associated with:

1.
retinal hemorrhage.
2.
orbital rim fracture.
3.
subarachnoid hemorrhage.
4.
periorbital cellulitis.

orbital rim fractures

A 70-year-old male presents with paresthesia of the lower extremities. On examination, the patient appears pale and shows a decreased vibratory sense. Laboratory tests reveal elevated indirect bilirubin; Hct = 30%; mean corpuscular volume (MCV) = 120 µm3 [normal = 87-103 µm3]. The most likely diagnosis is:

pernicious anemia

Obturator sign

RLQ pain with internal rotation of flexed right thigh = appendicitis

Kerley’s B lines on X-ray indicate

Pulmonary edema

MMR and pregnancy

Don’t get pregnant within the next 4 weeks

COPD patients may prolong their life by the use of?

Oxygen

Rheumatoid Arthritis s/s

Early morning stiffness, sausage joints. Symmetrical involvement. Longer stiffness than OA. Joint space narrowing. Pain, warm, tender, swollen, things.

Hip pain causes?

Legg-Calve-Perthes disease, SCFE, irritable hip

Hydrocele

Collection of serous fluid that causes painless scrotal swelling, easily recognized by transillumination.
Common in older men.

What is Caput Succedaneum

Diffuse edema of scalp due to intrauterine pressure (Cone head)

A 7-year-old child presents with group A streptococcal infection confirmed by throat culture. Past history includes treatment for positive streptococcal infection with erythromycin (EryPed oral suspension) 3 weeks ago. What is the most appropriate next intervention?

1.
Treat with amoxicillin for 10 days.
2.
Obtain culture for all household members.
3.
Treat with azithromycin (Zithromax) for 10 days.
4.
Intramuscular Penicillin.

treat with amoxicillin for 10 days

A 14-year-old patient who fell on an outstretched hand complains of proximal forearm pain. X-ray reveals a positive fat pad sign, and the patient is unable to fully extend the elbow. No definitive bony changes are seen on X-ray. The most likely working diagnosis is:

radial head fracture

Rovsing’s Sign

RLQ pain when pressure is applied to the LLQ = appendicitis

Depo provera can cause

menstrual irregularities and amenorrhea

caude equina syndrome

Varicocele

A palpable “nest of worms” scrotal mass that is only evident in a standing position.

COPD- other points of interest?

INSERT HERE

Rheumatoid Arthritis tx

: NSAIDS, steroids, DMARDS, TNF.

Legg-Calve-Perthes Disease

*Osteonecrosis of the capital femoral epiphysis due to interrupted vascular supply
*Common 3-12 years and more in boys
*Pain in hip or radiation to knee.
*Limp
*Trendelenburg Test – positive pelvic tilit

when rx hormones, consider fam hx of what cancers

uterine and breast

A 16-year-old female adolescent is brought to the nurse practitioner’s office by her mother, who is concerned about her daughter’s recent weight loss. History reveals that the daughter was consistently in the 50th percentile for weight, but is now in the 10th percentile. The mother states that she is concerned that her daughter is purging herself after meals since she often goes to the restroom after eating and remains there for a long period of time. The daughter denies any self-induced vomiting, starvation or excessive activity. She does state that she jogs five miles a day and is in good condition. In addition to a complete blood count with differential, which of the following laboratory tests will be most helpful for further assessment?

1.
Electrolytes, fasting blood sugar
2.
Creatine phosphokinase, follicle-stimulating hormone (FSH)
3.
Electrolytes, blood urea nitrogen (BUN)/creatinine, urinalysis
4.
Electrolytes, FSH, stool for occult blood

electrolytes, blood urea nitrogen (BUN)/creatinine, urinalysis

A 72-year-old female presents with a medical history unremarkable except for the occasional headache. She enjoyed emotional health throughout marriage and childrearing, although her family history is significant for strokes and nervous breakdowns. Since her husband died a year ago, however, she has been waking every morning at 3:00 A.M., is reluctant to go out although she has family members living nearby, and feels a continual urge to sleep. She cannot discuss her late husband without crying. Differential diagnoses would include:

depression, unresolved grief, and hypothyroidism.

Kerley B lines

Horizontal lines on CXR = heart failure

Phalen’s test

The test is performed by pushing the back of the hands together for 1 minute. This compresses the median nerve within the carpal tunnel. Characteristic symptoms (burning; tingling; numbness over the thumb, index, middle, and ring fingers) convey a positive test result.

Hirschsprung’s disease

A condition of the large intestine (colon) that causes difficulty passing stool.in first 48 hours of life, meconium

Testicular torsion

Characterized by scrotal pain and loss of the cremasteric reflex (Stroke inner thigh for contraction of crenatic muscle- testicle raise).
Urologic emergency.

Serous in middle ear may remain in Acute otitis media for weeks after the infection: any treatment needed?

no.

Swan Neck Deformity

deformed position of the finger, in which the joint closest to the fingertip is permanently bent toward the palm while the nearest joint to the palm is bent away from it (DIP flexion with PIP hyperextension). Associated with RA

SCFE

*hip/knee pain
*limp
*common in teens
*Trendelenburg test

Korsakoff Syndrome

chronic thiamine deficiency

An adult female who is homeless presents for an initial obstetric visit at 34 weeks of pregnancy. She is diagnosed with Chlamydia trachomatis infection. What complication will her newborn be most at risk?

1.
Conjunctivitis
2.
Hearing loss
3.
Pneumonitis
4.
Meningitis

conjunctivitis

A 66-year-old patient presents with bilateral otitis media with effusion and white patches in the mouth that do not rub off when wiped with a 4 × 4. The patient should be evaluated for:

HIV infection

Levine’s sign

Clenched fist over chest sign = ischemic chest pain

nuva ring does not affect menstrual cycle

Osteopenia is defined as T-score

-1.0 to -2.5

Phimosis

The foreskin cannot be pulled back to expose the glans. head of penis
Potential urologic emergency.

Acute otitis media tx?

alternative? INSERT HERE

Bouchard’s nodes

Swelling to PIP, in RA there will be Bouchard’s nodes (and not Heberden’s nodes)

Image: Bouchard's nodes

Transient Synovitis of the Hip

*”irritable hip”
*limp
*hip pain
*small effusion on US
*h/o URI 1 week prior
*no x-rays
*resolves in 1-2 weeks

implant can help with cramps

A 2-year-old male presents for reevaluation. Two days ago, he had four episodes of vomiting and six diarrheal stools. On physical examination today, his vital signs reveal P = 120 and capillary refill of 3 seconds. The patient’s eyes are sunken and his extremities are mottled and cool to the touch. The treatment plan would include:

1.
start oral electrolyte mixture every hour.
2.
hospitalization for IV fluid replacement.
3.
giving the child loperamide for each diarrheal stool.
4.
catheterizing the child to obtain accurate urine output.

hospitalization for IV fluid replacement

A 17-year-old female has never had her menses. She is at Tanner stage III of sexual development. Her physical examination is completely normal, and her weight is appropriate for her age and height. What is the most likely diagnosis?

Primary amenorrhea

Palm Sign

palm of the hand to the chest = ischemic chest pain

Osteoporosis is defined as T-score

-2.5 and less

Dystonia

involuntary repetitive muscle movements muscle spasms

Paraphimosis

Retracted foreskin that cannot be brought forward to cover the glans.
Potential urologic emergency.

S/s of retinal detachment?

Feels like curtian coming over vision, flashes of light OTHERS? TX?

Osteoarthritis s/s, first line tx

Large weight bearing joints. Early morning stiffness with inactivity. Has both nodes. FIRST LINE Acetaminophen.

Roseola

Human herpesvirus 6. High fever for 2-4 days, then abrupt cessation of fever with appearance of maculopapular rash but not on face

itraconazole – oral

A nurse practitioner places a 76-year-old patient on nifedipine (Procardia) 10 mg t.i.d. for angina. The patient is unable to remember to take the medication at the scheduled times. The practitioner should:

1.
discontinue the issue with the patient’s daughter.
2.
change the dose to extended release 30 mg daily.
3.
reinforce the importance of taking the medication.
4.
increase the dosage to 20 mg b.i.d.

change the dose to extended release 30mg daily

According to the American Diabetes Association, the newest standard for determining the presence of diabetes mellitus based on fasting plasma sugar level is a value equal to or greater than:

126 mg/dL.

Arm Sign

touching the left arm with right arm = ischemic chest pain

Thayer-Martin culture

Used to diagnose gonorrheal pharyngitis or proctitis

Dyskinesia

abnormal involuntary muscle rigidity

Cryptorchidism

Testicle located in inguinal canal or abdomen. Undescended testicle.

S/s of epididimitis?

???? Prinse test

Osteoarthritis recommended exercise

EXERCISE: Isometric exercises for knee OA. Non-weight bearing, like biking, swimming, stationary bike.

Fifth Disease

Parvovirus B19. “Slapped cheek” rash. Lacy. Macular rash.

griseofulvin

tinea capitis

A pediatric male is present with his mother, who states that he is extremely restless at night and constantly scratches his anal area. Which diagnostic procedure would confirm a diagnosis of pinworm infestation?

1.
The patch test
2.
Examination of bed linens
3.
Examination of transparent tape sample from perianal skin
4.
Blood tests for anemia and eosinophilia

examination of transparent tape sample from perianal skin

A 38-year-old male experiences a sudden onset of acute abdominal pain radiating to his groin. He is having difficulty walking and is nauseated. He denies any trauma or previous groin injuries. Examination reveals diffuse swelling of the left testicle and negative cremasteric reflex. The most likely diagnosis is:

testicular torsion.

Pointing Sign

pointing to chest with 1 finger = non-ischemic chest pain

Positive strep allergic to PCN

clarithromycin (biaxin) 250 mg bid x 10 days

Symptoms of initial hearing loss in adults

Loss of high frequency tones & harder to hear consonants than vowel sounds

Current guidelines recommend initiating antiretroviral therapy (ART) when?

in all individuals with HIV-1 infection, including those with early HIV-1 infection and all HIV-infected women, including pregnant women. It should be started regardless of the CD4 T lymphocyte cell count.

Varococele?

Sack of worms

Lachman’s: how to do, what does it test for

flex knee to 30 degrees, slightly externally rotate tibia, stabilize femur, pull up on tibia. If feel laxity (more forward movement) then there is laxity of ACL (instability of knee joint)

Rubella

Rubella virus. Maculopapular rash. Looks like measles rash. Remarkable lymphadenopathy. Macules on soft palate (Forchheimer spots)

miconazole – topical

Which of the following microorganisms are most frequently associated with acute bacterial rhino-sinusitis?

1.
Staphylococcus aureus and Mycoplasma pneumonia
2.
Staphylococcus aureus and Methicillin Resistant Staph aureus
3.
Streptococcus pneumoniae and Haemophilus influenzae
4.
Streptococcus pneumoniae and Pseudomonas aeruginosa

strep pneumo and haemophilus influenzae

Akathesia

intense need to move, restlessness

Babinski’s Sign

big toe remains extended or extends itself and other toes fan out when the sole of the foot is stimulated (normal in children until age 2). If seen child over 2 years or adult = CNS disorder (ALS, brain tumor/injury, meningitis, MS, spinal cord injury/defect, stroke)

A 10-month-old presents with a rash, runny nose, and cough. Examination reveals a cluster of tiny white papules with an erythematous base on the buccal mucosa. What does this suggest?

Measles

Jarisch-Herxheimer reaction

Infections caused by a spirochete such as syphilis or lyme. They dying bacteria release antigens that causes an host reaction. (occurs treating early stage syphilis. Symptoms are headache, myaligas, rigors, sweat, hypotension, and worsening of rash. Resolves on its own.

Cancer of testes?

nodule on teste
OTHERS?

Anterior drawer sign

pulling up on tibia more than expected shows tear/laxity in ACL. Flex hip to 45 degrees, flex knee to 90 degrees – sit on foot.

Image: Anterior drawer sign

Rubeola (measles)

Rubeola virus. Maculopapular “brick red” rash. Starts on head and neck, spreads centrifugally to trunk and extremities. reportable disease. 3 C’s (conjunctivitis, coryza, cough). Koplik spots (1-3mm whitish, bluish or gray elevations on the buccal mucosa and hard/soft palate)

What is the goal of antiretroviral therapy (ART)?

suppress plasma viral RNA to undetectable levels and can be important in preventing and managing opportunistic infections.

Akinesia

reduced voluntary muscle

The parent of a 13-year-old diagnosed with seizure disorder calls to report that the child is exhibiting symptoms of a cold, but has no fever. The nurse practitioner should advise the parent that the development of fever may:

lower the seizure threshold.

Battle Sign

bruising around the mastoid process (behind ears) = serious head / brain injury

A child is being treated with methylphenidate (Ritalin) for attention-deficit disorder. The following are all side effects of Ritalin EXCEPT:

1.
muscle cramps
2.
mild irritability.
3.
increased heart rate.
4.
slight increase in growth velocity.

slight increase in growth velocity

Dx: Of BV

Amsel criteria with microscopy: (need 3)
-white thick adherent discharge
-vaginal pH >4.5
-positive whiff-amine test, KOH
-clue cells on saline wet mount (greater than 20%) – epithelium, “bread crumb dip cells”
gram stain, DNA probe. (Epithelial cells dotted with with large numbers of bacteria that obscure cell borders)

S/s of gonorrhea?

Treatment?

Posterior drawer sign

flex knee 90 degrees. sits on toes of the tested extremity to help stabilize it. The examiner grasps the proximal lower leg, approximately at the tibial plateau or joint line, and attempts to translate the lower leg posteriorly. The test is considered positive if there is a lack of end feel or excessive posterior translation

Image: Posterior drawer sign

Chickenpox

Herpes virus. Vesicular lesions on erythematous base appearing in crops

16-24 hours sexually active screen for chlamydia is reportable There is also EPT= expedited partner therapy

additional testing= for STI risk reduction
GC/Chlamydia
HIV
syphilis
HAV
HBV
HCV
HSV-2

Agnosia

inability to recognize familiar people or objects

A geriatric female presents for her annual examination. She has been on antihypertensive medications for over 20 years, with good control. Laboratory values are within normal ranges. The nurse practitioner is concerned about the patient’s cardiac health risks, due her to weight and her waist circumference. According to the AHA guidelines, which of the following goals is expected for this patient?

1.
Physical activity for 60 minutes daily, for a minimum of 6 days a week
2.
Physical activity for 30 minutes daily, 7 days a week
3.
Physical activity for 30 minutes daily, for a minimum of 5 days a week
4.
Physical activity for 60 minutes daily, 7 days a week

physical activity for 30 minutes daily, for a minimum of 5 days a week

After a 3-week camping trip, an 11-year-old is seen for a target lesion with central clearing, located in the inguinal area. The patient has had a severe headache, malaise, fatigue, and generalized musculoskeletal pain for several days. Pharmacologic management of this condition includes:

doxycycline (Doryx).

Blue Dot Sign

tender nodule with blue discoloration on the upper pole of the testicle = testicular torsion

vulvovaginitis or STI dx with clinical findings or microscopic examination
1-Bacterial Vaginosis
2-Candid vulvovaginitis
3-Nongonococcap cervicitis/vaginitis

A-Clue Cells with alkaline PH
B- Pseudohyphae
C- Abundant White cells

BV= Clue cells with alkaline ph
Candida Vulvovaginitis= Pseudohyphae
Nongonococcal cervicitis/vaginitis= Abundant white blood cells

Legal scope of practice

defined by the nurse practice act in the state where you practice.

Torsion of testes?

S/S

McMurray’s test – how to do, what does it test for

McMurrays test- “CLICK” medial meniscus (valgus) EXAM. LateRal meniscus (vaRus).

With the patient supine the examiner holds the knee and palpates the joint line with one hand, thumb on one side and fingers on the other, whilst the other hand holds the sole of the foot and acts to support the limb and provide the required movement through range. The examiner then applies a valgus stress to the knee whilst the other hand rotates the leg externally and extends the knee. Pain and/or an audible click while performing this maneuver can indicate a torn medial meniscus. To examine the lateral meniscus the examiner repeats this process from full flexion but applies a varus stress to the knee and medial rotation to the tibia prior to extending the knee once again.

Image: McMurray's test - how to do, what does it test for

Herpangina

Coxsackie A virus

Astereognosis

inability recognize object placed in palm

An 18-year-old patient presents with complaints of maxillary facial pain and yellow nasal discharge for 14 days. What is the appropriate initial pharmacologic intervention?

Amoxicillin-Clavulanate (Augmentin)

Cremaster Reflex

lightly stroke superior and medial thigh – normal response is elevation of the ipsilateral testicle = negative elevation seen in testicular torsion (not 100%)

The management of COPD in the elderly is best guided by:

1.
spirometry.
2.
arterial blood gases.
3.
radiologic imaging.
4.
symptomatology.

symptomatology

Osteoporosis

Kyphosis causes curvature of the spine that causes rounding of the back, slouching posture. Severe thinning of bones.

Hand, foot and mouth

Coxsackie A virus

An intervention in vulvovaginitis-
1-Candida vulvovaginitis
2-Trichomoniasis
3-BV

A-Clotrimazole Cream
B-Oral Metronidazole
C-Metronidazole gel
D-Clindamycin cream

1-Candida vulvovaginitis
2-Trichomoniasis
3-BV

A-Clotrimazole Cream= Candida
B-Oral Metronidazole= Trich and BV
C-Metronidazole gel= BV
D-Clindamycin cream= BV

Rotator cuff tendonitis

after repetitive overhead activities (throwing, raking), abduction and elevation of shoulder joint worsen symptoms.

disturbed sleep because of pain, arm weakness, dull ache

Image: Rotator cuff tendonitis

Pt with pulmonary embolus suspected. After x ray what is next test?

helical CT pulmonary angiogram

Scarlet fever

exotoxin rash secondary to group A strep. Sandpaper like rash that ultimately desquamates

Hook test

bicep tendon rupture, normally with the elbow in flexion and arm supinated, you can hook your finger under the biceps tendon at the level of the AC and it will meet with resistance as you pull your finger against the tendon. A distal bicep tendon tear means that your finger will not feel resistance or be able to hook under the tendon in a hook test. (positive).

Symogi effect v dawn phenomenon:

Symogi: hypoglycemia causes hyperglycemia
Dawn phenomen: increase in insulin due to growth hormones, cortisol etc

Treatments for STI- matching

External genital warts in a 25 yo adult
Gonococcal urethritis in a 22 year old adult
Syphilis in a 45 yo adult
External warts in a 28yo pregnant woman

Ceftriaxone
Injectable PCN
Imiquimod
Trichloroacetic acid TCA

Ceftriaxone= Gonococcal urethritis in a 22 year old adult
Injectable PCN= Syphilis in a 45 yo adult
Imiquimod= External genital warts in a 25 yo adult NEVER in A PREGNANT WOMAN

Trichloroacetic acid TCA= External warts on the 25 yo and thee 28 yo who is pregnant

Ataxia

difficulty cording voluntary movement

Finkelstein’s Test

grasp the thumb and ulnar deviate the hand sharp and quick towards pinky = + De Quervain’s tenosynovitis

A 40-year-old male presents with a 2-week history of rectal pain and itching. He reports a past history of constipation and finding spots of bright red blood on toilet paper several times a week. Rectal examination reveals a tender, swollen, bluish, ovoid mass. The stool guaiac test is negative. Which of the following actions should the nurse practitioner take?

Prescribe bulk-forming agents and hydrocortisone suppositories.

A young, athletic adult presents with shoulder pain and inability to raise the arm above the shoulder. The most likely diagnosis is:

1.
frozen shoulder syndrome.
2.
cervical radiculopathy.
3.
rotator cuff tear.
4.
polymyositis.

rotator cuff tear

Thiazide diuretics

Favorable effect in pts with osteopenia and osteoporosis. They slow down the kidney’s excretion of calcium and increase distal tubule Ca+ reabsorption. Decreases bone demineralization.

Apraxia

difficulty to remember learned motor skill

A 50-year-old male presents with bruises and abrasions on the left leg and forearm, claiming to have had a number of minor accidents at home and at work. The patient reports general feelings of depression, forgetfulness, and, despite the fact that he is regularly employed, a mounting collection of unpaid and overdue bills. In taking the patient’s history, a significant pattern of parental alcohol abuse, stroke, TIA, and MI emerges. During questioning to elicit a more thorough diagnosis, the patient becomes hostile and leaves the clinic without further treatment. The most likely explanation for this behavior is:

1.
Alzheimer’s disease.
2.
chronic alcoholism.
3.
cancer.
4.
carotid artery stenosis.

chronic alcoholism

A nurse practitioner is suturing a simple laceration on an 11-year-old patient. The use of lidocaine with epinephrine is contraindicated in all of the following areas EXCEPT the:

scalp

Homan’s Sign

pain in the calf on forceful and abrupt dorsiflexion of the patient’s foot at the ankle = possible DVT

Which signs and symptoms are associated with-epididymoorchitis
acute bacterial prostatitis
or both

Irritative voiding symptoms
perineal pain (between scrotum and anus)
Prehn’s sign
Usually presents with fever
Scrotal swelling

Irritative voiding symptoms- BOTH
perineal pain (between scrotum and anus)- Prostatiits
Prehn’s sign =Epididymoorchitis
Usually presents with fever=BOTH
Scrotal swelling-Epididymoorchitis

Lachman’s maneuver and sign

Knee joint laxity. Suggestive of ACL damage of the knee.

How to test s1 nerve function?

achillies tendon reflex

bicep tendon rupture – looks like

bicep is curled up into a giant ball

herpangina

painful vesicles on the soft palate and mouth

hand foot and mouth resolves when

2-3 days

blood tests for sickle cell

screening is CBC, diagnosis is HGB electrophoresis
– HbS and HbF elevated, Reticulocytosis, hemolytic anemia
– Normocytic/Normochromic
– shortened life-span of 10-20d (norm 120d)

How to test s2 function? S3, s4? etc?

?

Cancers of the Female Reproductive Tract

– cervical cancer caused by HPV types=abnormal vag bleeding, vag discomfort during intercourse, malodorous dc and dysuria- symptomatic w 1st abnormal pap
pap test followed by colposcopy and biopsy
pelvic ct/MRI or PET
Tx based on disease progression surgery radiation chemo

– uterine or endometrial cancer- older age , estrogen tx, nulliparity, obesity, tamoxoifin use, DM family hx-abnormal vag bleeding, Diagnostics – transvag ultrasonography hydroutrsonography endometrial biopsy and D&C

– ovarian cancer- post menopausal nuliparity 1st preg after 35 fertitlity drugs Braca + has shared etiology with breast ca
minimal non specific symptoms bloating bladder pressure constipation vag bleeding indigestion, SOB, lethargy weight loss
Diagnostics- CT w/ contrast of abdomen,
MRI,US tumor markers 125
surgery followed by chemo

Drawer sign

A test for knee stability. A diagnostic sign of a torn or ruptured ligament. The positive anterior drawer sign is the test for the anterior cruciate ligament (ACL). The posterior drawer sign is the test for the posterior cruciate ligament (PCL).

Anomia

form of aphasia, unable to recall names of everyday objects

Pel Epstein fever

cyclic fever = rarely seen in Hodgkin’s lymphoma

A 16-year-old female in the first month of taking Ortho-Novum 7/7/7 complains of midcycle spotting. She has not missed any doses and uses no other medication. Which of the following is appropriate?

Providing reassurance

A 28-year-old patient presents with profuse yellow vaginal discharge, odor, and local irritation. She reports completing a 7-day course of oral metronidazole (Flagyl), 500 mg b.i.d. 4 weeks ago. She has had multiple recurrences over the last 18 months. Microscopic examination reveals presence of clue cells. What is the most appropriate treatment intervention?

1.
Oral metronidazole (Flagyl), 500 mg b.i.d., plus metronidazole vaginal gel (MetroGel), for 5 days
2.
Treat both patient and partner(s) with oral metronidazole (Flagyl), 500 mg b.i.d., for 7 days
3.
Metronidazole vaginal gel (MetroGel) twice weekly for 4 to 6 months
4.
Treat patient with metronidazole vaginal gel (MetroGel) b.i.d. and partner(s) with tetracycline, 250 mg q.i.d., for 7 days

Treat both patient and partner(s) with oral metronidazole (Flagyl), 500 mg b.i.d., for 7 days

Aphasia

difficulty verbalizing

An adult female who recently returned from a business trip to Japan presents for a recheck appointment. The only remarkable laboratory result is for thyroid-stimulating hormone (TSH), at 0.3 microunits/mL (normal = 0.4-6 microunits/mL). The patient reports that her neck hurts; examination reveals thyroid tenderness. Which of the following laboratory tests should the nurse practitioner order now?

1.
Triiodothyronine (T3) and free thyroxine (FT4)
2.
Triiodothyronine (T3) only
3.
Triiodothyronine (T3) resin uptake assay
4.
Triiodothyronine (T3) and free triiodothyronine (FT3)

Triiodothyronine (T3) and free thyroxine (FT4)

Research findings have shown that, in order to improve the longevity of a patient who has COPD, the treatment of choice is:

oxygen

vaccination for sickle cell pts

all vax given Preventive strategies that decrease the risk of infection are the routine use of daily antibiotics until five years of age, immunization of children with the 7-valent pneumococcal conjugate vaccine in addition to the 23-valent polysaccharide pneumococcal vaccine, annual influenza vaccination after six months of age, and meningococcal vaccination after two years of age

Finkelstein’s test

identifies deQuervain’s tenosynovitis which is caused by an inflammation of the tendon and its sheath, which is located at the base of the thumb. Test is positive if there is pain and tenderness on the wrist (thumb sign) upon ulnar deviation.

Lidocaine and epinephrine can be used on fingers, earlobes, nose?

False- may lack blood supply due to epinephrine.

match the following cancer types

1-cervical cancer

2-endometrial cancer

3-ovarian cancer

A-Risk factors include obesity and personal history of PCOs (do not shed lining of uterus obesity increases estrogen

B-Long term infection with HPV19 and or-18

C-BRACA1 and BRACA2 gene mutation

D-presentation includes unexplained abdominal bloating and constipation

E-Presentation includes abnormal vaginal bleeding

F-Often without significant clinical signs and symptoms until later disease

G-Screening include PAP

No specific screening tests available or recommended

Risk factors include obesity and personal history of PCOs (do not shed lining of uterus obesity increases estrogen= Endometrial Ca

Long term infection with HPV19 and or-18+Cervical Ca

BRACA1 and BRACA2 gene mutation= Ovarian Ca

presentation includes unexplained abdominal bloating and constipation=Ovarian

Presentation includes abnormal vaginal bleeding= Cervical and Endometrial Ca

Often without significant clinical signs and symptoms until later disease=Cervical, ovarian and endometrial

Screening include PAP=Cervical

No specific screening tests available or recommended =Endo and ovarian ca

otitis media

*Cloudy, bulging TM with impaired mobility
*Effusion – fluid collection w/o infection
*PCV has decreased AOM
*Viral – RSV, influenza
*Bacterial – strep pneumoniae, H. influenzae, M. Catarrhalis

McMurray Test

knee pain and click upon manipulation of the knee, injury to medial meniscus, (KNEE FEELS LIKE LOCKING UP) DX gold standard test for joint damage is the MRI (supine raise knee twisting knee inward/outward while extending in/out)

In treating a pregnant female with migraine headaches, which of the following drugs is in Category X?

1.
Ergotamine tartrate (Ergomar)
2.
Sumitriptan succinate (Imitrex)
3.
Frovatriptan (Frova)
4.
Amitriptyline (Elavil)

ergotamine tartate (ergomar)

Sacrcopenia

loss of muscle mass related to aging

A routine laboratory assessment of a 12-year-old patient with a family history of thalassemia and anemia reveals Tanner stage II presentation and Hct = 35%. In addition to a complete blood count (CBC), the nurse practitioner should order which of the following?

Hemoglobin electrophoresis

Cervical Screening Guidelines

screening should begin at age 21
21-65 it should be every 3 years.
21-29 every 3 yrs.
30-65 – every 5 + HPV.
> 65 after 3 normal paps – and negative HPV or terminate paps
NO NEED FOR ANNUAL SCREENING FOR AVERAGE RISK WOMEN AT ANY AGE.
WOMEN WITH HYSTERECTOMY with cervix removed – NO SCREENING.

scarlet fever: s/s and treatment and alternative ABT for those who cannot accept 1st line ABT.

???

lead poisoning anemia type, s/s

microcytic hypochromic anemia. s/s n/v, fatigue, loss of appetite, abdomen and joint pain, slowed growth, mental disability.

AOM management

* <6 months: ABX
* 6 months to 2 years – ABX ASAP if bilateral or severe illness. Otherwise observe if unilateral or mild
* >2 years – ABX if severe, observe if not severe or uncertain of diagnosis
* Amoxicillin.
* Recent ABX – Augmentin
* Mild PCN reaction – cefdinir, ceftriaxone, cefuroxime, cefpodoxime
* Severe PCN allergy – Zpack or clindamycin

Prostatitis- first test to dx?

urine and culture.

Varus stress test (knee)

LCL ligament stability — test should be done at both 0 and 30 degrees.

1) Examiner places fingers of one hand over the lateral joint line assessing for increase in joint line opening while applying a varus stress to the other.
2) Must be done bilaterally for comparison in order to asses laxity/disruption of ligament.

Positive test is that one joint line is opening is larger than the other side. Indicates possible LCL ligament laxity on the side of the larger joint line opening.

Cachexia

underlying illness involving marked weight loss and muscle loss

A common rule to follow when prescribing many medications for the elderly is to:

start at a lower dose than what is commonly prescribed for adults, and increase the dose slowly.

A nurse practitioner is evaluating an infant for possible colic. Which of the following could indicate the need for a more extensive evaluation?

1.
3-oz (85 g) weight gain over the past 2 weeks
2.
Stool negative for occult blood
3.
Moist mucus membranes and flat fontanels
4.
Onset at 4 weeks of age

3-oz (85 g) weight gain over the past 2 weeks

thalassemia alpha – which population

African, Asian

Which of the following would be an appropriate alternative to erythromycin therapy in an 18-year-old patient with Mycoplasma pneumoniae infection?

Azithromycin (Zithromax)

Which of the following is a hallmark of lumbar spinal stenosis?

1.
Incontinence of bowel and/or bladder
2.
Point tenderness of the lumbar spine
3.
Leg, buttock or back pain precipitated by walking.
4.
Bilateral leg pain with sitting.

leg, buttock or back pain precipitated by walking

thalassemia beta – which population

African, Mediterranean, Middle Eastern

Macular Degeneration

loss of center vision, loss of visual acuity, contrast sensitivity but still have peripheral vision, may find Drusen bodies

Valgus Stress Test

application of a medial force to the lateral aspect of a joint in an attempt to create a gap in the medial joint line, thereby testing the stability of the medial aspect of the joint. Medial collateral ligament

Prostitis s/s, tx?

???

thalassemia type of anemia

– Genetic d/o that bone marrow produces abnormal hemoglobin
– microcytic, hypochromic, ferritin/iron normal or high, high RDW
– Gold standard: Hgb electrophoresis: abnormal in beta
– Blood smear: microcytosis, anisocytosis, poikilocytosis
– Ferritin and iron normal
– Provide genetic counseling
– Minor trait does not require treatment

Acute Bronchitis with purulent discharge with fever tx? (NOT PCN!)

Primary Options:

azithromycin : 500 mg orally as a single dose on day one, followed by 250 mg once daily for 4 days, or 500 mg once daily for 3 days

Secondary Options

erythromycin base : 250-500 mg orally every 6 hours for 7-14 days

clarithromycin : 250 mg orally every 12 hours for 7-14 days

doxycycline : 100 mg orally every 12 hours on day one, followed by 100 mg once daily for 7-14 days

trimethoprim/sulfamethoxazole : 160/800 mg orally every 12 hours for 7-14 days

cephalexin : 250-500 mg orally every 6 hours for 7-14 days

NSAIDS

effect the renal prostaglandins and the effectiveness of the diuretic. NSAIDS inhibit the vasodilatory effect of PG, which predisposes the kidney to ischemia. Decrease blood flow to kidney.

Cataracts

opacity in lens, difficulty with glare, halos around light, blurred vision, gradual onset of decreased night vision, red reflex disappears (Red reflex is now opaque gray instead of orange red glow

A patient with a diagnosis of diverticulosis presents with localized left lower quadrant discomfort, a palpable mass, mild leukocytosis, and T = 100°F (37.8°C). The patient does not appear toxic and can tolerate fluids. An appropriate plan should include:

clear liquids and oral antibiotics.

A 70-year-old patient presents to the clinic complaining of dyspnea, palpitations, and fatigue. The patient reports a 2-week history of blackened stools, which the patient attributes to drinking berry juice. Assessment reveals vital signs of BP = 110/60, P = 100, R = 24; Hgb = 4.5 g/dL; Hct = 16%. What is the most appropriate immediate intervention?

1.
Order serum iron, total iron-binding capacity (TIBC), and ferritin.
2.
Refer to a gastroenterologist.
3.
Send to the emergency room.
4.
Order a complete blood count (CBC) with differential.

Send to emergency room

6 months hx of 39 year old pt who has daily swelling of wrists?

rheumatoid arthritis

treatment of iron deficiency anemia

– 325 mg ferrous sulfate daily treat 3-6 months, take w Vit C for better absorption
– Increase fiber and fluid to prevent constipation. May notice black-colored stools.
– Red meat, beans, green leafy vegetables
– Do not take w antacids, dairy, FQ, tetracyclines
– recheck reticulocytes/CBC after 2 weeks after tx started to make sure the problem isn’t in the bone marrow (the reticulocytes should be higher).

The symptoms of benign prostatic hypertrophy are often overlooked because the older male:

1.
understands that surgery is the only cure for this condition.
2.
considers urinary problems to be a normal part of aging.
3.
is fearful of the side effects of additional medication.
4.
is concerned that he will no longer be able to obtain an erection.

considers urinary problems to be normal part of aging

The most common bacteria responsible for pneumonia in older adults residing in the community is:

Streptococcus pneumoniae.

Pancreatic Cancer Dx and Tx?

Dx: AST ALT bilirubin lipase amylase
Refer to GI surgeon for Whipple procedure

Preeclampsia recommendations

Bed rest with bathroon privi, weight and BP monitoring, follow urine protein and serum protein, Cr and plt counts.

Lichen sclerosis

Characterized by thinning of the epidermis and fibrosis (sclerosis) of the dermis; presents as a white patch (leukoplakia) with parchment-like vulvar skin. Small white spots that grow in patches. Itchy, discomfort, bleeding + blisters. In older women.

how to differential b/t B12 and folate deficiency – both are macrocytic

B12 comes with neuro symptoms tingling numbness of hands/feet, starts peripherally migrates centrally, difficulty walking, difficulty fine motor

Multiple myeloma- cancer of the bone marrow, bone pain, weakness… how to dx & tx?

Dx: CBC, FOBT,
Refer to hematologist affects African race

Which class of antihypertensive agents has been associated with acute renal failure and is contraindicated in patients with bilateral renal artery stenosis?

Angiotensin-converting enzyme (ACE) inhibitors

Upon admission to a nursing home, an 85-year-old patient receives a negative result on the purified protein derivative (PPD) test. To determine TB status in the frail elderly, a second PPD should be performed in:

1.
72 hours.
2.
1 week.
3.
3 months.
4.
6 months.

1 week

Rheumatoid arthritis dx?

Polymyalgia Rheumatica

inflammatory disorder causing muscle and joint pain and stiffness (shoulders/hips), aching stiffness about the upper arms, neck, lower back and thighs, symptoms develop quickly, and worse in the morning

An 18-month-old child presents with a bulging, immobile tympanic membrane; T = 103°F (39.4°C). Assessment also reveals a grade II/VI systolic murmur at the left sternal border. After initiation of treatment for otitis media, the most appropriate intervention is to:

reevaluate the patient in 10 days.

Which of the following is NOT used for therapy of an acute attack of gout?

1.
Colchicine (Colcrys)
2.
Indomethacin (Indocin)
3.
Methylprednisolone (Solumedrol)
4.
Allopurinol (Zyloprim)

allopurinol (zyloprim)

Tanner stage 1 girls

-Preadolescent, no pubic hair, other than fine hair similar to abdomen
-No breast dev.

Reiters syndrome?

pernicious anemia cause/tx

destruction of parietal cells in stomach leads to decreased intrinsic factor leads to B12 not being absorbed.

causes of b12 deficiency

– Macrocytic Normochromic
no intake of foods of animal origin, bariatric surgery, antacids, h2 blockers, proton pump inhibitors, metformin

Gouty arthritis?

Tanner stage 2 girls

-sparse growth of long slightly pigmented, downy, straight
-small area of glandular tissue w/ surrounding areola that begins to widen around age 10-11. Breast Bud

Polymyalgia Rheumatica dx & tx?

DX elevated sed rate, Creactive protein, (Low hemoglobin)
line TX oral prednisone

The management of choice for polymyalgia rheumatica includes:

prednisone.

A 26-year-old female presents with abdominal distention, bloating, and intermittent crampy abdominal pain relieved by defecation. She has four to six loose stools a day when stressed. Between bouts of diarrhea, she often has constipation. Physical examination results are normal. The CBC was normal. The most likely diagnosis is:

1.
diverticulitis.
2.
viral gastroenteritis.
3.
inflammatory bowel disease.
4.
irritable bowel syndrome

irritable bowel syndrome

how does fever affect seizure threshold

fever lowers seizure threshold

A mother has just noticed that her 15-month-old, 15-kg child has recently ingested an unknown quantity of iron tablets. The mother has ipecac syrup and activated charcoal on hand. The mother should be advised to:

1.
take the child to the primary care provider.
2.
give ipecac per recommendation on the bottle label.
3.
take the child to the emergency room.
4.
give the child 30 g of activated charcoal diluted with water

take child to emergency room

Club foot (Other name for?)

talipes equinovarus,

Which of the following pharmacotherapeutics would be most important to administer to a patient who has a corneal abrasion?

Gentamicin ophthalmic (Genoptic)

s/s of brain tumor:

gets worse when lying down and neurological focal deficits

Tanner stage 3 girls

Breast enlargement without separate nipple contour. One mound.
Pubic hair fills out but is straight, starts to curl

A 13-year-old is concerned because she has not yet begun to menstruate. Physical examination indicates that the patient is at Tanner stage IV and is of average height and weight. Which of the following would be the most appropriate response to this patient?

1.
“We’ll need to refer you to an endocrinologist for a complete workup.”
2.
“We’ll need to do some tests to find out why you are not having periods.”
3.
“I’ll give you some pills that will make your periods start.”
4.
“Your development is exactly as expected for your age; you’ll probably begin to have periods within a year.”

“Your development is exactly as expected for your age; you’ll probably begin to have periods within a year.”

ABSENCE SEIZURE other name, define

PETIT MAL SEIZURE: SUDDEN BRIEF LAPSES OF IN ATTENTION. SEEN ON

Migraine s/s? Tx? differ from tension, cluster, stroke?

?

Tanner stage 4 girls

Areola and breast projects as secondary mound
Pubic hair is more thicker, darker, coarse – starts to curl

Bouchards nodes can be found in?

RA & OA

A patient with type 1 diabetes mellitus who is on NPH and regular insulin split-dosing presents with complaints of early morning rise in fingerstick blood glucose. A review of an at-home glucose test reveals increased morning levels. After an increase in the evening insulin dose, the problem worsens. This is most likely an example of:

the Somogyi effect.

A 14-year-old male with bronchitis is being treated with fluids and expectorants. He returns to the clinic with a fever of 103°F (39.4°C), right pleuritic chest pain, and green sputum. Which of the following examination results would be expected?

Right lower lobe crackles

mini mental status exam MMSE – what is the cut off for dementia

MMSE- <24 dementia. The lower your score the worse off you are

Most common cause of death cancer Women? Men?

Lungs!

Tanner 1 Boys

Prepuberty/none

Heberden’s nodes can be found in?

OA

The most common bacterial cause of cellulitis in patients with diabetes mellitus is:

1.
Pseudomonas aeruginosa.
2.
group B beta-hemolytic Streptococcus.
3.
Staphylococcus aureus.
4.
Staphylococcus saprophyticus.

Staphylococcus aureus

subdural hematoma

skateboarding concussion. Diagnosed with CT.

List of cancers incidence?

skin: melatonin cancer most deadly?
Basal cell cancer: most incidence?

Tanner 2 Boys

testes enlarge/few straight fine hairs

Tubercle

small round projection on bone or surface

The follow-up examination of a 12-month-old treated 2 weeks ago for bilateral otitis media reveals a normal left tympanic membrane and a right tympanic membrane with visible serous fluid levels and decreased mobility. An appropriate plan should include a:

follow-up evaluation of the ears in 2 months.

A 46-year-old female found a lump in her breast this morning. History includes: no prior breast disease; G2P2 (first birth, age 22); maternal aunt diagnosed with breast cancer at age 72; last menstrual period was 2.5 weeks ago. On examination, a nurse practitioner palpates a 2-cm round, soft, mobile, tender lesion in the upper outer quadrant of the right breast. Mammogram is negative. Which of the following actions is most appropriate?

1.
Reassure the patient that no further action is necessary.
2.
Repeat unilateral right mammogram in 3 months.
3.
Schedule a right-breast ultrasound.
4.
Refer for genetic testing if the lump increases in size.

schedule a right breast ultrasound

fibromyalgia

11/18 points. Widespread pain for at least three months

Image: fibromyalgia

Cancer death lists?

Tanner 3 (boys)

Penis lengthens, darker coarse hair starts to curl

Colles fx

fracture of distal radius of the forearm, DINNER FORK fracture, most common type wrist fracture

A 50-year-old male presents with a chief complaint of malaise. Further questioning reveals that his primary concern is delayed ejaculation. He is currently taking the following medications: atenolol (Tenormin), 50 mg daily; paroxetine (Paxil), 20 mg daily; loratadine (Claritin-D), 1 tablet daily; and hydrochlorothiazide (HCTZ), 25 mg daily. The most likely cause of the patient’s concern would be:

paroxetine (Paxil).

Risk factors for genital herpes include:

1.
multiple sexual partners and years of sexual activity.
2.
genetic transmission.
3.
infection with other sexually transmitted diseases before the age of 12.
4.
multiple sexual partners and frequent use of public spas.

multiple sexual partners and years of sexual activity

A 68-year-old female presents with a new onset of left-sided throbbing headache. She has noticed some spots in her visual fields that come and go. She is being treated with an NSAID for arthritis. Sedimentation rate is elevated, but all other laboratory values are within normal limits. The headache is most likely due to:

temporal arteritis.

On physically assessing a newborn’s head, you note an enlarged, fluid-filled area that is subperiosteal and does not cross suture lines. This finding indicates the presence of:

1.
macrocephaly.
2.
subgaleal hemorrhage.
3.
congenital hypothyroidism.
4.
cephalohematoma.

cephalohematoma

BPPV

Dix Hallpike maneuver-

Diverticulitis

small pouch like herniations, acute onset of fever, LLQ pain, anorexia, N/V,
Dx CBC, leukocytosis, shift bands to left,
TX fluoroquinolone, flagyl, , if severe refer to ER

Tanner 4 boys

penis widens, thicker, darker, coarse, curly hair

Sexual organs cancer deaths?

Ovaries in women
What about men? Prostate?

cluster headaches: tx, abortive and prophylaxis

tx High dose O2 via Mask 12L 100%, imitrex (abort) and a CCB for prophylaxis. VERAPAMIL

s/s of skin cancer:

tanner 5 boys/girls

Adult pattern

Pancreatitis symptom

midepigastric pain radiates to midback

Which of the following is the most serious outcome of Barrett’s esophagus?

Esophageal adenocarcinoma

A 65-year-old patient being treated with digoxin (Lanoxin) for chronic heart failure (HF) presents with complaints of palpitations. Three days ago, the patient was placed on clarithromycin (Biaxin) by another provider for community-acquired pneumonia. The appropriate course of action at this point would be to:

1.
increase the digoxin dosage by 25%.
2.
obtain a digoxin level and change the antibiotic to azithromycin (Zithromax).
3.
obtain a chest X-ray.
4.
obtain a serum potassium level and perform a 12-lead EKG.

obtain a digoxin level and change the antibiotic to azithromycin (zithromax)

A 39-year-old patient was diagnosed with acute bronchitis in the emergency department and treated with acetaminophen, dextromethorphan, and metaproterenol (Alupent). The patient’s history reveals a smoking habit of 1 pack per day. The patient now presents to a nurse practitioner’s office with a fever of 101.2°F (39.4°C) and a cough productive of thick, yellow-green, foul-smelling sputum. The nurse practitioner should encourage smoking cessation and prescribe:

a macrolide antibiotic.

A 54-year-old male who is 30 pounds overweight states that he awakens at night with heartburn and the taste of hot acid in his mouth. Stress makes his condition worse, yet baking soda seems to provide some relief. Findings on examination are normal and the stool for occult blood is negative. The preliminary diagnosis(es) should be:

1.
esophagitis.
2.
esophageal spasm.
3.
helicobacter pylori.
4.
gastroesophageal reflux disease.

GERD

migraine tx: abortive and prophylaxis

Triptains for abortive.
Prophylaxis:
– (propranolol)
-TCA (Amitriptyline)
– Anticonvulsants (gabapentin, topiramate).

Lichen Sclerosis

whitish patches on genitals, lesions chronic affect vulva/anal in elderly patients, risk for cancer

eGFR

test used to monitor kidney function and evaluate chronic kidney disease

Anemia: pernicious anemia:

neurological deficits

Bells Palsy, which nerve

CN VII- Facial BELLS

Other anemia: s/s, tx, differentiation

Signs of depression

anhedonia, weight loss or gain, fatigue, change in appetite, insomnia or hypersomnia, feeling of guilt or suicide.

Thrichomonas

yellow-greenish discharge with dyspareunia, pelvic exam reveals strawberry patches on cervix

A 15-year-old patient returns for contraceptive services 2 weeks after a diagnosis of trichomonas vaginitis and treatment with 2 g of metronidazole (Flagyl). She reports that discharge and itching are gone, but she is urinating frequently, accompanied with a lot of burning. The patient has not resumed sexual activity and has menstruated since her last visit. Examination reveals mild suprapubic tenderness, no leukorrhea, and a normal wet mount. Gonococcal culture and chlamydia tests are negative. Which diagnostic test should be performed immediately?

Microscopic examination of urine

A late adolescent patient presents with complaints of acute-onset unilateral right eye pain, visual changes, seeing halos around lights, abdominal pain, and intermittent nausea. Physical findings include an erythematous right eye without discharge, a cloudy cornea, and a moderately dilated right pupil that is nonreactive to light. The nurse practitioner should immediately:

1.
apply steroid drops to the right eye.
2.
apply mydriatic drops to the eye.
3.
refer to the emergency department.
4.
make an appointment for him to see an ophthalmologist.

refer to emergency department

A nurse practitioner orders pulmonary rehabilitation for a 75-year-old with COPD. Expected outcomes of this program include all of the following EXCEPT:

increased lung capacity.

In a 6-year-old child, which of the following is the preferred drug for stage 1 Lyme disease?

1.
Trimethoprim-sulfamethoxazole (Bactrim)
2.
Azithromycin (Zithromax)
3.
Doxycycline (Doryx)
4.
Cefuroxime (Ceftin)

cefuroxime (ceftin)

Anemia of chronic disease

Trichimonas

dx is wet mount/prep, flagella, partner needs treated too TX flagyl PO for 7 days

ectopic pregnancy

Hx of amenorrhea and new onset of bloody vaginal spotting, left adnexal tenderness and cervical motion tenderness

Which CN control eye MOVEMENT

EOM- CN III, IV, VI. (LR6SO4) 3-pupil/accommodation `

A 4-year-old girl with a history of recurrent urinary tract infections returns for follow-up after diagnosis of the most recent infection. During the physical examination, the 4-year-old tells the nurse practitioner that her brother (age 9) lies on top of her and rubs his penis near her pubic area. After completing the physical examination, the nurse practitioner must:

report suspicions to the appropriate child protective services agency.

An 86-year-old patient is experiencing radiculopathy-associated lower back pain that has not improved over the past 4 weeks. The medical history includes a lumbar discectomy. The best type of imaging study for evaluating this patient would be a/an:

1.
MRI.
2.
spinal X-ray.
3.
myelography.
4.
nuclear bone scan.

MRI

Fitz-Hugh-Curtis syndrome

Perihepatic infection which results in liver capsule inflammation from pelvic infections such as gonorrhea & chlamydia. PID infections. Sharp pain on the right upper quadrant, with PID symptoms.

PID

DX NAAT TX ceftriaxone, doxy, flagyl

Folic deficiency anemia

UTI urine culture positive when….

KNOW THAT UTI IS 100,000 (10 x 5) CFU’S TO BE DIAGNOSED in non preg patient and 10 x 3 in preg patient

3+ protein in urine, how do you follow up ?

do 24 hour urine for protein and creat clear

Beta anemia

Chlamydia Symptoms, Dx, & Tx

Fitz-Hugh syndrome, asymptomatic, mucopurulent, vaginal discharge, bleeding
Dx: gold standard, nucleic acid amplification test NAAT, gen probe, antigen detection (swab), urinalysis, wet prep >20 WBC
Azithro 1 dose or doxycycline for 7 days annual screenings

A 2-year-old child is diagnosed with radial head subluxation (“nursemaid’s elbow”). After closed manipulation, the best indication of successful treatment is that:

the child quickly begins to use the affected arm.

A 30-month-old with a history of frequent ear infections was seen in September after a symptom-free summer. Examination revealed bilateral bulging eardrums. After two separate courses of antibiotics, fluid is noted in both ears. The parent reports speech difficulties. The most likely diagnosis is:

1.
otitis media with effusion.
2.
myringitis.
3.
recurrent acute otitis.
4.
Esutachian tube dysfunction.

otiti media with effusion

12 weeks

uterine fundus first rises above the symphysis pubis.

Any baby 2-24 months with UTI and fever. how do you follow up?

– do renal and bladder ultrasound for first febrile UTI

cycle cell anemia

Wernicke’s aphasia

difficulty with comprehension but none with speech

A 67-year-old patient with type 2 diabetes mellitus, congestive heart failure (CHF), and mild coronary artery disease is currently taking digoxin, 0.25 mg daily; hydrochlorothiazide, 25 mg daily; metformin, 500 mg daily; glipizide, 10 mg daily; and atorvastatin (Lipitor), 20 mg h.s. Which of the following is an accurate statement regarding this regimen?

The hydrochlorothiazide will predispose the patient to digoxin toxicity.

A 19-year-old patient who has used oral contraceptives for 3 years plans to discontinue the pill at the end of her current cycle to become pregnant. Which of the following daily supplements would receive the highest priority?

1.
Calcium carbonate
2.
Folic acid
3.
Ferrous sulfate
4.
Multivitamin

folic acid

16 weeks

UF is between symphysis pubis and the umbilicus

RBC casts in urine, what is possible diagosis

glomerulonephritis

iron deficient anemia

20 weeks

UF at level of umbilicus

Broca’s aphasia

comprehends speech but unable to verbalize speech

During development of a treatment plan for an 84-year-old patient with hypothyroidism, the nurse practitioner must keep in mind the possibility of the worsening of:

osteoporosis.

A frail elderly widow presents to the senior clinic with a 3- to 4-day history of confusion that became worse the evening before but currently seems a little better, although her speech is rambling and difficult to follow. During the examination, the patient is anxious and agitated. She is on cardiac medications, an antidepressant, and insulin. The most likely diagnosis is:

1.
mild stroke.
2.
polypharmacy.
3.
electrolyte imbalance.
4.
agitated depression.

polypharmacy

First choice Atb for UTI

Nitrofurantoin (Macrobid)

Hpylori negative ulcer , tx

s: h2 first bedtime, may combine them with PPI. Do 6-8 weeks. PUNT after.

pregnancy anemia

20 to 35 weeks

Measure the distance between upper edge of pubic symphysis and the top of the uterine fundus using a tape measure. + or – 2 cm. 32 30-34 weeks

Initial therapy for a 3-week-old infant with uncomplicated symptoms of gastroesophageal reflux disease includes which of the following positions and change in diet?

Smaller, more frequent feedings while holding infant

A 46-year-old female presents with a complaint of amenorrhea for 3 months. The first-line diagnostic workup is:

1.
urine pregnancy test.
2.
progesterone challenge.
3.
serum prolactin level.
4.
thyroid-stimulating hormone (TSH).

urine pregnancy test

HBsAg means what

patient current HAS HbSAg (the test has the word “HAS” in it)

Define fremitis:

When is it increased?
10. Fremitus is enhanced by consolidation.

11. Fremitus is decreased by pleural fluid.

12. Fremitus is decreased by pneumothorax.

10. TRUE. Fremitus is enhanced by consolidation.

11. TRUE. Fremitus is markedly decreased by pleural fluid.

12. TRUE. Fremitus is decreased by pneumothorax.

CHF

S1, S2 and S3

Temporal arteritis

The cause of the blood vessel inflammation is unknown. In some cases, the swelling affects just part of an artery with sections of normal vessel in between. Symptoms include acute onset of headaches located on one temple, skin over artery is indurated, jaw pain, vision loss, fever, and weight loss, anorexia, fatigue.

Anti-HBs positive

immunity either from previous disease or vaccine

Bronchitis:

s/s, Tx,

RF for suicide

elderly white man after death of spouse, past Hx of suicide, family Hx of suicide, gender males have a higher success rate, hx of depression or bipolar

Temporal arteritis dx & tx

Dx usually requires biopsy of the temporal artery. Elevated sed rate and c reactive protein, high risk with polymyalgia rheumatica, the condition needs Prompt treatment with steroid medications to prevent permanent vision loss

Zollinger Ellison syndrome

tumors causes stomach to produce too much acid resulting in PUD

HBeAg positive

E- EEEEEEEnfectious! It is an indicator of active viral replication; this means the person infected with Hepatitis B can likely transmit the virus on to another person

Alpha fetoprotein

produced by fetal liver, high for twins, neural tube defect. Low AFP RF down syndrome. For either order the triple screen

Barrets esophagus:

Increases risk of cancer of esophagus

Somogyi

too much insulin in the blood at night causes a rebound increasing blood sugar in the AM

Anti-HBc

c is for caught it. either has or HAD hep B. The presence of
anti-HBc indicates previous
or ongoing infection with
hepatitis B virus in an
undefi ned time frame.

HIV

Varicella contrainindicated

Sedimentation rate is elevated and patient has visual spots and left sided-headache:

temporal arteritis

Anti-HCV positive. What is next step?

order HCV RNA. Biopsy of liver to check stage.

Psoriasis diagnosis s/s and tx:

sharply defined plaques with silvery scales: use auspitz sign when scales are pealed away and capillary bleeding. Sun helps pt. OTHER INFO?
Chronic inflammatory skin disease characterized by extensive erythematous, circumscribed scaly papules, and plaques.

Runs a variable course and seldom completely subsides. Severity is aggravated by genetic, infectious, emotional, and environmental factors.

Lesions are red, inflamed, silvery-white scaly, and circumscribed papules and plaques on elbows, knees, extensor limbs, and scalp. Psoriatic nails have a pitted surface and/or hypertrophic (subungual) changes.

Diagnosis is usually clinical.

Mild or limited psoriasis is treated with topical corticosteroids and/or vitamin D analogs.

Moderate to severe and/or extensive psoriasis may require phototherapy, oral retinoids, methotrexate, biological agents, or cyclosporine.

Inevitable abortion

vaginal bleeding with pain and cervical dilation and/or effacement.

Dawn Phenomenon

early morning increase in blood sugar between 2-8am

threatened abortion

vaginal bleeding without dilation of the cervix or passage of tissue. minimal pain

Pulses paradoxes

is an abnormally large decrease in stroke volume, systolic blood pressure and pulse wave amplitude during inspiration

s/s of preeclapsia:

headache, visual disturbances, edema.
Tests:
Treatments: metoprolol

Pencil like stool

think colon cancer: Tenesmus or the feeling of having to defecate without having stools, pain upon defecation, or sciatica can be symptoms of rectal cancer

S/s Intussusception and age it ususally occurs

Intussecption- sausage shaped mass in upper right quad. Currant jelly stool. Ur bowels prolapse into another part of ur intestine. Barium or air enema can help to reduce this. Previously healthy then they get sick. Usually before 2 years

incomplete abortion

moderate to diffuse bleeding with the passage of tissue and painful uterine cramping

Contraindications for Thiazides

Serious sulfa allergies, hypotension, gout, renal failure due to loss of sodium, hypokalemia, may worsen DM, lead to metabolic alkalosis, by action of the renin angiotensin system, decreased volume and arterial pressure. May cause hyperglycemia due to insulin resistance and decreased insulin release

How to palpate for rotator cuff injury:

??

Age of pyloric stenosis

Pyloric stenosis- 3-12 weeks

How to assess for knee injury:

Lachman
Drawer test:
Miniscus versus cruciable ligament tear

Presumptive signs of pregnancy

signs felt by women, amenorrhea, nausea, breast tenderness, N/V, urinary frequency,

Thiazides are beneficial for:

osteoporosis, reduces calcium excretion by kidneys, and stimulates osteoblasts which help with bone growth

Glimeperide

Don’t use this DM med with insulin

medications that can increase GERD

BB, CCB, HTN meds increase GERD.

when to refer ankle injury?

Grade of strain?

Probable signs of pregnancy

examiner suspects a women is pregnant. such as enlarged uterus, hegar’s sign(softening and compressibility of the lower uterus), chadwick’s sign (deepened violet-bluish color of the cervix and vaginal mucosa), goodell’s sign(softening of the cervical tip), ballottement, braxton hicks, positive pregnancy test, fetal outline felt by examiner.

what time of day should H2 blockers be taken

before bed

primary amenorrhea:

Primary amenorrhea: lack of menses by age 15 years in a patient with appropriate development of secondary sexual characteristics, or absent menses by age 13 years and no other pubertal maturation.

Secondary amenorrhea: lack of menses in a non-pregnant female for at least 3 cycles of her previous interval, or lack of menses for 6 months in a patient who was previously menstruating.

A1C result for Dx of DM

6.5 or greater and confirmed on a different day

Positive signs of pregnancy

audible fetal heartbeat, fetal movement felt by examiner, ultrasound visualization of fetus, cardiac activity on U/S.

intermittent claudication

Initial evaulation check the ankle and brachial blood pressure before and after exercise.

A1C results for Dx of Pre-DM

5.7-6.4

radial head fracture:

caused by fall onto wrist: x rays may show positive flat pad sign also known as sail sign.

epigastric pain that radiates to the back

pancreatitis

Random glucose that confirms DM

200 or higher

different between amylase and lipase

lipase more specific to alcoholic pancreatitis, starts to rise 4-8 hours after symptoms begin

amylase starts to rise 2-12 hours after onset of symptoms

Theophylline

contraindicated drugs are erythromycin, phenytoin and cimetidine.

Nursemaid elbow:

after reduction the child quickly resumes activity

treatment for high triglycerides.

High Triglycerides- causes pancreatitis >500.
If >500 treat with Niacin or Fibrate or Niaspan.
If your patient is already on NIACIN you can add a fibrate like (LOPID/TRICOR).
Apparently an insulin infusion works also

Webber test:

used to test for both conductive and neural hearing loss.

Eye tests

The cover/uncover test screens for strabismus. Visual acuity is the snellen. Cataracts screened by using the red reflex. (positive by white relaxation.

Which medications increase the risk of developing Type 2 DM?

Glucocorticosteroids
HCTZ
Atypical antipsychotics
HMG Co-A reductase inhibitors

Mitral Regurge murmur:

How to diagnose: Mitral regurgitation (MR) may present with dyspnea, usually on exertion, palpitations, and/or decreased exercise tolerance.

Typically, presents as a holosystolic blowing murmur at the apex, radiating to axilla.

1st Tests To Order

transthoracic echo
ECG

Koilonychia

spoon shaped nails associated with IDA.

CCBs (Verapamil)

Med that can make GERD worse?

Fructosamine test

checks sugar for past 2-4 weeks, high results = high blood sugar

hyperparathyroidism – labs and causes

Elevated PTH with primary- hypercalcemia or secondary-hypocalcemia
Primary hyperparathyroidism- benign adenoma (most common) or familial;
Secondary- is a response to hypocalcemia (renal failure, drugs)

s/s of hemorrhoids and tx:

Bulk forming agents with hydrocortisone tx

s3

opening snap, CHF, normal variant in healthy young adults and athelets

Negligence

When a practitioner fails to exercise the care that a reasonable person would exercise (Injury does not have to happen)

Duodenal ulcer

Type of ulcer where symptoms occur 2-5 hours after eating. (Symptoms can be relieved by antacids).

Herberden’s nodes

Hard nodules or enlargements
of the distal interphalangeal
joints of the fingers. Found in degenerative joint disease

s/s hyperparathyroidism

Fragile bones that easily fracture (osteoporosis)
Kidney stones
Excessive urination
Abdominal pain
Tiring easily or weakness
Depression or forgetfulness
Bone and joint pain

s/s of colon cancer:

herbal remedies commonly used:

Fever few: migraines, migraines during menstration.

DTAP, Tdap, Td

Dtap is the pediatric version you receive from 2 mo to 6 yrs. Adults begin with Tdap. Td as booster every 10 yes.

Gastric ulcer

Type of ulcer where symptoms occur within minutes after eating (less relief from antacids)

treatment for hyperparathyroidism

Primary: surgical parathyroidectomy if symptomatic or biphosphonates and monitoring (BP, renal, bones) if asymptomatic
Secondary: Vitamin D and calcium & treat cause

Addison’s :electrolytes, symptoms, treatment

Addison’s- adrenal deficiency – usually autoimmune cause. Deficient in cortisol (low blood sugar cuz cortisol stimulates gluconeogenesis in the liver so low cortisol = hypoglycemia in times of stress)… hyperpigmentation (Kennedy’s perpetual tan) and low aldosterone (leads to low Na in blood — SALT CRAVINGS! — high K) , You must give cortisol. (Diagnosis Plasma Cortisol <5 mcg/dl @ 0800.) *You have ADD cortisol because cortisol is low in ADDisons

Image: Addison's :electrolytes, symptoms, treatment

Mini-mental status exam:

Also called folstein mini-mental status exam- used for memory or- attention and calculation, recall, language, ability to follow simple commands and orientation.

Menses

The onset of menses occurs between Tanner stages III and IV.

Hypertrophied left ventricle

A common early finding in pt with chronic aortic regurgitation.

Tzank, PCR, or DFA

Tests that can be used to dx shingles

Delerium

reversible acute confusion, hours to days, short attention span, memory loss and disorientated.

Causes:prescription meds, substance abuse, dru-drug interactions, withdrawl, infections, sepsis, electrolyte inbalances

Alzheihmer does not cause it

Cushing syndrome

Central obesity, moon face, purple striae, hairy, hypertension, elevated plasma CORTISOL in AM. “INC BS, SODIUM” Dec K. You must draw cortisol levels in the morning.

Image: Cushing syndrome

Fever:

decreases the threshold of seizures.

when to start basal insulin

If you are already on TWO oral drugs for diabetes and A1c is 9 or higher, start BASAL insulin. If you cannot tolerate metformin and your A1c is 9 or higher start BASAL insulin.

Beta thal anemia:

electrophoresis

Implanon

progestin-only method and provides three yrs of pregnancy protection.

Diarrhea, abdominal pain, N/V

Classic symptoms of Cdiff

parathyroid gland and PTH – role in the body

– PTH is responsible for calcium loss or gain from bones, kidneys, and GI tract.

Neutral Protamine Hagedorn (NPH)

intermediate acting insulin

E Coli

The Most common pathogen with Pyleonephritis

HIV monitor progress of disease?

CD4 and viral load tests

3-4 weeks ago

A patient with a primary case of scabies was probably infected how long ago?

Legg-calve-perthens

Referral for aggressive treatment if they are 7 yrs of age

best test to differentiate iron deficiency from other anemias?

ferritin

Hypothyroidism- lab findings, diseases associated with, tx

High TSH LOW Free T4/T3, However, Free T4 is much more specific to this disease. Hashimotos (autoimmune) think of everything in ur body is slowing down. Synthroid.

Hypokalemia

A pt taking HCTZ and complaining of muscle cramps probably has?

Primary amenorrhea

-no menarche by age 16
-usually due to anatomic or genetic causes

trigeminal neuralgia:

Facial pain syndrome in the distribution of ≥1 divisions of the trigeminal nerve.

Diagnosis is clinical, with a history of paroxysms of sharp, stabbing, intense pain lasting up to 2 minutes.

First-line therapy is medical, to which the majority of patients are partially responsive.

Primary Options

carbamazepine : 200 mg/day orally initially given in 1-2 divided doses, usual maintenance dose is 400-1200 mg/day given in 2 divided doses

oxcarbazepine : 300 mg/day orally initially, usual maintenance dose is 600-1200 mg/day, maximum 1200 mg/day given in 2 divided doses

Which tx for hyperthyroidism is preferred in pregnancy

PTU PREFERED IN PREGNANCY

HTN

AV Nicking is seen in what disease?

Dubowitz and Ballard method

tests the elasticity of cartilage in the ears only.

Acute bacterial conjunctivitis in infants:

Primary Options

azithromycin ophthalmic : (1%) 1 drop into the affected eye(s) twice for one day, then once daily for 4 days

erythromycin ophthalmic : (0.5%) apply to the affected eye(s) four times daily

polymyxin B/trimethoprim ophthalmic : (10,000 units/mg) 1 drop into the affected eye(s) four times daily

hyperthyroidism

Graves disease-autoimmune. Lid lag, exophthalmos, everything is hyper (body wise). Treatment: PTU/Tapazole.

Image: hyperthyroidism

chorionic villus sampling (CVS)

CVS 10- 12 weeks
Amniocentesis 15-18 weeks

Complications of Cellulitis with a diabetic patient

OSTEOMYELITIS

Beta Blockers, CCB, & TCAs

Medications used for migraine prophylaxis

HCTZ interacts with digoxin:

Digoxin toxicity

Hashimotos, TSH and Free T3/t4 , plus test to connfirm

first order TSH and get high TSH. Then order thyroid panel and get low t3. The diagnosis is confirmed by the presence of thyroid autoantibodies in the blood, usually at high levels. Thyroid peroxidase antibodies (TPO) are more common that thyroglobulin antibodies

most common pathogens for community acquired pneumonia in older adults:

H.Influenza and S. Pneumoniae
Pneumocystis jiroveci can cause pneumonia in people whose immune system is not working well.
Streptococcus pneumoniae, or Pneumococcus, continues to be responsible for 20 to 50 percent of CAP, and is the single most common pathogen causing CAP.

Mycoplasma, Chlamydophila, and Legionella, collectively known as the “atypical” CAP pathogens, cause 20 – 30% of CAP. Legionella is commonly implicated in severe CAP, while Mycoplasma and Chlamydophila more often result in mild clinical disease.

Influenza is the most common respiratory virus resulting in CAP. Influenza infection also predisposes to subsequent development of bacterial CAP, referred to as “post-influenza pneumonia”.

Hemophilus and Moraxella are common pathogens, particularly in patients with COPD or intrinsic lung disease.

Mammograms

UPSTK end at 75
ACOG up to 74

Hyyphae

A skin lesion fluoresces under a Wood’s lamp. What microscopic finding is consistent with this?

Latent TB treated with

Latent TB usually treated with INH

Most common cause of acute bacterial sinusitis:

H.Influenza and S. Pneumoniae

Elderly PPD

Neg PPD, a second test PPD should be performed in 1 week.

Calcium

A supplement known to cause constipation?

s/s and testing for PCOS:

Key historic findings
female of reproductive age
irregular menstruation
infertility
hirsutism

Other Factors

scalp hair loss
oily skin or excessive sweating
acne
overweight or obesity
hypertension
acanthosis nigricans

1st Tests To Order

serum total and free testosterone
serum dehydroepiandrosterone sulfate (DHEAS)
serum 17-hydroxyprogesterone
serum prolactin
serum TSH
oral glucose tolerance test
fasting lipid panel

Other Tests to Consider

serum androstenedione
pelvic ultrasound
basal body temperature monitoring
luteal phase progesterone measurement
serum LH and FSH
Tx:
Acute
with infertility and desiring fertility

weight loss
metformin
clomiphene
metformin
dexamethasone
gonadotropins
metformin
in vitro fertilization
metformin
laparoscopic ovarian drilling

Ongoing
not desiring current fertility

with hyperandrogenic features alone
oral contraceptive pill
metformin
mechanical hair removal or topical therapy
antiandrogen
antiandrogen plus oral contraceptive pill
long-acting GnRH analog plus oral contraceptive pill
with oligoamenorrhea alone
weight loss
oral contraceptive pill
metformin
cyclic progestin
with hyperandrogenic features plus oligoamenorrhea
weight loss
oral contraceptive pill
metformin
mechanical hair removal or topical therapy
antiandrogen plus oral contraceptive pill
long-acting GnRH analog plus oral contraceptive pill

Diagnosing TB, which is gold standard for dx of TB

ACTIVE TB order, NAAT, C&S, AFB. The AFB is not diagnostic. SPUTUM FOR C & S is gold standard.

Ribavirin se

hemolytic amemia

Improve Urine flow

An elderly patient will take Saw Palmetto because he thinks it will help with what?

symptomatolgy

management of COPD for the elderly

60 yo

CDC recommends Zoster vaccine in what age of immunocompetent pt?

Mantoux test results, what size induration is considered positive.

>5mm-think immunocompromised or person in close contacts.
>10 think Immigrants, working status (nurses), drug users, home life.
>15 Think no risks

28 week old pregnant patient has single episode of vaginal bleeding without other s/s: What to do?

Ultrasound

What will you hear with percussion of emphysema lungs, what will you see on CXR, what will you notice about their breathing

Emphysema Lungs- Percussion-HYPERENNOSANCE tactile frem + egophony- dec. CXR- flattened diaphragms with hyperinflation. Inc. AP diameter, accessory muscles, pursed-lip breathing, weight loss.

TIA V. Stroke:

Tia has absence of residual s/s.

Biaxin and digoxin

Biaxin raises digoxin level

Alcohol

If a pt is taking metronidazole, they should avoid what?

Terbinafine, atorvastatin, simvastatin

Which 3 medications should have liver function test prior to starting?

McBurneys point: rebound tenderness-

sign for acute appendicitis

MANTRELS score [41]

Score is based on clinical characteristics of the patients. The higher the score out of a possible total of 10, the greater the chance of having acute appendicitis.

M: Migration of pain to right lower quadrant = 1 point

A: Anorexia = 1 point

N: Nausea and vomiting = 1 point

T: Tenderness in right lower quadrant = 2 points

R: Rebound tenderness = 1 point

E: Elevated temperature = 1 point

L: Leukocytosis = 2 points

S: Shift of WBC count to left = 1 point

Or use apache 2 calculator.

Omezaprole

se headache, abdominal pain and gas

asthma patient on low-dose ICS still has symptoms, what is next step

IF LOW DOSE ICS, NEXT IS MEDIUM DOSE

intermittent asthma, day time symptom frequency, night time awakenings, tx

<2 / week day, <2 month – night, SABA

Polymyalgia rheumatica s/s, tests and tx:

Basics:
Typically a history of neck, shoulder girdle, and/or hip girdle stiffness and pain, occurring in patients age 50 years or older.

Patients complain of difficulty rising from seated or prone positions, varying degrees of muscle tenderness, shoulder/hip bursitis, and/or oligoarthritis.

More common in women.

About 15% to 20% of patients with polymyalgia rheumatica (PMR) have giant cell arteritis (GCA); 40% to 60% of GCA patients have PMR.

Diagnosis is made via history and with supportive laboratory tests indicating an elevated ESR or CRP.

Rapid improvement often occurs within 24 to 72 hours with low-dose prednisone.

s/s:
Key Factors

shoulder/hip girdle stiffness
shoulder/hip girdle pain
rapid response to corticosteroids

Other Factors

acute onset
low-grade fever
anorexia
weight loss
malaise
depression
asthenia
oligoarticular arthritis

dx:

Criteria: must have any 3 factors, or just 1 and a temporal artery biopsy positive for giant cell arteritis

Age over 65 years

Bilateral shoulder girdle pain

More than 1 hour morning stiffness

Symptom onset <2 weeks

ESR >40 mm/hour

Depression/weight loss

Upper arm tenderness, bilateral.

Tests: 1st Tests To Order

ESR
C-reactive protein (CRP)
CBC
ultrasound

Other Tests to Consider

TSH
MRI
serum protein electrophoresis
serum creatine phosphokinase
TX:
Acute
initial presentation

corticosteroid: prednisone : 10-20 mg orally once or twice daily
calcium + vitamin D + bisphosphonate
nonsteroidal anti-inflammatory drug (NSAID)
methotrexate
folic acid
tocilizumab

Ongoing
treatment-resistant or relapse or disease exacerbation

corticosteroid (increased dose)
calcium + vitamin D + bisphosphonate
methotrexate plus folic acid
tocilizumab or leflunomide

Nummular Eczema

pruritic inflammatory dz – young, old
fall/winter, coin-shaped plaques – common on lower leg extremities

tx: moisturizers or topical steroids

Mitral Valve porlapse

Which mitral disorder results fro redundancy of the mitral valve’s leaflets?

Mild persistent, daytime symptoms, night awakenings, tx

>2 but not daily symptoms, >2 per month, but not weekly, SABA + low dose ICS

interductal papilloma

is a small, benign tumor that forms in a milk duct in the breast. These tumors are made of gland and fibrous tissue as well as blood vessels. They most commonly occur in women between ages 35 and 55. Unilateral spontaneous serous or serosanguineous discharge

s/s of polymyalgia rheumatica:

pelvic girdle pain for 6 months, unintentional weight loss, pain with ROM, unintentional weight loss, low hemoglobin and elevated ESR

LDL

The lipid particle with the greatest atherogenic effect is

Mod Persistent, daytime symptom frequency, night awakenings, tx

daily daytime symptoms, night awakenings >1 per week, SABA+ICS+LABA or SABA+medium dose ICS

Fasting blood glucose of 126:

Diagnose Diabetes

Alzheimer’s disease

starts with short term memory loss

Check TSH

If a pt’s lipids are elevated, what would the NP do next?

s/s of hypoglycemia if patient is on a betablocker and insulin:

sweating. Not tachycardia

ACE inhibitors

Which class of medication is frequently used to improve long-term outcomes in pts with systolic dysfunction?

Cellulitis

Staph aureus most common bacteria with people DM

Severe persistent, daytime, night awakenings, tx

symptoms throughout the day, nightly asthma symptoms- SABA, Med ICS plus LABA.

pulsus paradoxus

Pulsus paradox Apical pulse can still be heard even though the radial pulse is no longer palpable. Certain issues cause impairment with diastolic filling, 10 or greater drop in the SYSTOLIC pressure. I think her patient had asthma and their pressure dropped by 10 etc.

Tx for trichomonas vaginitis:

metronidazole 2 grams (flagyl)
s/s: Key Factors

vaginal discharge
dysuria
discharge adherent to vaginal mucosa

Other Factors

prior episodes
fever
pruritus
vulvodynia
vaginal dryness
vaginal bleeding
abdominal pain
dyspareunia
erythema
pale epithelium
shiny epithelium
decreased elasticity
friable epithelium
strawberry cervix
testing:
1st Tests To Order

vaginal pH
amine “whiff” test of vaginal secretions
wet mount microscopy of vaginal secretions
Gram stain of vaginal secretions
HIV test
nucleic acid amplification test (NAAT)
VDRL
serum rapid plasma regain (RPR) test
Criteria:

Bacterial vaginosis:

Vaginal pH >4.5

Whiff test

Clue cells (vaginal epithelial cells with distinctive stippled appearance in saline wet mount by being covered with bacteria)

Adherent white vaginal discharge.

cepalohematoma

enlarged fluid filled area that is subperiosteal and does not cross the suture line

Iron Def. Anemia

Which hypochromic, microcytic anemia has elevated RDW?

What to check before starting statins

Must check LFT before starting Statin

normal changes with aging?

less gastric acid production.

Forced Vital Capacity (FVC)

The volume of air expired during a forced maximal expiration after a forced maximal inspiration.

Clinical Inertia

Providers who do not increase therapy even if test results indicate the need.

T wave inversion suggests:

acute coronary ischemia

Potassium supplements

A patient taking an ARB should avoid?

Gastic ulcer v peptic ulcer disease:

Peptic ulcer:
Usually presents as chronic, upper abdominal pain related to eating a meal (dyspepsia).

Use of nonsteroidal anti-inflammatory drugs (NSAIDs) and Helicobacter pylori infection are the most common causes.

There may be some epigastric tenderness, but often there are no other signs on physical examination.

Endoscopy is diagnostic and may show an ulcer in the stomach or proximal duodenum. H pylori infection should be sought.

In the absence of “alarm” (red flag) symptoms or signs, testing for and treating H pylori and/or empiric acid inhibition therapy is appropriate.

Most common complications are gastroduodenal bleeding and perforation, either of which may be the presenting symptom, particularly in patients taking NSAIDs.

Gastritis:
A histologic term for inflammation of the gastric mucosa.

Helicobacter pylori infection and use of nonsteroidal anti-inflammatory drugs (NSAIDs) or alcohol are the most common causes. Other causes include stress (secondary to mucosal ischemia) and autoimmune gastritis. Rare forms include phlegmonous gastritis (a rare bacterial infection).

Diagnosis is based on clinical history and characteristic histological findings. A variety of methods may be used to diagnose H pylori infection.

Presence of suspicious features suggestive of upper GI malignancy requires urgent endoscopy. These include GI bleeding, anemia, early satiety, unexplained weight loss (>10% body weight), progressive dysphagia, odynophagia, or persistent vomiting.

Treatment depends on the etiology. Options include H pylori-eradication therapy, reduction of NSAIDs or alcohol exposure, and symptomatic therapy with histamine-2 antagonists and/or proton-pump inhibitors.

If untreated, progression to peptic ulcer disease may occur. Other complications include gastric carcinoma and gastric lymphoma.

Who gets statins

1. Patients with any form of clinical ASCVD

2. Patients with primary LDL-C levels of 190 mg per dL or greater

3. Patients with diabetes mellitus, 40 to 75 years of age, with LDL-C levels of 70 to 189 mg per dL

4. Patients without diabetes, 40 to 75 years of age, with an estimated 10-year ASCVD risk ≥ 7.5%

Age

The primary risk factor for development of breast cancer in women of average risk is?

Who gets moderate intensity statin

1. Age > 75 years + ASCVD
2. DM + 70-189 LDL but low <7.5 ASCVD risk
3. healthy, 70-189 LDL, very low 5-<7.5 ASCVD risk
(4 healthy, 70-189 LDL, low >7.5 ASCVD risk, may choose moderate or high intensity)

To assess abstract thought:

ask meaning of common proverb.

Lyme disease

child 6 yrs old amoxicillin or cefuroxime

14 days or longer

How long should a pt be treated with atb if he has prostatits secondary to an STD?

Latent TB

INH for 9 months isoniazid

Thiazide diuretic c/i and side effects

no sulfa allergies,
MoNKey low: hypo Mg, Na, K
GLUT – hyperGlycermia, hyperLipidemia, hyperUricemia, hyperTriglyseridemia,

No satelite lesions with diaper rash:

contact dermatitis
Satelite lesions- candida infection

Chronic Venous Insufficiency

Impaired venous return. Achy legs relieved by elevation, edema after prolonged standing, night cramps, brownish discoloration, cold, ulcers. Etc. do support stockings.

pulmonary rehab goals include:

increase exercise capacity, decreased hospitalization, enhanced quality of life.

NOT increased lung capacity.

H and H testing infant

6, 9 and 12

Cetirizine

Which long-acting antihistamine is sedating?

peripheral artery disease (same as PVascularD)

Nocturnal pain relieved by lowering legs, poor pulses, dependent rubor, intermittent claudication, atrophy, shiny, hairless, cold feet. Initial do a pulse check, ABI 0.9 or less is PAD. Ateriography is the most DEFINITIVE test. Try to develop collateral circulation. Otherwise- Trental, Pletal ( a xanthine derivative used as a drug to treat muscle pain in people with peripheral artery disease)

devlopment warning signs

-visual delay
-does not pick up toy by 6 mo
-no reaction to noise
-no laugh
does not sit up
-does not raise head by 3 mo while on tummy

Gynecological & GI bleeding

What are the 2 most common causes of IDA in adults?

Tx for isolated systolic HTN

ccb

ADHD (Attention-Deficit Hyperactivity Disorder)

a psychological disorder marked by extreme inattention and/or hyperactivity and impulsivity
-s/s present before the age of 7 as late as 12
-s/s for at least 6 mo
-must occur in home, school and play

-Tx ritalin, concerta,metadate,focalin, adderall, vyanse

Costovertebral angle tenderness (CVA)

A pt with pyelonephritis will likely have what symptom?

S4 when

S4-LVH stiffening, Tennesse, late diastole. “Atrial kick/gallop”

menarche

breast dev stage 3 to 4, predominatly 4

No significant systemic effects. They are predictable.

Class effect of nasal steroids?

s3 when heard

S3- HF, Kentucky, early diastole. Abn >35. Bell

Spermarch

genital stage 3

Hyperopic

Farsighted (objects nearby are blurry)

VSD

thrill, felt at LLSB

Presbyopic

Age-related farsightedness

MVP how it sounds, when to treat and how to treat

MVP- S2 click, followed by systolic murmur. Asymptomatic. MVP with palpitations is treated with BB.

HTN meds that can cause heartburn

BB, CCB, alpha agonists. (HTN meds). EXAM

Myopic

Nearsighted

Tx sinusitis

AUGMENTIN. if pen allergic – doxy or levofloxacin (adults). kids get clinda+3rd gen cephalosporin (IDSA) adults get treated 5-7 days, kids get 10-14 days

In what condition are the turbinates dark red and swollen?

Acute rhinitis

test for mononucleosis

monospot is a heterophile antibody test

Gyrate lesions

Scabies

Otitis Externa (swimmers ear)- bacterial cause and tx

Pseudomonas aeruginosa. (other- S. aureus). External ear pain- d/c itching, hearing loss, tragus, green d/c. TREATMENT: Corticosporin, Cipro
Topical antimicrobials or antibiotics such as acetic acid, aminoglycosides, polymyxin B, and quinolones are the treatment of choice in uncomplicated cases

Most likely diagnosis for rash that starts on belly and spreads to the rest of the body including the face. Causes severe itching.

Varicella

Weber lateralization to which ear

Weber’s test is performed by softly striking a 512-Hz tuning fork and placing it midline on the patient’s scalp, or on the forehead, nasal bones, or teeth.
If the hearing loss is conductive, the sound will be heard best in the affected ear.
If the loss is sensorineural, the sound will be heard best in the normal ear.
The sound remains midline in patients with normal hearing

Pt: NP, I can’t hear in my left ear.
NP (wonders to herself): I wonder if it is conductive hearing loss.
There are three options:
Does Weber test and pt can hear best in left ear (affected ear) == “Sir, you have conductive hearing loss.”
Does Weber test and patient can hear best in RIGHT ear (normal ear) == “Sir, you have sensorineural hearing loss”
Hears both equally == “Sir, you do not have hearing loss.”

What skin condition is a raised reddened area with a dimpled look?

Basal cell carcinoma

Rinne test results and interpretation

AC>BC is normal. BC>AC is conductive hearing loss

Skin condition that can manifest when stressed. Itchy and dries up like cream colored sand paper before healing

Eczema

Incretins do what?

Signal the pancreas to increase insulin secretion & signals the liver to stop producing glucagon

AOM tx

Amox, If your patient is only PCN allergic do azithromycin or clarithromycin.

AOM bacterial cause

. Usually S. pneumo. (others: h influ, mor catarrhalis).

Lactic acidosis is a potential adverse side effect most commonly seen with which medication?

Metformin (Glucophage)

Koplik spots are a diagnostic indicator of

lusters sm. Size red papules w/ white centers in the buccal mucosa by lower molars -rubeola measles

What is a bubo?

a swollen, inflamed lymph node in the armpit or groin.

diabetic retinopathy

Diabetic Retinopathy-Cotton wool spots (moderate retinopathy), micro-aneurysms. ALSO RETINAL HEMORRHAGES ON CENTER OF EYE APPEAR ORANGE RED

Where is the site for most breast tumors in women?

Upper outer quadrant

hypertensive retinopathy

Hypertensive Retinopathy- Copper/silver wire arterioles. AV nicking(mild retinopathy). Retinal “flame” Hemorrhages

What is a positive prehn’s sign?

Lifting of the testes

papilledema appearance and indicates what

Papilledema- optic disc swollen w/ blurred edges due to increased ICP

Which cranial nerve is most responsible for papillary constriction?

Oculomotor

S/S of retinal detachment

Retinal Detachment- Floaters, curtain, flashes of light. Painless.

Which type of ulcer feels better right after a patient eats?

Duodenal ulcer

age-related macular degeneration: how affects sight, how test

Age-Related Macular Degeneration – Painless loss of “central vision” reports straight lines appear curved. Periphery is preserved. Give amsler grid.

The most critical assessment finding with acute abdomen with peritonitis with a bowel perforation is?

Rigidity

Which ECG changes would the provider see in a pt with classic angina?

ST depression

Cataracts, how affect elderly eye sight

in elderly night vision issues. Opaque

acute angle closure glaucoma: what patient will see, findings on physical exam, disposition

Acute Angle-closure glaucoma- acute/severe halos, cupping optic nerve, cloudy cornea, mid-dilated oval pupil. ER STAT

Dependent rubor is found in what disease?

Ateriosclerotic occlusvie disease

Primary open angle glaucoma, how affects sight

Primary Open Angle Glaucoma- CN2 gradual changes in peripheral vision LOST FIRST, then second central vision

What to test for with rash on hands and feet?

rash hands soles/feet think to test for secondary syphilis RPR then VDRL are screening, then dx FTA-ABS.

pityriasis rosea

pityriasis rosea itchy, herald patch, xmas tree pattern,

Image: pityriasis rosea

Wickham’s striae

fine, lace-like network of white lines in lichen planus

Image: Wickham's striae

lichen planus tx

topical steroids

Describe scarlet fever rash

Scarlet fever (Scarlantina)- “sandpaper textured-pink rash with sore throat” strawberry tongue, rash starts on head and neck, spreads to trunk. The skin THEN desquamates.

Image: Describe scarlet fever rash

Impetigo appearance

Impetigo-Gram positive. Itchy pink-red lesions, evolve into vesiculopustules that rupture

Treatment for impetigo, plus for allergies

. If bullous-large blisters. Severe- Keflex, dicloxacillin. PCN Allergic-Azithro, clinda.

acne rosacea: appearance and treatment

Acne Rosacea- chronic small acne like papules/pustules around nose mouth chin. TREATMENT- Metrogel, Azelex. Low dose tetracycline.

treatment mild acne

Acne Vulgaris (common acne)- ON EXAM
mild (topicals only) *open/closed comedone w/ or w/o sm. papules. Retin-A, acne worsens 4-6 weeks if no improvement in 8-12 weeks increase dose or add erythromycin, benzoyl peroxide.

treatment moderate acne

Moderate (topicals plus antibiotics)- papules, pustules w/ comedones. Continue with topicals combined with topical antibiotics. Then add ORAL antibiotics tetra, mino, doxy

treatment of severe acne

Severe- with painful indurated nodule, cysts, abscesses, pustules. Accutane- check LFTs, 2 forms of contraceptives, monthly prego testing, only prescribe 1 month supply.

varicella zoster

“contagious 48 h. before, until all lesions crusted over” low grade fever, generalized lymphadenopathy, intense itching, erythematous macules, papules develop over macules, then vesicles erupt. “initially on trunk, then scalp and face” TREATMENT supportive, antihistamines, acyclovir 20mg/kg 5xd. If given first 24 hours works best.

MRSA tx

MRSA TREATMENT: Bactrim, doxy, mino, clinda. If sulfa allergy do not use Bactrim.

actinic keratosis: s/s, gold standard dx, tx

Actinic Keratosis- Precursor to squamous cell carcinoma. “numerous dry round and pink to red lesions” with a rough and scaly texture. Does not heal. Slow growing in sun exposed areas. Diagnosis: BIOPSY Golden Standard. Treatment: Sm. (cryotherapy), Lrg. (5-FU cream)- which causes ur skin to ooze, crust, scab, redness.

Tinea Corporis

Tinea Corporis- “ring like itchy rash, slowly enlarge central clearing”-Treatment: most respond to topical antifungals, if severe do oral Lamisil.

Eczema, s/s, mediated by which immunoglobulin, tx

Atopic Dermatitis (eczema)- Inherited. Extremely itchy. On flexural folds, neck, hands. Inc. IgE. “small vesicles that rupture leaving painful, bright-red, weepy lesions” they become lichenified from itching. First line: Topical steroids. Avoid hot water/soaps. PO antihistamines.

Koebner phenomenon

(Koebner phenomenon- new psoriatic plaques form over skin trauma)

Psoriasis tx

TREATMENT: Topical steroids, Tar preps (mild). For (severe) do anti-TNF, or immunologic.

Melanoma: how it looks

Melanoma- Dark Moles, uneven texture, different colors, irregular, >6mm, could be itchy. EXAM

Lyme: name of rash, what is the rash like

Erythema Migraines- (stage 1 Lyme) Target bulls-eye, usually appears in 7-14 days POST being bitten by a deer tick. Rash is hot to touch with rough texture, flu like symptoms.

Lyme: how dx

DX: B. Burgdorferi via ELISA, then confirm with western blot. Increased ESR.

Lyme: tx

TREATMENT: Less than 7 Amoxicillin or cefuroxime axetil. Older than 7 Doxycycline.

Rocky mountain spotted fever: s/s

RMSF- Inc. fever, chills, N/v, photophobia, myalgia, arthralgias THEN 2-5 days later you develop a petechial rash on forearms, ankles, wrists, that spreads towards trunk and becomes generalized. Think rocky NC/OK/AK/TN/MO.

Rocky mountain spotted fever – dx

DX: PCR essay with Rickessetti Antigen

Rocky mountain spotted fever – tx

TREATMENT- doxycycline.

acute sinusitis
CP?
Diagnostic?
(DDx)
Pharm, NonPharm?
f/u
Recognize normal course? Refer?

– S Pneu. H. Flu. M. Cat. Viral
– Facial or upper molar pain. Nasal congestion >10d. Purulent nasal d/c or PND- awaken at night w cough. Fever (mostly w children not adults). Can be due to allergy flare-up, then boggy swollen nasal turbinate. Tender sinuses.
– H&P, Positive transillumination.
– If mild uncomplicated- topical decongestants (Afrin), Steroid and saline nasal spray, Mucolytic
– If severe, 1st: Augmentin
Alt: Levaquin, Doxy, Cefdinir, Cefin

Arcus Senilis

normal finding in elderly
white, grey, or blue opaque ring in the corneal margin (peripheral corneal opacity), or white ring in front of the periphery of the iris. It is present at birth but then fades; however, it is quite commonly present in the elderly.

Epistaxis

anterior -Kiesselbach plexus
risk- nose picking

posterior – Woodruff –HTN, atherosclerosis- can lead to hemorrhage- refer to ED

tx: Anterior — pressure sitting position leading forward -lessen swallowing of blood

Epiglottis

severe, life-threatening infection of the epiglottis and supraglottic structures that occurs most commonly in children between 2 and 12 years of age

Periorbital cellulitis

Strep pneumo, Strep pyogenes, H flu, Staph aureus, Staph epi, oral anaerobes

acute onset of erythematous swollen eyelid w proptosis (bulging of the eyeball) and eye pain. Bad EOM exam w pain. h/o recent rhinosinusitis or URI.
Caused by bacterial infection of the orbital contents (fat and ocular muscles).
More common in children than adults.
Serious complications. Refer to ED.

Strep throat pharyngitis

Grp A Strep

Fever, Sore throat, Tonsillar exudate, absence of cough

PCN V, Amoxicillin
Alt: Azithromycin
Ibuprofen/Tylenol

Complications:
Scarlet fever- Sandpaper texture maculopapular rash that desquamates. Strawberry tongue. Sore throat that can lead to Acute Rheumatic Fever (affects the heart and valves, joints, brain)
Poststrep glomerulonephritis- abrupt onset of proteinuria, hematuria, dark-colored urine, RBC casts a/b HTN and edema

Mono

AOM

OME

OE

macular degeneration (MD)

Retinopathy
HTN
DM

Herpetic whitlow

A VIRAL skin infection of the finger(s) that is caused by HERPES SIMPLEX (type 1 (ORAL) or type 2 (GENTIAL) virus infection, from DIRECT CONTACT with either a cold sore or genital herpes lesion.

Self-limited infection: ANALGESICS or nonsteroidal anti-inflammatory drugs (NSAIDs) for pain PRN.
SEVERE infections: Treat with ACYCLOVIR (Zovirax).

Image: Herpetic whitlow

A 65-year-old woman presents for a follow-up examination after a new patient visit. She has not seen a healthcare provider for several years. She is a smoker and her hypertension is now adequately controlled with medication. Her mother died at age 40 from a heart attack. The fasting lipid profile shows cholesterol = 240 mg/dL, HDL = 30, and LDL = 200. In addition to starting Therapeutic Lifestyle Changes, the nurse practitioner should start the patient on:

1.bile acid sequestrant.
2. a statin drug.
3. a cholesterol absorption inhibitor.
4. low-dose aspirin.

A statin drug

The most commonly prescribed medication for mild systemic lupus erythematosus (SLE) is:

1. azathioprine (AZA).
2. belimumab (Benlysta).
3. ibuprofen (Advil).
4. cyclophosphamide (Cytoxan).

ibuprofen (advil)

The most common sign of cervical cancer is:

1. postcoital bleeding.
2. strong odor from vaginal discharge.
3. itching in the vaginal area.
4. molluscum contagiosum.

postcoital bleeding

The nurse practitioner prescribes amitriptyline (Elavil) for a patient with neuropathic pain secondary to diabetes mellitus. On follow-up, the patient complains of urine retention and dry mouth. The practitioner would:

1. discontinue amitriptyline and begin ibuprofen (Motrin).
2. refer to physical therapy.
3. start methocarbamol (Robaxin).
4. discontinue amitriptyline and begin gabapentin (Neurontin).

discontinue amitriptyline and begin gabapentin (neurontin)

A 17-year-old male with rheumatoid arthritis is being treated with an NSAID and omeprazole (Prilosec). The patient complains of headache, abdominal pain, and gas. These symptoms are most likely:

1. associated with the omeprazole.
2. related to the underlying condition.
3. the result of the NSAID.
4. caused by viral gastroenteritis.

associated with the omeprazole

The medication of choice for the initial treatment of juvenile rheumatoid arthritis is:

1. acetaminophen.
2. prednisone.
3. aspirin.
4. ibuprofen.

ibuprofen

A 12-year-old with sickle cell anemia has recently experienced a sickle cell crisis and presents for a follow-up examination after a recent hospitalization. It is most important to continue monitoring growth, development, and:

1. white blood cell levels.
2. fecal occult blood test.
3. hemoglobin levels.
4. urine dipsticks.

hemoglobin levels

A 90-year-old female is brought to the clinic by her neighbor. She states that everything is fine, but the nurse practitioner notes that she has poor hygiene and bruises on her trunk. The neighbor is concerned that the patient often has no money to buy food, despite income from social security and a coal miner’s pension. The nurse practitioner suspects abuse. Which of the following is the nurse practitioner obligated to do next?
1. Report the case to the proper authorities.
2. Tell the neighbor to check on the woman daily and report back.
3. Document the data and report the information to risk management.
4. Call the patient’s family and inquire about the concerns.

report the case to proper authorities

In most cases, the first manifestation of Alzheimer’s disease is:

1. impaired judgment.
2. decrease in short-term memory.
3. disorientation in time and place.
4. decrease in long-term memory.

decrease in short-term memory

The optimal treatment for latent tuberculosis is:

1. rifampin (Rifadin) for 5 months.
2. isoniazid (Nydrazid) for 9 months.
3. pyrazinamide for 6 months.
4. ethambutol for 6 months.

osioniazid (nydrazid) for 9 months

Unilateral spontaneous serous or serosanguineous discharge from a single duct of a breast is most often caused by:

1. intraductal papilloma.
2. mucinous breast lesions.
3. Paget’s disease.
4. ductal carcinoma in situ.

intraductal papilloma

A young child with asthma presents for follow-up evaluation. After numerous changes in medications and doses, the parents report that the child continues to have difficulty with coughing, especially during the night. Which of the following conditions would be the most likely cause of the continued asthma symptoms?

1. Vocal cord dysfunction
2. Cystic fibrosis
3. Gastroesophageal reflux
4. Allergic rhinitis

gastroesophageal reflux

A pregnant woman with known HIV infection can reduce the risk of perinatal transmission through zidovudine (Retrovir) therapy. Based on current research, optimal therapy is to start daily dosing:

1. post amniocentesis.
2. after 14 weeks of gestation.
3. if premature rupture of membranes occurs.
4. if maternal viral loads are greater than 10,000.

after 14 weeks gestation

T-wave inversion with a normal ST segment on a 12-lead EKG may represent:

1. acute coronary ischemia.
2. right ventricular hypertrophy.
3. atrial hypertrophy.
4. hyperkalemia.

acute coronary ischemia

The 16-year-old mother of a 2-month-old presents the infant, reporting that the child is very irritable and does not feed well. During physical examination, the child’s head drops back and the child exhibits sudden flexing of the extremities. As the flexing stops, the child cries uncontrollably. Funduscopic examination reveals retinal hemorrhages. Which of the following diagnostic tests should be ordered?

1. Skull X-rays
2. MRI
3. CT scan
4. Pet scan

CT scan

The most important diagnostic factor in evaluating angina pectoris is the patient’s:

1. King of Heart’s monitor.
2. physical examination.
3. history.
4. echocardiogram.

history

There are three reasons for latex allergies. Which of the following does NOT cause the problem?

1. Immediate hypersensitivity
2. Irritant contact of dermatitis
3. Cytotoxic hypersensitivity
4. Immune complex reaction

cytotoxic hypersensitivity

A patient on warfarin (Coumadin) therapy for recurrent deep vein thrombosis (DVT) is about to have lumbar spinal fusion surgery. The patient’s warfarin is put on hold starting 5 days prior to the surgery and subcutaneous Lovenox has been ordered for DVT prophylaxis until the resumption of the warfarin. The nurse practitioner knows that the patient’s postoperative warfarin dose should be restarted based on the:

1. value of her morning Prothrombin time.
2. loading dose of 10 mg, plus the previous warfarin dose.
3. baseline PT and INR values.
4. target INR of 2.

caseline PT and INR values

An 87-year-old patient presents with round, pruritic plaques and small vesicles on the lower legs. The most likely diagnosis is:

1. allergic contact dermatitis.
2. plaque psoriasis.
3. cutaneous T-cell lymphoma.
4. nummular eczema

nummular eczema

A 3-year-old presents with a 2-day history of acute diarrhea with a total of 8 watery stools without blood or mucus, and 2 episodes of vomiting in the past 48 hours. Assessment reveals no current antibiotic therapy, dehydration <5%, soft abdomen with hyperactive bowel sounds, no masses or organomegaly; other physical findings are normal. Which of the following is the most appropriate management plan?

1. Stool culture; Immodium AD liquid.
2. No laboratory workup; instruct parents on signs and symptoms of dehydration; diet of clear liquids, advancing to bananas, rice cereal, Jell-O, and soup.
3. Start diet of clear liquids, advancing to bananas, rice cereal, Jell-O, and soup over next 72 hours; Imodium AD liquid, 1 mg/sml
4. Stool for ova and parasites; advise parents of likely sources of Giardia; prescribe furazolidone (Furoxone)

No laboratory workup; instruct parents on signs and symptoms of dehydration; diet of clear liquids, advancing to bananas, rice cereal, Jell-O, and soup.

A mother presents her 12-month-old child with concern because the child does not yet say “mama” or “dada.” The mother reports that in the first months of life, the child loved listening to music and being talked to. The past medical history is negative for ear infections. The nurse practitioner should:

1. evaluate for hearing loss.
2. refer to speech therapy.
3. re-evaluate in 3 months.
4. check for cerumen impaction.

evaluate for hearing loss

The most common side effect of the oral ribavirin used in the treatment of hepatitis C is:

1. hemolytic anemia.
2. weight loss.
3. depression.
4. hypothyroidism.

hemolytic anemia

An 86-year-old patient presents with pinguecula, a yellow triangular degenerative tissue thickening of the bulbar conjunctiva. This condition is:

1. cured by eye drops.
2. a genetic process.
3. a malignant growth.
4. a solar-induced lesion

a solar-induced lesion

During a well-child examination of an 18-month-old, premature tooth decay and inflamed gums, particularly of the maxillary incisors, are noted. The approach to this problem includes:

1. referring the child to a dentist, assessing for bottle feeding and fluoride in local water.
2. teaching the mother to brush the child’s teeth, encouraging vitamin supplements.
3. recommending fluoride mouthwash and toothpaste, suggesting celery sticks for snacks.
4. providing fluoride treatment, teaching dental hygiene, and eating foods high in calcium.

referring the child to a dentist, assessing for bottle feeding and fluoride in local water

Which of the following is typically observed in the GI system upon stimulation of the parasympathetic nervous system?

1. No overall effect
2. Increased sphincter tone
3. Increased peristalsis
4. Decreased secretions

increased peristalsis

Type 2 diabetes mellitus is characterized by which of the following?

1. Decreased production of exogenous glucagon-like peptide
2. Autoimmune beta-cell destruction
3. Relative deficiency of dipeptidyl peptidase-4
4. Decreased reabsorption of sodium-glucose transporters

decreased reabsorption of sodium-glucose transporters

In pulmonary function testing, forced vital capacity represents the:

1. maximum volume of air that can be forcefully exhaled after maximum inspiration.
2. total volume of air that is exhaled after normal inspiration.
3. total volume of air that the lungs can hold, minus the expiratory reserve volume.
4. volume of air that can be forcefully exhaled in 1 second.

maximum volume of air that can be forcefully exhaled after maximum inspiration

An adult male presents as first patient of the day after awaking about 5:00 AM with indigestion and chest pressure. On checking his pulse, he found no change from previous measurements. He states that, while waiting for the nurse practitioner to arrive, he began to feel a little clammy. A 12-lead EKG records the following patterns. On the basis of these EKG readings, the diagnosis is:

1. anterior-septal acute myocardial infarction.
2. lateral acute myocardial infarction.
3. inferior acute myocardial infarction.
4. anterior acute myocardial infarction.

inferior acute myocardial infarction

During the past 24 hours, a 62-year-old has experienced abdominal pain that radiates to the back. The patient also reports several episodes of nausea and vomiting, a low-grade temperature, and a history of excessive drinking. Physical examination reveals a distended abdomen. Laboratory serum values indicate elevated alkaline phosphatase, amylase, and serum lipase. The most likely diagnosis is:

1. alcoholic liver disease.
2. acute mesenteric ischemia.
3. viral hepatitis.
4. acute pancreatitis.

acute pancreatitis

An otherwise healthy adult female presents complaining of vaginal burning and itching during urination. She has a social history of two sexual partners. She denies fever or vaginal discharge, but has noted a lesion on one labium. The nurse practitioner should order all of the following EXCEPT:

1. wet mount and potassium hydroxide.
2. dark-field microscopy of fluid from the lesion.
3. nucleic acid amplification testing for gonorrhea.
4. serology for Haemophilus ducreyi.

dark-field microscopy of fluid from lesion

A 3-year-old female has been diagnosed with bacterial meningitis. She attends preschool daily at a local church day care program. The day before her diagnosis, she ate lunch with her mother at a local restaurant. For which contact(s) would chemoprophylaxis with rifampin be recommended?

1. Preschool contacts in the past 7 days
2. A friend of the patient’s sister who visited after school yesterday
3. The checkout employee at the grocery store yesterday
4. Diners at the restaurant during lunch the day the patient ate there

preschool contacts in past 7 days

Congenital dysplasia of the hip:

1. more commonly affects both hips.
2. occurs more commonly in females.
3. is correctable at any age.
4. typically self-corrects by 12 weeks of age.

more commonly affects both hips

Anticholinergic agents such as ipratropium (Atrovent) and tiotropium (Spiriva) are used in COPD primarily to:

1. induce bronchodilation.
2. decrease airway inflammation.
3. expand the lung fields.
4. treat hypoxemia.

induce bronchodilation

The clinical presentation of placenta previa that develops during the third trimester includes:

1. intermittent dark-red spotting.
2. occult bleeding with abdominal or back pain.
3. sudden onset of painless and profuse bleeding.
4. painless occult bleeding that becomes visible upon onset of labor.

sudden onset of painless and profuse bleeding

A 27-year-old male patient, who works as a janitor, presents with a 6-month history of an intermittent rash on his hands. History reveals itching and occasional burning. Examination reveals irregularly-distributed scaly maculopapular erythematous patches extending from the dorsum of the hand several inches up the forearms, and dry palms with no nail involvement. The most likely diagnosis is:

1. contact dermatitis.
2. scabies infestation.
3. psoriasis.
4. eczema.

contact dermatitis

Anorexia nervosa occurs most commonly in which of the following?
1. Painters
2. Only children
3. High-level athletes
4. Individuals from large families

high level athletes

A frail elderly patient presents with constipation. Which of the following normal physiologic changes seen with aging is the most likely cause?

1. Decreased bowel muscle tone
2. Increased bile secretion
3. Increased absorption of calcium
4. Decreased pancreatic secretions

decreased bowel muscle tone

A patient reports to your office, having been struck on the right cheek with a baseball bat. On examination, you notice tenderness and swelling over the cheek. The patient is unable to gaze laterally with his left eye, and he complains of double vision when he attempts to do so. This is most likely associated with:

1. retinal hemorrhage.
2. orbital rim fracture.
3. subarachnoid hemorrhage.
4. periorbital cellulitis.

orbital rim fractures

A 7-year-old child presents with group A streptococcal infection confirmed by throat culture. Past history includes treatment for positive streptococcal infection with erythromycin (EryPed oral suspension) 3 weeks ago. What is the most appropriate next intervention?

1. Treat with amoxicillin for 10 days.
2. Obtain culture for all household members.
3. Treat with azithromycin (Zithromax) for 10 days.
4. Intramuscular Penicillin.

treat with amoxicillin for 10 days

A 16-year-old female adolescent is brought to the nurse practitioner’s office by her mother, who is concerned about her daughter’s recent weight loss. History reveals that the daughter was consistently in the 50th percentile for weight, but is now in the 10th percentile. The mother states that she is concerned that her daughter is purging herself after meals since she often goes to the restroom after eating and remains there for a long period of time. The daughter denies any self-induced vomiting, starvation or excessive activity. She does state that she jogs five miles a day and is in good condition. In addition to a complete blood count with differential, which of the following laboratory tests will be most helpful for further assessment?

1. Electrolytes, fasting blood sugar
2. Creatine phosphokinase, follicle-stimulating hormone (FSH)
3. Electrolytes, blood urea nitrogen (BUN)/creatinine, urinalysis
4. Electrolytes, FSH, stool for occult blood

electrolytes, blood urea nitrogen (BUN)/creatinine, urinalysis

An adult female who is homeless presents for an initial obstetric visit at 34 weeks of pregnancy. She is diagnosed with Chlamydia trachomatis infection. What complication will her newborn be most at risk?

1. Conjunctivitis
2. Hearing loss
3. Pneumonitis
4. Meningitis

conjunctivitis

A 2-year-old male presents for reevaluation. Two days ago, he had four episodes of vomiting and six diarrheal stools. On physical examination today, his vital signs reveal P = 120 and capillary refill of 3 seconds. The patient’s eyes are sunken and his extremities are mottled and cool to the touch. The treatment plan would include:

1. start oral electrolyte mixture every hour.
2. hospitalization for IV fluid replacement.
3. giving the child loperamide for each diarrheal stool.
4. catheterizing the child to obtain accurate urine output.

hospitalization for IV fluid replacement

A nurse practitioner places a 76-year-old patient on nifedipine (Procardia) 10 mg t.i.d. for angina. The patient is unable to remember to take the medication at the scheduled times. The practitioner should:

1. discontinue the issue with the patient’s daughter.
2. change the dose to extended release 30 mg daily.
3. reinforce the importance of taking the medication.
4. increase the dosage to 20 mg b.i.d.

change the dose to extended release 30mg daily

A pediatric male is present with his mother, who states that he is extremely restless at night and constantly scratches his anal area. Which diagnostic procedure would confirm a diagnosis of pinworm infestation?

1. The patch test
2. Examination of bed linens
3. Examination of transparent tape sample from perianal skin
4. Blood tests for anemia and eosinophilia

examination of transparent tape sample from perianal skin

Which of the following microorganisms are most frequently associated with acute bacterial rhino-sinusitis?

1. Staphylococcus aureus and Mycoplasma pneumonia
2. Staphylococcus aureus and Methicillin Resistant Staph aureus
3. Streptococcus pneumoniae and Haemophilus influenzae
4. Streptococcus pneumoniae and Pseudomonas aeruginosa

strep pneumo and haemophilus influenzae

A child is being treated with methylphenidate (Ritalin) for attention-deficit disorder. The following are all side effects of Ritalin EXCEPT:

1. muscle cramps
2. mild irritability.
3. increased heart rate.
4. slight increase in growth velocity.

slight increase in growth velocity

A geriatric female presents for her annual examination. She has been on antihypertensive medications for over 20 years, with good control. Laboratory values are within normal ranges. The nurse practitioner is concerned about the patient’s cardiac health risks, due her to weight and her waist circumference. According to the AHA guidelines, which of the following goals is expected for this patient?

1. Physical activity for 60 minutes daily, for a minimum of 6 days a week
2. Physical activity for 30 minutes daily, 7 days a week
3. Physical activity for 30 minutes daily, for a minimum of 5 days a week
4. Physical activity for 60 minutes daily, 7 days a week

physical activity for 30 minutes daily, for a minimum of 5 days a week

The management of COPD in the elderly is best guided by:

1. spirometry.
2. arterial blood gases.
3. radiologic imaging.
4. symptomatology.

symptomatology

A young, athletic adult presents with shoulder pain and inability to raise the arm above the shoulder. The most likely diagnosis is:

1. frozen shoulder syndrome.
2. cervical radiculopathy.
3. rotator cuff tear.
4. polymyositis.

rotator cuff tear

A 50-year-old male presents with bruises and abrasions on the left leg and forearm, claiming to have had a number of minor accidents at home and at work. The patient reports general feelings of depression, forgetfulness, and, despite the fact that he is regularly employed, a mounting collection of unpaid and overdue bills. In taking the patient’s history, a significant pattern of parental alcohol abuse, stroke, TIA, and MI emerges. During questioning to elicit a more thorough diagnosis, the patient becomes hostile and leaves the clinic without further treatment. The most likely explanation for this behavior is:

1. Alzheimer’s disease.
2. chronic alcoholism.
3. cancer.
4. carotid artery stenosis.

chronic alcoholism

A 28-year-old patient presents with profuse yellow vaginal discharge, odor, and local irritation. She reports completing a 7-day course of oral metronidazole (Flagyl), 500 mg b.i.d. 4 weeks ago. She has had multiple recurrences over the last 18 months. Microscopic examination reveals presence of clue cells. What is the most appropriate treatment intervention?

1. Oral metronidazole (Flagyl), 500 mg b.i.d., plus metronidazole vaginal gel (MetroGel), for 5 days
2. Treat both patient and partner(s) with oral metronidazole (Flagyl), 500 mg b.i.d., for 7 days
3. Metronidazole vaginal gel (MetroGel) twice weekly for 4 to 6 months
4. Treat patient with metronidazole vaginal gel (MetroGel) b.i.d. and partner(s) with tetracycline, 250 mg q.i.d., for 7 days

Treat both patient and partner(s) with oral metronidazole (Flagyl), 500 mg b.i.d., for 7 days

An adult female who recently returned from a business trip to Japan presents for a recheck appointment. The only remarkable laboratory result is for thyroid-stimulating hormone (TSH), at 0.3 microunits/mL (normal = 0.4-6 microunits/mL). The patient reports that her neck hurts; examination reveals thyroid tenderness. Which of the following laboratory tests should the nurse practitioner order now?

1. Triiodothyronine (T3) and free thyroxine (FT4)
2. Triiodothyronine (T3) only
3. Triiodothyronine (T3) resin uptake assay
4. Triiodothyronine (T3) and free triiodothyronine (FT3)

Triiodothyronine (T3) and free thyroxine (FT4)

In treating a pregnant female with migraine headaches, which of the following drugs is in Category X?

1. Ergotamine tartrate (Ergomar)
2. Sumitriptan succinate (Imitrex)
3. Frovatriptan (Frova)
4. Amitriptyline (Elavil)

ergotamine tartate (ergomar)

A nurse practitioner is evaluating an infant for possible colic. Which of the following could indicate the need for a more extensive evaluation?

1. 3-oz (85 g) weight gain over the past 2 weeks
2. Stool negative for occult blood
3. Moist mucus membranes and flat fontanels
4. Onset at 4 weeks of age

3-oz (85 g) weight gain over the past 2 weeks

Which of the following is a hallmark of lumbar spinal stenosis?

1. Incontinence of bowel and/or bladder
2. Point tenderness of the lumbar spine
3. Leg, buttock or back pain precipitated by walking.
4. Bilateral leg pain with sitting.

leg, buttock or back pain precipitated by walking

A 70-year-old patient presents to the clinic complaining of dyspnea, palpitations, and fatigue. The patient reports a 2-week history of blackened stools, which the patient attributes to drinking berry juice. Assessment reveals vital signs of BP = 110/60, P = 100, R = 24; Hgb = 4.5 g/dL; Hct = 16%. What is the most appropriate immediate intervention?

1. Order serum iron, total iron-binding capacity (TIBC), and ferritin.
2. Refer to a gastroenterologist.
3. Send to the emergency room.
4. Order a complete blood count (CBC) with differential.

Send to emergency room

The symptoms of benign prostatic hypertrophy are often overlooked because the older male:

1. understands that surgery is the only cure for this condition.
2. considers urinary problems to be a normal part of aging.
3. is fearful of the side effects of additional medication.
4. is concerned that he will no longer be able to obtain an erection.

considers urinary problems to be normal part of aging

Upon admission to a nursing home, an 85-year-old patient receives a negative result on the purified protein derivative (PPD) test. To determine TB status in the frail elderly, a second PPD should be performed in:

1. 72 hours.
2. 1 week.
3. 3 months.
4. 6 months.

1 week

Which of the following is NOT used for therapy of an acute attack of gout?

1. Colchicine (Colcrys)
2. Indomethacin (Indocin)
3. Methylprednisolone (Solumedrol)
4. Allopurinol (Zyloprim)

allopurinol (zyloprim)

A 26-year-old female presents with abdominal distention, bloating, and intermittent crampy abdominal pain relieved by defecation. She has four to six loose stools a day when stressed. Between bouts of diarrhea, she often has constipation. Physical examination results are normal. The CBC was normal. The most likely diagnosis is:

1. diverticulitis.
2. viral gastroenteritis.
3. inflammatory bowel disease.
4. irritable bowel syndrome

irritable bowel syndrome

A mother has just noticed that her 15-month-old, 15-kg child has recently ingested an unknown quantity of iron tablets. The mother has ipecac syrup and activated charcoal on hand. The mother should be advised to:

1. take the child to the primary care provider.
2. give ipecac per recommendation on the bottle label.
3. take the child to the emergency room.
4. give the child 30 g of activated charcoal diluted with water

take child to emergency room

A 13-year-old is concerned because she has not yet begun to menstruate. Physical examination indicates that the patient is at Tanner stage IV and is of average height and weight. Which of the following would be the most appropriate response to this patient?

1. “We’ll need to refer you to an endocrinologist for a complete workup.”
2. “We’ll need to do some tests to find out why you are not having periods.”
3. “I’ll give you some pills that will make your periods start.”
4. “Your development is exactly as expected for your age; you’ll probably begin to have periods within a year.”

“Your development is exactly as expected for your age; you’ll probably begin to have periods within a year.”

The most common bacterial cause of cellulitis in patients with diabetes mellitus is:

1. Pseudomonas aeruginosa.
2. group B beta-hemolytic Streptococcus.
3. Staphylococcus aureus.
4. Staphylococcus saprophyticus.

Staphylococcus aureus

A 46-year-old female found a lump in her breast this morning. History includes: no prior breast disease; G2P2 (first birth, age 22); maternal aunt diagnosed with breast cancer at age 72; last menstrual period was 2.5 weeks ago. On examination, a nurse practitioner palpates a 2-cm round, soft, mobile, tender lesion in the upper outer quadrant of the right breast. Mammogram is negative. Which of the following actions is most appropriate?

1. Reassure the patient that no further action is necessary.
2. Repeat unilateral right mammogram in 3 months.
3. Schedule a right-breast ultrasound.
4. Refer for genetic testing if the lump increases in size.

schedule a right breast ultrasound

Risk factors for genital herpes include:

1. multiple sexual partners and years of sexual activity.
2. genetic transmission.
3. infection with other sexually transmitted diseases before the age of 12.
4. multiple sexual partners and frequent use of public spas.

multiple sexual partners and years of sexual activity

On physically assessing a newborn’s head, you note an enlarged, fluid-filled area that is subperiosteal and does not cross suture lines. This finding indicates the presence of:

1. macrocephaly.
2. subgaleal hemorrhage.
3. congenital hypothyroidism.
4. cephalohematoma.

cephalohematoma

A 65-year-old patient being treated with digoxin (Lanoxin) for chronic heart failure (HF) presents with complaints of palpitations. Three days ago, the patient was placed on clarithromycin (Biaxin) by another provider for community-acquired pneumonia. The appropriate course of action at this point would be to:

1. increase the digoxin dosage by 25%.
2. obtain a digoxin level and change the antibiotic to azithromycin (Zithromax).
3. obtain a chest X-ray.
4. obtain a serum potassium level and perform a 12-lead EKG.

obtain a digoxin level and change the antibiotic to azithromycin (zithromax)

A 54-year-old male who is 30 pounds overweight states that he awakens at night with heartburn and the taste of hot acid in his mouth. Stress makes his condition worse, yet baking soda seems to provide some relief. Findings on examination are normal and the stool for occult blood is negative. The preliminary diagnosis(es) should be:

1. esophagitis.
2. esophageal spasm.
3. helicobacter pylori.
4. gastroesophageal reflux disease.

GERD

A late adolescent patient presents with complaints of acute-onset unilateral right eye pain, visual changes, seeing halos around lights, abdominal pain, and intermittent nausea. Physical findings include an erythematous right eye without discharge, a cloudy cornea, and a moderately dilated right pupil that is nonreactive to light. The nurse practitioner should immediately:

1. apply steroid drops to the right eye.
2. apply mydriatic drops to the eye.
3. refer to the emergency department.
4. make an appointment for him to see an ophthalmologist.

refer to emergency department

In a 6-year-old child, which of the following is the preferred drug for stage 1 Lyme disease?

1. Trimethoprim-sulfamethoxazole (Bactrim)
2. Azithromycin (Zithromax)
3. Doxycycline (Doryx)
4. Cefuroxime (Ceftin)

cefuroxime (ceftin)

An 86-year-old patient is experiencing radiculopathy-associated lower back pain that has not improved over the past 4 weeks. The medical history includes a lumbar discectomy. The best type of imaging study for evaluating this patient would be a/an:

1. MRI.
2. spinal X-ray.
3. myelography.
4. nuclear bone scan.

MRI

A 30-month-old with a history of frequent ear infections was seen in September after a symptom-free summer. Examination revealed bilateral bulging eardrums. After two separate courses of antibiotics, fluid is noted in both ears. The parent reports speech difficulties. The most likely diagnosis is:

1. otitis media with effusion.
2. myringitis.
3. recurrent acute otitis.
4. Esutachian tube dysfunction.

otiti media with effusion

A 19-year-old patient who has used oral contraceptives for 3 years plans to discontinue the pill at the end of her current cycle to become pregnant. Which of the following daily supplements would receive the highest priority?

1. Calcium carbonate
2. Folic acid
3. Ferrous sulfate
4. Multivitamin

folic acid

A frail elderly widow presents to the senior clinic with a 3- to 4-day history of confusion that became worse the evening before but currently seems a little better, although her speech is rambling and difficult to follow. During the examination, the patient is anxious and agitated. She is on cardiac medications, an antidepressant, and insulin. The most likely diagnosis is:

1. mild stroke.
2. polypharmacy.
3. electrolyte imbalance.
4. agitated depression.

polypharmacy

A 46-year-old female presents with a complaint of amenorrhea for 3 months. The first-line diagnostic workup is:

1. urine pregnancy test.
2. progesterone challenge.
3. serum prolactin level.
4. thyroid-stimulating hormone (TSH).

urine pregnancy test

Leave a Comment

Scroll to Top